Anda di halaman 1dari 372

Biology

Parti
Physiology
Sections I-V
Section I Nerve and Muscle

Section II

Heart and Lung


Section III Gastrointestinal Tract

and Kidney
Section IV

Reproduction and Development


f Section V

Endocrinology
and Immunology

The

Berkeley
JUr*e*v*i*e*w
Specializing in MCAT Preparation

ERKELEY
P.O. Box 40140, Berkeley, California
Phone:
Internet:

R E V ?^ E W
843-8378
(510)

94704-0140
THE-TEST

(510)

MCATprep@berkeleyrcview.com

http://www.berkeleyreview.com

The Berkeley Review and The Berkeley Review logo are registered trademarks of The Berkeley Review.

art was created using numerous graphics programs designed tor use on Macintosh computers. The majority of the
text type and display type was set in Times Roman and Palatine

This publication for The Berkeley Review was written, edited, and composed on a desktop publishing system using Apple Macintosh computers and Microsoft Word. Pages were created on theApple LaserWrite Pro. Line

Cover Design by MacGraphics.

Copyright 2011, 2010, 2009, 2007, 2005, 2003,2001,2000,1995,1994,1993,1992 by The Berkeley Review. All rights
reserved.

copyright owner.

means, electronic, mechanical, photocopying, recording, or otherwise, without the prior written permission of the

No part of this publication may be reproduced, stored in a retrieval system, or transmitted, in any form or by any

Biology
Section I
Nerve and Muscle
The Nervous System
1. Types of Tissues
2. Membrane Potentials
3. Action Potentials

4. The Neuromuscular Junction


5. The Sarcomere

6. Nervous System Components 7. Control of Body Activity

8. Receptors and Sensory Input

Practice Passages and Answers

BERKELEY
Specializing in MCAT Preparation

Nerve and Muscle


Top 10 Section Goals
Be familiar with the different types of tissues.

*
Q jja

The four general classes of biological tissues to consider are epithelial tissue, connective tissue,
muscle tissue, and nerve tissue.

) '^ Understand the concept of a membrane potential.


Know the relative intracellular and extracellular concentrations of ions like Na, K, and Cle. Be

familiar with the Nernst equation and how to calculate a membrane voltage.

Know the different phases of the action potential. Understand depolarization, repolarization, hyperpolarization, and refractory period. Be familiar with
the differences oetween action potentials in skeletal muscle and cardiac muscle.

O -im Be familiar with the neuromuscular junction. Understand how a signal is propagated from thepresynaptic membrane to thepostsynaptic membrane.
Know how neurotransmitters are released and how they are degraded.

0 v^ Be familiar with muscle contraction and the sarcomere.

the differences between actin, myosin, troponin, tropomyosin, and their interactions with Ca2.

Be aware of how the different components of muscle interact in order to contract and relax. Know

@^

Be familiar with the components of the nervous system.


The three primary divisions of the brain are the forebrain, midbrain, and hindbrain. Know the components of each and how they relate to the body as a whole.

q ^^ Be able to distinguish between consciously directed and involuntary muscle activity.

^5f Understand the differences between motor and sensory neurons and how they relate to the spinal
^ cord. Be able to describe the structure and function of a simple reflex arc, and of smooth versus
skeletal muscle.

9! p

Know the different divisions of the nervous system.

The two major divisions are the central nervous system (CNS) and the peripheral nervous system
(PNS). Know the subdivisions of the PNS and their actions.

q ^^ Know the types of neurotransmitters that are released from the CNS and PWS.
These neurotransmitters include acetylcholine, epinephrine, and norepinephrine. Understand why
some neurotransmitters or hormones are called amines, peptides, or steroids. Be familiar with the different types of sensory input received by the brain.

This sensory input can come from mechanoreceptors, chemoreceptors, thermoreceptors, and photoreceptors. Be familiar with the action ofeach receptor type.

Biology

Nerve fif Muscle

Types Of Tissues

The Nervous System


Types Of Tissues
Let's consider a series of discussions on cellular physiology. Forexample, we will consider how muscle and nerve cells function. How does the chemical energy of
ATP (which was generated in glycolysis, the Krebs cycle, and oxidative

phosphorylation) become converted into the mechanical movement of, say,


muscle cells? How is it that the chemical energy of ATP is converted into an
electrical signal that allows various nerves to communicate with those muscles?

Before we can discuss the cellular mechanisms of muscles and nerves, we first

need to consider some of thegeneral characteristics ofcells, tissues, and organs.


The general body plan of an animal is fairly simple and can be divided into a number ofsystems that represent a variety oforgans working in concert with one another. For example, one body system you are probably quite familiar with is

the skeletal system. Another is the muscular system. Others are the circulatory,
integumentary (skin), endocrine, nervous, and digestive systems, to name but a
few.

The digestive system is formed by an alimentary canal (gastrointestinal "tube") that begins at the mouth andends at the anus. This system issuspended within a body cavity referred to as the coelom. The coelom is separated into a thoracic cavity (upper) and anabdominal cavity (lower). These two cavities are separated by the dome-shaped mass of skeletal muscle called the diaphragm. Within the thoracic cavity, one finds the lungs and the heart. The abdominal cavity contains
the liver, stomach, and intestines.

As we examine these various systems, we will find different levels of

organization. There are individual cells, and then there are cells of a particular type which coalesce to form tissue. One example of a tissue is the layer of epithelial cells that line one of the principal organs of the alimentary canal, the
stomach. Some of the simple epithelial cells within the stomach secrete

hydrochloric acid (pH 1) to aid in the digestion offood. Other epithelial cells of the stomach secrete mucus to help prevent that acid from digesting the lining of the stomach. Still other epithelial cells secrete enzymes. These epithelial cells are
just one type of tissue that is involved in forming the stomach. The stomach is

also composed of other types of tissue. For example, nervous tissue helps to innervate thestomach, connective tissue helps to hold the stomach in its proper
position, and muscle tissue helps to propel food through the stomach. Thus, these four groups of primary tissue (epithelial, connective, muscle, and nerve)

have the ability to form the various organs of the body. An organ is a structure

that is composed oftwo or more tissues that act in such a way as to perform a specific
function.

Epithelial Tissues Let's examine the epithelial cells in a little more detail. The epithelial tissue that constitutes the various organs of the body can be either simple epithelium (consisting of a single layer of cells) or stratified epithelium (consisting of two or more layers of cells). These epithelial cellscome in a variety of shapes and sizes. For example, there are squamous (flat), cuboidal, and columnar epithelial cells
(refer to Figure 1-1).
Copyright by The Berkeley Review
The Berkeley Review

Specializing in MCAT Preparation

Biology

Nerve & Muscle

Types Of Tissues

On the lumenal side of the simple epithelial cells are projections called microvilli (singular, microvillussee Figure 1-1 and Figure 1-2). These

projections increase the total absorptive area of the cell (sometimes by as much as 25%). Sometimes you find specialized structures called cilia {singular, ciiium)

projecting outward on the apical surface of these cells. For example, in the respiratory tract these hair-like appendages move in a coordinated unidirectional
wave to move foreign particles out of the mucous lining of the lungs and
bronchial tubes.

microvilli

Basal

Simple squamous epithelial cells

lamina

Simple columnar epithelial cells

Basal lamina

Cuboidal and columnar

epithelial cells

Stratified squamous epithelial cells (non-keratinized)

Figure 1-1 Types of epithelial cells.

These cells are bounded by a number of specialized junctions. For example, tight junctions act as a permeability barrier (see Figure 1-2). Not only do they prevent the transport of protein molecules from the lumenal side of the cell towards the basolateral side of the cell, but they also act to hold neighboring cells together.

Epithelial cells are also held together by structures called desmosomes (see Figure 1-2) One type of desmosome joins the epithelial cell to a structure on the
basal side of the cell called the basal lamina (or basement membrane). The basal lamina is in close contact with connective tissue that helps to anchor the cells in place.

Gap junctions provide a means for water-soluble molecules to pass from the cytoplasm of one cell to the cytoplasm of another cell (see Figure 1-2). These junctions allow for equilibration within the connected epithelial cells and
therefore allow those cells to function as a unit. For example, the beating cilia

appear to be coordinated by waves of calcium, which flow in the plane of the


juxtaposed epithelial cells.

Copyright by The Berkeley Review

The Berkeley Review

Specializing in MCAT Preparation

Biology
LUMEN
Microvilli

Nerve fie Muscle

Types Of Tissues

Apical surface

^Zl Microvillus
\
Tight junctions'
Gap junction
Desmosome

I Tight junctions
Gap junction

Desmosomes

Basal lateral IZ^


surface

I Basal lamina
(basement membrane)

BLOOD
Figure 1-2 Different components of an epithelial cell.

As we have mentioned, epithelial cells can secrete substances into a lumenal space. For example, hydrochloric acid can be secreted into the lumen of the stomach. If a cell secretes a substance into the lumen by way of a duct, it is referred to as an exocrine gland. Endocrine glands secrete substances into the blood. For example, insulin is a protein hormone secreted into the blood by clusters of specialized epithelial cells in the pancreas.
Dead skin cells

SI (keratinized)

Epidermal
cells

Basement
membrane

^
*

Connective

Collagen fibers fN
Figure 1-3
Stratified squamous epithelial cells.

tissue

Stratified squamous epithelium usually has a protective function. Your skin is composed of many layers of stratified squamous epithelial cells. The outer cells of your skin are dead, and they contain a large amount of the fibrous protein keratin (Figure 1-3). These cells are constantly being lost and replaced, as cells begin to move toward the surface from below.

Copyright by The Berkeley Review

The Berkeley Review Specializing in MCAT Preparation

Biology

Nerve fie Muscle

Types Of Tissues

Consider a segment of skin. This organ comprises about 15% of your total body weight. The epidermal region contains stratified epithelial cells that act to protect the deeper layers of the skin. Below the epidermis is the dermis. Within the dermis are a variety of structures. Surrounding the hair follicles are erector muscles, which act to straighten the hair shaft. This causes the skin close to the hair follicle to become depressed and gives the characteristic appearance of "goose pimples." Those erector muscles are innervated by nerves which cause
them to contract at specific times (e.g., when it is cold outside). The skin is also a highly vascularized organ. When it is hot outside, the blood is shunted towards the surface of the skin where it can dissipate some of its heat to the outside
environment. Below the dermis is the subcutaneous tissue. This is where one

finds adipose deposits.


Connective Tissues

This type of tissue helps to anchor and support the various structures of the body. There are a variety of types of connective tissues, a few of which are structural, blood cells, mast cells, adipose cells, and melanocytes.

Many of the proteins that make up structural connective tissue are secreted by cells called fibroblasts. Collagen, reticulin, and elastin are structural proteins which are secreted by these cells. Collagen is a triple-stranded, insoluble, fibrous protein (see Figure 1-3) that is highly cross-linked, a feature that makes these fibers quite strong and rather flexible. Besides having a very high tensile strength, collagen is also the most abundant protein found in mammals. Reticulin is a thin fiber found in the spleen and lymph nodes. It is not as highly coiled as collagen. Elastin is also a highly cross-linked protein found associated with organs that require some degree of elasticity (like the lungs, skin, and blood
vessels).

Another type of structural connective tissue, cartilage, is secreted by a specialized fibroblast cell called a chondrocyte. There are different types of cartilage, but in general it is found in places where there is a certain amount of stress placed on the body. For example, cartilage can be found in the nose, on the articulating surfaces of bones (including the intervertebral discs of the vertebral
column), and in the external ear.

Bone is also a structural connective tissue. About one-third of the weight of bone comes from organic material such as collagen, while the remaining two-thirds is inorganic material such as calcium phosphate and calcium carbonate. The collagen found in bone matrix is secreted by specialized fibroblast cells called osteoblasts. Collagen lends flexibility to bone, while the inorganic crystals lend
rigidity. Within the central cavity of bone, we find a spongy marrow where red
blood cells and white blood cells are formed. Towards the surface of bone the

cellular arrangement is more compact. [As a comparison, the main structural component of chitin (found in the exoskeleton of insects) consists of specially modified glucose residues linked to one another to form long polymers.

Associated with these polymers is calcium carbonate (CaCC>3). This combination adds rigidity to the exoskeleton,but offers little in the way of flexibility.]
We mentioned that blood cells and mast cells are kinds of connective tissue. We

will discuss blood cells in a separate lecture. Mast cells can be found in the respiratory tract, as well as in the gastrointestinal tract. Mast cells can release

Copyright by The Berkeley Review

The Berkeley Review


Specializing in MCAT Preparation

Biology

Nerve & Muscle

Types Of Tissues

histamines in response to an allergic reaction, an infection, or even an injury.


Histamine causes an increase in blood flow to the blood vessels of the affected

region.

Other types of connective tissue involve adipose cells and melanocytes. Adipose cells are simply cells that store fat whereas melanocytes are cells which store
pigments.
Muscle Tissues

We will be discussing various types of muscle in future lectures. For example,


when we examine skeletal muscle we will find that it is voluntary muscle. That is, we can generally control its action. Cardiac muscle and smooth muscle are examples of involuntary muscles.
Nervous Tissues

The nervous systems allow one to adapt rather quickly to external stimuli. For example, consider a simple reflex arc. If someone were to tap on your knee with a rubber hammer, your lower leg would extend outward. As the hammer

impinged upon the patellar tendon in your knee, an electrical impulse was generated and traveled via a sensory nerve to your spinal cord. That sensory neuron synapsed with a motor neuron, which returned the impulse to the
muscle that was initially stimulated and caused it to contract. We will come back to this example and examine it in a bit more detail later. First, let's consider some terminology.

Dendrites
Axon

Neurotransmitters
are released from

synaptic bulbs -

A Typical Neuron
Cell body
Figure 1-4
The major components of a neuron.

Nerve cells and associated supporting cells make up the nervous system. Nerve cells are also called neurons, and they are the basic structural unit that make up the nervous system. The major anatomical features of a neuron are the cell body (involved in integration of information), the dendrites (involved in receiving and transmitting information towards the cell body), and the axon (involved in conducting information away from the cell body). When a neuron becomes excited and receives electrical information in the form of a stimulus, the cell body processes that information and transmits it down the axon in the form of a nerve impulse called an action potential. When that action potential reaches the endjof
the axon (referred to as the synaptic bulb or bouton terminal), it causes the
Copyright by The Berkeley Review

The Berkeley Review Specializing in MCAT Preparation

Biology

Nerve fif Muscle

Types Of Tissues

release of a chemical substance called a neurotransmitter (see Figure 1-4). The neurotransmitter diffuses across the synaptic cleft and induces an identical action potential in an adjoining neuron, muscle cell, or gland cell. The junction between two such cells is called a synapse.

Copyright by The Berkeley Review

The Berkeley Review


Specializing in MCAT Preparation

Biology
Membrane Potentials

Nerve & Muscle

Membrane Potentials

The generation of electrical signals in the nervous system is concerned with the diffusion of ions from a high to a low concentration (see Figure 1-5). In other words, charged ions diffuse down their concentration gradient. In the

extracellular space of vertebrates the concentration of Na is about 150 mM, while that of K is about 5 mM. The concentration of Cle is about 130 mM, while that of HC03e is about 25 mM. Note that the concentrations of the cations (Na and K) and the anions (Cle and HCC>3e) balance one another. In other words, we find 155 mM of the cations and 155 mM of the anions. This represents
electroneutrality.
Inside Cell
>

Outside Cell

K+ = 120 mM to 140 mM *
Cations

K+ = 5 mM
Na+=150mM

Na+=10mMtol5mM

Cr=5mMto40mM

^
> Anions

Cr=130mM

HC03-=12mMto25mM
Proteins" .;

HC03- = 25 mM

Figure 1-5
Typical cellular concentrations of the common ions.

Within the cell, we find a high concentration of K (about 120 mM to 140 mM) and a low concentration of Na (about 10 mM to 15 mM). We also find a lower

concentration of Cle (about 5 mM to 40 mM) and usually a lower concentration of HCC>3e (about12mM to 25 mM). There are manynegatively charged proteins
within the cell. Electroneutrality can also be found on the inside of the cell as
well.

If you look at the distribution of ions across the cell's membrane, you will find it to be asymmetrical. Let's consider a resting nerve (i.e., a nerve that is waiting to generate an action potential) that has a permeability to potassium which is much

greater than its permeability to sodium. In other words, PK > PNa (where "P" refers to permeability). Because the concentration of K is higher inside the
cell than outside the cell, potassium will diffuse down its concentration gradient

and leave the cell (see Figure 1-6). As the positive K cation leaves the cell, there
is correspondingly less positive charge remaining inside the cell. In other words, the inside of the cell is now more negatively charged with respect to the outside

of the cell. This sets up a voltage that is positive on the side of the cell to which potassium is trying to diffuse to (i.e., the outside of the cell). As that positive voltage begins to build up, it tends to push potassium back into the cell (remember, like charges repel each other).

Copyright by The Berkeley Review

The Berkeley Review Specializing in MCAT Preparation

Biology

Nerve S Muscle

Membrane Potentials

Those two forces (chemical and electrical) do not exactly match each other. It turns out that the force of diffusion is a little larger than the electrical force. This results in a little bit of leakage of K out of the cell, as well as a little bit of

leakage ofNa into the cell. The K thatleaked out of thecell has tobepumped back into the cell, and theNa that leaked into the cell has to be pumped out of the cell. The pumping action of these two ions is provided for by the Na/K
ATPase pump. This pump is responsible for the generation of the asymmetrical concentration gradient of Na and K across the cell'smembrane.
Cell membrane

Inside K*

cell

Voltage ^

1 -<-|

Diffusion

Outside

K+

cell Where

<^Na+
-87mV
Na+

PK > PNa
+ 87mV
and the anions are

f(I
Figure 1-6 Cellular gradients where PK > PNa.

^
K+

impermeable

We can calculate the membrane voltage (the potential difference) across the cell's membrane using the Nernst equation as shown in (1-1). In this equation, V is the

voltage in millivolts (1 mV = 10"3 volts), i refers to inside, o refers to outside, R is


the gas constant, T is the temperature in Kelvin, Z is the ion's valance, and F is

the Faraday constant. If we let the cell's membrane be permeable to just K, we find that the voltage is -87 mV inside the cell with respect to outside the cell. Remember, this is if potassium is the only permeable ion. It is the gradient of potassium alone across the cell's membrane that is able to generate this potential.

Vio = 2.3 RT log


ZF

[K+]0 [K+]i

(M)

Vio =60 log [5mM]


Vi0 = - 87 mV

[140mM]i

(1-2)

(1-3)

Copyright by The Berkeley Review

10

The Berkeley Review Specializing in MCAT Preparation

Biology

Nerve &? Muscle

Membrane Potentials

If we stimulate a nerve, it leaves its resting state and enters an active state in which the cell's membrane is more permeable to sodium (Na) than it is to
potassium. In other words, PNa PK. Since the concentration of sodium is higher outside the cell than inside the cell, sodium will tend to diffuse down its concentration gradient and into the cell.

Once again, we can establish a separation of charge. As the Na ions enter the cell with their positive charges, there is that much less positive charge outside the cell. The outside of the cell becomes more negatively charged than it was before Na started to diffuse into the cell. Similarly, as the Na ions diffuse into the cell, the inside of the cell begins to accumulate more positive charge. The inside of the cell becomes more positively charged than it was before the Na ions started to diffuse in (see Figure 1-7). As the Na ions diffuse into the cell down their concentration gradient, a chemical and an electrical equilibrium is being
established.

Cell membrane

K
Inside cell

<

1
i
i ^ 4
4

^
K+

Where |
Na

Outside cell

# Diffusion

Na"1

Na+

Voltage

+ 60mV
Figure 1-7
Cellular gradients, where PNa PK.

-60mV

We can calculate the magnitude of the potential across the cell's membrane by using the Nernst equation. We find that the potential difference is +60 mV (inside the cell with respect to outside the cell).

d-4)

Vio =601og[150mM]
[15mM]i

(1-5)

Vio = + 60 mV
Copyright by The Berkeley Review

(1-6)
11

The Berkeley Review Specializing in MCAT Preparation

Biology

Nerve & Muscle

Membrane Potentials

Suppose we were to measure the potential across the plasma membrane of a neuron (nerve cell). We can do this by using two electrodes. A microelectrode is inserted into the neuron itself while another electrode is placed in the fluid that surrounds the neuron (see Figure 1-8). If we connect these two electrodes to a voltmeter that can read the potential difference between the two environments, we will find that the inside of neuron registers about -80 mV (with respect to the

extracellular space). Note that this voltage is quite close to the voltage that we

would get (-87 mV) if this plasma membrane were permeable only to K. The
reason that the inside of the neuron is not -87 mV is because the plasma membrane is not exclusively permeable to K.

80

\
microelectrode
Voltmeter

Extracellular Fluid

Axon

Figure 1-8 Measuring electrical potential across a plasma membrane.

Copyright by The Berkeley Review

12

The Berkeley Review Specializing in MCAT Preparation

Biology
Action Potentials

Nerve 8? Muscle

Action Potentials

We can reduce the plasma membrane potential by transiently increasing the

membrane's permeability to Na. This can be accomplished by stimulating the


nerve. The transient reduction in the membrane potential is referred to as a depolarization of the membrane. If this stimulus is strong enough and the initial depolarization exceeds a specific minimum value characteristic of the cell (called the threshold potential), then within milliseconds a burst of Na ions will enter the cell and generate an action potential (see Figure 1-9). As the depolarization

of the membrane continues, its permeability to Na becomes much greater than its permeability to K (i.e., PNa PK). What we find is that the membrane
potential goes from a resting membrane potential of about -80 mV to a

membrane potential that approaches that for Na. In this example, at the height
of the wave of depolarization, the membrane potential would be about +40 mV. [Why isn't the membrane potential +60 mV as would be calculated using equation (1-4)?]

Na+ equilibrium potential


**

( }/ V(C)
Action

(B) Depolarization

(A) Stimulus

(C) Repolarization
(E) Refractory

(D) Hyperpolarization

**PNa>PK

* ^K >:> FNa

Potential^ Threshold potential H* /

~f" (A)
2

\ - - -V - - - -- - -----^ (D) \!^^%

Resting potential
K+equilibrium potential

Time (msec)

Figure 1-9 A typical action potential.

Once the membrane potential reaches the equilibrium potential for Na there is no more influx of Na ions into the cell. At this point (indicated by the peak of
the action potential) the membrane potential, which is +40 mV inside the cell, is

balanced by the concentration gradient of Na. Recall that the concentration of Na outside the cellis greater than the concentration of Na inside the cell.
One millisecond after the burst of Na into the cell, the ion channels

(ionophores) that let Na into the cell close and become temporarily inactive.
When the Na channels are temporarily inactive they are said to be in a refractory period, which usually lasts for several milliseconds. During this time a

neuron will not be able to generate another action potential, because the Na
channels cannot open to allow the cell membrane to depolarize.

Copyright by The Berkeley Review

13

The Berkeley Review Specializing in MCAT Preparation

Biology

Nerve &Muscle

Action Potentials

Once the Na channels have closed, the K channels begin to open and K
begins to leave the cell. This phase of the action potential is referred to as

repolarization. As the cell membrane's permeability to K increases, a massive

amount ofK flows outofthecell and themembrane potential passes its original
resting state of about -80 mV. This event is called hyperpolarization (see Figure 1-9). Eventually the cell will reach equilibrium, and once again an appropriate stimulus will generate an action potential.

The generation of an action potential is said to be an all-or-none phenomenon. What does this mean? If we apply a stimulus to a nerve (see point (A) in Figure 1-9), the plasma membrane of the neuron will become depolarized. The membrane potential will become less negatively charged during the increase in the permeability of the plasma membrane to Na ions. Remember,Na wants to flow down its concentration gradient and into the cell. As Na enters the cell, the membrane becomes more depolarized. This increase in depolarization tends to

induce neighboring Na channels to open, thus letting more Na ions into the
cell. However, we know that the cell is dynamic rather than static; K ions are leaving the cell and flowingdown their concentration gradient. This action has a

tendency to repolarize the plasma membrane. In other words, the generation of an action potential depends on the ratio of membrane permeability to these two ions. If there is not a sufficient depolarization (i.e., influx of Na ions) of the
membrane, then there will be no action potential. However, if there is a sufficient

depolarization of the membrane because there are more Na ions entering the

cellthan K ions leaving the cell, then anaction potential will begenerated ifthat
depolarization exceeds some threshold value. Once an action potential is generated in a nerve cell it will always have the same magnitude. Thus, an action
potential is either all-or none. It either happens or it doesn't.

In Figure 1-9 we have indicated how an action potential is generated. We mentioned that in order to generate an action potential there must be a depolarization of the membrane abovesome threshold value, a repolarizationof the membrane, a hyperpolarization, and a refractory period that allows the membrane potential to return to its resting value before a new action potential can be generated. Now let's examine how an actionpotential propagates along a
nerve.

Consider the charge separation across the plasma membrane (see Figure 1-10) of
a nerve as an action potential moves from, say, the center of the axon outwards. Suppose we have a stimulus that exceeds a given threshold value. As the action potential is generated a separation of charge will be established. Because Na ions are rapidly entering the cell, the inside of the cell will become more positively charged (+) with respect to the outside of the cell. Since there is a

deficit of positive charge on the outside of the cell (compared to the original resting condition), we say that the outside of the cell is now more negatively charged (-) with respect to the inside of the cell. In other words, at that particular
place on the axon, where the action potential has been generated, there is a reversal in the membrane's polarity. This is shown in Figure l-10a. The depolarization of the central region of an axon spreads outward in both

directions as moreNa ionsenter the cell. In Figure l-10a the arrows representa
local flow of current between the area of the plasma membrane that has been depolarized and the adjacent areas of the membrane which are still at their
Copyright by The Berkeley Review 14 The Berkeley Review Specializing in MCAT Preparation

Biology

Nerve Si Muscle

Action Potentials

resting potential. The flow of positive charge within the cell causes the adjacent areas to become more depolarized. In other words, the polarity of the membrane is being decreased in front of the propagating action potentials. Because the

membrane potential in these adjacent areas is becoming depolarized, more Na channels begin to open. This lets more Na into the cell, which increases the
depolarization until an action potential is generated.

Axon
+ + + + + + + -^+^++ + + + + + + + +

= Na+ Channels

Figure 1-10
Action potential propagation.

Note that as the action potentials spread outward, they create a region which is refractory (see Figure l-10b). Recall that the refractory period is due to inactivation of the Na channels. If the plasma membrane has been depolarized from an action potential, then another action potential cannot be generated until that membrane has once again reached its resting membrane potential.

During the state of hyperpolarization, the K channels are wide open and K is
pouring out of the cell. Even though an action potential could be generated during this time (of hyperpolarization), it is rather difficult unless a strong enough stimulus is applied. Also, the action potentials in Figure l-10b appear backwards compared to the action potential in Figure 1-9, because the action potential in Figure 1-9 shows a fixed point on the axon and the membrane's potential variation with time at that point. The action potentials in Figure l-10b show a distribution of the membrane potential along an axon (not at a fixed
point) at a particular instant in time.

There are a many kinds of nerve fibers in the body, and they do not all conduct action potentials at the same rate. For example, large diameter neurons will
Copyright by The Berkeley Review
15

The Berkeley Review Specializing in MCAT Preparation

Biology

Nerve & Muscle

Action Potentials

conduct a depolarization or action potential further and faster than a small diameter neuron. What this means is that the ability of a given neuron to conduct a current (the flow of charge represents a current) depends on the cross-sectional area of that neuron. The larger the cross-sectional area of a neuron, the more cytosol in that neuron, and the more ions there will be to conduct that current.
The more ions available to conduct the current, the faster and farther that current

will spread.

The nerves which we have been discussing so far are referred to as unmyelinated nerves. Myelinated nerves have the ability to greatly increase the rate at which action potentials are conducted. Myelin is a specialized membrane

that is composed of just a few types of proteins and a large proportion of phospholipids. A specialized type of cell (called a glial cell) attaches itself to a section of an unmyelinated axon and begins to rotate itself around that axon a number of times. During this process a myelin sheath is deposited on the axon. Sometimes this myelin sheath is thicker than the axon itself. The deposition of

myelin along the axon is interrupted by areas where there is no myelin. These
areas are called the nodes of Ranvier. See Figure 1-11.
Action potential

JV

Myelin sheath
(membranes)

0-

Axon

tf
Plasma membrane

(contains a high density of Na+ channels)


Figure 1-11 Action potential propagation from node to node.

Node of Ranvier

Later we will learn that the body's nervous system is separated into two divisions (based on function). The central nervous system is made up of the brain and spinal cord while the peripheral nervous system represents the nerves in the periphery. The point here is that the glial cells in the central nervous system are called oligodendrocytes, while the glial cells in the peripheral nervous system
are called Schwann cells.

We mentioned that myelinated nerves increase the rate at which action potentials are conducted. The myelin that surrounds the axon acts as an electrical insulator and prevents the transfer of ions across the plasma membrane of the axon. The only region of the axon where ions can pass across the plasma membrane is at

the nodes of Ranvier. Thesenodes contain a high density of Na channels. As an action potential is generated, there is a flow of current through the cytoplasm of
the axon (called the axoplasm) and the extracellular fluid from node to node. The

plasma membrane at each of these nodes is in turn depolarized enough to


Copyright by The Berkeley Review
16

The Berkeley Review

Specializing in MCAT Preparation

Biology

Nerve &Muscle

Action Potentials

generate an action potential. The nerve impulse seems to "jump" from node to node along the axon. Thus, this type of impulse transmission is referred to as saltatory conduction (from the Latin saltatoriuspertaining to dancing or leaping). See Figure 1-11.
Nerve fibers can be grouped into different classes. The nerve fibers in some of these classes are myelinated, while those in other classes are unmyelinated. Some of the myelinated fibers that impinge on skeletal muscle can transmit motor impulses quite rapidly, about 60 to 120 m/sec. In contrast, some of the unmyelinated fibers which transmit sensory impulses of certain types of pain, do so rather slowly (relative to a myelinated fiber), about 0.6 to 2 m/sec. As you can see, myelinated nerve fibers greatly increase the rate at which information, in the form of action potentials, is conducted. So far, we have been considering the propagation of an action potential within a single nerve cell. Let's see how this information is transmitted to other cells.

Copyright by The Berkeley Review

17

The Berkeley Review


Specializing in MCAT Preparation

Biology

Nerve & Muscle

The Neuromuscular Junction

The Neuromuscular Junction


At the end of the axon is a specialized area called the terminal bouton (or

synaptic bulb, synaptic knob, or even terminal foot). This specialized end region
of the axon can impinge or synapse on another axon of a different neuron, on a

dendrite, or even on some type of cell body (like a muscle cell). The space between this synapticjunctionis called the synaptic cleft. The plasma membrane at the terminal bouton is called the pre-synaptic membrane while the plasma membrane that the synapse is being made to is called the post-synaptic
membrane. See Figure 1-12.

Presynaptic
membrane *\

Synaptic cleft
a.

Postsynaptic
membrane

Y
Action

potential
ACh

ACh receptor complex

ACh

ACh

ACh

Terminal bouton

Acetlycholinestrase
Choline

Figure 1-12
Presynaptic and postsynaptic membranes.

Let's consider a neuromuscular junction. This is the synapse between the


terminal bouton of an axon and a muscle fiber. Within the terminal bouton of this

junction are hundreds of thousands of synaptic vesicles that contain the excitatory neurotransmitter acetylcholine (abbreviated ACh). There may be thousands of molecules of acetylcholine in each synaptic vesicle. Acetylcholine itself is synthesized in the cytosol of the neuron from the molecules acetyl-CoA
(which caries an acetate moiety) and choline.

As the action potential reaches the terminal bouton, it triggers the opening of

calcium (Ca^) channels. A transient influx of Ca^ into the terminal region
causes the synaptic vesicles to fuse with the presynaptic membrane and release their neurotransmitters into the synaptic cleft (via exocytosis). Calcium is pumped out of the cytosol and back into the extracellular fluid. The membranes of the synaptic vesicles that fused with the presynaptic membrane are recycled
Copyright by The Berkeley Review
18

The Berkeley Review Specializing in MCAT Preparation

Biology

Nerve & Muscle

The Neuromuscular Junction

(via endocytosis). The released neurotransmitters diffuse through the synaptic cleft and bind to specific postsynaptic membrane receptors. Binding of acetylcholine to this postsynaptic membrane receptor conformational^ changes that receptor into a channel (ionophore) that is large enough to allow cations

such as Na through. [Note that this channel is not a voltage-activated channel


like the channels we examined in the axon itself. This channel is a ligandactivated channel. In this case, the ligand (a molecule or ion bound to a protein)

is acetylcholine.] As Na enters the postsynaptic membrane, the muscle fiber


begins to depolarize and an action potential is eventually generated. If acetylcholine were to remain in the synaptic cleft, then it would continue to bind to the postsynaptic membrane and action potentials would continually be generated. This could result in prolonged muscle spasms. However, the enzyme acetylcholinesterase, which is bound to the surface of the postsynaptic membrane, hydrolyzes acetylcholine to acetate and choline. These two products are then recycled as they are transported back into the presynaptic terminal where they are used in the synthesis of acetylcholine (Figure 1-12). There are many different types of neurotransmitters. For example, the amino acid glutamate and the amino acid derivatives epinephrine, norepinephrine, dopamine, and serotonin can act as neurotransmitters. There are also drugs that can act at the level of these postsynaptic receptors.
EPSPs and IPSPs

We said that the binding of acetylcholine to its postsynaptic membrane receptor elicits an excitatory response that generates an action potential. These types of

potentials are referred to as excitatory postsynaptic potentials (EPSPs). There can also be inhibitory postsynaptic potentials (IPSPs) as well.

(b)

Excitatory Post Synaptic Potcntia


The result of

Inhibitory Post Synaptic Potential

ftp Na
Depolarization

synapse with < *


A and B !

ftp CI
Hyperpolarization

Figure 1-13
EPSPs and IPSPs.

Suppose neuron A synapses with neuron C, as shown in Figure l-13a. If the synaptic connection between these two neurons is excitatory, then the same
Copyright by The Berkeley Review
19

The Berkeley Review Specializing in MCAT Preparation

Biology

Nerve & Muscle

The neuromuscular Junction

number of action potentials that pass down the axon of neuron A will also pass down the axon of neuron C. In the case of an excitatory postsynaptic potential, there is an increase in the permeability of the postsynaptic membrane to Na (i.e., neuron C will depolarize).
Suppose neuron B, which is inhibitory, also synapses with neuron C, as shown in Figure l-13b. In the case of an inhibitory postsynaptic potential, there is an

increase in the postsynaptic membrane to K and to CI" ions. Neuron C will hyperpolarize. Remember, becoming more hyperpolarized does not generate an action potential. In fact, it will keep neuron C quiescent. In other words, if we just excited neuron B alone, then nothing would happen to neuron C.
However, if we integrated the action potentials that neuron C receives from both neuron A and neuron B, we find that neuron C only generates 2 action potentials (Figure l-13b). In this case, neuron B will decrease the excitability of neuron C to
the stimulation it is receiving from neuron A.

Consider two action potentials coming toward one another, as shown in Figure 1-14. When those two action potentials meet what will happen? Will they continue past one another? Will their amplitudes add together? The region in front of each action potentialis continually being depolarized, while the region
behind the action potential is in its refractory period for a few milliseconds and cannot immediately be depolarized. Therefore, when the two action potentials meet one another they will stop their propagation along the axon.

Refractory period

Refractory period

Figure 1-14
Propagation of action potentials.

Copyright by The Berkeley Review

20

The Berkeley Review Specializing in MCAT Preparation

Biology
The Sarcomere

Nerve & Muscle

The Sarcomere

We mentioned that the synapse in Figure 1-12 involved a neuromuscular junction. We have considered the neuronal portion of that junction already. Let's now examine the muscular portion and follow the fate of the action potential that was generated.

If you were to examine a typical muscle, you would find that in most cases it is attached to a specific area of the skeleton by tendons at either end of the muscle. The region between the tendons is referred to as the belly of the muscle. A crosssection of the belly of the muscle shows a great number of multinucleated muscle fibers (cells), as shown in Figure 1-15. These muscle cells do not divide to produce new muscle cells. However, if you were to exercise your muscles, those cells would just get larger.

Myofibril

Actin

J~l Myosin
^ Heads

Z-line

H-zonc

Z-line

Sarcomere

Figure 1-15
Muscle anatomy.

Closer examination of skeletal muscle fibers shows striations in both the

transverse and longitudinal directions. The striations in the longitudinal


Copyright by The Berkeley Review
21

The Berkeley Review Specializing in MCAT Preparation

Biology

Nerve fif Muscle

The Sarcomere

direction are due to myofibrils. The myofibrils contain the contractile units of the muscle. Each contractile unit (Figure 1-16),bounded by a structure referred to

as the Z-line, is referred to as a sarcomere and each sarcomere contains two types
of contractile proteins. The thin contractile protein is called actin and the thick

contractile protein is called myosin. The myosin thick filament comprises the Aband and the region between two A-bands, where there is just the thin actin filaments, is the I-band. The H-zone is that region in the center of the A-band
and between the ends of the actin filaments.

Sarcomere

Sarcomere

Figure 1-16
Sarcomere details.

Myosin filaments arearranged toward thecenter of thesarcomere. They give rise to the characteristic dark bands one sees when examining muscle tissue. Actin filaments are attached to the Z-lines. The actin and myosin filaments interact with each other through projections stemmingfrom the myosin filaments. Those projections are sometimes referred to as myosin heads. The myosin thick filament does not have any of these head groups in its central region but rather
concentrates those head groups towards its terminal regions. The actin thin filament is composed of a protein subunit called G actin ("G" for

globular), which is roughly spherical in shape. The actin filaments can grow by the addition of G actin to the ends of the already existing filament. Each actin
filament is composed of two rows of G actin monomers wound around each
other to form a helix.

Actin and Myosin


Let's consider how myosin and actin interact with each other at the molecular

level. When a muscle is in its relaxed state, ATP is bound to the myosin head
groups. The myosin head is not bound to the actin filament, because ATP

reduces myosin's affinity for actin. When ATP is bound to the myosin head, the myosin head itself is at about a 45 angle with respect to the actin filament. Since actin is not bound to myosin, the ATP on the myosin heads is hydrolyzed to ADP and Pi (inorganic phosphate). These hydrolysis products remain on the myosin head. More importantly, the myosin head now undergoes a conformational change, so that it is situated at a 90 angle with respect to the actin filament. This high energy, stable, myosin-ADP-Pi head complex binds to

the actin filament. As wewill see, this step is dependent on Ca2 being present.
Copyright by The Berkeley Review
22

The Berkeley Review Specializing in MCAT Preparation

Biology

Nerve & Muscle

The Sarcomere

The interaction between the actin filaments and myosin head groups causes the release of Pj and then ADP from the myosin heads. This process causes a conformational change in the myosin heads, so that they shift by about 45 in a direction that is away from the Z-lines. This step, in which the actin filaments move relative to the myosin filaments, is called the power stroke, and the product of this step is referred to as the rigor complex or rigor state. See Figure
1-17.

ATP binds and

Actin

causes myosin head


to release actin ATP is

hydrolyzed

Actin filament
moves

Actin and

myosin bind

Figure 1-17 The contraction cycle.

After the myosin heads have swiveled and have pulled the actin filament past
the myosin filament, the myosin heads remain bound to the actin filaments. This is the so-called rigor state. In order for the myosin heads to dissociate from the actin filaments, ATP needs to bind to those myosin heads. Remember, ATP reduces myosin's affinity for actin. Without ATP, muscles remain in a state of rigor for a given period of time. This is what happens to muscles in your body when you die (i.e., rigor mortis). The myosin head groups can no longer bind ATP, because the metabolic pathways that generate this energy-rich nucleotide
have ceased to function.

Troponin, Tropomyosin, & Calcium


When we examined actin, we saw that it was composed of two long rows of roughly spherical protein subunits, which polymerized together and then wound about each other in a helical fashion. If we look closely at the helical structure of actin, we notice two grooves. Running the length of these grooves is a coiled protein called tropomyosin, composed of two helical polymers wound about each other (see Figure 1-18). When tropomyosin resides in the actin groove, it covers up the binding sites for the myosin heads and prevents those head groups from attaching to the actin filament.

Copyright by The Berkeley Review

23

The Berkeley Review Specializing in MCAT Preparation

Biology

Nerve & Muscle

The Sarcomere

In the case of striated (skeletal) muscle, this is often referred to as the actin-based regulation of muscle contraction. Cardiac muscle and smooth muscle are both controlled by myosin-based regulation.
Troponin Proticn Complex Tropomyosin

^2+ Ca-+ Binding Site

Actin

Relaxation
Removal of Calcium Addition of Calcium

Contraction

Myosin Head Binding Sites

Figure 1-18
Sarcomere detail.

Troponin, a multi-subunit binding protein, interacts with tropomyosin, actin,

and Ca2. When Ca2 is bound to a particular subunit of the troponin complex,
it causes tropomyosin to shift its position and expose the myosin head binding sites. The myosin heads then bind to the actin filaments (see Figure 1-17) and

muscle contraction follows. [In Figure 1-17, Ca2 would bind at Step 3.] IfCa2
is not in the medium, there will be none to bind to the troponin complex and tropomyosin will remain in the actin groves and cover the myosin head binding
sites. This is the relaxed state.

Surrounding each myofibril is a membranous structure, a modified version of the endoplasmic reticulum called the sarcoplasmic reticulum. Calcium is sequestered within this smooth membranous structure. Also surrounding each myofibril is an invagination of the sarcolemma (i.e., the plasma membrane) called
the transverse tubule (abbreviated as T-tubules). These T-tubules follow the Z-

lines of each myofibril. Selected anatomical features of the structures associated


with the sarcoplasmic reticulum are shown in Figure 1-19.

After an action potential crosses the last synaptic junction on its way to a muscle

cell, it passes down each T-tubule and somehow stimulates the release of Ca2
from the sarcoplasmic reticulum. [At the present time, it is thought that the
lumen of the T-tubules and the lumen of the sarcoplasmic reticulum are not

Copyright by The Berkeley Review

24

The Berkeley Review Specializing in MCAT Preparation

Biology

Nerve & Muscle

The Sarcomere

continuous.] As the Ca2 flows from a region of high concentration (the


sarcoplasmic reticulum) to a region of low concentration (the cytosol), it binds to
its binding site on the troponin complex. Each sarcomere contracts

simultaneously because of the way in which the nerve impulse is carried along
the sarcolemma and into the T-tubules.

Sarcolemma

Myofibrils

Transverse tubules

Figure 1-19
The sarcoplasmic reticulum.

Once contraction has taken place and the nerve impulse has ceased, the Ca2 in the cytosol is pumped back into the sarcoplasmic reticulum by a Ca2-ATPase pump. In the sarcoplasmic reticulum Ca2 associates with a specific binding
protein. When the next action potential passes down the T-tubules, the cycle will begin again and another muscle contraction will take place.
There are a number of ways in which the strength of muscle contraction can be varied. The strength of contraction can be varied by (1) the size of the motor unit (to be defined in a moment), (2) the number of available motor units, and (3) the amount of actin and myosin contained with each cell. A motor unit is simply a motor neuron and the muscle fibers that it innervates. We have already examined the muscle fibers. What is a motor neuron? Motor
neurons are nerve cells whose cell bodies are located in the central nervous

system (e.g., the spinal cord or brain stem), and whose myelinated axons innervate skeletal muscle. Recall that myelinated axons allow action potentials to be transmitted rapidly to the desired effector organ (in this case, a muscle). If you wanted to precisely move a muscle (e.g., the muscles associated with your eyes or fingers), then you would need motor units which were rather small in size. The smaller the size of the motor unit, the smaller the strength of contraction. In contrast the postural muscles of the back or the legs require large
motor units to be effective. Not only can strength be controlled but precision can
be controlled as well.

Copyright by The Berkeley Review

25

The Berkeley Review

Specializing in MCAT Preparation

Biology

Nerve fie Muscle

The Sarcomere

The number of motor units also controls the strength of contraction. For example,

if you wantedto pickup a lightobject (e.g., a pencil), you would need to employ only a few motor units. However, if you wanted to pick up heavier objects (e.g., gym weights), you would need to utilize more motor units. When you lift weights regularly, the size of each muscle cell increases, because the amount of actin and myosin in each musclecellhas increased.The more actin and myosin in a muscle cell, the greater the strength of contraction that can be generated.
Even though the strength of muscle contraction can vary with the size of the
motor units, the number of motor units, and the amount of actin and myosin in
each muscle cell, the ultimate determinant of muscle contraction is the

concentration of ATP in your muscle cells. The energy for contraction comes from the hydrolysis of ATP to ADP and Pi (inorganic phosphate). See equation
(1-7):

ATP

> ADP + Pj + Energy

For muscle contraction to continue, ATP must be constantly regenerated. Recall from basic biology that under aerobic conditions (when O2 is not limited)

glucose will be oxidized to CO2 and H2O. Energy can be extracted (in the form of
36 molecules of ATP~if we use the glycerol-phosphate shuttle) during this catabolic process from glycolysis, the Krebs cycle, and oxidative phosphorylation/electron transport. See equation (1-8):
Aerobic

(Slow)

Glucose

> C02 + H20 + 36 ATP

(1-8)

During anaerobic conditions (when O2 is limited), glucose is metabolized to lactate (the anion of lactic acid). Only 2 ATP molecules are extracted for the use of energy in this process. See equation (1-9). [This equation is not balanced.]
Anaerobic

(Fast)

Glucose '

> Lactate + 2 ATP

(1-9)

Even though the efficiency of ATP production is greater for aerobic metabolism than for anaerobic metabolism (36 ATPs compared to 2 ATPs), ATP can be produced much more quickly through anaerobic metabolism than through aerobic metabolism, owing to enzyme regulation in the glycolytic pathway.

During anaerobic metabolism, the concentration of lactic acid begins to increase. This means that the pH of the medium will decrease (i.e., become more acidic). One of the key regulatory enzymes in the glycolytic pathway cannot function
well below a certain pH value. This enzyme has some optimum pH range at which it functions and if the pH falls below (or above) that range, the enzyme is

inhibited. Glycolysis comes to a halt and the ATP yield becomes insufficient to
carry out normal metabolic processes. In other words, fatigue sets in.

Copyright by The Berkeley Review

26

The Berkeley Review


Specializing in MCAT Preparation

Biology

Nerve & Muscle

Nervous System Components

Nervous System Components


The nervous system in its simplest form can be found among the cnidarians. All
the neurons in these creatures are linked together in a nerve net (much like a

web). Stimulation of the nerve net causes muscle contraction. This simple
procedure is referred to as a reflex arc. In the case of the cnidarians there is no

associative activity, meaning that the transmission of the action potential is not influenced by other neuronal activity.
Associative neurons are found in higher animals (Platyhelminthes and above). Here, different neurons can interact to produce a given response. When a number of these associative neurons are grouped together, the nervous system expands. A grouping of nerve cells is called a ganglion (or a nucleus or nuclei).

Groupings of neurons in higher animals also lead to more elaborate sense organs, differentiation into a central region of cells (the central nervoussystem, or CNS)
and a peripheral region of cells (the peripheral nervous system, or PNS), various
associative areas, and a brain.

The three basic anatomical divisions of the vertebrate brain can beseen in Figure
1-20. Those divisions are: (1) the forebrain (prosencephalon), (2) the midbrain (mesencephalon), and (3) the hindbrain (rhombencephalon).
Prosencephalon
A

Rhombencephalon

nal cord

Mesencephalon
Figure 1-20
Anatomical divisions of the vertebrate brain.

Inferior to (meaning "below") the brain is the spinal cord. Recall that the brain and the spinal cord together are the central nervous system (CNS). From the spinal cord, nerves extend into the limbs and extremities (the periphery) of the body. These nerves represent the peripheral nervous system (PNS). If a neuron carries
information into the spinal cord and brain, that neuron is said to be an afferent (sensory) neuron. If a neuron carries the information away from the brain or spinal cord, that neuron is said to be an efferent (motor) neuron.

The forebrain has several anatomical subdivisions, including the cerebrum,


thalamus, and hypothalamus (see Figure l-21a). The cerebrum is divided into right and left cerebral hemispheres, joined by the corpus callosum. The cerebral hemispheres are divided into the frontal lobes (associated with movement and personality), parietal lobes (associated with touch and stretch sensation), occipital lobes (associated with vision), and temporal lobes (associated with hearing) as shown in Figure l-21b.

Copyright by The Berkeley Review

27

The Berkeley Review Specializing in MCAT Preparation

Biology
(a)

Nerve St Muscle

nervous System Components

Thalamus

(b)
Cerebral
Cortex

Central Sulcus

Corpus
Callosum

Frontal

Temporal

Hypothalamus
Pituitary^/
Gland

^
The Cerebrum and its Four Lobes

Medulla

\\ Spinal
Cord

Cerebellum

Figure 1-21
Anatomical divisions of the vertebrate brain.

The outermost layer of the cerebrum is called the cerebral cortex. It consists of gray matter, which is simply nerve cell bodies and their dendrites. Beneath the gray matter is the white matter, or myelinated axons of the nerve cells. In the spinal cord, the situation is reversed: The gray matter is more centralized, while
the white matter is more peripheral.

The cerebral cortex has many important landmarks, one being the central sulcus, a prominent groove that separates the frontal lobes and the parietal lobes.
Anterior to this sulcus is the motor cortex, which controls the movement of

individual muscles. Posterior to this sulcus is the (somatic) sensory cortex, which detects sensations in various parts of the body.

Somatic receptors send their information into the spinal cord via afferent nerve fibers, which either cross over to the opposite side of the spinal cord and then ascend to the sensory cortex in the brain, or ascend in the spinal cord and then cross over in the brainstem before reaching the sensory cortex.
Homunculus

Wilder Penfield, a Canadian neurosurgeon, was able to map the sensory and motor areas of the brain by electrically stimulating certain regions of the brains of his patients during surgery and observing their actions. Throughout this

procedure his patients wereconscious but were unable to feel any pain because thereare no sensory receptors for pain in the cerebrum. He was able to show that
the largest number of cortical neurons found in the sensory cortex register sensation in the fingers, hands, lips, and tongue. This is represented as the

sensory homunculus (a schematic model of a human beingmapped out on the


sensory cortex) shown in Figure 1-22.

Penfieldwas also able to show that the largest number of corticalneurons found
in the motor cortex control individual movement of the fingers, hands, and

speech. Groups of muscles are controlled by an area just anterior to the motor cortex called the premotor cortex. This is an association area. Stemming from
Copyright by The Berkeley Review
28

The Berkeley Review Specializing in MCAT Preparation

Biology

Nerve fie Muscle

Nervous System Components

this association area are neuronal connections with the thalamus and cerebellum. Together these structures plan specific movements that the motor cortex then

executes. It is thought that cognitive functioning like speech, writing, and reading are localized in the lefthemisphere of thebrain, while intuitive functions are localized in the right hemisphere of the brain. This is not proven, only
theoretical.

l*H Tongue

Sensory receptors on the right side of the body project to the


somatosensory cortex on the left hemisphere
of the brain and vice versa
Left

Hemisphere

Right Hemisphere

Figure 1-22
The homunculus.

The thalamus is a relay station for much of the visual and auditory information that we receive from our environment. The hypothalamus is concerned with the visceral activities of the body. The pituitary gland is the master endocrine gland of the body. It receives information from the hypothalamus and sends out
information to regulate different parts of the body.
The brainstem contains such anatomical features as the midbrain, cerebellum, pons, medulla, and the reticular formation. These areas coordinate motor and
visceral activities. The midbrain detects movement and can direct the head and

eyes towards that movement. The midbrain can also sense pleasure and pain. The cerebellum is responsible for the bulk of regulation and coordination of muscular activity. The pons and medulla coordinate visceral activities. The reticular formation, which is the core of the brainstem, is essentially an activating
system designed to alert the brain. The reticular formation also inhibits motor

and sensory impulses and can induce sleep. Below the medulla is the spinal cord.

Copyright by The Berkeley Review

29

The Berkeley Review Specializing in MCAT Preparation

Biology

Nerve & Muscle

Control Of Body Activity

Control Of Body Activity


Recall that when we first mentioned the nervous system, we looked at a simple reflex arc. If someone were to tap you on your knee with a rubber-headed hammer, your lower leg would extend outward. The mechanism behind this is

quite simple. As the hammer impinges on the patellar tendon, sensory neurons
located in the tendon of the quadriceps muscle are excited. These impulses travel along an axon that enters the spinal cord through the dorsal root ganglion and synapses with two neurons (Figure 1-23).
This area, even though it is supposed to be gray matter, is shown as being

i clear so you can see the synapses, j


Biceps ('hamstring")
muscle is a flexor

Gray Matter

Spinal Cord
Polysynaptic J '
Reflex Arc

Tibia

T. Interncuron

Figure 1-23
The reflex arc.

One of the synapses is to a motor neuron that immediately leaves the spinal cord and returns to the quadriceps muscle, causing a contraction. As this muscle contracts, the leg straightens (extends) at the knee joint. Because it elicits this kind of action, the quadriceps muscle is termed an extensor muscle. The type of synapse just described (making just one synaptic connection), is referred to as a monosynaptic reflex arc. The other synapse is made to an interneuron which, in this case, is inhibitory. This interneuron in turn synapses with a motor neuron that innervates the bicep ("hamstring") muscle in the back of the leg. When the bicep muscle contracts, the lower portion of the leg bends or flexes at the knee joint. We call this kind of muscle a flexor muscle, and this type of synapse (because it makes at least two synaptic connections) is referred to as a polysynaptic reflex arc. If contraction of the biceps muscle is inhibited as the quadriceps muscle
contracts, then one observes a smooth and coordinated movement at the knee

joint, as the lower portion of the leg extends outward after stimulation by the tapping of the hammer on the patellar tendon.

Copyright by The Berkeley Review

30

The Berkeley Review

Specializing in MCAT Preparation

Biology
Neurovisceral Control

Nerve fit Muscle

Control Of Body Activity

The autonomic nervous system, which is part of the efferent division of the peripheral nervous system, can be subdivided into the sympathetic and the

parasympathetic systems. Nerve fibers from the autonomic nervous system leave the spinal cord to innervate various glands, smooth muscle, and cardiac muscle.
Let's examine these two subdivisions.

Parasympathetic Division The parasympathetic division of the autonomic nervous system has nerve fibers
which leave from the sacral portion of the spinal cord and from the midbrain (mesencephalon) and medulla (part of the rhombencephalon), as shown in Figure 1-24. Parasympathetic nerve impulses tend to increase the rate of digestion and lower the heart rate. The blood pressure is also lowered, and the pupils constrict (contract). In general, the parasympathetic division conserves energy and helps in the restoration of various bodily functions (e.g., by aiding in the digestion of food for later use in metabolic processes).

Preganglionic ^-Ganglion
nerve fiber f
Neurotransmitter is

acetylcholine

Midbrain 1

Medulla J

Lacrimal glands

Lungs and
Bronchi

Large intestines

Figure 1-24 Parasympathetic nerves.

The parasympathetic division has both preganglionic and postganglionic nerve fibers. The cell bodies of the preganglionic neurons are found in the sacral region
Copyright by The Berkeley Review
31

The Berkeley Review

Specializing in MCAT Preparation

Biology

Nerve &Muscle

Control Of Body Activity

of the spinal cord and in the brainstem. The ganglia of the parasympathetic division lie near or in the organs that are to be innervated. Therefore, the preganglionic nerve fibers are rather long, while the postganglionic nerve fibers are rather short. Both the preganglionic and the postganglionic nerve fibers in the parasympathetic division release acetylcholine (ACh) as the
neurotransmitter. [Nerve fibers that release acetylcholine as their

neurotransmitter are called cholinergic nerve fibers.]

The most prominent nerve in the parasympathetic division is the vagus nerve [also called the (tenth) X cranial nerve]. Roughly three-quarters of all the neurons in the parasympathetic division can be found in the vagus nerve. The reason for this should become obvious, if you look at Figure 1-24 The vagus nerve innervates the heart, lungs, stomach, liver (not shown), small intestine, large intestine, and kidneys (not shown), among other organs.

Sympathetic Division
The sympathetic division of the autonomic nervous system has nerve fibers branching off from the thoracic and lumbar regions of the spinal cord, as shown in Figure 1-25. Sympathetic nerve fibers tend to condition the body for a "fightor-flight" response (a term first proposed by the Harvard physiologist Walter Cannon in the early 1900s). The heart rate increases, blood pressure elevates, pupils dilate (open wider), and the digestive functions decrease, all as a result of sympathetic nervous innervation.

The sympathetic nerves that leave the thoracic and lumbar regions of the spinal cord first enter chains of ganglia connected by nerve fibers on either side of the spinal column. These chains of ganglia are collectively called the sympathetic trunk. As these spinal nerves, called preganglionic nerve fibers, enter the sympathetic trunk, they can either (a) pass through this collection of ganglia to synapse with other ganglia outside the sympathetic trunk, (b) pass into the sympathetic trunk and ascend or descend to synapse with ganglia at other levels, or (c) pass into the sympathetic trunk and directly synapse with a given
ganglion.

The nerve fibers leaving a synapse in a given ganglion are referred to as post ganglionic nerve fibers. In the caseof the sympatheticdivision the preganglionic nerve fibers tend to be short, while the postganglionic nerve fibers tend to be

longer. The preganglionic fibers release acetylcholine, while the postganglionic


fibers release norepinephrine as their neurotransmitters. [Nerve fibers that release norepinephrine (or epinephrine (adrenaline)) are called adrenergic nerve
fibers.]

One set of spinal nerves that originates in the lower thoracic regionof the spinal cord send long preganglionic nerve fibers to the adrenal medulla, a region within the adrenal glands (located on the superior surface of the kidneys). These nerve fibers synapse directly on the ganglia in the adrenal medulla. There are no postganglionic nerve fibers. When the adrenal medulla is stimulated, both norepinephrine and epinephrine are released directly into the bloodstream.
Because these substances are released into the bloodstream, we can refer to them

as hormones. Therefore, the adrenal medulla can be considered as an endocrine

gland. These hormones, distributed throughout the body by the circulatory system,can increase the heart rate and cause the pupils to dilate.

Copyright by The Berkeley Review

32

Specializing in MCAT Preparation

The Berkeley Review

Biology

Nerve &? Muscle

Control Of Body Activity

Postganglionic
nerve fiber

Neurotransmitters are

acetylcholine and norepinephrine

Preganglionic,
nerve fiber
Thoracic

Lumbar

Large
intestine

Figure 1-25 Sympathetic nerves.

Somatic vs Autonomic Nervous System


Let's review the basic differences between the somatic nervous system and the autonomic nervous system. In the somatic nervous system, we find that (a) once the nerve fibers leave the central nervous system, they do not make a synapse until they have reached their effector organ. When the synapse is made at the effector organ, (b) the neurotransmitter that is released is acetylcholine. The
somatic nervous system (c) innervates skeletal muscle. Innervation of that
skeletal muscle (d) leads to excitation of the muscle itself.

In the autonomic nervous system, we find that (a) once the nerve fibers leave the central nervous system, they synapse with a ganglion before they make the final synapse with their effector organ. The preganglionic fibers in both the parasympathetic and sympathetic divisions release (b) acetylcholine as the neurotransmitter. The postganglionic fibers in the parasympathetic division release acetylcholine; in the sympathetic division, norepinephrine is released. The autonomic nervous system (c) innervates glands, and smooth and cardiac muscle. The cells innervated by the autonomic nervous system can (d) be either
excitatory or inhibitory.

Copyright by The Berkeley Review

33

The Berkeley Review

Specializing in MCAT Preparation

Biology

Nerve St Muscle

Control Of Body Activity

The adrenal medulla, a specialized ganglion in the sympathetic division of the autonomic nervous system, is directly stimulated by a preganglionic fiber. This nerve fiber releases acetylcholine, which causes the cells of the adrenal medulla to release two types of hormones into the blood. The major hormone released is epinephrine. The minor hormone released into the blood is norepinephrine.
These differences are illustrated in Figure 1-26.

CNS PNS
Somatic Nervous
Skeletal muscle
^F & V V V

System

Acetylcholine

Glands, cardiac
and smooth muscle

Postganglionic nerve fiber


r

Acetylcholine
Autonomic

<P!l
Norepinephrine

CO
V-

3
Sr
a.
o

Nervous -^
System

*\% Jd^ Hormone C=^ Blood CZ^ Organ


^ ' (Epineprhine)
Adrenal Medulla Preganglionic nerve fiber Ganglion

Glands, cardiac
and smooth muscle
X"A A A A A A v

^
JS

^.

Acetylcholine

Figure 1-26
CNS and PNS review.

Copyright by The Berkeley Review

34

The Berkeley Review

Specializing in MCAT Preparation

Biology

Nerve St Muscle

Receptors and Sensory Input

Receptors and Sensory Input


There are many different types of receptors receiving sensory information from the environment and passing that information to the nervous system. For example, there are mechanoreceptors, which are concerned with pressure, hearing, balance and blood pressure. Nociceptors sense pain. Mechanoreceptors respond to a change in their configuration. Some can respond to a light pressure, while others respond to a deeper pressure. There are specialized regions on the sides offish called lateral lineorgans that respond to a change in the pressure of
the water. There are pressure receptors in the walls of the aorta that are able to
sense an increase in blood pressure.

Thermoreceptors detect cold and warmth, while chemoreceptors can detect taste, smell, oxygen, carbon dioxide, hydrogen ions, and blood glucose levels.
The taste receptors in the tongue can distinguish between food which is bitter, sour, salty, or sweet. Olfactory cells in the nasal cavity can distinguish hundreds of different odors. Photoreceptors in the retina of the eye can respond to the presence of a single photon of light. There are also receptors concerned with electricity and magnetism. For example, catfish have electrical receptors that help them detect prey, and birds have magnetic receptors that help them to
navigate.

The sensory information that a receptor receives is specific for that particular receptor. The stimulus that is received by that receptor changes (converts or transduces) the potential (Vm) across the receptor's membrane. This change in membrane potential is called a receptor potential. If this receptor potential were to exceed a specific threshold, an action potential would be generated. We also find that if the pressure on this receptor is great and the change in membrane
potential is large, then the receptor potential will exceed the threshold level. The

result is an increase in the frequency of the action potentials being generated. The amplitude of the action potentials will not be change, only the frequency.
Receptors and Transducers Let's consider the receptor potential for a special type of mechanoreceptor called a pacinian corpuscle. A pacinian corpuscle has an unmyelinated nerve ending which is encapsulated in layers of connective tissue. However, the afferent nerve that leaves this encapsulated ending is myelinated.
Saltatory
Pressure

Conduction

Figure 1-27
Pacinian corpuscle.

Copyright by The Berkeley Review

35

The Berkeley Review Specializing in MCAT Preparation

Biology

Nerve St Muscle

Receptors and Sensory Input

When the nerveendingis depressed, a localdeformation causessodium channels to open and Na ions to rush into the nerve ending. An action potential is not generated in the receptor region. Instead, the establishment of this receptor potential causes a localized flow of current to be propagated along the nerve
fiber. If the threshold has been reached, then this local current flow will initiate

an action potential at the first node of Ranvier, located at the outer edge of the pacinian corpuscle. The action potential canspread down the afferent nerve and
towards the central nervous system by saltatory conduction, as shown in Figure
1-27.

Suppose we were to depress the pacinian corpuscle just a small amount. If the
threshold is not reached, an action potential will not be generated. This means one would not feel the pressure that is being applied. If we apply a second stimulus, we might depolarize the membrane even more. If we were to apply a
third stimulus that was even stronger, we could exceed threshold and an action

potential would be generated. If we maintain this pressure such that we are just above threshold by a certain amount, we will continue to generate action potentials at the rate of, say, 2 action potentials every 10 milliseconds.
2 action potentials per 10 milliseconds
+40
>

3 action potentials per 10 milliseconds

Action Potential

rh
WWN-WWW

Potential

ft
1st

/\

Time (ms)

^ Strength of
stimulus

Weak 2nd

Strong
4th

Figure 1-28 Action potential frequencies.

What would happen if we apply a fourth stimulus, much stronger than the previous stimuli? The membrane would become more depolarized, and we would move further above the threshold level. Once this happens, the frequency of action potentials generated would increase, say, to 3 action potentials every 10 milliseconds. This is shown in Figure 1-28. Note that in each case the amplitude of the action potentials remains the same. What changes is the frequency of the action potentials propagating along the axon. It is the frequency of action potentials that signals to the central nervous system the magnitude of the stimulus being received.All receptors function on this basic principle.

Copyright by The Berkeley Review

36

The Berkeley Review

Specializing in MCAT Preparation

Biology

Nerve St Muscle

Receptors and Sensory Input

Adaptation When you wake up in the morning and take a cool shower, you feel the coolness

of the water on your skin. When you get dressed, you feel the clothes touching your skin. In each case, though, after a brief period of time you get used to the water or the clothes touching your body. These are examples of sensory adaptation. One type of receptor that undergoes sensory adaptation is the pressure receptor. If we plot the frequency of action potentials (as they travel back to the central nervous system along a given axon) as a function of the time of a sustained stimulus, we would find that pressure receptors adapt rapidly. However, pain receptors do not adapt rapidly, as is shown in Figure 1-29. If the
pressure receptors did not adapt to the touch of the clothes that we wear, it

would prove to be rather inconvenient. [On the other hand, the body does not adapt as readily to the sensation of pain-a good thing, evolutionarily speaking, since pain is a warning of potential damage to the body's tissue and not
something to be responded to as a matter of convenience, but as a matter of
survival.]
pain

pressure

10

Time of sustained stimulus (sec)


Figure 1-29 Adaptation to pressure and to pain.

Tactile Discrimination

Recall that the end of a neuron can be divided into many branches. Each of these branches in turn can end at a receptor (such as a pacinian corpuscle). These axonal branches and their corresponding receptors constitute a receptive field. Depending on which area of the body one is describing, there can be many receptive fields, some of which overlap, or there can be a few receptive fields, some of which do not overlap. Let's consider a set of receptive fields that overlap in order to determine how two points in space can be distinguished from one another by the same touch.
Suppose we have a set of receptive fields like the ones shown in Figure l-30a. What one generally finds is that a greater frequency of action potentials will be generated (assuming threshold has been reached) if the central region rather than the periphery of a given receptive field is stimulated. In other words, there seem to be more receptors in the central region of a receptive field than in itsperiphery.
What would happen if receptive fields overlapped, as shown in Figure 1-30? If we were to stimulate the receptive field represented by (b) in Figure 1-30 vigorously enough, then because of the field overlap, we would also stimulate the receptive fields in (a) and (c). It would feel to a subject as if three different areas of the body were being stimulated when, in fact, just the receptive field in (b) is being stimulated. How can the information being received from these sensory afferent neurons be fine-tuned to let us know that only field (b) is being
Copyright by The Berkeley Review
37

The Berkeley Review Specializing in MCAT Preparation

Biology

Nerve & Muscle

Receptors and Sensory Input

stimulated, and not field (a) or field (c)? This is accomplished by a process called lateral inhibition, mediated through interneurons within the spinal cord.

In Figure 1-30, the axon that leads away from field (b) has lateral connections to interneurons that inhibit the impulses being sent down the axons from fields (a) and (c). In other words, because of lateral inhibition the impulses generated in

receptive fields (a) and (c) are suppressed, allowing the impulses from the receptive field in (b) reach the proper spinal tracts that ascend to the brain.
To Brain

Receptive
Field

Action
Potentials
A.

Ganglion

19
9
9

Axon

Axon

Axon

Lateral Inhibition

(interneuron)

Figure 1-30 Receptive fields.

Somatic Sensory Pathways Once the sensory afferent nerve fibers enter the spinal cord they cross to the side opposite from the side they entered, either in the spinal cord or in the brainstem. In other words, the sensory input from the right side of the body will be represented on the somatosensory cortex of the left cerebral hemisphere, and sensory input from the left side of the body will be represented on the somatosensory cortex of the right cerebral hemisphere. [In Figure 1-30, which side of the body are the receptive fields located, the right side or the left side? Will the sensory input go to the somatosensory cortex of the right or left cerebral hemisphere? How do you know?] Where do these sensory neurons synapse as they ascend to the brain? As a

general rule, there are three neurons involved in sensory pathways. In thecase of pressure, we find that the first-order neurons carrying information from the
receptive field(s) enters the spinal cord and synapse with second-order neurons that ascend on the oppositeside of the spinal cord to the thalamus. Here, another

synapse is made with third-order neurons that continue to ascend, until they reach a specific region of the somatosensory cortex of the cerebral hemisphere opposite to the side of the body in which the sensation was perceived. The
cerebral cortex itself contains cells that are organized into 6 horizontal layers. The sensations of pressure would be registered in cells arranged in columns that
cross these different layers.
The Berkeley Review

Copyright by The Berkeley Review

38

Specializing in MCAT Preparation

Nerve and Muscle

15 Passages
100 Questions

Passage Titles
I. II. III. IV. V. VI. VII. VIII. IX. X. XL XII. XIII. XIV. XV.

Questions
1 -8 9- 13 14-20 21 -26 27-33 34-40 41 -46 47-52 53-58 59-64 65-71 72-79 80-87 88-94 95- 100

Types of Transport Autonomic Nervous System


Action Potentials

Local Anesthetic

The Lens, the Iris, and Associated Muscles

Resting Membrane Potential Nicotine Replacement Therapy Retinal Projection Axonal Transport Huntington's Disease Photoreceptors
Sound Transmission in the Ear

Tryptophan and Serotonin Experiment Frog Muscle Experiment Skeletal Muscle Groups

Ilk

<

JLrR-E-V-I-E-W Specializing in MCAT Preparation

BERKELEYi* 1 ^

Suggestions
Thepassages that follow are designed to getyou to think in a conceptual manner about the processes of physiology at the organismal level. If you have a solid foundation in physiology, many of these answers will be straightforward. If you have not had a pleasantexperience with the topic, some of these answers might appear to comefrom the void past the Oortfield of the solar system.
Pick a few passage topics at random. For theseinitialfew passages,do not worry about the time. Just focus on what is expected of you. First, read the passage. Second, look at any diagrams, charts, or graphs. Third, read each question and the accompanying answers carefully. Fourth, answer the questions the best you can. Check the solutions and seehow you did. Whether you got the answersright or wrong, it is important to read the explanations and see if you understand (and agree with) what is being explained. Keep a record of your results.
After you feel comfortable with the format of those initial few passages, pick another block of passages and try them. Be aware that time is going to become important. Generally, you will have about 1 minute and 15 seconds to complete a question. Be a little more creative in how you approach this next group. If you feel comfortable with the outline presented above, fine. If not, then try different approaches to a passage. For example, you might feel well versed enough to read the questions first and then try to answer some of them, without ever having read the passage. Maybe you can answer some of the questions by just looking at the diagrams, charts, or graphs that are presented in a particular passage.
Remember, we are not clones of one another. You need to begin to develop a format that works best for you. Keeping a record of your results may be helpful.

The last block of passages might contain topics that are unfamiliar to you. Find a place where the level of distraction is at a minimum. Get out your watch and time yourself on these passages, either individually or as a group. It is important to have a feel for time, and how much is passing as you try to answer each question. Never let a question get you flustered. If you cannot figure out what the answer is from information given to you in the passage, or from your own knowledge-base, dump it and move on to the next question. As you do this, make a note of that pesky question and come back to it at the end, when you have more time. When you are finished, check your answers and make sure you understand the solutions. Be inquisitive. If you do not know the answer to something, look it up. The solution tends

to stay with you longer. (For example, what is the Oort field?)
The estimated score conversions for 100 questions are shown below. At best, these are rough approximations and should be used only to give one a feelfor which ballpark they are sitting in.

Section I

Estimated Score Conversions


Scaled Score
>12
10-11

Raw Score

86 - 100
79-85

8-9
7

65-78 59-64

6 5
<4

54-58
48-53
0-47

Biology
Passage I (Questions 1-8)

Types of Transport

Passage I

The transport of substances across a cellular

Animal cells have membranes containing a vast array


of lipids and proteins. Some of the transmembrane proteins in these membranes act as channels that allow for the simple diffusion {passive transport) of water-soluble molecules or ions of low molecular weight between the intracellular and extracellular regions of the cell. The ionic components of these two regions differ considerably
(see Table 1).

membrane by a carrier protein occurs either through a uniport, a symport, or an antiport mechanism.

2K+

Selected Cellular Ionic Components


Ion

[Intracellular] [Extracellular]
15 mM 140 mM 4mM 120mM 4mM 2mM

+Ei0n
+58 mV -92 mV

ATPase,
>

Na+

ATP

Ca2+ ADP + P

K+
cr

135 mM
4mM

'a.

-89 mV +129 mV Ca

Ca2+

2 x lO'4 mM
40 mM

Mg2+

*Ejon =Equilibrium potential for the ion


Table 1

Ca2+ 3Na+ tmq+

Glucose Na+

Other transmembrane proteins act as carriers to transport ions and molecules across the cell membrane. If the substance is too large (e.g., glucose), it will require a carrier protein. This type of carrier-mediated transport is called facilitated diffusion, and the rate of diffusion will increase until all of the carrier binding sites on the membrane transport protein are filled. If energy is required to move a substance across a membrane, the process is called active transport. Primary active transport uses the energy of ATP directly to move a substance against its concentration gradient (Figure 1). The carrier molecule is the membrane-bound protein ATPase. Only three primary active transport systems have been identified: one for Na and K ions, one for

Figure 1

1.

Compared to the extracellular fluid, the intracellular fluid will contain higher concentrations of all of the following substances EXCEPT: A. B.
D.

protein HCO3K

C. PO43-

Ca2 ions, and one for H ions.


Secondary active transport systems do not use the energy of ATP directly, but rather use the energy stored in a concentration gradient established across a cellular
membrane.

2.

All of the following will lead to inhibition of the


Na+/K+-ATPase EXCEPT a low:

A.

intracellular [ATP].

Transmembrane proteins that act as channels in nerve or muscle cells can be electrically or chemically activated.

B. C. D.

intracellular [Na]. extracellular [K]. extracellular [Na].

The resting membrane potential (Em) in these cells is


about -80 mV. The driving force that moves an ion through one of these channels is the difference between
Glucose can be carried across the cell membrane by

Em and Ejon, as shown in the following equation:

all of the following EXCEPT:


A. facilitated diffusion.

Force Moving Ion = E m

B. C. D.

primary active transport. a symport mechanism. secondary active transport.

Copyright by The Berkeley Review

41

The Berkeley Review Specializing in MCAT Preparation

Biology
4.

Types of Transport

Passage I

Glucose, despite its relatively high intracellular


concentration, can dissociate from its carrier protein

7.

All of the following characteristics are common to gap junctions that exist between cells EXCEPT: A. B.
C.

on the cytosolic side of the membrane and into the cytoplasm, because the:

cytoplasmic continuity. rapid transmission of action potentials.


coordination of muscle contraction in the heart.

A.

low intracellular [Na] allows sodium to be


released from the carrier protein.

B.
C. D.

high extracellular [Na] allows glucose to be


released from the carrier protein. affinity of the carrier protein for intracellular sodium is high. affinity of the carrier protein for glucose
decreases, once sodium binds to the carrier

D.

a high resistance electrical pathway.

8.

Which of the following will NOT occur across a cell's membrane, if the electron transport chain is
inhibited?

protein on the extracellular side of the


membrane.

5.

Which of the following graphs best represents the relationship between simple diffusion (SD) and
facilitated diffusion (FD)?
A. B.

A. B. C.
D.

Decreased primary active transport. Decreased secondary active transport. Increased sodium concentration gradient.
Increased intracellular calcium levels.

[Gradient]
C.
D.

[Gradient]

[Gradient]

[Gradient]

6.

The acetylcholine receptor within the postsynaptic membrane of a neuromuscular junction binds two

molecules of acetylcholine and allows Na and K


to flow through the membrane. The acetylcholine
receptor can best be described by its ability to allow:

A.
B. C.

Na to enterthecell, leading to repolarization.


more Na to enter the cell than K to leave
the cell.

more K to enter the cell than Na to leave


the cell.

D.

a change in the membrane potential to lead to


its activation.

Copyright by The Berkeley Review

42

The Berkeley Review Specializing in MCAT Preparation

Biology
Passage II (Questions 9-13)

Autonomic Nervous System

Passage II

10. Which of these situations would stimulate the SNS?


I.

The autonomic nervous system (ANS) is concerned primarily with the regulation of visceral function in response to external and internal stimuli. The two divisions of the ANS are the parasympathetic nervous system (PNS) and sympathetic nervous system (SNS).

Extreme heat.

n. m.
IV.
A.

Extreme cold.

Sleeping. Running a marathon on a sunny day


I and II only I, II, and III only III and IV only I, II, and IV only

The SNS is associated with readying the body for


stressful situations, while the PNS is involved in

B.

maintaining bodily functions at basal levels. These subgroups use a two-neuron system. A primary neuron will extend from the central nervous system (CNS) while a secondary neuron will extend from a ganglion to the
nerve's target organ.

c D.

The primary and secondary neurons for the PNS secrete the neurotransmitter acetylcholine (ACh). The
SNS, however, uses two different neurotransmitters. ACh

11. Which of the following symptoms would be considered a PNS response?


A. B.
C. D.

is secreted from the primary neuron, while norepinephrine (NE) is secreted from the secondary neuron. The SNS has large collections of ganglia that form an intricate network. The most prominent of these ganglia are the paravertebral ganglia, which are adjacent to the vertebral column. The prevertebral ganglia lie along the anterior surface of the great vessels of the abdominal cavity and on the adrenal medulla. The PNS has ganglia that are immediately adjacent to the organs that are being
innervated.

Blushing Gastrointestinal immotiltiy


Piloerection Penile erection

12. The adrenal gland is considered an extension of the


SNS. Stimulation of the SNS will initiate the release

of the hormones epinephrine and norepinephrine


from the adrenal medulla. Cortisol and aldosterone,

produced in the adrenal cortex, will also be released.


Which of the effects below can be attributed to the

hormones released by SNS stimulation of the adrenal


cortex ?

9.

The control of individual organs is considered an

important function of the ANS. However, the PNS component is thought to be more specific in its regulation than the SNS. Which statement(s) best
explain this theory ?

I. II. III. IV.


A. B. C.
D.

Gluconeogenesis Retention of ions and water in the kidney. Proteolysis Storage of glucose in the liver.
I and II only I, II, and in only Ill and IV only
I, II, m, and IV

I.
II.

A response to stressful stimuli needs a general


stimulation of the body, thus the SNS is less specific. The SNS developed much later in evolution
and thus is not as refined as the PNS.

III.
IV.

The regulation needed to maintain the bodily systems at equilibrium and after stress must be more specialized to each organ innervated. The location of the ganglia of the PNS allows it to regulate organ function more closely.
I only II, III, and IV only Ill and IV only I, III, and IV only

13. The vagus (10th cranial) nerve is a component of the


PNS. Which of the following effects is LEAST likely to be associated with the PNS?
A.
B.

A. B. C. D.

Constriction of the pupils.


Increased heart rate.

C.
D.

Increased gastric motility.


Increased secretion of HC1 in the stomach.

Copyright by The Berkeley Review

43

The Berkeley Review

Specializing in MCAT Preparation

Biology
Passage III (Questions 14-20)

Action Potentials

Passage III

Many animal cells have an intracellular fluid (ICF)


that is electrically negative relative to the extracellular fluid (ECF), creating a voltage difference across the cell's membrane called the resting membrane potential (Em). The Em in nerve cells and muscle cells is about -80 mV, while in epithelial and red blood cells it is about -30 mV. In nerve and muscle cells the concentration gradients

If a stimulus is strong enough so that the threshold potential of a cell is reached (about 15 mV from the Em), an action potential (Figure 2) will be generated that will propagate along the nerve cell until it reaches the presynaptic ending. A neurotransmitter is released and diffuses across the synaptic cleft, where it binds to the postsynaptic membrane. A postsynaptic potential of equal magnitude is generated in the postsynaptic cell and the signal continues. The response of the action potential is all-or-none. The phases are shown in Table 2.

and the permeability of the membrane to both Na and K determine the Em. Theconcentrations of these ions in
the ICF and ECF are shown in Table 1. They are

maintained by the Na/K-ATPase pump, a pump that transports 3 Na ions to the ECF for every 2 K ions
transported to the ICF. At rest, the relative permeability

of the membrane to K is roughly 10times greater than it


is to Na.

Table 1
Ion
Na+
K+

[Intracellular] [Extracellular]
15 mM

tEi(
Time (msec)
+58 mV -92 mV

140 mM
4mM

135 mM

Figure 2
Table 2

'Ejon = Equilibrium potential for the ion

1. Resting State: Thecell at itsEm.


If the Em transiently changes from its resting value, an electrical signal is generated in the nerve cell. There are two types of electrical signals: graded potentials and action potentials (AP). Graded potentials are signals that operate over short distances, while action potentials are signals that operate over long distances.
Along the plasma membrane of a nerve cell are
2. Threshold: Potential that generates an AP.

3. Depolarization: Due to influx of Na+.


4. Overshoot: AP with a positive membrane potential. 5. Repolarization: Due to efflux of K+. 6. Hyperpolarization: Membrane potential is more

negative thanthe E^
7. Absolute Refractory Period: AP cannot be generated. 8. Relative Refractory Period: AP can be generated if the stimulus is strong enough.

voltage-sensitive Na and K channels that contain

gates. TheNa channel has anmgate that opens quickly


and an h gate that closes slowly upon depolarization. The

K channel has an n gate that opens slowly upon depolarization. The Na gate can be blocked by tetrodotoxin (TTX), and the K gate can be blocked by
tetraethylammonium (TEA). In the resting state the Na and K channels have their gates arranged as shown in
Figure 1: 14. The threshold potential for the generation of an action potential can be found at that value of the Em
where the:

m **

ECF

n
A. influx of K balances the efflux of Na.

membrane

B.

efflux of K is 10times greater than the influx


ofNa. efflux of K balances the influx of Na.

1/\l

icf
K+

C.

D.

influx of Na is 10 times greater than the


efflux of K.

Figure 1 Copyright by The Berkeley Review


44

The Berkeley Review Specializing in MCAT Preparation

Biology
15.

Action Potentials

Passage m

The threshold potential for the action potential shown in Figure 1 is:
A. B.

20. Gamma-aminobutyric acid (GABA) is a major


inhibitory neurotransmitter in the central nervous

system. It is synthesized from glutamate, a major


-50 mV. -65 mV. -80 mV.

excitatory neurotransmitter in the brain.


COOH

C.
D.

-92 mV.

HjN-C-H
CH,
I

Enzyme

HjN-C-H

CH2
CH,
I

16.

During depolarization, the quick opening of the voltage dependent m gates results in all of the following EXCEPT:
A. an influx of Na ions.

CH,
I

COOH

COOH

Glutamic acid

GABA

B. C.
D.

an increase in the opening of fast m gates in

The enzyme that catalyzes this reaction is best


described as:

neighboring Na channels.
an increase in the opening of fast n gates in

neighboring K channels.
an absolute refractory period.

A. B. C. D.

GABA decarboxylase. glutamic acid decarboxylase. GABA carboxylase. glutamic acid carboxylase.

17.

Which of the following voltage gated positions best represents the repolarization phase?

A. B. C. D.

m gate open; h gate closed; n gate open. m gate closed; h gate closed; n gate open. m gate open; h gate open; n gate open. m gate closed; h gate open; n gate closed.

18.

During the last half of the relative refractory period shown in Figure 1, the membrane potential is
beginning to: A. B. C. D. repolarize. hyperpolarize. depolarize. hypopolarize.

19.

If TTX and TEA are added to a nerve cell

preparation and a depolarizing stimulus is applied to the presynaptic ending, a postsynaptic potential will
be produced. This shows that:
A. Na and K ions are needed for the release of
the neurotransmitter to occur.

B.
C.
D.

Na and K ions are not responsible for the


release of the neurotransmitter.

TTX and TEA can amplify the postsynaptic

potential by allowing neurotransmitter release.


an increase in ICF K ions and an increase in ECF Na ions inhibit the release of the neurotransmitter, unless a depolarizing stimulus is applied.

Copyright by The Berkeley Review

45

The Berkeley Review Specializing in MCAT Preparation

Biology
Passage IV (Questions 21-26)

Local Anesthetic

Passage IV

23.

Which of the following statements is LEAST compatible with the effects of local anesthetics?
A. Local anesthetics slow the rise of the action

Local anesthetics provide a reversible block in the conduction of impulses in nerve fibers. All local anesthetic drugs contain a lipophilic group, an
intermediate chain, and a hydrophilic group. The intermediate chain may be either an ester (as in procaine) or an amide (as in lidocaine). The activity and potency of any local anesthetic depends on several aspects, including its chemical structure, and the pKa of the substance. Local anesthetics are weak bases, so they are kept in an acid solution in which they form a water-soluble salt. The base is lipid-soluble and can penetrate various tissue
barriers. The concentration of base or cation in the

B.
C.

potential in nerve fibers. Local anesthetics slow the propagation of nerve impulses.
Local anesthetics decrease threshold for electrical stimulation in nerve fibers.

D.

Local anesthetics prevent the depolarization of


cell membranes.

24.

solution depends on the pKa of the local anesthetic. A


decrease in the amount of base facilitates removal of the

The pKa of lidocaine is somewhere between 7.6 and 7.8. The pKa of procaine is between 8.1 and 8.6. Based on this information, which of the following statements is the MOST likely conclusion? A. The onset of lidocaine is more rapid, because more exists in its base form at body pH. The onset of lidocaine is more rapid, because more exists in its acidic form at body pH.
The onset of lidocaine is slower, because more

local anesthetics, resulting in a shorter duration of action.


The duration of action of a local anesthetic is also affected

by the local vasodilation which most of them produce. Finally, experiments have shown that a nerve which has been recently and repetitively stimulated is more sensitive to a local anesthetic than a resting nerve.

B.
C.

exists in its base form at body pH.


D.
C,Hc

The onset of lidocaine is slower, because more

exists in its acid form at body pH.

/r~ v~ c~ CH2~ -N-~qH<i


CH,

25.

Lidocaine at pH = 9.0

If acidosis develops after a local anesthetic has penetrated a tissue barrier, it is most likely that:
A. the amount of ionized form will be decreased,

Figure 1

and the local anesthetic will freely cross the


tissue barrier.

21.

Local anesthetics penetrate the blood-brain barrier with great ease, most likely because of their: A. B. C. D. large molecular size and high lipid solubility. large molecular size and low lipid solubility. small molecular size and high lipid solubility. small molecular size and low lipid solubility.

B.

the amount of ionized form will be decreased,

and the local anesthetic will be trapped in the


tissue.

C.

the amount of ionized form will be increased, and the local anesthetic will freely cross the
tissue barrier.

D.

the amount of ionized form will be increased,

and the local anesthetic will be trapped in the


tissue.

22.

Based on the passage, which of the following statements is most likely true?
A. The higher the pKa of the local anesthetic, the
lower the concentration of base in the tissue.

26.

The structure of lidocaine is given in Figure 1. Lidocaine is administered through injection, rather than orally. This is because lidocaine:
A.
B.

B.

C.
D.

The lower the pKa of the local anesthetic, the higher the concentration of acid in the tissue. The higher the Ka of the local anesthetic, the
lower the concentration of base in the tissue.

will be deactivated by the acidic conditions of


the stomach. will not be absorbed across the intestinal wall.

C.

is biotransformed by reactions occurring in the


liver.

The lower the Ka of the local anesthetic, the


lower the concentration of acid in the tissue.

D.

is biotransformed by reactions occurring in the spleen.

Copyright by The Berkeley Review

46

The Berkeley Review

Specializing in MCAT Preparation

Biology
Passage V (Questions 27-33)

The Lens, the Iris, & Associated Muscles

Passage V

27. For a person watching a meteor shower at midnight,


what are the contraction states of the circular muscle

Figure 1 shows the lens, the iris, and the muscles of the eye.

of the iris, the radial muscle of the iris, and the ciliary muscle? A. The circular muscle of the iris is contracted, the radial muscle of the iris is relaxed, and the ciliary muscle is relaxed. The circular muscle of the iris is relaxed, the
radial muscle of the iris is contracted, and the

Pupil
Cornea

B.

ciliary muscle is contracted.


Suspensory Ligaments

C.

The circular muscle of the iris is relaxed, the radial muscle of the iris is relaxed, and the

Optic
Nerve

ciliary muscle is relaxed.


D. The circular muscle of the iris is relaxed, the radial muscle of the iris is contracted, and the

Figure 1

ciliary muscle is relaxed.

The opening of the pupil of the eye is controlled by two sets of muscles. The circularly arranged smooth muscle is under parasympathetic control, and the radially arranged smooth muscle is under sympathetic control. Figure 2 shows the relation between these muscles and the pupil of the eye. The size of the pupil reacts to the amount of light present.

28.

As people age, the lens becomes less llexible and therefore less able to change shape during accommodation. What happens to a person's vision with these age changes? A.
B.

Radial Smooth Muscle Fibers

Circular Smooth Muscle Fibers

C.
D.

The lens is less able to focus from near objects. The lens is more able to focus from near objects. The lens is more able to focus from far objects. The lens is less able to focus from far objects.

the light rays


the light rays

the light rays


the light rays

Dim Light

Bright Light

29.

Special solutions are used to dilate the pupil during a


retinal exam, so that the retina can be viewed. The

Figure 2

solution causes an inability to focus on near objects, reduces clear distance vision, and enlarges the pupil.
What is the stimulation or inhibition required to

The lens of the eye is suspended within and surrounded by a ring of tissue called the ciliary muscle. Suspensory ligaments connect the lens to the ciliary muscle. When the ciliary muscle is relaxed, the suspensory ligaments pull the lens taut and Hat. Since we spend most of our time in far-away vision (20 feet or
more), this is ideal. For closer work, the lens must accommodate and become thicker for focusing. In the

produce these changes in the eye?

A. B.
C. D.

Sympathetic nerves arc inhibited, and parasympathetic nerves are stimulated. Sympathetic nerves are stimulated, and parasympathetic nerves are inhibited.
Both are stimulated. Both are inhibited.

process, the ciliary muscle tightens, the suspensory ligaments become slack, and the lens thickens and
becomes more convex.

Copyright by The Berkeley Review

47

The Berkeley Review

Specializing in MCAT Preparation

Biology

The Lens, the Iris, & Associated Nusdes

Passage V

30. When light rays travel through the pupil and the lens, upon which part of the eye does the image
focus? A. B. Retina Cornea

C. D.

Optic nerve Vitreous body

31.

One eye exercise involves focusing on an object while it moves from arm's length to the closest point upon which you can focus. This causes the lens to undergo accommodation, that is, the lens must change shape to keep the object in focus as it moves closer. As the object moves nearer, what changes occur in the eye?

I. The pupil gradually contracts. II. The ciliary muscle gradually contracts. III. The suspensory ligaments gradually contract.
A. B. C. D. I only II only IH only II and III only

32. The iris is pigmented epithelial tissue. The color of the eye is determined by the amount of pigment. Blue eyes have the least pigment, brown eyes have more, and black eyes have the greatest amount of pigment. What color are the eyes of a person who has no pigment in her iris?
A. B. C. D. Black

White Pink Green

33.

Astigmatism is the condition of having a nonuniformly shaped lens or cornea. This means that parallel light rays do not focus, so that a sharp image is not formed. How could this eye condition be
corrected?

A. B. C. D.

By By By By

providing a convex corrective lens. providing a concave corrective lens. providing a non-uniform corrective lens. providing a uniform corrective lens.

Copyright by The Berkeley Review

48

The Berkeley Review Specializing in MCAT Preparation

Biology
Passage VI (Questions 34-40)

Resting Membrane Potential

Passage VI

34.

The cell membrane acts as an insulator, because the:

Separated electric charges of opposite sign have the capacity to do work if they are allowed to come together. This electric potential is more commonly known as simply the potential. All cells in a resting state have a difference in potential across their cell membranes, the inside being negative with respect to the outside.
The magnitude of the resting membrane potential is determined primarily by two factors. One is the typical
concentration of three kinds of ions within the

A.
B.

C.
D.

proteins making up the cell charged. proteins making up the cell uncharged. lipids making up the cell charged. lipids making up the cell uncharged.

membrane are
membrane are

membrane are
membrane are

intracellular and extracellular fluids (Table 1):


Extracellular Ion Intracellular

(mmol/L)
150 110 5

(mmol/L)
15 10 150

35.

The resting membrane potential of a typical neuron


is*

Na Cle K

A.

positive, with the excess charge representing a very large fraction of the total number of ions
inside and outside the cell.

B.

Table 1

positive, with the excess charge representing a very small fraction of the total number of ions
inside and outside the cell.

The second factor is the ability of each kind of ion to

penetrate the plasma membrane, an ability that fluctuates


from one moment to the next. In Table 1, notice that the

C.

negative, with the excess charge representing a very large fraction of the total number of ions
inside and outside the cell.

sodium and chloride ion concentrations are generally lower inside in the cell, while potassium ion concentration
is lower outside the cell. The concentration differences

D.

negative, with the excess charge representing a very small fraction of the total number of ions
inside and outside the cell.

between sodium and potassium are produced by a plasma membrane active transport system that simultaneously

pumps sodium out of the cell and potassium into the cell.
As an ion moves down its concentration gradient, an electric force is established that opposes the movement of that ion. The membrane potential that exists when the electric force is equal in magnitude but opposite in direction to the concentration force is the equilibrium

36.

Which of the following diagrams BEST represents the equilibrium potential for potassium across a
nerve cell membrane?
B.

potential for that ion. The equilibrium potential for each ion species is different in magnitude-sometimes even in
directionand can be calculated using the Nernst equation:

High
K
Low

c =.RT ln Co
ZF d

In a nerve cell at rest, the cell membrane is 50 to 75

times more permeable to potassium than to sodium. During the resting potential, there is a net diffusion of
ions into and out of the cell. The concentration gradients, which would eventually dissipate, are maintained through

the Nae/Ke pump. If the concentration gradients remain fixed and the permeabilities of ions do not change, the resting membrane potential remains constant.

Copyright by The Berkeley Review

49

The Berkeley Review Specializing in MCAT Preparation

Biology
37.

Resting Membrane Potential

Passage VI

The resting membrane potential lies above the equilibrium potential for potassium, because a small
number of: A. chloride ions diffuse into the cell.

B. C. D.

chloride ions diffuse out of the cell. sodium ions diffuse into the cell. sodium ions diffuse out of the cell.

38.

According to information in the passage, the concentration gradients of ions are maintained

through the Nae/ K pump. The ultimate source of


energy for this pump is the:
A. B. formation of ATP. breakdown of ATP. oxidation of nutrients. reduction of nutrients.

C. D.

39. The constancy of the resting membrane potential is


an example of: A. B. C.
D.

an equilibrium state. a dynamic state. a steady state.


a static state.

40. The plasma membrane of many cells is permeable to


chloride ions and does not contain chloride ion

pumps. The membrane potential set up by other ions thus acts on chloride ions. According to this information and information in the passage, the new resting membrane potential is:
A. B. C. D. changed only slightly by the chloride ion. changed significantly by the chloride ion. changed greatly by the chloride ion. not affected at all by the chloride ion.

Copyright by The Berkeley Review

50

The Berkeley Review

Specializing in MCAT Preparation

Biology
Passage VII (Questions 41-46)

Nicotine Replacement Therapy


42.

Passage VII

The following graph shows the results of a binding assay on the muscarinic acetylcholine receptor:

Acetylcholine is one of the body's most important neurotransmitters, responsible for the transmission of nerve impulses across synaptic junctions. There are two main classes of acetylcholine receptors. The nicotinic acetylcholine receptor responds to nicotine as an agonist and to curare as an antagonist. The muscarinic acetylcholine receptor responds to muscarine as an agonist and to scopolamine as an antagonist.
Nicotine is a psychoactive alkaloid extracted from tobacco plants. It is a toxic substance, one that places a
stress on the heart and the entire cardiovascular system. Because this drug is quite addictive, several techniques have been designed to cure nicotine dependence.
[Muscarine]

In a novel nicotine replacement therapy, a

Scopolamine, a competitive inhibitor of muscarine, is added to the solution and the assay is repeated. Which of the following graphs BEST represents the results of the second assay?

combination of physostigmine and scopolamine is administered. Physostigmine is an acetylcholinesterase


inhibitor at both muscarinic and nicotinic acetylcholine

receptors. It must be noted that scopolamine is functional in the central nervous system, but receptors in heart tissue are insensitive to scopolamine. Figure 1 schematically depicts the action of these substances in the autonomic
nervous system.
[Muscarine]
[Muscarine]

Parasympathetic division
C.

1 w "1 * 1
Nicotinic

Effector

receptor

Muscarinic receptor

organ

Sympatheti c

division
Effector organ

[Muscarine]

[Muscarine]

PSl^
/
Nicotinic

1 : 1 no :

Adrenergic
receptor

receptor

CNS = Central nervous system

43.

= Acetycholine

o = Norepinephrine

Figure 1

The nicotinic acetylcholine receptor, which is made of a pentamer of subunits, is essentially a sodium channel. Which of the following occurs directly after acetylcholine binds to a nicotinic acetylcholine receptor?

A.
41. Which of the following ions is responsible for the release of acetylcholine-filled vesicles from a presynaptic nerve terminal?
A. B. C. Na K Cle

B. C.
D.

An action potential is generated in the postsynaptic cell. An inhibitory postsynaptic potential is


generated.

An excitatory postsynaptic potential is


generated. There is no change in the resting membrane

potential of the postsynaptic cell.

D.

Ca2

Copyright by The Berkeley Review

51

The Berkeley Review

Specializing in MCAT Preparation

Biology
44.

Nicotine Replacement Therapy

Passage VII

The novel nicotine treatment described in the

passage theoretically should be superior to


traditional nicotine addiction treatment, because it

offers patients:
A. no net nicotine excitation.

B.
C.
D.

a net nicotinic and sympathetic nervous system


excitation.

a net nicotinic and parasympathetic nervous system excitation.


a net nicotinic and neuromuscular excitation.

45.

Cardiovascular stress is one common result of

ingesting nicotine by smoking cigarettes. This supports the idea that:


A. nicotine stimulation of the nicotinic

acetylcholine receptor is dominant in the sympathetic nervous system.


B. nicotine stimulation of the nicotinic

C.

D.

acetylcholine receptor is weaker in the sympathetic nervous system. nicotine acts directly on cardiac muscle to promote overstimulation and stress. nicotine acts indirectly through the central nervous system to produce cardiac stress.

46.

As part of an experiment, a molecule is introduced at the synapses between heart muscle and nerve fibers

of both the sympathetic and parasympathetic


nervous systems. The molecule moves in a retrograde fashion through both of these divisions.

By the time it reaches the next synapse, the molecule will have traveled through:
A. B. C. D. a dendrite, moving a greater distance in parasympathetic nervous system than in sympathetic nervous system. a dendrite, moving a shorter distance in parasympathetic nervous system than in sympathetic nervous system. an axon, moving a shorter distance in sympathetic nervous system than in parasympathetic nervous system. an axon, moving a greater distance in sympathetic nervous system than in parasympathetic nervous system. the. the the the the the the the

Copyright by The Berkeley Review

52

The Berkeley Review

Specializing in MCAT Preparation

Biology
Passage VIII (Questions 47-52)

Retinal Projections
49.

Passage VIII

The blind spot, located at the optic disc, is called so


because:

The optic nerve is formed from the axons of all retinal ganglion cells. The optic nerves from each eye join at the optic chiasm and eventually enters either the left or right optic tract. The optic tract projects to three subcortical areas. One is the lateral geniculate nucleus, which is responsible for processing visual information. One is the pretectal area, which produces pupillar reflexes based on information from the retina. Finally, the superior colliculus uses the information from the retina to
generate eye movement.

A.
B.

this is the region where ganglion cells leave


the retina. color vision is not available at this retinal
location.

C.
D.

this area is free of photoreceptors.


light is unable to reach this small area of the
retina.

When light is shone upon one eye, it causes constriction of the pupil in both eyes. Constriction of the eye in which the light is shone is the direct response while
constriction of the other is known as the consensual

50.

The neurotransmitter released by the axons in the Edinger-Westphal neurons is most likely: A. B. C. D. epinephrine. norepinephrine. acetylcholine. glutamine.

response. The pupillary reflexes are mediated through retinal ganglion neurons that project to the pretectal area which lies anterior to the superior colliculus. The cells in the pretectal area project bilaterally to preganglionic parasympathetic neurons in the Edinger-Westphal nucleus. This is also known as the accessory occulomotor nucleus. The preganglionic parasympathetic neurons in the Edinger-Westphal nucleus send axons through the occulomotor nerve to innervate the ciliary ganglion. The ciliary ganglion's postganglionic neuron innervates the smooth muscle of the pupillary sphincter.

51.

As part of a routine eye exam, the following is noticed: If light is shone directly into the patient's left eye, the patient exhibits a consensual but not a direct response. Which of the following is a likely

explanation?
A. 47. It has been determined that the frequency of action potentials increases dramatically in axons once they have left the optic nerve. The most likely explanation for this increase is: The optic nerve of the left eye is intact, but the efferent limb of the left eye is damaged. The optic nerve of the left eye is damaged, but the efferent of the left eye is intact. The optic nerve of the right eye is intact, but the efferent limb of the right eye is damaged. The optic nerve of the right eye is damaged, but the efferent limb of the right eye is intact.

B.
C. D.

A.
B.
C. D.

a higher density of sodium channels are found in the axons leaving the optic disc. a lower density of sodium channels are found in the axons leaving the optic disc. the axons are myelinated by Schwann cells. the axons are myelinated by oligodendrocytes.

52. In another patient, the following is observed: Light shone into the right eye does not elicit a response in either pupil. Light shone directly into the left eye causes a direct and consensual response. It can be 48. The right optic tract can be described as an:
concluded that there is a lesion in the:

A.
B.

efferent pathway, containing nerve axons from


only one eye.

efferent pathway, containing nerve axons from


both ways.

A. B. C.

left optic nerve. right optic nerve. left preganglionic parasympathetic neuron.

D.

right preganglionic parasympathetic neuron.

C.
D.

afferent pathway, containing nerve axons from


only one eye.

afferent pathway, containing nerve axons from


both eyes.

Copyright by The Berkeley Review

53

The Berkeley Review

Specializing in MCAT Preparation

Biology
Passage IX (Questions 53-58)

Axonal Transport

Passage IX

56. Glucose does not simply diffuse over the inner


mitochondrial membrane. Glucose enters the mitochondrial matrix as:

Movement of substances from the soma to the

synaptic endings of a nerve cell by simple diffusion is an inefficient process, because axons are often very long. However, components which originate in the soma must
be distributed along the axon. Axonal transport, a process which costs metabolic energy, is a special transport mechanism that accomplishes this function. Membranebound organelles are transported rapidly by fast axonal transport, while substances dissolved in cytoplasm are moved by slow axonal transport. Microtubules provide a pathway along which membrane-bound organelles can move. These organelles may interact with the microtubules through a linkage similar to that between
thick and thin filaments of skeletal muscle cells.

A. B. C.
D.

glucose-6-phosphate. phosphoenolpyruvate. pyruvate.


oxaloacetate.

57.

Phaseolus vulgaris leucoagglutinin is taken up by neurons and is transported anterogressively. This marker substance is most likely injected at the: A. B. C. D. synapse to trace the pathway of neuronal
axons.

Transport from the soma toward the axonal terminal is known as anterograde axonal transport. Retrograde axonal transport is transport in the opposite direction. In order to trace neural pathways experimentally, marker substances can be used that are transported either retrogressively or anterogressively.

synapse to trace the location of cell bodies. soma to trace the pathway of neuronal
dendrites.

soma to trace the synaptic endings of neurons.

58. The graph shown below represents the response of a postsynaptic cell to acetylcholine in the synaptic cleft. Each vertical line (I) represents an action
potential.

53.

A neurotransmitter traveling from the soma to the presynaptic terminal will travel:
time L

A. B. C. D.

anterogressively, via fast axonal transport. anterogressively, via slow axonal transport. retrogressively, via fast axonal transport. retrogressively, via slow axonal transport.

Which of the following graphs BEST represents the response following the addition of the enzyme
acetylcholinesterase?
A.

54.

Which of the following ions most likely triggers an interaction between organelles and microtubules?
A. B. Na K
time

C.

Ca2

B.

D.

Mg2
time

^>

55.

Neuropeptide packaging into vesicles will include all of the following EXCEPT the:

C.

ill
time

A. B. C. D.

signal recognition particle receptor. signal peptide. complete translation of the peptide in the cytoplasm. transport through the Golgi apparatus.

^>

D.

time

Copyright by The Berkeley Review

54

The Berkeley Review Specializing in MCAT Preparation

Biology
Passage X (Questions 59-64)

Huntington's Disease

Passage X

61.

The following molecule is referred to as GABA.


What is its IUPAC name?

Huntington's disease is a relatively rare disease that causes progressive degeneration of the cerebral cortex and the basal ganglia of the brain. The basal ganglia are masses of neuron cell bodies (gray matter) located deep within the cerebrum. Within the basal ganglia, certain types of neurons are destroyed, while others remain intact.
A.

H3N CH2- CH2- CH2- COO

y-amino butyrate
3-amino butanoic acid

Huntington's disease is a genetic disorder caused by an autosomal dominant genetic defect on the tip of chromosome 4. The symptoms of the disease are choreiform movements (rapid, uncontrolled, jerky
movements), mental deterioration, and emotional

B.

C.
D.

1-amino butyrate
4-aminobutanoic acid

disturbances. The disease is relentless and usually leads to complete dehabilitation and death within 15 years of its onset. The age of onset is usually from 35-50 years, usually after the patient has had children.
The proposed mechanism of the defect lies with
certain neurotransmitters and is twofold. First of all, there

62.

Which amino acid is the precursor to GABA?


A. B. Histidine Leucine

is a decreased level of y-aminobutyric acid, an inhibitory neurotransmitter, and a decreased level of the enzyme that synthesizes it, glutamic acid decarboxylase. Secondly, a deficiency of the enzyme choline acetylase leads to decreased levels of the excitatory neurotransmitter, acetylcholine. The delicate homeostatic balance between the two neurotransmitters is disrupted. These deficiencies are thought to lead to the observed characteristic movements and mental symptoms of Huntington's
disease.
63.

C.
D.

Tyrosine
Glutamate

After having one child, a 40-year old woman with a paternal family history of Huntington's disease is diagnosed as having the disease. Assuming that the father is not affected, what is the MOST probable likelihood that the child will also develop the
disease? A. 100%
75%

59.

Based on information in the passage, Huntington's


disease is:

A. B.

more common in men than in women. more common in women than in men.

B.

C.
D.

50%
25%

C.
D.

equally common in both men and women.


more common in children.

60.

Based on this passage, the basal ganglia most likely


control:

64.

In what structure of the neuron are neurotransmitters


stored?

A. B. C.
D.

pituitary gland secretion. speech. the parasympathetic nervous system.


voluntary movements.

A. B. C. D.

The synaptic cleft. The vesicles of presynaptic neurons. The receptors of postsynaptic neurons. The vesicles of postsynaptic neurons.

Copyright by The Berkeley Review

55

The Berkeley Review Specializing in MCAT Preparation

Biology
Passage XI (Questions 65-71)

Photoreceptors

Passage XI

65.

Which of the following retinal cells is LEAST responsible for vision in the dark?
A. B. Cone cells. Rod cells.

The retina of the human eye contains two types of photoreceptors known as rods and cones, both located at
the back of the retina. Upon stimulation by a photon, the sodium channels of these photoreceptor cells close. The highest density of cones found on the retina will give the greatest visual acuity, or highest visual precision. In addition to the photoreceptor cells, the retina houses four types of neurons. These cells are the bipolar, ganglion,
horizontal, and amacrine cells. The rods and cones

C. D.

Ganglion cells. Bipolar cells.

synapse with bipolar cells, which then go on to synapse with ganglion cells. The axons of the ganglion cells converge and leave the eyes as the optic nerve.
The visual field is the view seen by the two eyes without movement of the head. As shown in Figure 1, the left visual hemifield projects to the right side of the brain, while the right visual hemifield projects to the left half of
the brain.

66.

Stimulation of a photoreceptor by a single photon


will result in a:

A. B. C. D.

monopolarization of the photoreceptor. depolarization of the photoreceptor. hyperpolarization of the photoreceptor. micropolarization of the photoreceptor.

67.
Left Visual
Hemifield

Right
Visual Hemifield

To view an object with greatest acutiy, one will focus which of the following structures on that object?
A. B. Fovea centralis Cornea

C.
D.

Optic disc
Choroid

68.

An experiment involved the removal of the lateral geniculate nucleus of the thalamus. To determine the effects on processing of visual information,

electrodes were placed in which of the following


lobes of the brain?
A. B. Parietal Frontal

C. D.

Temporal Occipital

69.
Left Optic Tract

Light entering the eye will pass which of these cells


first?
A. Rod cells
Cone cells

Right Optic Tract

Figure 1

B.

C. D.

Bipolar cells Ganglion cells

Copyright by The Berkeley Review

56

The Berkeley Review Specializing in NCAT Preparation

Biology

Photoreceptors

Passage XI

70. The region of the retina where the axons of the

ganglion cells converge and leave as the optic nerve


is best described as the region: of highest visual precision, because of the dense axonal population. II. of the retina most sensitive to stereovision, because of a high density of photoreceptors. III. where no vision is possible, because of a lack of photoreceptors. I.
A. B. C. D. I only I and II only Ill only II only

71. According to Figure 1, light originating from the left


hemivisual field will strike the nasal hemiretina of
the:

A.

B.
C. D.

right eye and the temporal hemiretina of the left eye. right eye and the temporal hemiretina of the right eye. left eye and the temporal hemiretina of the right eye. left eye and temporal hemiretinal of the left
eye.

Copyright by The Berkeley Review

57

The Berkeley Review

Specializing in MCAT Preparation

Biology
Passage XII (Questions 72-79)

Sound Transmission In The Ear

Passage XII

72.

Sound is created by disturbances within a medium

The human ear normally responds to a range of sound covering about 120 dB. The loudest sound pressure level that can be heard over this range is:
A. B. ten thousand times less than the reference
pressure.

resulting in the production of pressure waves detected by the ear. These pressure waves consist of alternating compressions and rarefactions of the surrounding
medium. The loudness of a sound is determined by the

one million times less than the reference


pressure.

amplitude of these pressure waves and is measured on the


decibel (db) scale. The relationship is as follows:
Sound Pressure Level (SPL) = 20 logio ?t / Pr

C.

ten thousand times greater than the reference


pressure.

D.

one million times greater than the reference


pressure.

where Pt is the test pressure and Pr is the reference

pressure. Pr has the value of20 micro-Newtons/m2. The


pressure waves travel through the external ear canal and
will cause the tympanic membrane of the middle ear to
vibrate.

73.

According to Figure 1, the tympanic membrane has

A set of small bones is responsible for transmitting this vibration throughout the middle ear, and the last of these bones (the stapes) is attached to the oval window.
The oval window is the connection between the middle

an area greater than the oval window. A result of


this design is that the total:

and internal ear, which is composed of the cochlea and the vestibular apparatus (Figure 1).
Scala Media

A. B. C. D.

force acting on the oval window is increased. force/unit area acting on the oval window is
decreased.

force acting on the oval window is decreased. force/unit area acting on the oval window is
increased.

6. i " o'

,-.

Incus

Malleus ~Z*WTwH4 \' StaPes

. ,

. 1- \ Scala VestibulL
74.

Tympanic
Membrane

The relationship between the speed of sound (C), wavelength (k), and frequency (v) is:
C = (X)(v)

Auditory
Canal

. ,

indow

Tympani

Internal Ear Middle Ear

where C = 340 m/s. What is the period of a sound

wave with a wavelength of 3.4x 10"4 km?


A. B. C. D. Ixl0"6s. 1 x 10"3 s. 1 x 103 s. 1 x 106 s.

Figure 1

The transduction of pressure waves into electrical

signals occurs in the internal ear. The vibration of the stapes produces pressure waves in the fluid of the scala vestibuli and scala tympani. This wave motion in the paralymph naturally sets up oscillations in the endolymph,
located in the scala media. The transduction organ (organ of Corti) is located on the basement membrane of the scala media. Oscillating movements of this organ excite

and inhibit sensory transduction cells, which transmit impulses to the brain.

75.

As a result of movement in the organ of Corti, the

sensory transducing cells most likely repond with a


potential that is: A. B. C. depolarizing. hyperpolarizing. oscillating depolarizing-hyperpolarizing.

Inducing a vibration in the temporal bone will cause


sounds to reach the cochlea. This sound transmission is

unique in that the middle ear is bypassed. While this is an


inefficient method of energy transfer, such a method is clinically useful for diagnosing auditory problems.

D.

returning to its resting membrane potential. The Berkeley Review

Copyright by The Berkeley Review

58

Specializing in MCAT Preparation

Biology
76.

Sound Transmission In The Ear

Passage XII

The following picture represents the


membrane uncoiled and stretched out flat.

basilar

78. A patient complains of auditory problems and so a tuning fork is set into vibration and placed at the
patient's ear. When the patient indicates the sound is

inaudible, the fork is placed on the temporal bone. If the sound becomes audible to the patient, the
damage can be narrowed down to the:
m

A. B. C.

external ear. middle ear. inner ear.

D.
Flexible

auditory cortex.

region

According to the resonance theory, different areas of the basilar membrane are affected by varying sound frequencies. High frequency sounds resonate best in which region?
A. B. C. D. Region 1. Region 2. Region 3. Region 4.

79. It has been discovered that every hair cell responds maximally to a particular frequency. Which of the
following graphs best represents this idea?

A.

B.

77.

The following circuit model attempts to explain


away the problem that sound arrives at the two ears
at different times. Cells will fire with maximum

E <

Frequency (kHz)

Frequency (kHz)

output when bilateral inputs arrive at the same time.


C.
Cells

D.

Input from right ear

<2>
<Z>
Input from
left ear

CL

E
<

Frequency (kHz)

Frequency (kHz)

If sound to the right ear were delayed relative to the left, which of the following cell(s) would most likely
fire?

A. B. C. D.

Cells 1 and 2. Cell 3. Cells 3 and 4. Cell 5.

Copyright by The Berkeley Review

59

The Berkeley Review Specializing in MCAT Preparation

Biology
Passage XIII (Questions 80-87)

Tryptophan and Serotonin Experiment

Passage XIII

Table 2 shows the study results for the animals receiving protein first (Trial 2). Serum tryptophan and serum
LNAA are in umol/1, and cortex tryptophan is in umol/g.

The change in tryptophan levels in the brain and in the synthesis rate of serotonin (a neurotransmitter) from tryptophan in the brain after the ingestion of two meals is examined in several experimental animals. The synthesis
and release of serotonin by brain neurons is known to be

Group
No Food

Serum Serum Trp LNAA


142 162* 165*
190*

Cortex

Trp
26

strongly influenced by the local tryptophan concentration


near these neurons.

511
497

CHO-CHO

33*

Brain tryptophan (Trp) concentration reflects the uptake of tryptophan from the blood into the brain. This uptake is accomplished by a transporter in the blood-brain
barrier that is shared by the large, neutral amino acids (LNAAs): leucine, isoleucine, tyrosine, phenylalanine, and tryptophan. Food ingestion can alter levels of serotonin in the brain by altering blood concentrations of tryptophan and its LNAA competitors. The design of the experiment requires that two meals given two hours apart be fed to laboratory rats that have fasted overnight. The five experimental diets fed to the
rats contain varying ratios of protein and carbohydrate, with fat held at a low and constant percentage in all of the diets. The diets are designated as CHO (all carbohydrate, no protein), 6% protein, 12% protein, 24% protein, and 40% protein. For the sake of reference, standard rat chow is about 10% protein, and the typical human diet is about 12-15% protein. Rats in Trial 1 are given the all-carbohydrate diet, followed by one of the protein diets two hours later. Those in Trial 2 receive one of the protein diets, followed by the CHO diet two hours later. The rats are sacrificed
two hours after the second meal, and their brain tissue is

6% Protein - CHO
12% Protein-CHO 24% Protein - CHO 40% Protein - CHO

533
621

31*
27*
28

184*

743 814

194*

25

*p = 0.05 versus no food


Table 2.

80.

The diagram below shows the molecular structure of tryptophan and the chemical reaction by which it
becomes serotonin.
COOH

Tryptophan

Serotonin

examined for tryptophan concentration. Table 1 shows the study results for the animals receiving CHO first (Trial 1). Serum tryptophan and serum LNAA are in umol/1, and cortex tryptophan is in
umol/g.

Tryptophan is converted into serotonin by first undergoing:


A. B. C. D. reduction, followed by carboxylation. hydroxylation, followed by decarboxylation. amidation, followed by hydroxylation. amination, followed by decarboxylation.

Group
No Food

Serum Serum Trp LNAA


96
140*

Cortex

Trp
28

81.

Which of the following statements about the data in


Table 1 and Table 2 is FALSE? A. Serum LNAA concentration increases as

482
376

CHO-CHO CHO - 6% Protein


CHO-12% Protein CHO - 24% Protein

36*
36* 35*
28

150*
156* 160* 162*

385
437 675 976

B.

dietary protein content increases. A 24% protein meal at two hours following a
CHO meal attenuates the increase in serotonin

CHO - 40% Protein

23

*p = 0.05 versus no food


Table 1.

C. D.

due to tryptophan. At higher levels of protein, the serum LNAAs


are converted to serotonin.

Serum tryptophan concentrations increase as dietary protein content increases.

Copyright by The Berkeley Review

60

The Berkeley Review

Specializing in MCAT Preparation

Biology
82.
data in Table 1 and Table 2?

Tryptophan and Serotonin Experiment


85.

Passage XIII

Which of these statements can be inferred from the

In gauging the metabolic effect of eating a large


amount of pure tryptophan as a dietary supplement, we should conclude that dietary tryptophan alone
would:

I.

In Trial 1, the higher protein levels (24% and 40%) cause competition between tryptophan
and the other LNAAs.

A.
B.

not be absorbed readily in the small intestine,


so it would have no metabolic effect.

II.

In Trial 2, the higher protein levels (12%, 24%, and 40%) cause competition between
tryptophan and the other LNAAs.

increase brain serotonin concentrations, since

few LNAAs would be competing.


C.
D.

III. In Trials 1 and 2, serum tryptophan increase with feeding.


A. B. C.
D.

increase the synthesis of all classes of


neurotransmitters.

I only II only II and III only


I, II, and III

cause more alert behavior, due to the production of serotonin.

83. It is accurate to describe tryptophan as:


I. II. III. a neurotransmitter. an essential amino acid. an acidic amino acid.

86.

The LNAAs described in the passage are all


essential amino acids. If the diet of an adult human

being is deficient in phenylalanine, which of the following non-essential amino acids now becomes required in the diet?
A. B.
C.

A. B. C.
D.

I only II only I and II only


I, II, and III

Asparagine Cysteine
Serine

D.

Tyrosine

84. As part of this experiment, researchers administer an inhibitor of one of the enzymes that participates in the conversion pathway of tryptophan to serotonin in the brain. Examining which factor would provide
the BEST estimate of the new rate of serotonin

87. The selectivity of the blood-brain barrier is due to:


A.

synthesis caused by this inhibitor?


A. Accumulation of an intermediate metabolite in
the brain

B.

tight junctions, gap junctions,


desmosomes. adherens.

C.
D.

B.
C. D.

Concentration of tryptophan in the blood


Concentration of the inhibitor in the brain Accumulation of the inhibitor in the blood
61

Copyright by The Berkeley Review

The Berkeley Review Specializing in MCAT Preparation

Biology
Passage XIV (Questions 88-94)

Frog Muscle Experiment


88.

Passage XIV

The muscle is stretched more and more as heavier

The gastrocnemius muscle along with the sciatic nerve from a frog's leg were dissected and attached to a device to measure action potentials and record them on chart paper. Figure 1 is a diagram of the experimental
setup:

weights are added to the experimental apparatus. How is the intensity of the muscle stretch indicated by the activity of the nerve fiber? A. B. C. D. A larger amplitude of the action potentials indicates greater intensity. A larger amplitude of the action potentials
indicates lower intensity.

A greater frequency of the action potentials indicates greater intensity. A greater frequency of the action potentials indicates lower intensity.

89.

Two more trials were performed on the same muscle preparation. A 75-g and a 100-g weight were used to stretch the muscle. The printouts of the action potentials looked exactly like the 50-g trial. What is the explanation for this? A. The action potentials had increased to a maximum with the 50-g weight and increased no further, due to the refractory period of the
neuron.

Figure

Weights (1, 2, 5, 10, 20, and 50 gm) were hung

opposite the muscle, so that a known set of tensions were applied to it. The action potentials generated by each weight were recorded on the chart paper. This is illustrated in Figure 2. Each vertical ([) line represents an
action potential.

B.

C.
D.

The nerve and muscle were damaged by the weights, and the nerve therefore slowed its transmission of action potentials. The speed of the chart recorder could not keep up with the increased action potentials. The action potentials were identical, due to a
neurotransmitter defect.

90.
1 gm

Figure 3 indicates the output of action potentials after a 5-mg weight was added to the muscle preparation. What is the explanation for this effect?

5 gm

10 gm
time

:>
Figure 3

20 gm

50 gm
A.
time

:>
B.
C.

Figure 2

The weight was not heavy enough to exceed the action-potential threshold. The weight was too heavy and suppressed the action-potential threshold.

The weight must produce an action potential,


so the recording equipment must be damaged. The size of the action potential was too small to read on the printout.

The purpose of this experiment was to learn more


about the interaction between muscle and nerve, and how muscle stretch is communicated.

D.

Copyright by The Berkeley Review

62

The Berkeley Review Specializing in MCAT Preparation

Biology
through which structure?

Frog Muscle Experiment

Passage XIV

91. Actions potentials are conducted by the nerve cell


A.
B.

Cytoplasm
Schwann cell membrane

C.
D.

Endoplasmic reticulum
Plasma membrane

92. When an action potential is communicated, what is the sequence and direction of movement of sodium
and potassium?
A. Potassium moves from the intracellular fluid to

the outside of the cell, and then sodium moves


B.
from the extracellular fluid into the cell. Sodium moves from the extracellular fluid into

the cell, and then potassium moves from the


C.
intracellular fluid to the outside of the cell. Sodium moves from the intracellular fluid to

the outside of the cell, and then potassium


D. moves from the extracellular fluid into the cell. Potassium moves from the extracellular fluid into the cell, and then sodium moves from the intracellular fluid to the outside of the cell.

93. What type of nerve function is studied using this


experimental design?
A. B. Efferent function Afferent function

C. D.

Sympathetic function Parasympathetic function

94. The sodium-potassium pump, an active transport


system, restores the proper balance of intracellular

and extracellular ions. When an organism is poisoned with cyanide, which halts ATP production, what happens to neural action potentials? A. B. Action potentials stop immediately. Action potentials will continue, even though
ATP production stops.

C.

Action potentials continue briefly but stop


eventually, due to a lack of ATP.

D.

Action potentials will generate ATP during


cyanide administration.

Copyright by The Berkeley Review

63

The Berkeley Review

Specializing in MCAT Preparation

Biology
Passage XV (Questions 95-100)

Skeletal Muscle Groups

Passage XV

II.

Hamstring Group

Made up of three muscles. All originate from the


Muscles of the lower extremities aid in the movement

of bones such as the femur, tibia, and fibula. They help

maintain an erect skeletal system and keep the body in balance by acting as an antagonist to gravity. These muscles provide the pumping action that helps circulate bloodand lymphthrough the lowerextremities. A diagram of the major bones found in the lower
extremity is shown in Figure 1.
III.

ischium and run along the posterior side of the femur. Two of the muscles of the hamstring group insert onto the medial condyle of the tibia, while the third inserts onto the lateral condyle of the tibia and the head of the fibula. The hamstrings are innervated by branches of the sciatic nerve.

Superior
ilium

ft

Adductor Group Several muscles comprise this group. All of these muscles originate from the pubic bone region and extend to insert along the medial and posterior
inferior shaft of the femur. The adductors are

innervated mainly by the obturator nerve.

<^U Posterior

Anterior C^>
pubic bone
femur

medial

condyle

95. Judging by where they attach, which of the following muscle groups would play the strongest role in flexing the hip joint (i.e., pulling the knee upwards
towards the chest)?

A. B.
C.

Quadriceps. Hamstrings.
Adductors.

D.

Quadriceps and hamstrings together.

Figure 1

A list of the major muscle groups of the thigh region of the lower extremity is provided below:
Muscles of the Thigh I. Quadriceps Group Made of four muscles, three of which originate

96. The pumping action of muscles in the lower extremity is crucial in order for blood to properly flow back to the heart. Which of the following
statements must be true if blood is to be returned to

the heart effectively while a person is standing? from the upper, anterior portion of the femur and one that originates from the anterior inferior iliac spine of the ilium. They all descend and insert into the broad patellar tendon which in turn crosses the knee joint and inserts into the tibial tuberosity of the upper, anterior tibia. These muscles are innervated by branches of the femoral nerve.
A. B. C. D. The The The The veins involved must have inelastic walls. arteries involved must have elastic walls. veins involved must have valves. arteries involved must contract under

sympathetic control.

Copyright by The Berkeley Review

64

The Berkeley Review Specializing in MCAT Preparation

Biology

Skeletal Muscle Groups

Passage XV

97. If an individual contracts the hamstring muscles, the:


A. B. C. D. quadriceps must contract. quadriceps must relax. quadriceps are unaffected. obturator nerve is responsible.

98. Paralysis of the femoral nerve would MOST STRONGLY affect the ability:
A. B.
C. D.

to stand on tip-toes. to extend the hip posteriorly.


to kick a ball. of the adductor muscles to contract.

99. Often when a person suffers a herniated vertebral


disk in the lower back, the sciatic nerve is

compressed and damaged. Which of the following would most likely NOT be a symptom of such a
condition?

A.

Difficulty walking.

B. C.
D.

Difficulty moving the lower leg posteriorly. Pain radiating down the back of the leg.
Weakness in the adductor muscles.

100. Claudication is a medical condition in which arteries

which feed the muscles of the lower extremity become partially occluded. In these patients, moderate exercise might cause all of the following in
the affected muscles EXCEPT:

A. B. C. D.

ATP deprivation. lactic acid buildup. lower-than-normal CO2 levels. O2 deprivation.

Copyright by The Berkeley Review

65

The Berkeley Review

Specializing in MCAT Preparation

Biology
Passage 1(1 - 8)

Nerve & Muscle

Section I Answers

Types of Transport

B is correct, HC03e. Roughly 60% of the body by weight is due to water, and all of the water can be divided into two compartments separated by a cellular membrane. One compartment contains the intracellular fluid (ICF) while the other compartment contains the extracellular fluid (ECF). The larger of the two compartments is the ICF compartment, and itcontains about 2/j ofall the water in the body. The ECF compartment isitself divided into two types of fluids: interstitial fluid and blood plasma. Table 1in the passage lists the major ions of the ICF and ECF. The first thing to note is that the ICF has a higher concentration of K than the ECF. This allows us to eliminate
choice D.

The remaining three choices are not mentioned in the passage and require a little thought. Phosphate (P043e) is
required not only in cellular DNA and RNA, but it is also required by a vast array of proteins that participate in phosphorylation reactions within the cell. Simply consider phosphorylating ADP with Pj to make ATP. Therefore, we would expect the levels of phosphate to be higher in the ICF than in the ECF. and they are. Phosphate levels in
the ICF are about 20 mM, while in the ECF they are about 4 mM. Eliminate choice C. We can use the same

reasoning for the relative levels of proteins. All of our cells are carrying out metabolic reactions, and to do the majority of these reactions enzymes (proteins) must be employed. This means that there will be more protein inside
a cell than outside a cell. Eliminate choice A. Cells which are catabolizing molecules are producing COt gas (e.g.,

via the Krebs cycle). Carbon dioxide can combine with water to form carbonic acid, which can then dissociate to form the bicarbonate ion (HC03e). Since CO2 is a waste and it needs to be eliminated via the blood, the cell wants
to get rid of it. We find that the levels of bicarbonate are higher in the ECF (i.e., the blood plasma) than in the ICF.
Bicarbonate levels in the ICF are about 10 mM, while in the ECF they are about 24 mM. The correct choice is B.

D is correct, extracellular [Na]. Extracellular refers to what is happening outside the cell and intracellular refers to what is happening inside the cell. The Na/K-ATPase requires ATP in order to function. If the levels of ATP
inside the cell are low, the ATPase activity will decrease and eventually be inhibited. The remaining ATP left within the cell will be diverted to reactions that are more important to the cell's survival. We can eliminate choice A. A low intracellular concentration of Na will also lead to inhibition of the ATPase, because if there is no Na to pump to the extracellular space, then the enzyme cannot catalyze the transport of sodium out of the cell and potassium into the cell. We can eliminate choice B. The same reasoning applies to low extracellular concentrations

of K. This allows us to eliminate choice C. A low extracellular concentration of Na will not directly affect the
ATPase, because the ATPase is located within the cell. It is not found on the outside of the cell. The correct
choice is D.

B is correct, primary active transport. In the second paragraph of the passage, we learned that glucose can cross the

cell's membrane by facilitated diffusion. At the bottom of Figure 1 In the passage, we see glucose and Na being
transported across the cell's membrane in the same direction. The protein carrier is acting as a symport. If the carrier were a uniport, only one molecule would be crossing the membrane. An antiport allows two molecules to cross the membrane, but each is crossing opposite to the other. Glucose is entering the cell along with sodium through a symport mechanism. The sodium was actively transported out of the cell. Sodium can re-enter the cell down its concentration gradient, allowing glucose to be transported with it by a secondary active transport process. A primary active transport system uses the energy of ATP directly to move a substance across a cell's membrane. We do not observe this happening in the case of glucose, especially since we read in the third paragraph that only

three primary active transport systems have been identified: one for Na and K ions, one for Ca- ions, and one
for H ions. The correct choice is B.

A is correct, low intracellular |Na) allows sodium to be released from the carrier protein. A high extracellular Na concentration is what allows glucose to bind to the symport on the extracellularside of the cell's membrane and

be transported into the cytosol. We can eliminate choice Band D. Ifthe affinity of the carrier protein for Na were
high on the cytosolic side of the membrane, then sodium and glucose could not be released into the cytosol. When Na is released into the cytosol from the carrier protein, the carrier protein's affinity for glucose is reduced. We can
now eliminate choice C. The correct choice is A.

A is correct. Simple diffusion (SD) involves a substance moving from a high concentration to a low concentration. Diffusion of lipid-solublc substances can occur through a lipid bilayer, while diffusion of water-soluble substances
will occur through a transmembrane channel. The lamer the gradient, the more a substance will diffuse across the

Copyright by The Berkeley Review

66

The Berkeley Review Specializing in MCAT Preparation

Biology

Nerve & Muscle

Section I Answers

membrane. We would expect to see a straight line showing that as the gradient of a substance increases, the flux of

the substance across the membrane increases as well. Facilitated diffusion (FD) requires a carrier protein. These carrier proteins have a limited number of carrier sites for the substance that is to be transported across the
membrane. Therefore, at a higher concentration gradient all of these carrier sites will be filled. The carrier will then

be saturated (think of Michaelis-Menten kinetics). At this point, the rate of diffusion of the substance across the

membrane will no longer increase with an increasing concentration gradient. We would expect to see a plateau in
the curve as soon as all of the carrier sites become saturated. The correct choice is A.

B is correct, more Na to enter the cell than K to leave the cell. The resting membrane potential of a
neuromuscular cell is about -HO mV. If a positively charged Na ion enters the cell, it will not make the inside of

the cell more negative, but rather make it more positive. The resting membrane potential will be depolarized, not repolarized. Eliminate choice A. Table 1 in the passage tells us that the concentration of K is higher in the cell
than it is outside the cell. It also tells us that the concentration of Na is higher outside the cell than it is in the cell. Therefore, when the acetylcholine receptors open and allow for the flow of K and Na ions, we find that Na ions

enterthe cell, while K ions leave the cell down their concentration gradients. Thisallows us to eliminate choice C.
The question tells us that two molecules of acetylcholine will bind to the acetylcholine receptor. This is a chemically mediated and activated response. The channel in the acetylcholine receptor is not activated (opened) by
an electrical stimulus (i.e., a depolarization). If that were the case, acetylcholine would not need to bind to the
receptor. We can eliminate choice D.

By the process of elimination, we arrive at choice B. But how do we know that more Na will enter the cell than

K will leave the cell? The electrochemical gradient for Na is greater than the electrochemical gradient for K.
The electrochemical gradient that allows an ion to pass through a membrane is simply the difference between the membrane potential (Em) and the equilibrium potentials for the ion in question (Ejon). The resting membrane

potential for our cell is about -80 mV. The resting membrane potential for K is about -92 mV, while the resting membrane potential for Na is about +58 mV. Clearly, the electrochemical gradient for Na is greater than that for
K. The correct choice is B.

D is correct, a high-resistance electrical pathway. Gap junctions join the cytoplasm of one cell to the cytoplasm of a neighboring cell through intramembrane proteins. These proteins (connexons) contain 6 subunits that form a central pore that allows molecules with molecular weight up to 1500 to pass through. Molecules like ATP and ions like

Na can easily pass through these channels. This helps to establish a cytoplasmic continuity between the cells.
When an action potential reaches a gap junction, electrical coupling between the two cells occurs, and the action propagates to the next cell. This type of coupling is important in heart tissue, where entire collections of cells must contract in a coordinated fashion. Therefore, a gap junction must provide a low-resistance (not a high-resistance) electrical pathway that enables current to pass from one cell to the next. The correct choice is D.

C is correct, increased sodium concentration gradient. Inhibition of the electron transport chain means that the amount of ATP being synthesized is substantially reduced. If the amount of ATP is reduced, then the amount of energy transferred from ATP to the membrane ATPases is reduced. The primary active transport systems (those

involving the Na/K-ATPase, Ca2-ATPase, and H-ATPase) all show a decrease in their activity. In secondary
active transport, the energy that was stored in the Na concentration gradient across the cell's membrane (high extracellular [Na 1and low intracellular [Na]) is used to transport molecules like glucose and amino acids into the cell and calcium out of the cell. If the synthesis of ATP decreases, the Na concentration gradient across the
cell's membrane is not as great (i.e., it decreases, not increases), and the rate of secondary active transportdecreases.

Low levels of ATP also mean that the Ca--ATPase cannot adequately pump calcium out of the cell. Since calcium
is also transported out of the cell (against its concentration gradient) by a secondary active transport system that utilizes the sodium concentration gradient (which is now rather low), the levels of intracellular calcium begin to
increase. Calcium is able to diffuse back into the cell though calcium channels. The correct choice is C.

Passage II (9 - 13)
9.

Autonomic Nervous System

D is correct, I, III, and IV only. The sympathetic system is important in getting the body ready for a perceived stressful situation. This response is a general response, affecting all parts of the body almost simultaneously. This is essential to the survival of the animal. You wouldn't want to have to wait for each organ or muscle to determine that this is a stressful situation while a lion is running at you! But the parasympathetic system has the luxury of determining specifically for each structure it innervates whether the job is done during this time of stress and when
67

Copyright by The Berkeley Review

The Berkeley Review

Specializing in MCAT Preparation

Biology

Nerve & Muscle

Section I Answers

it's time to return to basal levels. The parasympathetic system can do this type of individual monitoring, partially because of the proximal location of its ganglia next to the structure in question. The correct choice is D. 10. D is correct I, II, and IV only. All three situations are interpreted by the body as demands to increase particular functions beyond their normal basal levels. In other words, they represent a stress to the body. Sweating on a warm day is a response to release heat and cool down the external body. On cold days shivering and piloerection are stimulated to increase heat in the body. Both of these reactions are instigated by the sympathetic system. Furthermore, running is a stressor on the body. The correct choice is D. D is correct, penile erection. Penile erection can be caused by direct stimulation involving penile mechanoreceptors, or it can be caused by central nervous system activity stemming from sights and smells or even thoughts and emotions. In either case, input reaches the neurons of the penis. Parasympathetic nervous system activity is increased while sympathetic nervous system activity is decreased. When parasympathetic activity increases and sympathetic activity decreases, vascular dilation of the arterioles occurs so that the tissues of the penis become engorged with blood. As erection continues, the veins leaving the penis become compressed and little blood is allowed to leave, thus maintaining the penis in the erect state. The correct choice is D.
B is correct, I, II, and III only. The adrenal gland sits on top of the kidney. Cortisol, a glucocorticoid produced by the adrenal cortex, controls various aspects of metabolism. An increase in plasma concentrations of Cortisol will lead to an increase in gluconeogenesis (i.e., the synthesis of glucose from precursor molecules like lactate), a decreased uptake of glucose by cells, an increase in protein catabolism, and an increase in triacylglycerol breakdown (i.e., the release of free fatty acids). Aldosterone, a mineralocorticoid produced by the adrenal cortex, controls electrolyte balance by stimulating sodium reabsorption and potassium secretion in the kidney. Sodium reabsorption also leads to water reabsoiption in the kidney. The correct choice is B. B is correct, increased heart rate. The vagus (10th cranial) nerve is a component of the parasympathetic nervous system. Think of the parasympathetic system as being passive. The iris muscle of the eye will contract, making the pupil smaller. The motility and secretions (e.g., HC1 from the parietal (oxyntic) cells) of the stomach will increase, especially after a meal. An increased heart rate and contractility is due to the sympathetic nervous system (think of thefight-or-flight response). The correct choice is B. Action Potentials

11.

12.

13.

Passage III (14-20)


14.

C is correct, efflux of K balances the influx of Na. A small depolarization in the membrane will allow some Na to enter the cytoplasm of the cell. At the end of the second paragraph in the passage we see that the relative permeability of the membrane to K is roughly 10 times greater than it is to Na. Since there is more K in the
ICF than Na (from Table 1), we see that more K will flow out of the cell than Na into the cell. Therefore, if

there is a small depolarization, then the influx of Na will be more than balanced by the efflux of K. If the efflux of K is greater than the influx of Na, the explosive nature of an action potential will not result. What this means is that the threshold potential for the generation of an action potential is that point where the influx of Na exactly matches the efflux of K. Remember, there is a higher concentration of Na in the ECF and a higher concentration
of K in the ICF. The nanosecond that there is more of an influx of Na than there is an efflux of K, an action
potential will be generated. The correct choice is C.

15.

B is correct, -65 mV. As stated in the fourth paragraph of the passage, the threshold is about 15 mV away from the Em. In Figure 2 of the passage, the Em is at -80 mV. The threshold is given by the dashed line labeled with the number 2. Therefore, the threshold potential must be -65 mV. The correct choice is B.

16.

C is correct, an increase in the opening of fast /; gates in neighboring K channels. As depolarization takes place, Na rapidly enters the cell (Table 2 in the passage). In order for Na to enter the cell, the fast acting /;/ gate must open to allow Na to rush into the cell down its concentration gradient. As Na enters the cell, it causes further
depolarization and allows more m gates to open. This is the explosive nature of the action potential. The absolute refractory period begins at about the time the depolarization of the membrane takes place. Another action potential cannot be generated at this time because of the opening of the /;/ gates from the first action potential. This allows us to eliminate choices A, B, and D. Depolarization of the membrane will lead to an eventual opening of the // gates. The problem is that the n gates are slow and not fast. The correct choice is C.

17.

A is correct, m gate open; h gate closed; n gate open. Table 2 in the passage says that during the repolarization

phase K is flowing out of the cell and down its concentration gradient. If K is flowing out of the cell, the n gate
Copyright by The Berkeley Review
68

The Berkeley Review Specializing in MCAT Preparation

Biology
eliminate choice D.

Nerve 8t Muscle

Section I Answers

must be open. Looking through our choice of answers, we see that the n gate is closed in choice D. Therefore, we

We know that during repolarization, we do not want Na entering the cell. If Na enters the cell as K leaves, we
will not be able to obtain the resting membrane potential of the cell as quickly as we would like. Which one of the

Na gates is closed, or are they both closed? The last paragraph of the passage tells us that during depolarization
the h gate begins to slowly close. Let's assume that the h gate has closed as repolarization begins. If this is the case,
then we can eliminate choice C.

However, all through depolarization Na is entering the cell, which means that the in gate is remaining open. By
the time the peak of the action potential has occurred, the /; gate has closed, the mgate is still open but beginning to
close, and the // gate is open. The correct choice is A.

18.

A is correct repolarize. An action potential will initially depolarize the resting membrane potential (Em) and cause

that potential to be less negative. As the Na gates close and the K gates open, the membrane potential is being
repolarized toward the Em of the cell. Because of the high permeability to K during repolarization, the membrane potential becomes even more negative than the Em. This is called hyperpolarization. Once the cell has completed hyperpolarized, it will return to the Em value of the cell. This is what is occurring during the last half of the relative refractory period. Whenever a membrane potential returns to its true Em, the process is referred to as a
repolarization. The correct choice is A.

19.

B is correct, Na and K ions are not responsible for the release of the neurotransmitter. The passage states that TTX blocks the Na channels, while TEA blocks the K channels. If this is the case, then the depolarization of the
nerve cell membrane will not occur. However, if the presynaptic ending is stimulated with a depolarizing signal, a postsynaptic potential is produced. The last paragraph of the passage says that a neurotransmitter is released from the presynaptic terminal, diffuses across the synaptic cleft, binds to the postsynaptic terminal, and produces a postsynaptic potential. Since we are using a stimulus (signal) of depolarization to cause the release of a

neurotransmitter, it must mean that Na and K are not responsible for the release of the neurotransmitter. If they
were needed, then the stimulus that we applied would have no effect on the generation of a postsynaptic potential.
We can eliminate choice A.

As mentioned above, TTX and TEA block Na and K ion channels, respectively. They do not allow
neurotransmitter release. If they did. we would not have needed the depolarizing stimulus. We can eliminate choice

C. Because the channels for Na and K are blocked, it does not (necessarily) mean that the concentrations of ICF K and ECF Na are going to increase. Remember, the membrane is permeable to both Na and K (see the
second paragraph in the passage). These ions will tend to diffuse down their concentration gradients and the

Na/K-ATPase pump will redistribute them again. Blocking the Na and K channels is analogous to not having
a stimulus. The nerve cell will be at its resting state. We can eliminate choice D. The correct choice is B.

20.

B is correct, glutamic acid decarboxylase. The a-carboxyl group of glutamic acid is being removed. When CCb is removed from a molecule, the reaction is called a decarboxylation and is carried out by a decarboxylase enzyme. If CCb were added to a molecule, it would be a carboxylation reaction, which would be carried out by a carboxylase enzyme. Since the CCb is being removed from glutamic acid, the enzyme that catalyzes this reaction is a glutamic acid decarboxylase. Note that once GABA is formed, the numbering of the carbon atoms changes. What was the aposition becomes the y-position. There is no carboxyl group at the y-position. Instead, there is an amino group. If we were to remove the amino group of GABA, the four carbon compound would be butyric acid. Hence the name,
y-aminobutyric acid. The correct choice is B.
Local Anesthetic

Passage IV (21 - 26)


21.

C is correct, small molecular size and high lipid solubility. The passage tells us that the base form of the local anesthetic, being lipid-soluble, can penetrate various tissue barriers. The blood brain barrier is certainly one of these tissues. The reason lipid soluble substances are able to penetrate the blood brain barrier is that the leaflets of the cell membrane, including those of the capillary endothelium, are composed of lipid molecules. Lipid soluble molecules can thus diffuse through the two-dimensional liquid and through the membrane. Furthermore, it is logical that the
smaller the molecule, the greater its ease in crossina a membrane. The correct choice is C.

Copyright by The Berkeley Review

69

The Berkeley Review Specializing in MCAT Preparation

Biology
22.

Nerve & Muscle

Section 1 Answers

A is correct, the higher the pKa of the local anesthetic, the lower the concentration of base in the tissue. The passage tells us that the concentration of base or cation in the solution depends on the pKa of the local anesthetic.
Remember that local anesthetics are weak bases. Look at the structure of lidocaine. When it is protonated, it carries

a positive charge. In other words, the cation is the acid form. Therefore, when we have a high pKa, we have a small Ka. This, of course, means the reaction does not go very far to the right. In other words, we have a large
concentration of acid and a small concentration of base in the tissue. The correct choice is A.

23.

C is correct, local anesthetics decrease threshold for electrical stimulation in nerve fibers. We are looking for a

statement that is least compatible with the way in which local anesthetics act. Remember that local anesthetics act

to produce a reversible, local nerve conduction block. Therefore, we are looking for a statement that contradicts this
goal. A decrease in the threshold value indicates it is now easier to initiate a nerve action potential. Creating conditions where action potentials are formed more easily does certainly contradict the goal of the local anesthetic.
The correct choice is C.

24.

A is correct, the onset of lidocaine is more rapid, because more exists in its base form at body pH. We know from the passage that the base form is lipid-soluble, and therefore it penetrates the tissue. One can logically conclude that
the local anesthetic that exists more in its base form (as a percentage) at tissue pH will have a quicker onset of action because more penetration of tissue will occur. That is why the pKas of these local anesthetics are important. The local with the lower pKa will have a higher percentage of molecules in the base form and thus have a more rapid onset. Lidocaine has a more rapid onset because more of it exists in its base form. The correct choice is A.

25.

D is correct, the amount of ionized form will be increased, and the local anesthetic will be trapped in the tissue. Again, the local anesthetic penetrates the tissue in its neutral, base form. If acidosis (lowering of the pH) occurred in tissue after penetration of the local anesthetic, this would drive the base form back into its acidic form. In other words, a lower percentage of the molecule would exist in the base form. The problem with this is that the acidic form is charged and thus cannot escape the tissue barrier. The trapping of the charged form of the local anesthetic can contribute to an increase in the degree of toxicity. The correct choice is D. C is correct, is biotransformed by reactions occurring in the liver. Lidocaine is injected (rather than taken orally) to avoid biotransformation reactions which occur in the liver. If the drug is taken orally, it is not destroyed by the harsh conditions of the stomach. For example, the drug is not a protein, so there is no concern over the protease content of the stomach. After absorption across the intestinal wall occurs (most will probably occur in the small intestine), the drug enters into the hepatic portal system and is transported to the liver. Remember that the liver contains enzymes (we do not need to concern ourselves with the exact reactions) which are involved in modification of substances. Such modification can render the local anesthetic inactive. Furthermore, we wish to avoid having a local anesthetic circulating in the blood stream. The correct choice is C. The Lens, the Ins, St Associated Muscles

26.

Passage V (27 - 33)


27.

D is correct, The circular muscle of the iris is relaxed, the radial muscle of the iris is contracted, and the ciliary muscle is relaxed. Although this answer seems complicated, we can figure it out. First, figure out what the iris is doing. In the dark, the pupil will be large to let in light. The radial muscle will contract, and the circular muscle will relax. This immediately eliminates choices A and C. We must now figure out what the ciliary muscle is doing to choose between choices B and D. The ciliary muscle, as we are told in the passage, is relaxed for vision beyond about 20 feet. Since the night sky extends beyond 20 feet, the ciliary muscle is relaxed. Eliminate choice B. The
correct choice is D.

28.

A is correct, the lens is less able to focus the light rays from near objects. Remember, the "default" shape of the lens in the relaxed state is flatter, for far vision. This really does not change with age. Choices C and D are
incorrect. Near vision requires a flexible lens that can plump up into a convex shape to allow focusing on near objects. A less flexible lens is less able to focus on near objects. Choice B is incorrect. The correct choice is A

29.

B is correct, sympathetic nerves are stimulated, and parasympathetic nerves are inhibited. What situation leads to large pupils? The radial muscles must contract under sympathetic stimulation. The circular muscles must be relaxed under parasympathetic inhibition. Choice A is backwards and thus incorrect. Two different stimuli are required, so choices C and D are incorrect. The correct choice is B. A is correct, retina. Rays converge and focus on the retina. The cornea is a clear, tough membrane covering the front of the eyeball. Choice B is incorrect. The optic nerve is responsible for transmitting visual information to the brain, but it does not focus the image. Choice C is incorrect. The vitreous body is the inert stuff inside the eyeball.
Choice D is incorrect. The correct choice is A.

30.

Copyright by The Berkeley Review

70

The Berkeley Review Specializing in MCAT Preparation

Biology
31.

Nerve & Muscle

Section 1 Answers

B is correct, II only. As the object moves closer, the lens must become more convex. This is accomplished by contraction of the ciliary muscle. Choice II is correct. The ligaments do not contract, they merely pull the lens taut by default. Choice III is incorrect. The pupil responds to changes in light, not in focusing. Choice I is incorrect.
Since choice III is false, choice D is incorrect as well. The correct choice is B.

32.

C is correct, pink. The condition of not having epithelial pigment is call albinism. A person or animal with albinism is called an albino. Most strains of laboratory rats are albinos. They have pink eyes, due to the color of the blood vessels visible in their iris. Also, you may have seen white bunnies with pink eyes. Pigment covers up the color of the blood vessels in people and animals who don't have albinism. Black eyes contain the most pigment.
Choice A is incorrect. Green eyes contain some pigment, as well. Choice D is incorrect. You may not know about the lab rats, but a person would not have white eyes due to the presence of blood vessels in the iris. Choice B is
incorrect. The correct choice is C.

33.

C is correct, by providing a non-uniform corrective lens. Since the person has an irregularly shaped lens or cornea, the corrective lenses used should be shaped to correct these problems and allow focusing of the image on the retina. This would require that the corrective lens have a compatible non-uniform shape. Convex lenses correct farsightedness. Choice A is incorrect. Concave lenses correct nearsightedness. Choice B is incorrect. A uniform lens would not help the person with astigmatism. Choice D is incorrect. The correct choice is C.

Passage VI (34 - 40)


34.

Resting Membrane Potential

D is correct, lipids making up the cell membrane are uncharged. Cell membranes are made up primarily of lipids. Lipids contain very few charged groups, cannot carry current, and have a high electrical resistance. Materials with high electrical resistance are known as insulators. The lipid layers of the plasma membrane are regions of high electrical resistance separating two water compartments of low resistance. The correct choice is D. D is correct, negative, with the excess charge representing a very small fraction of the total number of ions inside and outside the cell. This problem requires outside knowledge about the resting membrane potential of a typical neuron. The typical resting membrane potential lies somewhere between -40 mV to -75 mV. The minus sign indicates that the potential is negative with reference to the inside of the cell. In other words, there is an excess of negative charge inside the cell relative to the outside. This eliminates choices A and B. Now, do these excess charges represent a small or large fraction of the total ions in the cell? The excess charges are but a very small
fraction of the total number of ions inside and outside the cell. The correct choice is D.

35.

36.

B is correct. We are looking for a picture that represents the potassium equilibrium potential. The equilibrium potential will exist where the force due to the concentration gradient is equal to the electrical force gradient.
Potassium ions have a higher concentration inside the cell, eliminating choices C and D. To discriminate between choice A and choice B, we simply look at the magnitude of the arrows. Again, at the equilibrium potential, the two forces will be equal. Answer choice A represents the resting membrane potential. The correct choice is B.

37.

C is correct, sodium ions diffuse into the cell. One can arrive at this answer by thinking about the membrane

potential and the direction in which it is going. We are told the resting membrane potential lies above that for the potassium equilibrium potential. This means that some positive charge is coming back into the cell, making the potential higher. Of the possible answers, the only way this is going to happen (given the true ion concentrations) is
to have sodium coming into the cell. This is indeed the case. There is a permeability to sodium ions over the

plasma membrane, and this contribution causes a rise in the membrane potential. Chloride ions have a higher
concentration outside of the cell, so if they move down their gradient, they will decrease the membrane potential.
The correct choice is C.

38.

C is correct, oxidation of nutrients. The pump that transports sodium and potassium ions does breakdown ATP to

couple the energy of that reaction to the work it must perform in transporting the ions. However, the question alludes to the ultimate source of the energy. Therefore, we need to think about how we are getting the ATP that is used by this pump. This answer comes from our knowledge of metabolism and an understanding that food is oxidized in the course of glycolysis and the Krebs cycle. The electrons released are eventually transported down the
electron transport chain and are coupled to a proton gradient, which results in the formation of ATP. The correct
choice is C.

39.

C is correct, a steady state. The constancy of the resting membrane potential is best described as a steady-state potential. The answer really comes down to two choices. Is it an equilibrium, or is it a steady state? With an equilibrium, no energy input is needed to maintain the state. Is this the case? No. We use the energy of ATP to run

the Na/K ATPase pump that maintains the concentration gradients of the these ions. Since there is energy used,
this cannot be described as an equilibrium. When a state is constant, but energy is used to maintain that state, this is termed a steady-state system. The correct choice is C.
Copyright by The Berkeley Review
71

The Berkeley Review Specializing in MCAT Preparation

Biology
40.

Nerve St Muscle

Section I Answers

D is correct, the new resting membrane potential is not affected at all by the chloride ion. We are told that the plasma membrane of many cells are permeable to chloride ions and do not contain chloride-ion pumps. Therefore,
in these cells, the membrane potential set up by other ions will act on chloride ions. The inside negativity moves chloride out of the cell until a concentration gradient develops. Looking at Table 1. we see the concentration of chloride is larger extracellular!}'. This concentration gradient will force chloride ions into the cell. However, the diffusion force will be exactly equal to the electrical force pushing chloride ions out of the cell. The result is that the equilibrium potential for chloride ions is equal to the resting membrane potential, and the chloride ion makes no contribution to the magnitude of the membrane potential. The correct choice is D.

Passage VII (41 -46)


41.

Nicotine Replacement

D is correct, Ca-. Calcium ions are the link between depolarization of the presynaptic membrane and
neurotransmitter release. Depolarization of the terminal causes voltage-sensitive calcium channels in the membrane

to open, and calcium diffuses into the axon terminal from the extracellular fluid, including that in the synaptic cleft.
The increase in the calcium level in the terminal causes vesicles filled with neurotransmitter to fuse with the

presynaptic membrane and release their contents into the synaptic cleft. The correct choice is D.

42.

C is correct. In the question, we are given a graph of a binding assay for the muscarinic acetylcholine receptor. where the substrate is muscarine. The question tells us that scopolamine, a competitive inhibitor, is added to the solution and the assay is run again. A competitive inhibitor will compete for the same site on the receptor as muscarine. It will not remove or incapacitate any of the receptors (this would be a non-competitive inhibitor). Since the total number of receptors remain in place, the level of maximum binding will not change. This eliminates choices A and B. What will change is the apparent affinity of the receptor for muscarine, which can be represented by half-maximum binding. The apparent affinity of the receptor for muscarine will decrease, because the inhibitor is competing for the same site. Therefore, we will see an increase in the half-maximum binding site of muscarine, because it now takes a higher concentration of muscarine to reach the half-maximum binding site. Remember, the
tell-tale sign of a competitive inhibitor is that if we increase the concentration of either substrate, one of them should

be able to out-compete the other and reach maximum binding. The correct choice is C.

43.

C is correct, an excitatory post-synaptic potential is generated. We arc told from the question that the nicotinic acetylcholine receptor is essentially a sodium channel. Therefore, when acetylcholine binds to a nicotinic acetylcholine receptor, we are essentially activating a sodium channel. When a sodium channel opens, what happens? Since sodium always has an extracellular concentration higher than its intracellular concentration, sodium comes into the cell and causes a net movement of positive ions in the cell. At this point, we can eliminate choice D. This causes a slight depolarization and is known as a excitatory post-synaptic potential (EPSP). Eliminate choice B.

This is a graded potential, and it is not an all-or-nothing action potential. In order to generate an action potential, the depolarization must be beyond a threshold value set by that particular cell. Usually, several EPSPs added together will eventually generate an action potential at an area known as the axon hillock, where a high concentration of
sodium channels are found. The correct choice is C.

44.

C is correct, a net nicotinic and parasympathetic nervous system excitation. We want to offer the person who is
addicted to nicotine the "high." without the cardiovascular risks associated with nicotine. Therefore, we will want to

stimulate the nicotinic receptors in the central nervous system to achieve this sensation. The problem is that nicotinic receptors arc located in the pre-ganglionic synapse of both the sympathetic and parasympathetic nervous systems. We might think that since both are stimulated, they simply cancel each other out. However, we know that nicotine causes stress on the heart, so the nicotinic receptor in the sympathetic nervous system must be dominant. With this in mind, let us look at the two drugs added. The first is physostigmine (eserine). This acetylcholinesterase inhibitor will cause increased levels of acetylcholine in both nicotinic and muscarinic receptors. This will give us the nicotine "high" that we want. Yet wc do not want to stimulate all the muscarinic receptors, so we add scopolamine. This acts to block all muscarinic receptors. This is what we want. We get the familiar feeling caused by nicotine and have blocked all muscarinic receptors. We are not done, though. We are still left to deal with the cardiovascular stress. The physostigmine will increase acetylcholine (ACh) levels at the first synapse in the
sympathetic nervous system. But in the parasympathetic system, it increases ACh levels at both the nicotinic and the muscarinic receptors (remember that scopolamine does not work directly upon heart tissue). Therefore, we have two stimulations in the parasympathetic system while we only have one in the sympathetic system. Therefore, we are left with a net nicotinic stimulation with a parasympathetic excitation. The correct choice is C.

45.

A is correct, nicotine stimulation of the nicotinic acetylcholine receptor is dominant in the sympathetic nervous system. We would think that because the first synapse in both the sympathetic and parasympathetic are nicotinic that nicotine would stimulate both of them equally, and therefore there would be no stress on the heart. However, we

Copyright by The Berkeley Review

72

The Berkeley Review Specializing in MCAT Preparation

Biology

Nerve &

Muscle

Section 1 Answers

know this not to be the case, as nicotine places considerable stress on the cardiovascular system. Therefore, nicotine stimulation of the nicotinic acetylcholine receptor in the sympathetic nervous system must be dominant over the one in the parasympathetic nervous system. There is no evidence in the passage for the claim made in answer choice C.

Cardiovascular stress is not brought about by activity in the central nervous system, but by activity in the peripheral
nervous system. We can eliminate choice D. The correct choice is A. 46. D is correct, axons, moving a greater distance in the sympathetic nervous system. The question tells us we have a molecule that will be moving in a retrograde (backward) fashion from the synapses to the heart between both sympathetic and parasympathetic nervous systems. The molecule will thus be moving through axons (eliminating choices A and B), because the axons, not the dendrites will be closest to the organ. In other words, an electrical signal reaching the organ will travel through a nerve's axon to reach the synapse between the nerve and the organ. So the next question becomes one of distance. Remember that in the sympathetic nervous system, the preganglionic fiber is short while the postganglionic fiber is long. The molecule will be traveling through the postganglionic fiber to reach the first synapse (the first synapse is the junction between the pre- and postganglionic fibers). Therefore, the molecule will travel a longer distance in the sympathetic nervous system. The correct choice is D.

Passage VIII (47 - 52)


47.

Retinal Projections

D is correct, the axons are myelinated by oligodendrocytes. This question calls on our knowledge of the nervous system outside of what is stated in the passage. We are looking for the most likely explanation for the increase in the frequency of the action potential. Myelinated nerves have the ability to increase the frequency of action potential conduction. Therefore, we can narrow the options down to choices C or D. The question then becomes: Which cells are responsible for the myelinalion? In both cases, glial cells are responsible for laying down the myelin sheath. In the central nervous system, these cells are called oligodendrocytes, while in the PNS they are called Schwann cells. Since we are talking about nerves located in the CNS, the best answer becomes choice D. The
correct choice is D.

48.

D is correct, afferent pathway, containing nerve axons from both eyes. Again, this question requires us to draw on our knowledge of the nervous system, and in particular, the eye. One must remember that the information leading towards the brain is found in the afferent nerves. Information leading away from the CNS is found in the efferent nerves. Therefore, answer choices A and B can be eliminated. Now. the question becomes whether the optic tract contains information from one eye or two. The answer is two. Information from the nasal hemiretina (medial half of the retina) of the left eye cross the optic chiasm and enters the right optic tract. The right optic tract is also made up of nerve fibers originating from the temporal hemiretina of the right eye. In that way, the right side of the brain processes information from the left side of the visual world. Because the tract contains information from both eyes,
D is the correct answer. The correct choice is D.

49.

C is correct, this area is free of photoreceptors. The question is fairly straightforward. It asks for the best explanation of the "blind spot." The blind spot is the area on the retina where the optic nerve bundle leaves the eye. That is not the reason why the area is termed the blind spot. The region is blind because it has no photoreceptors. If there are no photoreceptors to be found, there can be no transduction of light into a visual image. The correct
choice is C.

50.

C is correct, acetylcholine. We are told from the passage that the neurons which make up the Edinger-Westphal nucleus are parasympathetic neurons. Therefore, this question is really testing one's knowledge of the neurotransmitter used by parasympathetic neurons. We cannot be expected to know from the question alone which neurotransmitter these neurons use. However, we are supposed to be aware that neurons that are parasympathetic
use the neurotransmitter acetylcholine. The correct choice is C.

51.

A is correct, the optic nerve of the left eye is intact, but the efferent limb of the left eye is damaged. The question tells us we see a consensual but not a direct response. We can therefore conclude that the optic nerve of the left eye is intact because the optic nerve of the right eye is not involved in the response. The response involves information going down the left optic nerve to the pretectal area. Prom the pretectal area, neurons project bilaterally to the Edinger-Westphal nucleus. Axons from neurons in the nucleus innervate the ciliary ganglion. We see a consensual response in the right eye. We can therefore conclude that the bilateral projection and the efferent pathway to the right eye arc unharmed. In addition, we can conclude that there is some problem with the efferent pathway to the left eye. We are not seeing a constriction of the pupil in response to the light being shone. We are left with the following conclusion: The left optic nerve is intact, but the efferent pathway of the left eye is somewhere and
somehow damaged. The correct choice is A.

52.

B is correct, right optic nerve. The light shone in the right eye elicits neither a direct or consensual response. This most likely means that there is damage to the right optic nerve. The result of the damage is that the information
73

Copyright by The Berkeley Review

The Berkeley Review Specializing in MCAT Preparation

Biology

Nerve 8t Muscle

Section I Answers

leading away from the eye is unable to elicit a reflex which results in the constriction of both eyes. This can be the only explanation for the information given in the question, because light shone in the other eye elicits both a direct and a consensual response. The correct choice is B.

Passage IX (53 - 58)


53.

Axonal Transport

A is correct, anterogressively, via fast axonal transport. Recall from the passage that transport from the soma
towards the axon terminal is known as anterograde axonal transport. Therefore, this eliminates choices C and D. Next, is the neurotransmitter moving via fast or slow axonal transport? Recall that neuropeptides are packaged into membrane bound vesicles as they are formed near the soma. From the passage, we know that membrane-bound organelles are transported rapidly by fast axonal transport. While the vesicle may not be technically considered an organelle, given the choice between membrane-bound organelle and cytoplasm, a vesicle is best aligned with the membrane-bound organelle system. Our best choice includes fast axonal transport. The correct choice is A.

54.

C is correct, Ca-. We are told from the passage that organelles interact with microtubules through a linkage
similar to that between thick and thin filaments of skeletal muscle cells. Recall that thick filaments refer to myosin, while thin filaments refer to actin. Recall that calcium is needed for actin and myosin interaction, as calcium removes the troponin-tropomyosin complex which covers the actin binding sites. Since the interaction of the organelles with the microtubules is similar to that of actin/myosin, one can best conclude that calcium triggers the
interaction. The correct choice is C.

55.

C is correct, complete translation of the peptide in the cytoplasm. Recall that peptides that are produced by a cell for the purpose of secretion (neuropeptides are certainly an example) are processed through the rough endoplasmic reticulum. The peptide being translated will have a signal peptide that attracts a cytoplasmic signal receptor particle
(SRP). When the SRP attaches to the signal peptide, a pause in translation will occur. The translation does not begin again until the SRP complex (SRP/signal peptide/ribosome) binds to an SRP receptor located on the rough

ER. When the SRP complex binds to the SRP receptor, translation begins along with transport of the new protein into the lumen of the RER. At that point, the signal peptide is cleaved off and the proteins are modified in both the RER and the lumen of the Golgi apparatus. From the trans face of the Golgi, the peptide buds off as a vesicle. According to this hypothesis, there is no complete translation of the neuropeptide in the cytoplasm. The correct
choice is C.

56.

C is correct, pyruvate. Glucose will go through the glycolytic pathway and the end product of glycolysis is pyruvate. Recall that glycolysis occurs in the cytosol of cells. This is a very straightforward question asking us to
remember that pyruvate is transported over both the outer and inner mitochondrial membranes to enter the matrix.
The correct choice is C.

57.

D is correct, soma to trace the synaptic endings of neurons. The marker substance moves anterogressively. According to the passage, the marker substance will move from the soma toward the synaptic ending. If we are interested in tracing a neural pathway, we should use this marker to identify synaptic endings. In other words, we should inject this marker in a given cell body, and it will trace the axon and the synaptic endings of the neuron. If we inject the marker into the soma, it will not trace the dendrites because of its direction of movement. Therefore, we can eliminate choice C. Furthermore, we do not want to inject the marker by the synapse, because there is nothing left to trace. The correct choice is D.

58.

D is correct. We are told from the graph in the question that action potentials arise in a postsynaptic cell after the addition of acetylcholine. We then add acetylcholinesterase, which breaks down ACh. We must look at the change in the pattern of the action potential response. Why? Recall that action potentials are all-or-nothing events, so they will not change in amplitude. We can easily eliminate two of the choices. We then look at the pattern, and in this case, the frequency of response. If we are going to have lower levels of ACh due to the addition of acetylcholinesterase, we should see a less frequent train of action potentials in the postsynaptic cell. The correct
choice is D.

Passage X (59 - 64)


59.

Huntington's Disease

C is correct, equally common in both men and women. The passage tells you the defect is an autosomal dominant one. This means it is not sex-linked, so eliminate choices A and B as incorrect. We also learn from the passage that the age of onset is from 35 years of age. Eliminate choice D. The correct choice is C. D is correct, voluntary movements. The hypothalamus produces releasing hormones that affect the pituitary. Choice A is incorrect. The control of speech is localized in the cerebral cortex. Choice B is incorrect. The

60.

Copyright by The Berkeley Review

74

The Berkeley Review Specializing in MCAT Preparation

Biology

Nerve &

Muscle

Section I Answers

parasympathetic nervous system (PNS) is not controlled by the basal ganglia. In the second paragraph, we learn that symptoms of the disease include rapid, uncontrolled, jerky movements. The correct choice is D. 61. D is correct, 4-aminobutanoic acid. This molecule is a 4-carbon organic acid. The skeletal structure is called butanoic acid. The carbon of the carboxyl group is designated 1. This means the amino group is attached to the 4th carbon. This gives the name 4-aminobutanoic acid. The correct choice is D. D is correct, glutamate. This information is given in the second paragraph of the passage. A key missing enzyme that leads to GABA deficiency is glutamate decarboxylase. If we correctly named the previous molecule, GABA, then we can imagine adding on a carboxyl group on the C-4 carbon to make glutamate. Choice A is incorrect: histidine is the precursor of histamine. Choice B is incorrect. Choice C is incorrect: tyrosine is the precursor of epinephrine and norepinephrine. The correct choice is D. C is correct, 50%. Since the gene is autosomal dominant and the father is not affected, he is homozygous recessive for this disease. The gene is rare, and only one of the woman's parents shows a familial history for the disease. The woman is probably heterozygous. Therefore, she either passed on the dominant, disease-causing gene or the recessive gene. The chance is 50/50. since she has 2 X-chromosomes and only one is affected. The correct choice
isC.

62.

63.

64.

B is correct, the vesicles of presynaptic neurons. Neurotransmitters are synthesized in presynaptic neurons, stored in vesicles there, and released by the arrival of a neural impulse. The released neurotransmitter crosses the synaptic cleft and interacts with the receptors on the postsynaptic neuron's membrane. The correct choice is B.

Passage XI (65 - 71)


65.

Photoreceptors

A is correct, cone cells. This answer can be arrived at using previous knowledge, and not information specifically

from the passage. The fact is that cones are used for color vision, while rods are used for black-and-white vision. The question asks about seeing in the dark, which definitely qualifies for a lack of color. Of course, one may see
some color in the dark and the cones would be responsible, but the question asks which cell is most likely not involved. Consider the other answers. We have already discussed that rods play a role in black-and-white vision,

making their role in dark vision likely. The other cells are involved in taking information from the photoreceptors to the brain. It is stated in the passage that the photoreceptors synapse with the bipolar cells, which then synapse with the ganglion cells. Therefore, for both color and black/white vision, these cell types will be involved in the
transmission of visual information. The correct choice is A.

66.

C is corrrect, hyperpolarization of the photoreceptor. It is stated in the passage that upon stimulation of a photoreceptor cell by a photon, the sodium channels close. This implies that when there is no light stimulation, the sodium channels are open. When sodium channels are open, the sodium ion comes in (following its gradient) and

causes the cell to be depolarized. In an unstimualted cell, a very active Na/K pump is constantly restoring the
sodium gradient. Again, the net result of this is that a photoreceptor cell in the dark is normally depolarized. When a photon hits, the sodium channels close and the pump continues to work. Both of the these events cause a hyperpolarization of the photoreceptor cell. This is a very special case for photoreceptors, and it is important to be aware of this phenomonon. Based on this information, the other answers can easily be eliminated. The correct
choice is C.

67.

A is correct, fovea centralis. The area where there is the highest density of cones on the retina will give the greatest

acuity, or precision. At this point, one must recall the anatomy of the eye. The area of highest cone density is the
fovea centralis, and when we look at objects, we move our eyes to focus light onto this region of the retina. This answer could not be obtained from the passage, but we thought it would be nice to review some eye anatomy. Look at the other answers. The cornea is the transparent structure located at the front of the eye and is the structure through which light rays enter. The optic disc is where the optic nerve leaves the eye and the retinal blood vessels enter. Since there are no photoreceptors overlying this disk, it is known as the blind spot. The choroid is a pigmented layer behind the retina, which contains many of the blood vessels that nourish structures in the eye. It also absorbs light not taken up by photoreceptors. It absorbs rather than reflects, so as not to alter the photoreceptor responses to
other light. The correct choice is A.

68.

D is correct, occipital. We need to recall our knowledge of the cortices of the brain. The experiment removed the thalamus. The thalamus, in particular the lateral geniculate nucleus, is a relay station for visual information. From this nucleus, information then goes on to the visual cortex, located in the occipital lobe of the brain. The occipital lobe is located above the cerebellum. Based on this, the only possible answer choice is D. As a very brief review, one can associate the frontal lobe with association processes, the temporal lobe with hearing, and the parietal lobe with high level sensory and motor control. The correct choice is D.
75

Copyright by The Berkeley Review

The Berkeley Review Specializing in MCAT Preparation

Biology
69.

Nerve St Muscle

Section I Answers

70.

D is correct, ganglion cells. The answer can be obtained by either previous knowledge, or by reading the passage carefully. It is stated that both photoreceptorsare located at the back of the retina. It clearly states that the four other types of cells (bipolar, etc.) are part of the retina. Since the photoreceptors synapse with the bipolar cells, and the bipolar cells synapse with the ganglia, one could create a picture in their head where the ganglion cells are furthest away from the photoreceptor cells, or the first layer in the retina. Based on this thought process, light entering the eye will first pass over the ganglion cells. The correct choice is D. C is correct, III only. The place where the axons of the ganglion cells leave as the optic nerve is called the optic disc. There are no photoreceptors at this region of the retina, because the axons are leaving and blood vessels are entering. If there are no photoreceptors, then vision is not possible. This is of course one's "blind spot." Consider the other possibilities. The region of highest acuity is where one finds the highest density of cones. This is certainly not
the case. In the case of stereovision, it is not the structure of one eye that allows for depth perception, but the fact that we have two eyes looking at one object from different angles. This gives us stereovision. Therefore, statement II does not apply. Statement III is the only applicable statement. The correct choice is C.

71.

C is correct, left eye and the temporal hemiretina of the right eye. If light originates in the left visual hemifield, then we known that our right side of the brain will be processing this information. In order to get to the right side of the brain, we need to have light strike the temporal hemiretina of the right eye. If you follow the optic nerve, you will see it goes to the right side of the brain. In addition, light from the left side will strike the nasal hemiretina of the left eye. The optic nerve from this part of the retina will cross over at the optic chiasm and move on to the right side of the brain. Draw some lines of light from the left visual hemifield; it may augment your understanding of how light originating from one side of the visual field becomes processed in the opposite side of your brain. The correct
choice is C.

Passage XII (72 - 79)


72.

Sound Transmission in the Ear

D is correct, one million times greater than the reference pressure. The question asks for the relationship between the loudest sound pressure level and the reference pressure. We know from the question that the upper available dB

range is 120. Inorder to arrive at this figure, we must have a test pressure that is 106 times as large as the reference
pressure, because 20 log 10" = 120. The correct choice is D.

73.

D is correct, force/unit area acting on the oval window is increased. We know the sound wave is conducted through the set of small bones in the middle ear. Since the passage does not mention any significant loss of force during this
transmission, we can assume the force associated with the wave does not increase or decrease. Well, how about the

pressure? Pressure is the force/unit area. The force does not change. However, the area becomes smaller. Therefore, the total force/unit area acting on the oval window is increased. The correct choice is D.

74.

B is correct, 1x 10"3 s. The question isasking us to carry out some calculations. First, letus convert the 3.4 x 10"4
kilometers into meters. This gives us 0.34 m. Based on this value and the speed of sound, we get a frequency of 1000 Hz. Recall that the period of a wave is the inverse of its frequency. Therefore, the period of this wave is

1/1000 Hz, or 1 x 10"3 seconds. The correct choiceis B.


75. C is correct, oscillating depolarizing-hyperpolarizing. The passage tells us that oscillating movements of the organ of Corti both excite and inhibit sensory transducing cells, because the hair cells attached to these cells are moving back and forth. In one direction, these hair cells cause cation channels to open. This will cause a depolarization.
However, in the other direction, the hair cells cause a closure of these channels.
choice is C.

This is associated with a

hyperpolarization. Therefore, the cell undergoes an oscillating depolarizing-hyperpolarizing potential. The correct 76. A is correct, region 1. The question is asking which region of the basilar membrane would be associated with highfrequency sounds. High-frequency sounds are associated with short wavelengths and will be more energetic than lower-frequency sound waves. We can assume that the broad flexible region of the basilar membrane will not be able to resonate with these short energetic movements. In other words, the broad flexible region will not be able to receive high frequency sound as well. Such movements are achieved by the short, stiff region of the membrane.
The correct choice is A.

77.

D is correct, cell 5. Let us look at the circuit model. We know that information is arriving at the left ear at a time before that of the right ear. We also know that these cells fire maximally when the inputs from both sides arrive simultaneously. For example, if sound arrived at both ears at the same time, cell 3 would fire maximally, because input would arrive at that neuron simultaneously. Based on that logic, if information is arriving at the left ear before the right, it will travel up the entire circuit before the information from the right ear will arrive. If the two inputs arrive at any cell at the same time, that cell must be located in the upper region of the model. Therefore, cell 5 is the
best answer. The correct choice is D.

Copyright by The Berkeley Review

76

The Berkeley Review Specializing in MCAT Preparation

Biology
78.

Nerve St Muscle

Section I Answers

B is correct, middle ear. The passage tells us that conducting sound through the temporal bone is one way to bypass the middle ear. The passage also tells us that this means of transmitting sound is quite inefficient. Therefore, when the fork is placed on the patient's temporal bone and the patient hears the sound (after they no longer heard the sound with the fork at their ear), we can narrow down the problem to conduction of air in the middle ear. The
correct choice is B.

79.

C is correct, graph C. The Y-axis is amplitude and the X-axis is frequency. If cells have a characteristic frequency, the data should reach only one point on the X-axis. Some graphs show two. Choices A and D can be eliminated. Now that we know that only one characteristic frequency point should be reached, which direction will the curve go? At a cell's best frequency, does the loudness of a sound need to be high or low to elicit a response? It is low. The amplitude of the sound does not need to be high, if we are at the cell's characteristic frequency. However, if we move away from that frequency in either direction, the amplitude of the sound needed to elicit a response increases. Based on this information, graph C is best. The correct choice is C.

Passage XIII (80 - 87)


80.

Tryptophan St Serotonin Experiment

B is correct. Choice A can be eliminated, because a carboxyl group is not being added to serotonin. Choice C can be eliminated, because amidation is the process of forming an amide (CONH). There are no amides (think of a peptide bond) associated with serotonin. Choice D can be eliminated, because amination involves the addition of an
amino (NH2) group to a molecule.
COOH

Tryptophan

5-1lydroxy tryptophan

Serotonin

Comparing the two molecules shown in the question, we see that a hydroxyl (OH) group has been added to the C-5 carbon of the indole ring system, and a carboxyl (COOH) group has been removed from the side chain. As shown in the reaction sequence above, a hydroxylase enzyme adds a hydroxyl group to make the molecule 5-

hydroxytryptophan. A decarboxylase enzyme next removes the carboxyl group to give serotonin, making choice B
the best answer. The correct choice is B.
81.

C is correct. We are looking for the false answer. Choices A and D appear to be the easiest choices to eliminate first. Serum large, neutral amino acid (LNAA) concentration increases in both trials as dietary protein content increases. This can be seen by reading the values in the Serum LNAA columns in Table 1 and Table 2. Therefore,
choice A is a true statement and can be eliminated as the best answer. Serum tryptophan levels also show an

increasing trend as dietary protein content increases. This can be seen by reading the values in the Scrum Trp
columns in Table 1 and Table 2. Therefore, choice D is a true statement and can be eliminated as the best answer. In

choice B, a 24% protein meal is given to the rats two hours after they were fed an all-carbohydrate meal. This means we are just considering the information in Tabic 1, because that reflects Trial 1. To attenuate means to reduce, weaken, or diminish. The 24% protein meal in Trial 1 docs attenuate the increase in serotonin due to tryptophan. We know from the first paragraph that serotonin is synthesized from tryptophan. If the diet is higher in protein (read down the Group column in Table 1), there will be more competition from LNAAs at the transporters in the bloodbrain barrier. If there is more competition at the transporters, there should be less tryptophan crossing the bloodbrain barrier. If there is less tryptophan near the neurons (see the column Cortex Trp in Table 1), there will be less synthesis of serotonin. In other words, the levels of serotonin are attenuated. Therefore, choice B is true statement and can be eliminated as the best answer. Choice C is false, because there is no indication in the passage that higher levels of serum LNAAs lead to an increase in the synthesis of serotonin. In fact, based on the last two columns of each table, it appears that an increase in LNAAs actually leads to a decrease in serotonin synthesis. The correct
choice is C.
82.

D is correct. The question is asking us to draw a conclusion from the data in Table 1 and Table 2. One aspect of
both tables that stands out is that the serum LNAAs increase with higher protein diets. If the LNAAs other than tryptophan (e.g., leucine, isoleucine, tyrosine, and phenylalanine) were competing with tryptophan for the LNAA receptors associated with the blood-brain barrier, then the tryptophan levels in the brain should decrease as serum

protein levels increase. This is especially true for the higher protein diets in both tables. In Table 1, the levels of 24% protein and 40% protein cause this competition. In Table 2, the levels of 12% protein, 24% protein, and 40%
Copyright by The Berkeley Review
77

The Berkeley Review Specializing in MCAT Preparation

Biology

Nerve St Muscle

Section I Answers

protein cause thiscompetition. In bothtrials, thelevel of serum tryptophan increased for the group of laboratory rats that fasted overnight. Therefore, because statements I, II, and III are correct, we can eliminate all of the answer choices except choice D. The correct choice is D.
83. B is correct. In the first paragraph we read that serotonin, a neurotransmitter, is synthesized from tryptophan. If we assume that one neurotransmitter is not synthesized from an existing neurotransmitter, we could conclude that tryptophan is not a neurotransmitter. Also, paragraph two states that all of the LNAAs, including tryptophan, are amino acids. Statement I is eliminated, which means that choices A, C, and D are all eliminated. Support for the elimination of choice D comes from paragraph two where we read that tryptophan is a neutral amino acid, not an acidic amino acid. By the process of elimination, this leaves us with choice B as the best answer. Essential amino acids are amino acids that cannot be synthesized de novo by human beings. Even though tryptophan is indeed an essential amino acid, there is no indication in the passage as to whether it is an essential amino acid. Tryptophan is found in most protein-based foods, like red meat, poultry, fish, milk, and eggs. It is also found in chocolate. The
correct choice is B.

84.

A is correct. If one of the enzymes in a pathway (see below) is blocked by an inhibitor, then some intermediate metabolite is not processed through to the final product. This means the intermediate form may remain and
accumulate in the tissue.

Tryptophan

Hydroxylase

5-Hydroxytryptophan

Decarboxylase

Serotonin

If an inhibitor of the tryptophan hydroxylase enzyme were added to rat brain tissue, the intermediate (i.e., 5hydroxytryptophan) between tryptophan and serotonin accumulates. The factor to be examined in order to estimate the new rate of serotonin synthesis is not the concentration of the inhibitor itself, either in the blood or the brain, just whether it is working or not, so choices C and D are incorrect. Exclude choice B, also: The tryptophan concentration in the blood would not provide any information about the rate of the conversion of tryptophan to serotonin in the brain. Remember, the LNAAs, including tryptophan, have to cross the blood-brain barrier to reach the brain. The
correct choice is A.

85.

B is correct. The small intestine, which includes the duodenum, jejunum, and ileum, is where the majority of digestion and absorption of food takes place. Whether tryptophan is obtained from the food we eat or in the form of a dietary supplement, makes no difference in terms of its metabolic effect. Either way, it is easily absorbed in the small intestine by a carrier-mediated process, so eliminate choice A. Some common neurotransmitters include aspartate, glutamate, gamma-aminobutyric acid, acetylcholine, dopamine, norepinephrine, and serotonin, to name but a few. Even though tryptophan leads to the synthesis of the neurotransmitter serotonin, it does not lead to the synthesis of the other neurotransmitters just mentioned. For example, acetylcholine is an ester of acetic acid and choline. It does not involve the indole ring structure like that found in tryptophan and serotonin, so choice C is
eliminated.

We are now left with choices B and D. To determine whether choice D is the best answer, we would need to bring in some outside information. About 10% of the human body's serotonin concentration is located in the central nervous system (CNS); the reminder can be found in the gastrointestinal system. As a neurotransmitter, serotonin is believed to be involved with a wide variety of body functions. It is thought to be involved with some social behaviors, as well as memory, learning, mood, sleep, appetite, sexual desire, and temperature regulation. A deficiency in serotonin is thought to be involved in depression, whereas an increase in serotonin above normal levels can lead to anxiety, insomnia, and gastrointestinal disturbances. An excess of serotonin, usually brought about by a drug-induced augmentation of the neurotransmitter, includes an elevated body temperature and sweating, nausea, vomiting, a rapid heart beat, blood pressure fluctuations, loss of coordination, and hallucinations. This hyperserotonergic state is referred to as serotonin syndrome and is a potentially life-threatening condition. These conditions do not mimic one of alertness, so we can rule out choice D as the best answer. A careful reading of the passage does not help us learn whether dietary tryptophan, when converted to serotonin, causes more alert behavior. However, based on the information in paragraph two about the uptake of LNAAs, we could conclude that if a large dietary supplement of tryptophan alone were ingested, without the presence of any other LNAAs, then tryptophan would be the primary amino acid being transported across the blood-brain barrier. In this case, the brain concentrations of serotonin would increase, making choice B the best answer. The correct choice is B.
86. D is correct. The nine essential amino acids for adult human beings are Valine, Methionine, Histidine, Leucine, Phenylalanine, Threonine, Isoleucine, Lysine, and Tryptophan. A helpful mnemonic is: Very Many Happy Little Pigs Take Iced Lemon Tea. The degradation and catabolism of amino acids occurs as a result of protein turnover. We can assume from the question that the metabolism of phenylalanine leads to the production of one of the amino

Copyright by The Berkeley Review

78

The Berkeley Review Specializing in MCAT Preparation

Biology

Nerve & Muscle

Section I Answers

acids listed in the answer choices, because if there were a deficiency in phenylalanine in the diet, then one of those amino acids would need to be supplemented in the diet. The easiest way to arrive at the best answer is by structural analogy between the amino acids. Phenylalanine has a phenyl group (i.e., a ring structure) attached to an alanine

component. The only amino acid among the answerchoices that has a ring structure is tyrosine. The correct choice
isD.

87.

A is correct. There are four types of cellular junctions in vertebrates: tight junctions, gap junctions, desmosomes, and adherens. Tight junctions help to prevent the passage of molecules and ions between cells. Gap junctions connect the cytoplasm of two cells, and allows the passage of selected molecules and ions between those cells. Desmosomes are specialized structures that allow for cell-to-cell adhesion on the lateral sides of plasma membranes. They help resist the shearing forces associated with cells. Adherens appear as actin bands that can encircle a cell, helping to create a strong attachment between cells. Since these junctions are all associated with a variety of organ systems in the body, we need to know a little more about the blood-brain-barrier in order to answer the question. The primary function of the blood-brain barrier is to separate circulating blood from the cerebrospinal fluid of the central nervous system. This barrier to the central nervous system is due to the selectivity of the tight junctions found between the endothelial cells that make up the capillaries supplying nutrients to the brain. The correct
choice is A.

Passage XIV (B8 - 994)


88.

Frog Muscle Expenment

C is correct, a greater frequency of the action potentials indicates greater intensity. Action potentials act in an "allor-none" fashion. There is no amplitude modulation. Choices A and B are incorrect. Identical action potentials are produced more frequently to indicate a stronger stimulus, such as the stretch caused by a set of increasing weights.
The correct choice is C.

89.

A is correct, the action potentials had increased to a maximum with the 50-g weight and increased no further, due to

the refractory period of the neuron. The action potentials were limited by the nonconducting refractory period of the neuron. Increasing the weight could not speed up the "recovery time." The nerve was not damaged by the weight.
Choice B is incorrect. Do not assume equipment failure when a biological mechanism is being studied. Choice C is incorrect. A neurotransmitter defect would probably mean no action potentials rather than slowed ones, exactly
identical to another weight. Choice D is incorrect. The correct choice is A.

90.

A is correct, The weight was not heavy enough to exceed the action-potential threshold. 5 mg (1/200) of the 1 gram weight caused only a few action potentials. Since there was not an action potential generated, this weight must have been too small to overcome the threshold for generating action potentials. The weight was not too heavy, so choice B is incorrect. The weight does not have to produce an action potential, if its stimulus is lower than the threshold. Choice C is incorrect. All the action potentials were identical in size, so frequency was modulated, not the amplitude. Choice D is incorrect. The correct choice is A.

91.

D is correct, plasma membrane. The cytoplasm of a neuron is a poor conductor. Choice A is incorrect. The
Schwann cell acts to insulate nerve fibers and to provide gaps between the conducting regions. Choice B is incorrect. The endoplasmic reticulum does not function as a conductor. Choice C is incorrect. The plasma membrane is the site of conduction of action potentials. The correct choice is D. B is correct, sodium moves from the extracellular fluid into the cell, and then potassium moves from the intracellular fluid to the outside of the cell. This is an easy question, but all the words make it easy to make a mistake. First, eliminate wrong answers based on ion location. Sodium is the principle extracellular ion, while

92.

potassium is the principle intracellular ion. Choices C and D are incorrect based on this fact. Then eliminate
choices based on order of action. The sodium channels open first, followed by the potassium channels. Choice A is
incorrect. The correct choice is B.

93.

B is correct, afferent function. Afferent pathways carry information from the tissues to the nervous system. Information of muscle stretch was being conveyed back toward the (no-longer-present) nervous system. Efferent
function would be tested, if a nerve were stimulated with electrodes to make a muscle twitch. Choice B is incorrect.

Since the nerve and muscle under study are isolated from the intact frog, there is no parasympathetic or sympathetic
function. The correct choice is B.

94.

C is correct, action potentials continue briefly but stop eventually, due to a lack of ATP. The Na/K pump
requires ATP to perform active transport. The cell has a small reserve of ATP that would provide brief function of the pump if ATP production stopped. Choice A is incorrect. Action potentials stop when all the ATP is gone. Choice B is incorrect. Action potentials require ATP, they do not generate it. Choice D is incorrect. The correct
choice is C.

Copyright by The Berkeley Review

79

The Berkeley Review Specializing in MCAT Preparation

Biology
Passage XV (95 -100)
95.

Nerve St Muscle

Section I Answers

Skeletal Muscle Groups

A is correct, quadriceps. According to the passage, the quadriceps group has at least one muscle that crosses the hip joint anteriorly. It attaches above the hip joint at the anterior inferior iliac spine and below the hip joint at the tibial tuberosity. Thus, as the muscle shortens it will aid in flexion of the hip by pulling the femur and tibia upwards towards the chest. Refer to the skeleton diagram and try to visualize exactly how this works. The other answer
choices can be eliminated after consideration. The hamstrings cross the hip joint behind the femur. When they shorten, they would pull the femur backwards. This is hip extension, not flexion; eliminate choice B. Choice D can also be ruled out on the basis of this information. The adductors mainly insert along the medial and posterior shaft of the femur. When they contract, they pull the thigh inwards. This is adduction; eliminate choice C. The correct
choice is A.

96.

C is correct, the veins involved must have valves. How can blood be returned from the lower extremity against gravity and under the low pressure that is normally found in the venous system? Muscles of the thigh and lower leg "massage" the elastic veins, pumping blood in one direction: upwards. Blood can only go in this direction because veins in the lower extremity have one-way valves which prevent backflow of blood away from the heart. These are very important in maintaining proper circulation. Varicose veins result from defects in these valves. Let's consider
the other answer choices. Eliminate choices B and D because arteries do not return blood to the heart, veins do.

Eliminate choice A because if veins had inelastic walls, they couldn't be squeezed by muscles and the process would
not work. The correct choice is C.

97.

B is correct, the quadriceps must relax. The quadriceps and the hamstrings are opposing muscle groups with opposite actions. The hamstrings extend the hip posteriorly while the quadriceps flex the hip anteriorly. In order for one of these antagonistic muscle groups to work, the other must relax. If both contract simultaneously, nothing would get accomplished because the two actions would cancel each other out. Knowing these should lead us to eliminate choices B and C. Choice D can be ruled out because the obturator nerve, according to the passage,
innervates the adductor muscles, which aren't involved here. The correct choice is B.

98.

C is correct, to kick a ball. The passage states that the femoral nerve innervates the quadriceps muscles. The quadriceps are responsible for flexing the hip anteriorly (see prior question) and also for extending the knee. In other words, since the quadriceps crosses the knee joint, contraction pulls the lower leg and foot anteriorly (a kicking action, known as extension of the knee joint). Paralysis of the femoral nerve might therefore affect the ability to kick a ball, among other things. Let's consider the other answer choices. Choice A can be eliminated because the quadriceps muscle doesn't cross the ankle joint and hence can't plantar flex the foot (i.e., cause a person to stand on tip-toes). Choice B can be ruled out because extending the hip posteriorly is the job of the hamstrings, not the quadriceps. Choice D can be eliminated because the adductor muscles aren't innervated by the femoral nerve; the
obturator innervates them. The correct choice is C.

99.

D is correct, weakness in the adductor muscles. What would happen if an intervertebral disk herniated (squeezed out of its sheath) and smashed the adjacent sciatic nerve? From the passage, we learn that the sciatic nerve innervates the hamstring muscles. These muscles are normally responsible for extending the hip posteriorly and flexing the knee (i.e., bringing the heel upwards posteriorly). These actions of the hamstring group can be explained by looking at where they attach. The hamstrings attach to the ischium posteriorly, cross the hip joint, and attach interiorly to the femur, tibia, and fibula. They therefore cross both the hip joint and the knee joint. Shortening of the hamstrings results in flexion of the knee and extension of the hip posteriorly. Since the hamstrings are innervated by the sciatic nerve, damage to this nerve would affect walking (i.e., hip extension), difficulty moving the leg posteriorly (knee flexion), and sensory stimulus along the back of the thigh and lower leg (which manifests as pain).
The combined conditions are known as sciatica. Choice D is our answer because the adductor muscles are not

innervated by the sciatic nerve. According to the passage, they are innervated by the obturator nerve. We would not expect them to be affected by damage to the sciatic. The correct choice is D.
100. C is correct, lower-than-normal CCb levels. What effect would reduced blood flow have on exercising muscle

cells? To start, Cb (which would be in high demand in active muscle) would not reach muscle fast enough due to poor circulation. This would lead to a decline in aerobic respiration (oxidative phosphorylation, etc.) resulting in ATP deprivation. In order to partially compensate, anaerobic respiration (glycolysis and lactic acid fermentation) would be stepped up. resulting in a buildup of the by-product lactic acid (possibly causing severe pain). CCb levels would actually rise because it could not be carried away effectively by the blood. The correct choice is C.

Copyright by The Berkeley Review

80

The Berkeley Review Specializing in MCAT Preparation

Biology
Section II
Heart and
A. The Heart

1. 2.
3. 4. 5.

Cardiovascular Anatomy Cardiac Output


Poiseuille's Law Diffusion Osmosis

Lung

6. Lymphatics 7. Blood Clotting B. The Lung


1. Gases

2.

Gas Exchange

Practice Passages and Answers

f?EBKELEY
Specializing in MCAT Preparation

Heart and Lung


Top 10 Section Goals

.^
?
J

Be familiar with the basics of cardiac anatomy.

Know how to describe the heart in the anatomical position, and understand its relationship to the other major organs of the body.

Qlftfc Be able to trace the path of blood flow through the heart. Starting with the vena cava, bloodflows intothe rightatrium, to the right ventricle, to the rightand
left lungs, to the left atrium, to the left ventricle, and then out to the systemiccirculation.

Q&* Understand how blood flows through the systemic circulation. jar Blood leaving the heart passes through the aorta, toarteries, arterioles, capillaries, venules, and back
to the veins beforeentering the heart againby way of the vena cavas.

O^i Be familiar withthe electrical activity of the heart.

]Br Know the actions of the sympathetic and parasympathetic nervous systems on the heart. Be able to
* trace the waves ofelectrical activity through theheart, and knowhow they relate to heart function.

'fm Understand the cardiac cycle in relation to pressure and volume curves.

jfir Understand the relationship between the opening and closing of the valves of the heart and how
theyrespondto pressure and volume changes.

%% Have a basic understanding of the lymphatic system.


Befamiliar with the concept of edema.

Understand the relationship ofthelymphatic system to thesystemic and pulmonary circulations.

amiliar with the basics of respiratory anatomy. o*m Bef; ^5r Have a mental picture of how the thoracic cage, diaphragm, visceral pleura, parietal pleura, and Have
J infra

intrapleural space relate to one another. Review the mechanics of inspiration and expiration.

(*)-* Understand the process of gas exchange at the level of the alveoli. ]5r Have a general feel for the differentpartial pressures ofO2 and CO2, both at the level ofthe lung
* and tissues. Be familiar with the gradients orexchange foreach ofthese gases.

() -% Understand the mechanics behind the oxygen-hemoglobin saturation curve. Know why the siemoidal saturation curve shifts to the rightor to the left. Be ableto relate thisshift to theeffects ofpH,[H], pO^ PCO2, temperature, and [2,3-BPG].
Know the basis of the Bohr effect and how this relates to ventilation.

This is tied into the concept ofmetabolic acidosis, metabolic alkalosis, respiratory acidosis, and
respiratory alkalosis. Beable to read and interpret graphs that describethese events.

Biology

Heart St Lungs

Cardiovascular Anatomy

The Heart
Cardiovascular Anatomy
Within living creatures there are two types of circulatory systems. In an open circulatory system the circulating fluid within the body is mixed with the body fluid itself. In other words, the blood in these creatures does not circulate entirely
within the confines of vessels like arteries and veins. Most mollusks and

arthropods have an open circulatory system. Conversely, in a closed circulatory system the blood that flows throughout the body is confined to vessels like arteries and veins. It does not freely mix with the fluid of the body. Annelids and mammals like ourselves have a closed circulatory system.

The function of the circulatory system is to bring nutrients and oxygen to the tissues of the body while simultaneously removing waste products from those very tissues. Because the circulatory system is continually flowing it helps to
maintain body temperature. Also, the circulatory system can act as a means to

transport hormones to various locations within the body. Ultimately, though, as the blood enters into the smallest vessels, the capillaries, there will be diffusion
between those capillaries and the cells in the immediate environment.
General Anatomical Features

Let's examine the general anatomical features of the mammalian cardiovascular system. The foundation of the circulatory system is the heart. The heart can be thought of as being divided into two halves. These two halves are often referred

to as the right heart and the left heart. When we look at a picture of a heart we must remember that we are viewing that heart in its anatomical position. In the anatomical position the body is standing upright, the arms to the sides, hands with plams up, and the face positioned forward. Thus, when we look at a diagram of a heart on paper, the right side of that paper really represents the left
side of the heart and vice versa.

The right heart pumps blood to the lungs and back to the left heart. The left heart pumps that blood to the remaining tissues of the body and back to the right heart. The blood that is pumped from the right heart to the lungs and back to the

left heart is called the pulmonary circulation while the blood that is pumped from the left heart to the rest of the tissues and back to the right heart is called the systemic circulation. Both the pulmonary and systemic circulations lie in series
with one another.

We can pick a starting point in the circulatory system and follow a red blood cell as it migrates through the pulmonary and systemic circulations. Let's start with a red blood cell in the right heart. The right heart is composed of the right atrium and right ventricle. As deoxygenated blood passes from the right atrium into the right ventricle it is pumped into the pulmonary artery and to the lungs where it is oxygenated. The oxygenated blood returns to the left heart by way of the (left and right) pulmonary veins. The blood enters the left atrium and passes into the left ventricle where it is pumped out the aorta and to the branching arteries, arterioles, and capillaries. It is at the level of the capillaries that the blood exchanges nutrients and oxygen for waste products created by metabolism. Deoxygenated blood passes from the capillaries to venules and then to larger veins and eventually to the superior and inferior vena cava which enter the right
Copyright by The Berkeley Review
83

The Berkeley Review Specializing in MCAT Preparation

Biology

Heart St Lungs

Cardiovascular Anatomy

atrium. The circulation of our red blood cell can now start again. As you are studying this circuit it is important to note that arteries carry blood away from the heart while veins carry blood from the tissues and lungs towards the heart.
This can be seen in the diagram shown in Figure 2-1.
Capillaries of Head and Arms

Superior Vena
Cava

Pulmonary Artery

Pulmonary
Vein

Inferior Vena Cava

Left
Atrium

Left Ventricle

Capillaries of Abdomen and Legs


Figure 2-1 Blood flow through the cardiovascular system.

Aorta and Arteries

As shown in Figure 2-1, blood leaves the left ventricle of the heart and exits through the aorta and passes to both the superior and inferior portions of the

body. The total cross-sectional area of the aorta is about 5 cm2. Under resting
conditions the velocity of blood flow in the aorta is about 30 cm/sec. The blood pressure fluctuates between 120 and 80 mmHg with an average being about 100 mmHg. The volume of blood found in the aorta and arteries at any given time is
about 16% to 20%.

As the left ventricle contracts (systole) it propels blood out the aorta and into the arteries with a pressure of about 120 mmHg. At sea level this force is enough to raise a column of mercury 120 mm above the ground. After contraction takes

Copyright by The Berkeley Review

84

The Berkeley Review

Specializing in MCAT Preparation

Biology
in the arteries is about 80 mmHg.

Heart & Lungs

Cardiovascular Anatomy

placeand the ventricles begin to relax (diastole) and fill with blood, the pressure
Measurement of Blood Pressure: When you have your blood pressure taken a

pressure cuff is placed around your upper arm and a stethoscope is placed over the antecubital artery (the artery at the bend between your upper and lower
arm). The cuff is inflated above arterial systolic pressure. This causes the artery in

your arm to collapse thus stopping the flow of blood. The pressure in the cuff is decreased and the pressure needle monitored. As soon as the pressure in the cuff is below the systolic pressure, blood will jet through the small opening in the artery. The flow through the artery is turbulent because of the great pressure pushing the blood through such a narrow opening. The "tapping" sounds that are heard in the stethoscope approximately correspond to the systolic pressure in the heart. The cuff is continually deflated and the artery slowly regains its original shape. The blood has an easier time passing through the vessel and the flow begins to change from turbulent to laminar (smooth). When the turbulent sounds disappear it means that you are recording the diastolic pressure of the heart. Blood pressure readings are given as systolic/diastolic (e.g., 120 mmHg/80 mmHg). These are the approximate readings that the baroreceptors are sensing as they monitor your blood pressure.
The blood that moves through the arteries is under a lot of pressure. Therefore, those arteries have to be rather durable. They have thick walls and are composed of smooth muscle and connective tissue that contain both collagenous and elastic fibers. The elasticity of the arteries prevents the blood pressure from

becoming too high when it is ejected out of the heart and it alsomaintains a high arterial pressure between the systolic and diastolic phases of the heart. This
allows blood to flow to the rest of the circulatory system without a sudden loss of
pressure.

The lumen of all the blood vessels in the body are lined with epithelial cells and because these cells are inside the cardiovascular system they are referred to as

endothelial cells. Damage to these endothelial cells by the pulsating arterial

pressure or even by abrasive substances in the blood can lead to the disease
known as atherosclerosis. Once these cells are damaged cholesterol can build up at the site of the lesion and a plaque will form. During the later stages of the disease the arteries become "hardened" from layer upon layer of deposit. This is referred to as "hardening of the arteries" or arteriosclerosis.

Regulation of the circulatory system is controlled (in part) by the sympathetic and parasympathetic divisions of the autonomic nervous system. The sympathetic division is the more important ofthe two. Besides nervous control of
blood flow there is also humoral control from the action of ions or hormones and local control at the level of the individual tissues from various metabolites.

How are metabolites important? If you were to start exercising a particular muscle, you would notice that the blood flow to that muscle would increase. Recall that when muscles contract ATP is hydrolyzed to ADP and Pi, lactic acid
is formed, and other such metabolites circulate in the tissue. When these
metabolites diffuse from the tissue out to the arterioles the smooth muscle dilates

and increases the flow of blood to that particular area. The blood brings not only nutrients and oxygen to the working tissue, but it carries away the waste
products of metabolism as well.
Copyright by The Berkeley Review
85

The Berkeley Review

Specializing in MCAT Preparation

Biology

Heart St Lungs
Arterioles

Cardiovascular Anatomy

After the aorta branches to form a variety of arteries, the arteries themselves

branch to form arterioles. The arterioles are important because they represent the major area of resistance in the cardiovascular system. Arterioles have strong muscular walls and are regulated as mentioned above.

Capillaries
Diffusion takes place at the level of the capillaries. The total cross sectional area

of the capillaries is about 3000 cm2. The velocity of blood flow has beenreduced
to about 2 cm/sec. The pressure is roughly 25 mmHg and the total volume of blood is about 5% to 7%. The wall of the capillaries are composed of a unicellular layer of endothelial cells. Surrounding these cells is a basement membrane. However, there is no connective tissue or smooth muscle. The capillary itself is just large enough for a red blood cell to squeeze through. At the entrance to the capillary bed is a precapillary sphincter composed of smooth muscle which helps to regulates the flow of blood to the area.
Veins

Once the blood flows through the capillaries and reaches the veins the blood pressure has been reduced to a value between 0 mmHg and 25 mmHg. The velocity of blood flow through the veins is about 20 cm/sec and the total cross

sectional area isroughly 7cm2. Roughly 50% ofthe total blood volume ispresent
in the veins at any given time.

Figure 2-2 A valve in a vein showing directionality.

Since there is not much pressure in the veins the amount of smooth muscle and elastic tissue surrounding the veins is reduced. However, they are under control of the sympathetic nerves. Note that the rate of blood flow in the veins is similar to that in the arteries. This is due to specialized valves that allow the blood to flow in only one direction (see Figure 2-2). As the muscles that surround the veins in your body contract they squeeze the blood back towards the heart. Since these valves are "one-way valves" blood is prevented from flowing backward. If the valves become damaged and blood is allowed to flow backwards, pressure in the veins can increase. This has a tendency to cause varicose veins, which are
protrusions of the dilated veins beneath the skin.
Blood Flow in the Heart

We have mentioned that deoxygenated blood returns from the tissues and enters the right atrium via the superior and inferior vena cava. As the right ventricle begins to relax blood from the right atrium is pumped into the right ventricle. The right ventricle contracts and forces blood out the pulmonary artery and to the lungs where it is oxygenated. Blood returning to the heart enters the left atrium via the right and left pulmonary veins. As the left ventricle begins to relax
Copyright by The Berkeley Review
86

The Berkeley Review Specializing in MCAT Preparation

Biology

Heart St Lungs

Cardiovascular Anatomy

blood from the left atrium is pumped into the left ventricle. Contraction forces the blood in the left ventricle out the aorta and to the tissues of the body. See Figure 2-3.

Pulmonary i!*^
Artery
Pulmonary
Veins 1"I

^r3!iM

Pulmonary Artery
Pulmonary
Veins

Superior

Left

Vena Cava
Pulmonary Valves

g^

Atrium
Aortic Valves

(semilunar)

-y

(semilunar)
L. Atrioventricular

Right
Atrium
R. Atrioventricular

Valves (mitral)

Valves (tricuspid)
Inferior
Vena Cava

Left Ventricle
Bundle of His

gives rise to
L. fir R. Bundle Branches

Chordae tendineae

i/am(mIa

Ventricle

Descending
Aorta

...

>i

Papillary
Muscle

(further aiviaes into irunner divides in

purkmje Fibers)

Figure 2-3
Anatomical landmarks of the heart.

Within the heart there are valves between the right atrium and right ventricle

(the right atrioventricular valve), between theright ventricle and thepulmonary artery (the pulmonary valve, or tricuspid), between the left atrium and the left ventricle (theleft atrioventricular valve, or mitral), and between the leftventricle
and the aorta (the aorticvalve). Oncethe ventricles are filled with blood and they

begin to contract the valves between the atria and the ventricles close. This prevents any backflow ofblood into the atria and ensures that the blood will be directed to the pulmonary and systemic systems of the body. The closing of the
atrioventricular valves between the atria and the ventricles as the ventricles are

contracting gives the characteristic "lub" sound when listening to the heart. As theblood flows out of the pulmonary artery and the aorta, the pulmonary valve and the aortic valve close, giving the characteristic "dub" sound. The closing of these valves as blood is pumped either from the atria intothe ventricles or from theventricles to the pulmonary or systemic tissues prevents backflow intoeither
Copyright by The Berkeley Review
87

The Berkeley Review

Specializing in MCAT Preparation

Biology

Heart & Lungs

Cardiovascular Anatomy

the atria or the ventricles, respectively. The valves between the atria and the
ventricles themselves do not invert because of tendinous cords, called chordae

tendineae, which hold them in place.

Electrical Activity of the Heart


Located near the junction of the superior vena cava and the right atrium is a specialized region of myocardium called the sinoatrial node (SA node) or the pacemaker of the heart (Figure 2-4).

Figure 2-4
The S A and A V nodes of the heart.

The SA node is the point of origin for the electrical impulse that propagates through the rest of the heart. This electrical impulse spreads out over the atria causing them to contract and fill the ventricles. Located in the lower portion of
the right atrium and near the right ventricle is the atrioventricular node (AV node). Impulses from the SA node also spread to the AV node and then from the AV node through a collection of fibers called the bundle of His. Branches of the
bundle of His surround the ventricles, and when this bundle receives an

impulses it causes the ventricles to contract and eject blood to the pulmonary and systemic systems. Anatomically speaking, ventricular contraction is from the apex of the heart towards the base of the heart. It is interesting to note that if the SA node is damaged, the AV node takes over and slows the heart down to about 40 beats per minute.
Cardiovascular Control

The average blood pressure leaving the aorta is about 100 mmHg. This blood pressure is monitored by baroreceptors and chemoreceptors located in the carotid arteries and in the aortic arch. The baroreceptors (pressure receptors) are
continually monitoring how much the aortic and carotid arteries are expanding
Copyright by The Berkeley Review
88

The Berkeley Review Specializing in MCAT Preparation

Biology

Heart St Lungs

Cardiovascular Anatomy

and contracting.Suppose we were to decrease the arterial pressure. There would be less stretch on the aortic and carotid arteries. The baroreceptors would sense this and send impulses to the medulla in the brain stem. The medulla responds

byactivating thesympathetic nerves of the autonomic nervous system. Impulses


are sent via the sympathetic nerves and norepinephrine is released at the SA node to increase the rate of the heart. This actionhelps to increasethe contraction

of the heart, which in turn will elevate the blood pressure. Sympathetic nerve fibers will also stimulate the adrenal medulla to release epinephrine (adrenaline)
into the blood. This hormone acts to increase the rate of the heart, therefore

increasing the contraction of the heart. Both of these actions act in a negative feedback manner to "negate" the initial loss of pressure due to, say,
hemorrhaging. The sympathetic nerves will also cause constriction of the blood vessels that lead

to the gastrointestinal system and the kidneys. During hemorrhaging the brain and heart receive first priority in termsof blood. Therest of the organsystems of the body see a decreasein the flow of blood until the problemis corrected.

Copyright by The Berkeley Review

89

The Berkeley Review Specializing in MCAT Preparation

Biology

Heart St Lungs

Cardiac Output

Cardiac Output
Every time your heart beats, a certain volume of blood is pushed out into the circulatory system. The cardiac outputis that amount ofblood which is pumped per minute by each of the two individual ventricles of the heart. We can define
the cardiac output (for either ventricle in liters/minute) as being equal to the heart rate (in beats/minute) times the stroke volume (in liters/beat). This is shownin equation (2-1). The stroke volume is simply the amountof bloodejected by each ventricle during one beat of the heart.

If the average heart rate is 72 beatsper minute and the average stroke volume is 70 milliliters (or 0.07 liters) per beat, then the cardiac output would be about 5 liters per minute. This value is for the average resting adult male. As we will see later, the volume of the cardiac output can be influenced by the diameter of the blood vessels in the periphery, the amount of blood returned to the heart by the superior and inferior vena cava (i.e., return of the venous blood), and the heart
rate and force of ventricular contraction.

Cardiac Output = (Stroke Volume) (Heart Rate)

(2-1)

Copyright by The Berkeley Review

90

The Berkeley Review

Specializing in MCAT Preparation

Biology
Poiseuille's Law

Heart St Lungs

Poiseuille's Law

Not all of the arteries and veins in the circulatory system have the same diameter. Jean Poiseuille, a French physician studying the flow of blood in blood vessels,

established a relationship between the radius and length of a tube, the change in pressure between the two ends of the tube, the viscosity, and the flow rate of a fluid in that tube. This relationship, known as Poiseuille's law, is given in
equation (2-2).

Flow =AP Ri= (P, - P2 ) i


8ilL to\L

(2-2)

In this equation (2-2), AP is the pressure drop between the two ends of the tube

(i.e., Pi - P2), R is the radius of the tube, eta (r|) is the coefficient of viscosity, L is the length of the tube, and rc/8 is a proportionality constant adjusting for the cross-sectional area of the tube. What can we say about this equation?

(a) Notice that the flow rate is proportional to R4. This tells us that the rate of
blood flow is extremely dependent on the radius of the vessel. If the radius of the vessel were reduced by a factor of 2, then the rate of blood flow would be reduced by a factor of 16. Similarly, if the radius of the vessel were

increased by a factor of 1.5, then the flow rate would increase by a factor of
5.1.

(b) The flow rate is also inversely proportional to the length of the vessel. In
other words, the longer the vessel, the slower the rate of flow. The shorter
the vessel, the faster the rate of flow.

(c) The flow rate is also inversely proportional to the viscosity of the solution. This tells us that a high viscosity gives a low flow rate.
(d) The value of AP is provided for by the strength of the heart's contraction. In other words, the difference in the pressure is what drives the blood in the cardiovascular system.

Copyright by The Berkeley Review

91

The Berkeley Review Specializing in MCAT Preparation

Biology

Heart St Lungs

Diffusion

Diffusion is simply the process by which molecules randomly move from one place to the next. The molecular weightof a molecule and the temperature of the
medium have a lot to do with the velocity at which a molecule moves. Smaller molecules tend to move faster than larger molecules. Similarly, a higher temperature provides more energy to a system and therefore imparts more thermal motion to the various molecules in that system. You might think that because smaller molecules move relatively fast they would have no problem

traversing their environment. However, don't forget that there are millions and
millions of molecules within a given system and that each of those molecules, even thought they are moving at a given velocity, collide with their neighbors. These collisions tend to alter the path of the molecules, thus confining them to a random walk through their medium.

Consider an imaginary sphere of water with a given radius. Suppose we place some dye outside this sphere of water and ask how long it will take to reach the center. The answer clearly depends on the radius of that sphere. If the radius were 7 microns, it would take the dye about 5.4 seconds to reach the center.
However, if the radius were 1 centimeter, it would take about 11,000 seconds or a little more than 3 hours to reach the center. [As a reference a red blood cell is

about 7 microns in length. There are about 1,400 red blood cells end to end in 1 centimeter.] What this is telling us is that simple diffusion is a rather poor way
for a molecule to trek across long distances.

The law that governs diffusion is given in equation (2-3) where J is the net flux or net rate of diffusion (in moles per unit time, usually in seconds; that is, mol/sec), D is the proportionality constantcalled the diffusioncoefficient, A is the area of

the plane ofinterest (in cm2), and AC/Ax is the concentration gradient across that

plane (in mol/cm4, because concentration isinunits ofmol/cm3 and distance isin
units of cm). Since the net flux always proceeds down a concentration gradient, from a high concentration to a low concentration, we need to add a minus sign in front of this equation. [The minus sign indicates the direction of the flux or diffusion.] This equation is sometimesreferred to as Fick's law (after the German physiologist who postulated it in the 19th century). In equation (2-3), what are
the units of the diffusion coefficient, D?

J = - (D)(A)

AC _ (D)(A)(Cout - Cin)
Ax
Ax

(2-3)

Copyright by The Berkeley Review

92

The Berkeley Review

Specializing in MCAT Preparation

Biology
Osmosis

Heart St Lungs

Osmosis

In order to understand hydrostatic pressure we must first review osmosis. Recall

that osmosis is simply the net movement of water from a region of high concentrationto a region of low concentration. Generalchemistrytells us that the concentration of pure water is 55.5 moles/liter. If we had a beakerof pure water
and we added a solute to that beaker (say glucose), then we would decrease the
concentration of the water in the beaker. Remember, when we add a solute

molecule (or molecules) we are occupying a volume of space that was once
occupied by a water molecule. The more solute molecules we add to our beaker

of pure water, the more water molecules we will displace and the lower will be
the concentration of pure water.

Glucose, when added to a solution of pure water, does not ionize. It stays as glucose. However, if we add a molecule ofsodium chloride to a solution ofpure
water, it will ionize into a Na ion and a Cle ion. Because we now have added a

sodium chloride molecule which has dissociated into two ions in solution, we have displaced (lowered the concentration of) the water molecules twice as much

as we would if we had added a glucose molecule. What this is telling us is that


the concentration of water in a given solution depends on the number of solute particles (e.g., glucose, Na, or Cle) in that solution. We can define the total solute concentration in our solution as the osmolality, where one osmol is
simply one mole of a molecule that does not ionize. If we had a IM solution of

glucose, it would have a concentration of 1 osmol per liter. If we have a 1M


solution of sodium chloride, we would find that it would have a concentration of

2 osmols per liter (one from the Na ion and onefrom the Cle ion). If we have a
150mM concentration of NaCl, then after ionization we would have 150 mM of

Na ions and 150 mM of Cle ions ora total of 300 milliosmols (mosmols) per
liter. Therefore, the osmolarity refers to the concentration ofsolute particles that
we have in our solution.

(a)

(b)

>i

Jih
^

H2 <_ >- 1i2o

H2(1

Permeable Membrane

Semipermeable Membrane

Figure 2-5 Water movement across a permeable and semipermeable membrane.

Suppose we have a U-tube with water and a permeable membrane as shown in Figure 2-5a. Water is free to pass back and forth across this membrane and because of this the height of the water in each of the columns of the U-tube will be the same. However, suppose we now replace the permeable membrane with a

semipermeable membrane and then add some protein to the right side of the Utube as shown in Figure 2-5b. What willhappen? The level of water in the right
Copyright by The Berkeley Review
93

The Berkeley Review

Specializing in MCAT Preparation

Biology
drop. Why?

Heart St Lungs

Osmosis

side of the tube will rise and the level of water in the left side of the tube will

In order to understand this we must examine the semipermeable membrane and the solutes which we have added to the right side of the tube in a little more detail. First, the concentration of solute on the right side of the tube is greater
than the concentration of the solute on the left side of the tube. This established a

solute concentration gradientin which the solutes on the right side of the U-tube want to diffuse down their concentration gradient to the solution on the left side of the U-tube. This cannot happen, because we have said that the membrane is

semipermeable. In other words, the membrane will not allow these solutes to pass through only water. What aboub the concentration of the water? The
concentration of the water on the left side of the U-tube is greater than the

concentration of water on the right side of the U-tube. Again, a concentration

gradient has been established that will allow water to diffuse from theleft side of
the U-tube to the right side of the U-tube. Since the membrane is permeable to
water we find that water diffuses down its concentration gradient (from left to

right). This leads toan increase in the volume in therightsideof the U-tube and
a decrease in the volume in the left side of the U-tube. The effect is to decrease

the solute concentration in the right side of the U-tube. An equilibrium will

eventually be established when the concentrations of both the water and the
solute are equalon both sidesof the semipermeable membrane.

Once equilibrium isreached nomore water will flow from theleft side ofthe Utube to the right side of the U-tube. This is because the pressure has increased in the right side of the U-tube (because there is now a larger volume of solution pushing on the semipermeable membrane). The amount ofpressure thatstopped
osmosis is referred to as the osmotic pressure (abbreviated as rtosm). A direct

measure of theosmotic pressure is the difference in the levels of waterin the left andright sides ofthe U-tube. This difference, Ah, is referred toas thehydrostatic pressure (or fluid pressure) which canbe abbreviated as PH2O.

The osmotic pressure is proportional to the number of dissolved molecules in a solution and isrepresented in Figure 2-6. As we increase theconcentration of the protein in solution (i.e., solute in solution) we find that the osmotic pressure
increases as well.
<u
h-

3
t/3

<D '"i,

s
O
Cfl

J>
K
"-'

[Protein]
Figure 2-6

Relationship between osmotic pressure and dissolved particles.

Now, let's return tothe hydrostatic pressure generated by theheart which forces fluid out of the capillaries and into the interstitial space. Since the hydrostatic

pressure in the capillaries turns out to be a bit greater than the osmotic "pulling pressure" of the solutes in the blood, there is a net movement of fluid from the capillaries to the interstitial space and eventually into the lymphatic capillaries.
94

Copyright by The Berkeley Review

The Berkeley Review

Specializing in MCAT Preparation

Biology
Lymphatic System

Heart & Lungs

Lymphatic System

The lymphatic system lies parallel to the systemic and pulmonary circulations. This can be seen in Figure 2-7. The lymphatic system collects the excess fluid (about 4 liters per day) that leaks into the interstitial space from the capillaries and returns it by way of the vena cava back to the circulatory system. Lymph nodes located along the lymphatic system help to filter out foreign particles that couldpotentiallylead to disease. If the lymph flow through the lymphatic system were blocked, edema would result. This is simply an increase in the interstitial fluid (because it cannot be reabsorbed by the lymphatic system). Patients who have heart surgery usually have swollen legs and ankles. This is because the heart cannot pump the blood out fast enough, and as a result blood within the

heartbegins to back up. This translates to a back-up in theveins and eventually


in the lymphatic system. Since gravity pools fluid toward the lower extremities, edema results in the legs and ankles.

^D Lymphatic capillaries
Pulmonary capillaries

Artery

Valve IZ^

Lymph
node

Systemic capillaries

^3 Lymphatic capillaries

Figure 2-7 The lymphatic system.

Copyright by The Berkeley Review

95

The Berkeley Review

Specializing in MCAT Preparation

Biology

Heart St Lungs

Blood Clotting

Blood Clotting
Blood clotting occurs via a cascade process. Thrombin, which is involved in blood clotting, is a serine protease. We can achieve amplification of a very weak signal by a cascade process. If we consider blood clotting, we will find that there is an
intrinsic route (due to contact with some abnormal surface) and an extrinsic route (due to trauma to the tissue), both shown in Figure 2-8.

In the outline shown in Figure 2-8 we will use the Roman numerals to represent

the clotting factors. The subscript "a" means that we are dealing with the active form of the molecule. Again, this would be some type of conversion of a proenzyme to an enzyme. Factor IX will be converted to Factor IXa by some intrinsicfactor. Factor IXa will be the trigger which will convert Factor X to Factor Xa. From the extrinsic portion of this scheme we start with some tissue
factor which will convert Factor VII to the active form, Factor Vila. Factor Vila

can also bring about the conversion of FactorX to FactorXa. It is Factor Xa which is involved in the conversion of prothrombin (II) to thrombin (Ha). Thrombin (Ila) will convertfibrinogen (I) tofibrin (la). These fibrin fibers are then crossed-linked by FactorXHIa (which is an enzymecalled transglutaminase) to form the mature
cross-linked fibrin clot.

Some of the serine proteases mentioned in Figure 2-8 are Factors VII, Vila, Xa, II, and Ila. In the blood clotting process there are more than 15 different factors
involved, and about 8 or so of them are serine proteases.
Intrinsic Pathway

(Damaged Surface)

Extrinsic Pathway (Trauma)

Factor IX '

> FactorIXa
VIIL

FactorVIIa <= > Factor Xa


t

Jl
=> Factor VII

FactorX '

<

^ Factor X

Prothrombin (II)

> Thrombin (IIa)

Fibrinogen (I)

> Fibrin (Ia)

JI XIIIa (transglutaminase)
Crossed-linked Fibrin Clot

Figure 2-8
The intrinsic and extrinsic pathways.

A lot is known about the biochemistry of blood clotting. In the conversion of Factor Xto Factor Xa by Factor IXa, we find that we need an additional factor called Factor Villa. Individuals lacking Factor Villa havehemophilia (asex-linked recessive characteristic). Factor Villa is sometimes called the antihemophiliac
factor.

Copyright by The BerkeleyReview

96

The Berkeley Review

Specializing in MCAT Preparation

Biology
0 HO

Heart 8e Lungs

Blood Clotting

II
1 H

1
1 CH2

II
a

Carboxylase enzyme requiring Vitamin K

HO

II
1 H

1
1 CH2
1
C-H
1

II

CN CC

and HCO3-.
/

C N C C

| CH2
Cv

P
Y Y-Carboxyglutamate
residue

o=c' Nc-0
1 0 1
+Ca+
Prothrombin fragment
chelated with calcium.

Preprothrombin fragment

Figure 2-9 Chelating action.

Vitamin K is one of the fat-soluble vitamins that is found in green leafy


vegetables. In addition, our intestinal flora can make a form of Vitamin K. What is the importance of this vitamin? Prothrombin exists in even an earlier form

called preprothrombin. In preprothrombin there are certain Glu residues which

are carboxylated by a carboxylase enzyme. This carboxylase enzyme has an absolute requirement for Vitamin K. The carboxylase enzyme adds another
carboxyl group to the gamma position of the first ten Glu residues located in the amino terminal region of preprothrombin. Thus, in the presence of Vitamin K,

HCO3 and the carboxylase enzyme, we willbe able to convert preprothrombin to prothrombin. This structure now has a great affinity (a good chelating agent) for

divalent ions such as Ca2. This is shown in Figure 2-9. (Calcium ions are
essential for blood clotting.)

Platelet Membrane

following injury.

After cut is made,


thrombin is released. A

A section of prothrombin

Figure 2-10 Binding of prothrombin to a membrane.

Prothrombin is next converted to thrombin in the presence of Factor Xa. The conversion of prothrombin to thrombin in the presence of FactorXacan be pictured as shown in Figure 2-10. Blood platelets have phospholipid molecules in their membrane. The head of the phospholipid is negatively charged. This will allow
Copyright by The Berkeley Review
97

The Berkeley Review

Specializing in MCAT Preparation

Biology

Heart St Lungs

Blood Clotting

the y-carboxyglutamate residues ofprothrombin tobind via Ca2. Factor Xa also


has y-carboxyglutamate residues on it, and can also bind to the membrane via

interaction withCa2e. The enzyme Factor Xa willmake cuts (atArg-X residues)


in the prothrombin molecule as shown. The portion of the prothrombin molecule that is cut away is called thrombin. Thrombin will drift away and convert fibrinogen into fibrin in the vicinity of the damaged area. It is fibrin that will form the blood clot. There is also an auxiliary factor, Factor Va, which is involved in this process. So, what you need for this clotting process to take place

is the (a) platelet membrane, (b) enzyme, (c) Ca2 ions, (d) an auxiliary factor,
and (e) the substrate prothrombin. Thrombin can now act as a proteolytic enzyme that converts fibrinogen to fibrin. Fibrinogen is a large soluble protein. Its solubility is due to an excess of negatively charged amino acids (Glu, Asp, Tyr-SC^), particularly in the central domain of the molecule. The net charge in the central domain of the fibrinogen molecule is -8, while the net charge at the terminal ends is -4. If very large molecules have a net charge of zero, they will tend to come together. However, if we have an excess of negative charges or positive charges, the molecules will repel one another.

Fibrinogen

TfrTPmrm

Fibrinopeptides

Aggregation of
fibrin monomers
V

f . \ssssf +Wis/ - j f - Wsh/+ \pmmd - j

O000=0G
Fibrin Clot

Figure 2-11
Excess charge allows for aggregation.

Thus, one way to make things soluble is to have an excess of charge. In order to converta soluble proteinto an insoluble protein one must remove that portion of the molecule contributing all those negative (or positive) charges. When fibrinogen is converted to fibrin we find that 4 Arg-Gly bonds are brokenin the central domain. This releases four peptides containing an excess of negative
charge calledfibrinopeptides.

Copyright by The Berkeley Review

98

The Berkeley Review

Specializing in MCAT Preparation

Biology

Heart St Lungs

Blood Clotting

Once these fibrinopeptides are released, the overall net charge in the central
domain now becomes +5. The fibrin monomer that is now formed (due to the release of the fibrinopeptides) has the ability to interact with other fibrin
monomers through electrostatic interactions between the terminal and central

domain regions of the polypeptide. This is shown in Figure 2-12. This


aggregation of fibrin monomers leads to the formation of the fibrin clot.

The clot that is initially formed is called a soft clot. The hard clot involves a

further step, in which there is a cross-linking via the enzyme transglutaminase


(or Factor XHIa). Cross-links are formed between the individual subunits that

make up the aggregate. Thosecross-links occurbetweenGin and Lys residues as


shown in Figure 2-12. This type of reaction is referred to as a transamination
reaction.

Fibrin CH2

II CH2 C NH2
Gin

NH3-(CH2)4

Fibrin

Lys

Xllla (transglutaminase)

Fibrin CH2

CH2 C N- (CH2)4H

Fibrin

Cross-linked Fibrin Clot

Figure 2-12 Cross-linking leads to a hard clot.

Once the damaged area has been repaired, a serine protease called plasmin hydrolyzes specific regions in the fibrin clot in order to dissolve it into smaller peptide fragments (to remove the clot). Tissue plasminogen activator (TPA) converts plasminogen into this active protease.
Let's return to vitamin K for a moment. Vitamin K is one of the fat-soluble

vitamins that is found in green leafy vegetables. In addition, out intestinal flora can make a certain form of vitamin K. What is the importance of this vitamin? We have already discussed the importance of this vitamin in connection with blood clotting. If you were to have a vitamin K deficiency, you could suffer from bleeding disorders. One way to have a vitamin K deficiency is a failure to absorb lipids properly. Another way is to take a lot of antibiotics. Antibiotics have a tendency to kill off the intestinal flora that help synthesize vitamin K. Finally, there may not be enough of this vitamin in the diet.
There are also antagonists to vitamin K, like dicoumarol. In the 1920's many farmers in Wisconsin were having problems with their cows. They were feeding their cows various feeds, some of which were causing hemorrhaging in the cows. It turns out that dicoumarol is a naturally occurring material in the rotting clover
of the feeds that the cows consumed. Cows that ate the dicoumarol had an

abnormal prothrombin that did not bind Ca2.


Copyright by The Berkeley Review
99

The Berkeley Review Specializing in MCAT Preparation

Biology

Heart & Lungs

Blood Clotting

It occurred to some chemists that they might be able to find an inhibitor similar to dicoumarol which would not be toxic to cows, chickens, or even people, but

might have an effect on rats or mice. One of the things that farmers didn't like was rats and mice running around eating their chicken feed. The chemists at the Wisconsin AgriculturalResearch Station developed a vitamin K antagonist called Warfarin. If you mixed Warfarin with chicken feed, and that feed was eaten by
the rats and mice, then eventually they would die of hemorrhaging. It would take much greater levels of warfarin to be toxic to humans. It turns out that warfarin
is much more active in mammals than in birds, so the chickens were not affected.

Warfarin is a Vitamin K antagonist that is used as rat poison. The structures of


vitamin K, Warfarin, and dicoumarol are shown in Figure 2-13.

O,

CH3

11 CH2 - C= C- CH2 -f H

OH
Vitamin K2
O O

?H^\\ //
CH2
I
CH,
C=0

Warfarin

0. //
Dicoumarol

\\ . 0

CH2
OH OH

Figure 2-13
Chemical structures of vitamin K, Warfarin, and dicoumarol.

Copyright by The Berkeley Review

100

The Berkeley Review

Specializing in MCAT Preparation

Biology

Heart St Lungs

Gases

The Lungs
Gases

Glucose can be oxidized to carbon dioxide and water as shown by equation (2-4). The majority of carbon dioxide is generated in the Krebs cycle. Oxygen is used as the final electron acceptor in the electron transport chain. Water is generated in

that reaction. Out of this metabolic pathway we are able to extract energy in the
form of ATPand that ATP is ultimately used to keep the organismalive.

6 02 + C6H1206 i==> 6C02 + 6 H20 + Energy

(2-4)

In general, respiration is the process by which oxygen is brought to the cells of the tissues and carbon dioxide is removed as a waste product. As we breathe air into our lungs it first enters our system by way of the nose or mouth. The air passes form the oral cavity to the pharynx, into the larynx, and then down the

trachea. At the end of the trachea the air passes into two tubular passageways
called bronchi. One bronchus enters into each lung and continues to divide into smaller passageways called bronchioles, ending eventually in the functional

units of the lungs which are the alveoli. Each alveolus consists of a single layer of epithelial cells juxtaposed to a very thin basement membrane. Surrounding
each single alveolus is a capillary network. The epithelial cell layer of each alveolus and the endothelial layer of the capillaries are separated from each other by a very narrow interstitial space (if they are separated by an interstitial
space at all). This means that the air in each alveolus and the blood in the

capillaries are separated by a very small distance (about 0.2 to 0.3 |im, compared to the average diameter of an erythrocyte, which is about 7 (im). The two lungs
are composed of millions of alveoli, and if they were all laid flat on a surface,

their combined total surface area would be between 70 m2 to 100 m2 (which is about the size of a tennis court). Therefore, large quantities of oxygen in the alveoli of the lungs can quickly be equilibrated with the blood in the capillaries because of the large surface area and the thin barrier to diffusion for gas
exchange.

The air that we breathe on a normal day is composed of roughly 78% N2, 21% 02, 0.3% C02, and 0.7% H20. [These values vary slight from textbook to textbook.] Because all of these gas molecules are (normally) quite far apart, they tend not to interfere with one another. This means that the pressure exerted by one gas is independent of the pressure exerted by all of the other gases. In other words, the sum of each of these individual gas pressures equals the total pressure of this mixture of gas. The partial pressure of a gas (e.g., PN2, PO2, PCO2, etc.) is therefore a measure of the concentration of a given gas (such as O2) in a mixture of gases (i.e., the air). If we add up all the partial pressures of the individual
gases in the atmosphere, we will come up with the atmospheric pressure. At sea level the atmospheric pressure is 760 mmHg.

What is the partial pressure of oxygen gas (PO2) at sea level? Roughly 21% of the total atmospheric pressure at sea level is oxygen gas. Therefore, the partial pressure of oxygen gas at sea level is about 160 mmHg (from 0.21 x 760 mmHg = 160 mmHg). What happens to the partial pressure of oxygen as we ascend to the
Copyright by The Berkeley Review
101

The Berkeley Review Specializing in MCAT Preparation

Biology

Heart & Lungs

Gases

top of a high mountain? The concentration of oxygen in the air on this mountain top will still be about 21%. However, as we increase our altitude the atmospheric pressure begins to decrease. The result is a decrease in the partial pressure of
oxygen.

How does a gas behave in a liquid? Consider an open glass of water sitting on a table as shown in Figure 2-14. The surface of that water is constantly being bombarded with gas molecules from the air.

Figure 2-14
Gas movement across a water surface.

When a gas molecule such as O2 comes in contact with that water surface it can dissolve in the liquid. The number of O2 molecules which dissolve in the water is directly proportional to the partial pressure of the O2 gas. Just as O2 molecules in the air can hit the surface of the water and dissolve in the liquid so too can O2 molecules in the liquid hit the surface of the water and escape into the air. Therefore, at equilibrium the number of O2 molecules dissolving in the water will equal the number of O2 molecules leaving the water. In other words, the partial pressure of oxygen in the gas phase (air) is equal to the partial pressure of oxygen in the liquid phase (water), as shown in equation (2-5).

(Po2)gas - (Po2)liquid

(2-5)

We can say that the pressure of the air acting on the membranes of the epithelial cells in the alveoli of the lungs is the sum partial pressures of all the gases in the air. Thus, the total pressure for any given gas molecule is directly proportional to the concentration of that gas molecule in the air. For example, we can account for the many factors that affect the rate of diffusion of a gas into a liquid by considering equation (2-3). In this case J is the flux or diffusion rate of the gas
molecule across the membrane, D is the diffusion coefficient, A is the area of the

plane the molecule is diffusing across, C is the concentration of the molecule, and x is the distance of diffusion. Since the concentration of a gas is proportional to the pressure of the gas, we can write that as shown in equation (2-6).

j_ (DXAXCft -C22) a (DXAXP!02 -P202)


Ax Ax

(2-6)

Copyright by The Berkeley Review

102

The Berkeley Review Specializing in MCAT Preparation

Biology

Heart St Lungs

Gases

The respiratory passages have a unique anatomical structure. Let's consider a cross-section of one of these passageways and work our way from the lumen outward. The epithelial cells which line the lumen of the passagewaysto the end of the bronchioles have cilia which are continually beating towards the pharynx. Scattered among these epithelial cells are glands which secrete mucus. As the cilia beat towards the pharynx the mucus is moved upward towards the oral cavity. Any foreign matter that has become trapped in the mucus is eventually transported to the oral cavity where it is swallowed in a normal reflex action. Oneof the reasons that smokingis bad for you (besides its carcinogenic nature) is that it decreases the activity of the cilia and lowers the body's defenses against lung infection by bacteria that can enter the respiratory tract on airborne dust particles. Immediately beneath the layer of mucus is a layer of fluid in which the cilia operate. Air that flows within the passageways of the respiratory tract is
warmed and moistened.

The upper passageways of the respiratory tract maintain their opening by means of cartilage rings that surround most of the diameter of the passageways. By the time the bronchioles are reached the cartilage has disappeared. Smooth muscle is found in almost all areas of the respiratory tract where there is no cartilage. For
example, the walls of the bronchioles are mainly smooth muscle. The bronchioles of the lungs are innervated by nerve fibers from parasympathetic nerves (which travel in the vagus nerve).

Asthma is usually caused by an allergic hypersensitivity to airborne antigens which have entered the respiratory tract. The direct result is to cause the mast
cells within the bronchioles to release a number of different substances which

cause the smooth muscle surrounding bronchioles to spasm and constrict.

Mechanics of Breathing
The thoracic cage which contains the lungs is separated from the abdomen by a sheet of skeletal muscle and connective tissue called the diaphragm (see Figure
2-15). The lungs themselves are encased in a pleural membrane. The visceral pleura covers the lungs, while the parietal pleura adheres to the diaphragm and the chest wall. Between the visceral pleural and parietal pleura is the intrapleural space which contains a watery fluid. As the muscles of the diaphragm contract they pull the diaphragm itself downward. Simultaneously the muscles of the rib cage contract and cause the rib cage to move upward and outward. Both of these actions enlarge the area of the thoracic cage that contain the lungs.

Since the diaphragm is attached to the parietal pleura this pleura is also pulled downward. The watery fluid in the intrapleural space is rather indistensible and the pulling of the parietal pleura translates through the intrapleural fluid. It also pulls the visceral pleura downward as well. Enlargement of the thoracic cage expands the lungs and creates a subatmospheric pressure in the alveoli. Air rushes down its pressure gradient from 760 mmHg at sea level to whatever lower pressure is found in the lungs at inspiration. Once the muscles of inspiration stop contracting, the elastic tissue found within the thoracic cage and lungs returns to its normal length. Air within the alveoli is compressed and forced out through the passageways of the respiratory tract during expiration.

Copyright by The Berkeley Review

103

The Berkeley Review Specializing in MCAT Preparation

Biology

Heart & Lungs

Gases

If the lungs were to become separated from the visceral pleura, they would collapse. This is because they have no anatomical structures to maintain rigidity. One way to separate the lungs from the visceral pleura is to receive a very strong
blow to the chest area (as often happens during football games).
To the pharynx and oral cavity

Larynx ^^
Intrapleural Cartilage rings
space

Ribs

Parietal

pleural

Figure 2-15
The thoracic cage.

Copyright by The Berkeley Review

104

The Berkeley Review Specializing in MCAT Preparation

Biology
Gas Exchange

Heart St Lungs

Gas Exchange

It is important to remember that molecules will diffuse from a site of high


concentration to a site of low concentration. As deoxygenated blood (a dark red color) is coming back from the tissues it enters the right atrium via the superior and inferior vena cava and then passes into the right ventricle, where it is pumped out to the lungs. The PO2 is about 40 mmHg and the PCO2 is about 46 mmHg as this deoxygenated blood enters the capillaries surrounding the alveoli of the lungs. The PO2 and PCO2 within the alveoli are about 105 mmHg and 40 mmHg, respectively. Passage of the blood through the capillaries is relatively slow and an equilibration can be reached between the gas exchange in alveoli and the capillaries. Oxygen will diffuse down its concentration gradient from the alveoli to the capillaries, and carbon dioxide will diffuse down its concentration gradient from the capillaries to the alveoli. As the oxygenated blood (a bright red color) leaves the capillaries of the lungs and enters the left atrium of the heart, the PO2 is about 100 mmHg and the PCO2 is about 40 mmHg. See Figure 2-16.
P02 = 160
All values are in

PCO2 = 0.3

mmHg P02 = 105

PC02 =40 J Alveoli


P02 = 100 PCO2 = 40

P02 = 40 PC02 = 46

Capillaries at level of alveoli

Right Heart
Capillaries at level of tissues
P02 = 40 PC02 = 46

Left Heart

P02 = 100 PC02 = 40

PC02 > 46

Cells

Figure 2-16 Gas exchange at the level of the lungs and tissues.

Oxygenated blood will pass from the left atrium to the left ventricle of the heart and then be pumped to the tissue capillaries. At the levelof the tissues the PO2 in the cells (depending on which cells you are considering) is less than 40 mmHg,
while the PCO2 in those cells is greater than 46 mmHg. Oxygen will diffuse from the blood in the capillaries to the cells while carbon dioxide will diffuse from the cells to the blood in the capillaries. The deoxygenated blood that leaves the tissue capillaries returns to the right atrium of the heart via the venous system.
Copyright by The Berkeley Review
105

The Berkeley Review Specializing in MCAT Preparation

Biology

Heart & Lungs

Gas Exchange

Oxygen can travel in the blood by being dissolved in the blood itself or by being bound to a transport protein in the red blood cells (erythrocytes) called hemoglobin (abbreviated as Hb). Since oxygen is rather insoluble in water, not much is actually dissolved in the blood and transported in that manner. However, since hemoglobin has such a high affinity for oxygen, more than 98% of the oxygen in contact with hemoglobin is transported by this protein.
Hemoglobin is a protein that is composed of four polypeptide subunits. When these subunits interact with each other to form the hemoglobin molecule, they give hemoglobin a quaternary structure. Located within each of the four polypeptide subunits is a heme prosthetic group that has an iron atom in the

center which is in the ferrous (Fe2) oxidation state. Since one hemoglobin
molecule has four binding sites for oxygen, there is a potential for 4 O2 molecules to bind to one hemoglobin molecule. Every time hemoglobin takes up O2 from the blood more O2 can leave the gas phase in the alveoli and enter into the liquid phase in the blood. This increases oxygen's solubility in the blood.

Oxygen Saturation Curve for Hemoglobin


In Figure 2-17 we have a plot of the percent saturation of hemoglobin with oxygen as a function of the partial pressure of oxygen at various places in the body. Recall that when the PO2 is about 100 mmHg, we are at the level of the alveoli and when the PO2is about 40 mmHg, we are at the level of the tissues. [It is best to read this curve from right to left and from top to bottom.]
Venous Arterial

Blood
100-r

pH7.4

Blood

O2 released
O O

80-

to tissues

Extra O2

X 60
o
c

I
Shift due to:

released to tissues

-a

40

Decrease in pH Increase in temperature


Increase in 2,3-DPG

{20Resting Muscle

Working Muscle
40

P02 (mmHg)

100

Figure 2-17
Oxygen-hemoglobin dissociation curve.

At the level of the capillaries (PO2 ~ 100 mmHg) surrounding the alveoli roughly 98% of the hemoglobin is saturated with oxygen. As the blood passes to the tissues (PO2 = 40 mmHg) roughly 25% of the oxygen that was bound to hemoglobin is given up to the tissues. This can be seen in Figure 2-17.
Copyright by The Berkeley Review
106

The Berkeley Review Specializing in MCAT Preparation

Biology

Heart St Lungs

Gas Exchange

Effects of pH, Temperature, and 2,3-BPG


The upper oxygen dissociation curve shown in Figure 2-17 is for normal conditions in which the body temperature is about 37 "C and the blood pH is about 7.4. If we were to decrease the pH of the blood (to a pH of 7.2) or increase

the temperature, then the oxygen dissociation curve would shift to the right and downward as shown in the lower curve in Figure 2-17. This happens when you exercise. The same type of shift occurs when a by product of glycolysis, 2,3bisphosphoglycerate (abbreviated as 2,3-BPG), binds to hemoglobin. 2,3-BPG is usually synthesized in increased amounts when the body has been deprived of oxygen for an extended period of time (e.g., when you visit high altitudes).
All three of these interactions with hemoglobin (the lowering of the pH, the increase in temperature, and the binding of 2,3-BPG) cause hemoglobin to release more oxygen to the tissues at the same partial pressure of oxygen as in the standard case. The oxygen dissociation curve can be shifted to the left and upward by reversing these interactions.
Carbon Dioxide

How is CO2 carried in the blood? It can be carried by (a) dissolving in the plasma and the red blood cells, (b) binding to a specific site on the hemoglobin molecule,

or (c) in the form of bicarbonate ions (HC03e). About 70% of the carbon dioxide
is carried in the blood in the form of bicarbonate ions, roughly 20% is carried by the hemoglobin itself, and about 10% is dissolved in the plasma and red blood cells. See Figure 2-18.

70%

Plasma

/
/

\^^Red Blood Cell


Carbonic

^y^ ""\ HC03e

( C02 + H20

anhydrase

H2C03

H +HC03e)
5% C02 J
>^^dissolved/

Nl

20% Hb-C02

x \ > ^ 5% CO2 dissolved


P ^ CO2 from tissues

Figure 2-18 Gas exchange at the level of the tissues.

At the level of the tissues the PCO2 is greater than 46 mmHg. However, in the blood of the capillariesit is about 40 mmHg. Therefore, CO2 will diffuse down its concentration gradient and into the blood. Some of the carbon dioxide will dissolve in the blood plasma, some will dissolve in the red blood cell, and some will bind to hemoglobin. The remaining CO2 will react with water and be converted to carbonic acid (H2CO3) by the enzyme carbonic anhydrase. Carbonic acidwill ionizeto the bicarbonate ion and a proton (He). Bicarbonate will diffuse into the blood plasma and be carried by the circulatory system to the capillaries
of the lungs. See Figure 2-18.

Copyright by The Berkeley Review

107

The Berkeley Review Specializing in NCAT Preparation

Biology

Heart St Lungs

Gas Exchange

At the level of the capillaries in the lungs, the PCO2 in the blood is about 46 mmHg, while the pC02 in the alveoli of the lungs is about 40 mmHg. Once again, CO2 will diffuse down its concentration gradient and into the alveolar space. The CO2 that is dissolved in the plasma and in the red blood cell diffuses into the alveoli as does the CO2 that was bound to the hemoglobin. Bicarbonate ion in the plasma will diffuse into the red blood cell and combine with a proton to become carbonic acid. Carbonic anhydrase will convert carbonic acid to water and carbon dioxide. The CO2 diffuses into the lungs. This is shown in Figure 219.

Plasma

HC03<

Capillary

To the alveoli

Figure 2-19 Gas exchange at the level of the alveoli.

Control of Breathing The coordinated rhythm for breathing is generated by the medulla and the pons in the brainstem. Nerve impulses from these centers cause the respiratory muscles to contract. What is the stimulus that causes contraction? The major
stimulus is a change in the concentration of CO2 and H that affects chemo-

sensitive areas in the medulla. The concentration of O2 in the bloodis sensed by


chemoreceptors in the carotid arteries and in the arch of the aorta. These

chemoreceptors transmit signals to the respiratory center in the brainstem,


informing it of the levels of O2 in the blood.

If you were to breathe into a container that collected your exhaled CO2, you
would soon find yourself breathing deeper and faster. The increase of CO2 in your blood indicates to your respiratory center that C02 is building up in the body. One way to eliminate CO2 would be to breathe faster and deeper. What would happen if you were to breathe pure O2? The chemoreceptorsfor O2 would relay a message to the respiratory centers indicating that there is too much
oxygen in the blood. Impulses from the brain stem would be sent to the respiratory muscles in order to slow their action.

Copyright by The Berkeley Review

108

The Berkeley Review Specializing in MCAT Preparation

Heart and Lungs

15 Passages
100 Questions

Passage Titles
I. II. III. IV. V. VI. VII. VIII. IX. X. XI. XII. XIII. XIV. XV.

Questions
1 -7 8- 14 15-21 22-28 29-34 35-40 41 -47 48-54 55-60 61 -66 67 - 72 73-78 79-85 86-92 93- 100

Mechanics of Blood Plow Enzymes in Blood Clotting Cardiac Output and Venous Return Circulatory Pressure, Area, Velocity, and Volume
Thoracic Cavity Electrocardiogram Folate Experiment Measurement of Blood Pressure Aortic Compliance
Heart Muscle Action Potentials

Capillary Filtration Respiratory Calculations


Aspirin
Sickle Cell Anemia

Ventilation Regulation

fgERKELEY
Specializing in MCAT Preparation

Suggestions
The passages that follow are designed to getyouto think in a conceptual manner about the processes of physiology at the organismal level. If you have a solid foundation in physiology, many of these
answerswill be straightforward. If you have not had a pleasant experience with the topic, some of these answers might appear to come from the void past the Oortfield of the solarsystem. Picka few passagetopics at random. Forthese initial few passages, do not worry about the time. Just focus on what is expected of you. First, read the passage. Second, look at any diagrams, charts, or graphs. Third, read each question and the accompanying answers carefully. Fourth, answer the questions the best you can. Check the solutions and seehow you did. Whether you got the answersright
or wrong, it is important to read the explanations and see if you understand (and agree with) what is being explained. Keep a record of your results.
After you feel comfortable with the format of those initial few passages, pick another block of passages and try them. Be aware that time is going to become important. Generally, you will have about 1 minute and 15 seconds to complete a question. Be a little more creative in how you approach this next group. If you feel comfortable with the outline presented above, fine. If not, then try different approaches to a passage. For example, you might feel well versed enough to read the questions first and then try to answer some of them, without ever haying read the passage. Maybe you can answer some of the questions by just looking at the diagrams, charts, or graphs that are presented in a particular passage. Remember, we are not clones of one another. You need to begin to develop a format that works best for you. Keeping a record of your results may be helpful.

The last block of passages might contain topics that are unfamiliar to you. Find a place where the level of distraction is at a minimum. Get out your watch and time yourself on these passages, either individually or as a group. It is important to have a feel for time, and how much is passing as you try to answer each question. Never let a question get you flustered. If you cannot figure out what the answer is from information given to you in the passage, or from your own knowledge-base, dump it and move on to the next question. As you do this, make a note of that pesky question and come back to it at the end, when you have more time. When you are finished, check your answers and make sure you understand the solutions. Be inquisitive. If you do not know the answer to something, look it up. The solution tends
to stay with you longer. (For example, what is the Oort field?)

The estimated score conversions for 100 questions are shown below. At best, these are rough approximations and should be used only to give one a feel for which ballpark they are sitting in.

Section II

Estimated Score Conversions


Scaled Score
>12
10-11

Raw Score
86 - 100
79-85

8-9
7 6 5
<4

65-78 59-64
54-58

48-53 0-47

Biology
Passage I (Questions 1-7)

Mechanics Of Blood Flow

Passage I

Flow can be used in three different contexts: (1) as flow, (2) as average velocity, and (3) as the linear velocity
of a small element of fluid. The total volume of fluid

Which of the following equations BEST represents blood flow for the entire vascular system acting as a
unit?

passing a given point per time is known as the flow rate. The flow at a given point divided by the cross-sectional area is known as the average velocity offlow. The linear velocity is the distance traveled by small volume of blood per unit time. The flow rate (Q) must remain constant throughout the entire cardiovascular system. Blood flow through a blood vessel results from a driving force in the form of a pressure gradient. The perfusion pressure along the length of any blood vessel is given by the differences between the pressure at the proximal and distal end of the vessel. The following relationship between blood flow and perfusion pressure
can be established:

A. B. C. D.

Q x TPR = (CVP-MAP) MAP x Q = (TPR - CVP) Q x TPR = (MAP - CVP) CVPxMAP = (Q-TPR)

The increased viscosity of whole blood relative to plasma is best explained by the contribution of:
A. albumin.

B.
C. D.

glucose.
white blood cells. red blood cells.

BloodFlow = Perfusion Pressure


Flow Resistance

The structure of systemic veins differ from the structure of systemic arteries in that only:
A. the inner surface of veins is lined with a single endothelial layer. arteries contain a component of elastic tissue. veins contain valves that favor one-way blood
flow.

The meanarterialpressure (MAP) is a time-weighted average of the arterial pressure over the entire heart cycle. The net flow resistance of the systemic loop can be defined as the total peripheral resistance (TPR) while the entire venous pressure (entire vascular system acting as one unit) is termed the central venous pressure (CVP).

B. C. D.

arteries contain a layer of smooth muscle.

The viscosity of a fluid is a measure of the internal work necessary to make a fluid flow. The viscosity of blood is determined by the composition of blood, the
nature of the vessel in which it is flowing, and the mean

The most likely reason that the central venous

pressure (CVP) is not an average value similar to the


MAP is because the:

flow velocity. Plasma has a viscosity of about 3, twice


that of water, while whole blood has a viscosity A. cross-sectional area of the veins is larger than
the arteries.

approximately three times that of water.

B.

venous pressure does not change significantly


over the heart cycle.

1.

Capillary walls provide the minimum possible


barrier to diffusion. Relative to the arteries and

C.

arterial pressure does change significantlyover


the heart cycle.

veins, the blood in capillaries has a:

D. A. B. C. low average velocity because of a high total


cross-sectional area.

arterial pressure rises rapidly with increased


volume while the venous pressure remains

within a few mmHg with the same increase in


volume.

low average velocity because of a low total


cross-sectional area.

high average velocity because of a high total


cross-sectional area.

In the classical method of measuring blood pressure,

D.
2.

high average velocity because of a low total


cross-sectional area.

pressure from a cuff is applied to the brachial artery in the arm to collapse the artery and prevent blood flow. The pressure of the cuff in this initial step
must exceed:
A. B. CVP. MAP.

The diameter of a uniform blood vessel is reduced

by a factor of 2. The resistance of this blood vessel


increases by a factor of:
A. B. C. D. 2.
4.

C. D.

arterial diastolic pressure. arterial systolic pressure.

8. 16.

Copyright by The Berkeley Review

HI

The Berkeley Review

Specializing in MCAT Preparation

Biology
Passage II (Questions 8-14)

Enzymes In Blood Clotting

Passage II

Figure 1. Blood Clotting Cascade Intrinsic Pathway

Kallikrein <^

I Prekallikrein
Extrinsic Pathway
t

Hemostasis is the process by which bleeding is arrested. At the time of injury, enucleated blood cells called platelets bind to the exposed subendothelium of the damaged blood vessel. As platelets begin to aggregate to one another, a platelet plug is formed. Vasoconstriction of the blood vessel is stimulated by the release of

thromboxane A2 and serotonin from the platelets. The purpose of the platelet plug is to provide a surface that allows for the activation of blood coagulation factors (Table 1), many of which are synthesized in the liver. Activation of these coagulation factors initiates the blood clotting cascade (Figure 1). The end result of this cascade is to form a blood clot (thrombus) that will seal the damaged blood vessel. Many of the coagulation factors are glycoproteins that can be classified as being pre-enzymatic. They exist in their inactive form as zymogens of serine proteases, proteolytic enzymes that have a common catalytic mechanism based on a highly reactive serine residue at

the active site. Once activated these serine proteases


enhance the coagulation cascade. The blood clotting cascade can follow either the

intrinsic pathway, initiated by contact of an appropriate surface, or the extrinsic pathway, initiated by tissue trauma. Both pathways converge at factor Xa, the
activated form of factor X. Factor Xa initiates the final

sequence of events in the common pathway that will lead


, Thrombin

to the formation of a clot.

y Factor^-^-^ XITJ >


Xffla

n <J?
S~~^
f ' Fibrin
(soft clot)

Fibrinogen

Fibrin \
(hard clot)

Table 1. Blood Coagulation Factors


Common Name

Vitamin K is essential for the synthesis of many of the coagulation factors, including prothrombin. The reduced form of this cofactor, vitamin K hydroquinone, is involved in a carboxylation reaction that converts many of the glutamate residues located in the N-terminal segment of these clotting factors into y-carboxyglutamate. Vitamin K hydroquinone is regenerated in a two-step reaction that
can be inhibited by antagonists like dicoumarol and
warfarin.

Factor
I
II

Half-life*
90-120
50-120 N/A N/A 12-24
2-6

Fibrinogen
Prothrombin"!"*

y-carboxyglutamate is an excellentchelator of Ca2+


and, following injury, acts as an anchoring mechanism between calcium-dependent coagulation factors and the phospholipid membranes of platelets. The functional significance of this event is that it brings specific factors together that aid in the formation of a clot, thereby accelerating clot formation many fold.
Defects in the blood clotting cascade may be assessed using screening tests. Partial thromboplastin time (PTT) screens the intrinsic and common pathways while prothrombin time (PT) screens the extrinsic and common pathways. A deficiency in factor XIII cannot be detected by using these two tests.

Tissue factor (thromboplastin)


Calcium
Proaccelerin

III
IV V
VII

Proconvertin**

Antihemophilic factor
Christmas factor t*
Stuart-Prower factor t*

VIII IX
X

10-12
18-30
25-60

Plasma thromboplastin antecedent* Hageman factor*


Fibrin-stabilizing factor
Fletcher factor*

XI
XII XIII

45-80 40-70

70-200
48-52

Prekallikrein

Ca2+, Phospholipid Membrane


f Dependent on Vitamin K

A Reported in hours

$ Undergoes activation to Serine Protease

Copyright by The Berkeley Review

112

The Berkeley Review Specializing in MCAT Preparation

Biology
8.

Enzymes In Blood Clotting

Passage II

Mature red blood cells obtain their energy from:


A. B. C. D. electron transport. oxidative phosphorylation. Krebs cycle. glycolysis.

12. Hemophilia A results from a deficiency of factor VIII while hemophilia B results from a deficiency of factor IX. Genes coding for both hemophilias reside on the X chromosome and are genetically
transmitted as a sex-linked recessive trait. What is

the probability that a woman whose father has hemophilia, and who marries a normal man, will
have an affected son?

9.

Glutamate is converted to y-carboxyglutamate in the vitamin K dependent reaction shown below:


h o I n n-c-c-n
I

A. B.

1.00 0.75

C. D.

0.50 0.25

CH,
I

CH2

coo

Glutamate

13.
Vitamin K ,,

Which of the following compounds, when added to stored whole blood, BEST prevents clotting?
A.
coo
CH,

Hydroquinone

Vitamin K _^ Quinone "

B.

I
HO C- COO CH, COO

~ H O I ll HjN-C-C- O
CH,
I *

y-Carboxyglutamate
ooc coo

CH2 I
coo

Citrate

Glutamate

The enzyme which BEST regenerates vitamin K hydroquinone is called a:


A. B.
C.

C.

D.
H O

decarboxylase. carboxylase.
reductase.

coo

II

HjN-C-C" O
CH2

CH,

D.

protease.

CH2
I
coo

0 M 0
OOC COO

Succinate

Y-Carboxyglutamate

10.

The only coagulation factor listed below that DOES


NOT require vitamin K is:
A. Factor II.

B.
C. D.

thromboplastin.
Christmas Factor. Factor X.
14.

The most common oral anticoagulant in use at the

present time is warfarin. Administration of this compound begins with a daily dose of about 5 mg and adjustments are made 3 days later to maintain a
PT between 1.2-1.5 times the control. This is most

11.

All of the following mechanisms will limit clot


growth EXCEPT:

likely attributed to a decrease in:


A. B.
C. D.

A.
B. C.
D.

consumption of leafy green vegetables.


removal from circulation by the liver. inhibitors of serine proteases.
blood flow dilution.

prothrombin. proconvertin.
Christmas factor. Stuart-Prower factor.

Copyright by The Berkeley Review

113

The Berkeley Review Specializing in MCAT Preparation

Biology
Passage DI (Questions 15-21)

Cardiac Output St Venous Return

Passage III

If there is an increase in the venous pressure, then during diastole there will be more filling of the ventricles. This
stretches the muscle fibers in the heart tissue and leads to

The blood returning to the right heart from the venous system must equal the blood leaving the left heart to the arterial system over any extended period of time. In a normal individual the venous return and the cardiac output
is about 5 L/min.

a greater force of contraction during systole. This is known as Starling's law of the heart, and simply stated says that an increase in venous pressure leads to an increase in cardiac output.

The vascular function curve in Figure 1 depicts a relationship between venous pressure and venous return.
As the heart contracts, blood is removed from the vessels

of the venous system and pumped into the vessels of the arterial system.

15. The component of the circulatory system with the LEAST total percentage of blood volume is the
network of:

A.
B. C.

capillaries.
arterioles. arteries.

D.

small veins and venules.

-4

12

16

Venous pressure (mmHg)

16. The pH of the blood passing through the pulmonary artery, compared to the pH of the blood passing
through the renal artery, is:

Figure 1

If the cardiac output and the venous return is increased, the pressure in the arterial system will increase while the pressure in the venous system will decrease. This is due
to the increased transfer of blood from the venous to the

A. B. C. D.

lower due to the loss of O2. higher due to the loss of O2. lower due to the accumulationof CO2. higher due to the accumulationof CO2.

arterial system.

17. Which of the following curves BEST represents an If the heart were stopped and the cardiac output was
reduced to zero, the pressures in the venous and arterial systems would soon equilibrate. However, the pressure increase in only arteriolar resistance to blood flow?
A.
9 10

B.

in the venous system will be at its highest value, simply


because the veins now contain more blood than when the
s
Normal

10
Normal

heart was contracting. This pressure is referred to as the mean circulatory pressure (MCP) and varies with sympathetic stimulation and blood volume.

The cardiac function curve in Figure 2 depicts a relationship between venous pressure and cardiac output.
-4 0 4 8 12 16 -4 0 4 8 12 16

Venous pressure (mmHg) C.


-? 10.,

Venous pressure (mmHg) D.


-= 10

8-

E
-4 0 4 8 12 16

6
4

Venous pressure (mmHg)

2-1 NormaP
0 -4 0 4 8 12 16 -4 0 4 8 12 16

Figure 2

Venous pressure (mmHg)

Venous pressure (mmHg)

Copyright by The Berkeley Review

114

The Berkeley Review

Specializing in MCAT Preparation

Biology

Cardiac Output St Venous Return

Passage ID

18. Changes in blood pressure or blood volume are detected by stretch receptors in the atria and arterial baroreceptors. Which of the following pairs of hormones act directly to increase blood volume?

20. Sympathetic nerve fibers are supplied to all parts of


the heart while parasympathetic nerve fibers are located primarily at the sinoatrial (SA) node and the atrioventricular (AV) node. The neurotransmitter released by sympathetic heart nerves is:

A. B. C. D.

Renin and vasopressin Angiotensin II and aldosterone Renin and angiotensin I Vasopressin and aldosterone

A.

norepinephrine, because it opens Na and

Ca^ channels in myocardial cells.


B. acetylcholine, because it opensK channels in
myocardial cells.

19. Both the cardiac output (CO.) from the heart and the venous return (V.R.) to the heart depend on venous pressure. This allows the curves in Figure 1 and Figure 2 of the passage to be combined into one graph as shown below:

C.

norepinephrine, because it opens K channels


in myocardial cells.

D.

acetylcholine, because it opens Na and Ca^


channels in myocardial cells.

21. All of the following compensatory mechanisms would be expected to occur during a hemorrhage
EXCEPT:

A.
-4 0 4 8 12 16

increased sympathetic discharge to the


arterioles.

Venous pressure (mmHg)

B.
C. D.

decreased parasympathetic discharge to the


heart.

Which of the following graphs BEST represents activation of the sympathetic nervous system? (Solid

increased cardiac output. decreased arterial pressure.

lines represent normal curves; dashed lines represent


changes.)
A. B.

-4

12

16

-4

12

16

Venous pressure (mmHg)

Venous pressure (mmHg)

C. "s 10 -i-

D.
10

12

16

12

16

Venous pressure (mmHg)

Venous pressure (mmHg)

Copyright by The Berkeley Review

115

The Berkeley Review

Specializing in MCAT Preparation

Biology

Circulatory Pressure, Area, Velocity, St Volume

Passage IV

Passage IV (Questions 22-28)


The total blood volume in an average human being is about 5.5 L. Blood flows through the circulatory system, which is divided into the systemic and pulmonary circulatory systems. The systemic system supplies blood to all of the tissues of the body except the lungs. The pulmonary system supplies blood to the lungs.
The passage of blood through the circulatory system can be examined in terms of differences in the pressure, cross-sectional area, velocity, and total blood volume in different segments of this closed system.
100 00

The pulmonary veins return the oxygenated blood to the left atrium of the heart which then passes it to the left
ventricle. Contraction of the left ventricle sends the blood

into the aorta under high pressure. Blood passes down the descending aorta and into the systemic arteries, arterioles, capillaries, venules, and veins before returning to the right atrium by way of the superior and inferior vena cava (the great veins).

22. The flow of blood throughout the body is primarily


regulated at the level of the:
A.

80 -

6 6

veins.

60 40 -

B.

capillaries
arterioles.
aorta.

C. D.

20 -

20

40

60

80

100

Total Blood Volume (%)


5000 <n 4000 3000 < 2000

23. Which point on the graph of pressure versus total blood volume BEST represents blood flowing through the pulmonary artery?
A. B.
I

C.
D.

m
IV

1000 -

20

40

60

80

100

Total Blood Volume (%)

24. Which point on the graph of area versus total blood volume BEST represents blood flowing through the
lungs?
A. B. C. D.
I
n

in
rv

20

40

60

80

Total Blood Volume (%)

25. Which point on the graph of velocity versus total blood volume BEST represents blood flowing through the vena cava?
the the the the
A. B. C.
D.

Blood returns to the right atrium of the heart from systemic system under low pressure and is passed to right ventricle. Contraction of this ventricle pushes blood into the pulmonary arteries where it flows to lungs.

II
m IV

Copyright The Berkeley Review

116

The Berkeley Review Specializing in MCAT Preparation

BlOlOgy

Circulatory Pressure, Area, Velocity, fif Volume

Passage IV

26. During circulation of the blood, turbulent flow could


increase with a decrease in the:

A. B.
C.

viscosity of the blood. velocity of the blood.


diameter of the blood vessels.

D.

Reynold's number.

27. In the circulatory system, the vascular distensibility is


the LEAST for:

A. B. C. D.

systemic veins. pulmonary veins. systemic arteries. pulmonary arteries.

28. Which of the following is NOT characteristic of the systemic and pulmonary capillaries of an average
adult human?

A. B. C. D.

The diameter of a capillary is the smallest of any of the vessels. The thickness of a capillary wall is the thinnest of any of the vessels. The capillaries contain between 5% to 10% of
the total blood volume.

The capillaries contain about 20% of the total


blood volume.

Copyright The Berkeley Review

117

The Berkeley Review Specializing in MCAT Preparation

Biology
Passage V (Questions 29-34)

Thoracic Cavity

Passage V

uuo

The thoracic cavity of the body protects the heart and lungs and is important in the mechanics of respiration. The thoracic cage is composed of the ribs, costal cartilages, sternum, vertebral column, primary and
associated musculature.

Curve II //^S/^
80-

S
o

X
60-

/
1 1 Curve I

The walls of the thoracic cage are constructed of the


ribs. These are curved bones that articulate with the

4003 c

vertebral column posteriorly and extend anteriorly and medially to articulate with the sternum. Out of the 12 pairs of ribs, only the upper six to seven pairs of ribs will articulate directly to the sternum. The remaining ribs will attach by costal cartilages. Spanning the distance between each rib are the (external and internal) intercostal muscles. Attaching to the most inferior ribs and costal cartilages, and extending posterior to the vertebral column, is a large, flat, domeshaped muscle called the diaphragm. These muscles, along with several of the abdominal muscles, are important in the expansion and compression of the thoracic cage, which is necessary for the function of respiration.

60

20-

O
0 1 1 1 1

20

40

60

80

100

Partial Pressure of Oxygen


Figure 1

100-1
"so
o
<L>

80-

Curve I
60-

Lining the internal wall of the thoracic cage is a sheet of tissue referred to as the parietal pleura. Lining the external surface of the lungs is a similar sheet of tissue called the visceral pleura. Between these two layers of tissue is the pleural cavity, a space containing a small amount of fluid and registering a negative atmospheric
pressure.

3 3
08

40-

00
c 0)

60
>> X

20-

The functional unit of the lungs is referred to as the alveolus, a small sac lined with epithelial cells. Each alveolus is surrounded by a network of capillaries. The walls of the alveoli and the capillaries are quite thin, allowing for maximal efficiency in gas exchange between
the two structures.

020 40 60 80
100

Partial Pressure of Oxygen


Figure 2

Red blood cells are the primary means of gas transport between the lungs and the tissues of the body. The protein hemoglobin, a component of red blood cells, binds oxygen at the level of the alveoli and transports this
gas by way of the circulatory system to the tissues of the body. Carbon dioxide, a waste product of metabolizing tissues is transported back to the lungs by the circulatory
system.

29. As the intercostal muscles lift the thoracic cage, how will pressure in the pleural cavity change?
A. B. C.
D.

The transport of both gases is dependent not only on their partial pressures but also on the temperature and pH of their environment. Figure 1 and Figure 2 represent a relationship between the partial pressure of oxygen and the percent oxygen saturation of hemoglobin.

No change to pleural pressure. Pleural pressure will become more negative. Pleural pressure will become more positive.
None of the above.

Copyright by The Berkeley Review

118

The Berkeley Review Specializing in MCAT Preparation

Biology

Thoracic Cavity

Passage V

30. A pneumothorax is the presence of gas in the pleural


space of the thoracic cavity. This condition leads to a collapsed lung. In order for a collapsed lung to
occur, which structure associated with the thoracic

cavity needs to be punctured?


A. Internal intercostal muscles

33. In the graph shown in Figure 1, curve I represents a relationship between the partial pressure of oxygen and the percent oxygen saturation of hemoglobin at a pH of 7.40 and a temperature of 38 C. If the pH of the blood is increased, curve II depicts the result. Which of the statements below explains this
phenomenon?

B.
C. D.

Parietal pleura
Ribs Heart

A rise in pH requires a lower partial pressure of O2 to bind a given amount of O2. II. A rise in pH requires a greater partial pressure of O2 to bind a given amount of O2. HI. A rise in pH requires a lower partial pressure of CO2 to bind a given amount of O2.
A. B. C. D. I only II only Ill only I and ffl only

I.

31. Several muscles are involved in respiratory function.


If the external intercostal muscles are non-functional,

which muscle group will take over the primary functions of normal respiration?
I. Abdominal muscles

II.
HI.

Diaphragm
Internal intercostals

A. B. C. D.

I and LQ only II only in only Normal respiration cannot occur.

34. In the graph shown in Figure 2, curve I represents the relationship between the partial pressure of oxygen and the percent oxygen saturation of hemoglobin at a pH of 7.40 and a temperature of 38 C. If the temperature of the blood is increased, curve II depicts the result. Which of the statements below explains this phenomenon?

I.
32. The main gases that are exchanged in the lungs are carbon dioxide and oxygen. This exchange depends to a large extent on the partial pressures of these gases (pC02 and p02) in the capillaries and the alveoli. Which situation is ideal for normal gas exchange between the capillaries and the alveoli of the lungs as blood enters the lungs from the pulmonary arteries?

A rise in temperature requires a lower partial pressure of O2 to bind a given amount O2. II. A rise in temperature requires a greater partial pressure of O2 to bind a given amount of O2. III. A rise in temperature requires a lower partial pressure of CO2 to bind a given amount of O2.
A. B. C. D. I only II only Ill only II and III only

A.
B.

High pC02 in the capillaries; low p02 in the


capillaries.

Low pC>2 in the capillaries; low pC02 in the


capillaries.

C.
D.

High pC02 pressure in the lungs; high pC>2 in


the lungs.

Low pC>2 in the lungs; high pC02 in the lungs.

Copyright by The Berkeley Review

119

The Berkeley Review Specializing in NCAT Preparation

Biology
Passage VI (Questions 35-40)

Electrocardiogram
36.

Passage VI

Potential differences within the heart tissue are

The phase of contraction of the ventricles is called systole. Based on the passage, to which part of the ECG does systole best correspond?
A. P

conducted to the surface of the skin because the body is a

good conductor of electricity. A reading of these patterns provides a graph representing the electrical activity of the heart, called an electrocardiogram (ECG). During each heartbeat cycle, 3 distinct wave patterns are produced: P, QRS, and T waves. The waves represent changes in potential occurring across heart tissue as the result of many action potentials inside the myocardial
cells.

B.
C.

QRS
T

D.

T-P phase

37.

Fibrillation

involves

random

contraction

of

myocardial fibers at different times. It can be


monitored with an ECG. What is the effect on an

The P wave represents the spread of depolarization through the atria. The QRS wave represents the spread of depolarization into the ventricles. The T wave represents the repolarization of the ventricles. The graph shown in Figure 1 indicates a standard cycle of these waves.

individual who experiences ventricular fibrillation?


A. More heartbeats than usual.

B. C.

Fewer heartbeats than usual. Unaffected.

D.

Death, if a normal rhythm is not introduced.

R
1.0
-

38.

Which group of pacemaker cells provides the trigger for depolarization in the right atrium and initiates the
P wave?

0.5

T P

A. B.

Sinoatrial node Atrioventricular node

C.

Bundle of His

0.0

D.

Purkinje fibers

qV
0.5

s
1 '

39.
400
600

Why is the body a good conductor of electricity?

200

A.
B.

Tissue fluids contain ions that participate in


conduction.

Milliseconds

Tissue fluids are free of ions that hamper


conduction.

Figure 1

C.

Tissue fluids contain ions that hamper


conduction.

D.

Tissue fluids are free of ions that participate in


conduction.

40.

The ECG can also be used to count the number of

35.

Which of the following conditions could be


monitored with an ECG?

heartbeats per minute (the cardiac rate). Which person would most likely have the slowest cardiac
rate at rest?

A. B. C.
D.

Blood pressure Cardiac output Arrhythmias


Hematocrit

A. B.

An infant A marathon runner

C.
D.

A weight lifter
A sedentary adult

Copyright by The Berkeley Review

120

The Berkeley Review

Specializing in MCAT Preparation

Biology
Passage VII (Questions 41-47)

Folate Experiment

Passage VII

42.

A microbiological assay using a folate-dependent


strain of Lactobacillus casei is used to determine

Currently, the recommended dietary allowance for the B-vitamin folate is 200 |ig/day for non-pregnant women. One group of scientists explored the adequacy of the 200 Hg/day recommendation in the following experiment.
Experiment 1

levels of folate in the plasma and in the red blood cells. What type of laboratory technique is used to
separate the plasma and the red blood cells?

A. B.

Extraction with an organic solvent Centrifugation

30 non-pregnant women were assigned to 3 different dietary groups: either a diet containing 200 Jig/day, 300 Hg/day, or 400 Jig/day of folate. They followed the
prescribed diet for 10 weeks. Levels of folate in the plasma and in the red blood cells (RBCs) were measured at the beginning of the experiment and at the conclusion.
Results of these measurements are shown in Table 1.
43.

C.
D.

Thin layer chromatography


Dialysis

How would a microbiological assay for folate be


performed using Lactobacillus casei ? A. The subject's sample is added to a known
amount of bacteria, incubated for a time

period, and the turbidity of the medium is


Dosage per Day
Initial Plasma Folate Initial RBC Final Plasma Folate Final
measured.

RBC Folate

B.

(M*>
200 300

Folate

(ng/ml)
3.6 3.5 3.6

(ng/ml)
444

(ng/ml)
3.1 3.3

(ng/ml)
387

The subject's sample is added to a known amount of bacteria along with a known amount of folate, incubated for a time period, and the
turbidity of the medium is measured.

C.

The subject's sample is added to a known


amount of bacteria, incubated for a time

450

423
610

400

449

6.7

D.

period, and the amount of folate produced is measured by an enzyme activity assay. The subject's sample is added to a known
amount of bacteria, incubated for a time

Table 1: Folate concentrations in plasma and RBCs before and


after folate diets.

period, and the number of mutated colonies is


counted.

If plasma folate drops below 3.0 ng/ml, then a folate

deficiency is indicated. Further deficiency will deplete


RBC stores and could lead to anemia.

44.

Based on the results in Table 1, what conclusion

would you make about the levels of folate these women were consuming in their diets before the study began? A. They were consuming between 300 and 400 Hg/day. They were consuming more than 400 Jig/day. They were consuming between 200 and 300 M-g/day. They were consuming less than 200 (ig/day.

41. In this study, the outcome variables of plasma folate


and RBC folate were chosen to reflect the status of

two different body pools of folate. Which body pool


does each reflect?

B. C. D.

A.

B.

C.

D.

Plasma folate represents folate intake over the past year, while RBC folate represents folate intake over the past day. RBC folate represents folate intake over the past year, while plasma folate represents folate intake over the past day. Plasma folate represents folate intake over the past day, while RBC folate represents folate intake over the past few months. RBC folate represents folate intake over the past day, while plasma folate represents folate intake over the past few months.

45.

When folate is deficient, red blood cell synthesis is dramatically affected. Folate is required for synthesis of the nucleotide thymidylate. Developing cells will enlarge too much before they are mature
and are released from the bone marrow. What type
of anemia is this called?

A. B. C. D.

Microcytic anemia Megacytic anemia Millicytic anemia Macrocytic anemia

Copyright by The Berkeley Review

121

The Berkeley Review Specializing in MCAT Preparation

Biology
pregnant women?
A.

Folate Experiment

Passage VII

46. What could you infer about the folate needs in


Folate needs are still about 200 Jig/day in
pregnancy.

B. C.
D.

Folate needs increase during pregnancy due to

synthesis of many new cells.


Folate needs decrease during pregnancy due to synthesis of folate by the placenta. Folate needs of the mother are met by liver
stores of folate, since folate is a fat-soluble
vitamin.

47. Which of the following structures indicates the pyrimidine nitrogenous base thymine?

A.

H,N

C.

D.

0'

Copyright by The Berkeley Review

122

The Berkeley Review

Specializing in MCAT Preparation

Biology
Passage VIII (Questions 48-54)

Measurement of Blood Pressure

Passage VIII

49.

arms,

If a person has a large amount of fat tissue on their how will this change the blood pressure

The first measurement of blood pressure, in the early


1700s by Stephen Hales, used a cannula inserted into the artery of a horse. The pressure of the blood was determined by the height it achieved in a vertical tube attached to the cannula. Although this is a direct

reading?

technique, we currently use an indirect technique based on arterial sounds to measure blood pressure.
A pressure cuff is wrapped around the arm and

A.. B. C. D.

The reading will be falsely high. The reading will be falsely low. The reading will be accurate. One cannot tell from this passage.

inflated, compressing and closing the brachial artery. Using a stethoscope, the reader listens for the beginning
of blood flow as the pressure is released from the cuff.

The tapping sounds she hears are spurts of blood, pushing through the partially obstructed artery with each heartbeat.
She also listens for the final sound and notes at what

50.

What is laminar flow?

A. B. C. D.

A fluid moves as a group of particles, each


with its own random course.

pressure the first and last sounds occur. As the artery


becomes completely opened, no sounds are heard. The first sound is recorded as the systolic pressure, and the

final sound is recorded as the diastolic pressure. An attached meter called a sphygmomanometer is used to read the pressures.
In Table 1 are some average arterial (mm/Hg) at various ages for men and women.
pressures

A fluid moves as a series of individual layers, each of the same velocity. A fluid moves as single particles, each with its
own random course.

A fluid moves as a series of individual layers,


each of a different velocity.

Systolic Systolic Age


20-24

Diastolic
Men
76

Diastolic Women
72
74

Men
123

Women
116 117

25-29
30-34

125

78
78

51. Gravity plays a role in reading blood pressure. For this reason, blood pressure measurements are conventionally made with the cuff approximately level to the heart. For a 46-year-old woman, to what value would the systolic pressure change if it were
measured at the level of the ankle? Assume the

126
127 129 130

120 124 127


131

75 78
80

35-39
40-44 45-49 50-54 55-59 60-64

80
81 82

ankle is 100 cm below the heart, and gravity increases blood pressure 0.8 mm/Hg per cm.
A. 162

82 84

135
138
142

137
139
144

83
84 85

84 85

B. C.
D.

210 211
313

Table 1

52. 48. As the pressure in the cuff is released, what does the first sound, read as the systolic pressure, indicate?
A. B. C.
D.

Astronauts on the space shuttle routinely participate in research experiments. Where should the blood pressure cuff be placed for a reading of blood pressure on an astronaut in space?
A. The cuff must be located in the conventional

The minimum pressure of the blood following


a heartbeat.

position at the level of the heart.


B.
C.

The beginning of laminar flow in the artery. The maximum pressure of the blood following
a heartbeat. The heartbeat itself.

The cuff should be located at the ankle, as far

as possible from the level of the heart.


The location of the cuff does not matter.

D.

The cuff will not work in an anti-gravity


situation.

Copyright by The Berkeley Review

123

The Berkeley Review Specializing in MCAT Preparation

Biology
blood flow turbulent?

Measurement of Blood Pressure

Passage vm

53. When during the blood pressure measurement is the


I. Before the systolic measurement

II.
III.

Between

the

systolic

and

diastolic

measurements

After the diastolic measurement

A. B. C.
D.

I only II only II and m only


I, II, and IH

54. The pulse pressure is the difference between the diastolic pressure and the systolic pressure. It canbe
used to calculate the mean arterial pressure (MAP) during the cardiac cycle.

MAP = diastolic pressure + (pulse pressure/3)


What is the MAP, on average, for a 27-year-old
man? A. B. C. D. 51

62 78 94

Copyright by The Berkeley Review

124

The Berkeley Review

Specializing in MCAT Preparation

Biology
Passage IX (Questions 55-60)

Aortic Compliance

Passage IX

55. Compliance is best represented by which of the


following relationships?
A. B. C. D. AD/D AV/AP AP/AD D/AD

To obtain curves shown in Figure 1, aortas were

obtained at autopsy from individuals in different age groups. Successive volumes of liquid were injected into
this closed elastic system and after each increment of

volume the internal pressure was measured. The graph can be used to extract information about theeffects of age
on aortic compliance.

56. The elastic modulus is most likely:


A. constant over a lifetime.

not related to aortic compliance. C. directly proportional to aortic compliance. D. inversely proportional to aortic compliance.

B.

57. According to Figure 1, which curve most likely


represents the oldest age group?
A. B. C. D. Curve A. Curve B. Curve D. Curve E.

58. As compliance diminishes, it is most likely thatpeak


arterial pressure occurs progressively:
50 100 150

Pressure (mmHg)

A. B.
C. D.

earlier in systole. later in systole.


earlier in diastole. later in diastole.

Figure 1

The effects of aging on the elastic modulus (Ep) is


shown in Figure 2. The elastic modulus is defined as AP/ (AD/D). AP is the aortic pulse pressure, D is the mean

59. According to Figure 1, aortic compliance in the


youngest individuals is:

aortic diameter during the cardiac cycle, and AD is the maximum change in aortic diameter during the cardiac cycle. The fractional change in diameter (AD/D) of the aorta during the cardiac cycle is a reflection of the change in volumeduring left ventricular contraction. Finally, the heart is unable to eject its stroke volume into a rigid arterial system as rapidly as into a more compliant system.

A.
B. C.
D.

smaller over all pressures when compared to


other age groups.

greater over all pressures when compared to


other age groups.

greatest at very high and low pressures and least over the usual range of pressure
variations.

least at very high and low pressures and greatest over the usual range of pressure
variations.

60. According to Figure 1, it can most likely be concluded that compliance:


A. increases with age, a increased arterial rigidity. increases with age, a decreased arterial rigidity. decreases with age, a increased arterial rigidity. decreases with age, a decreased arterial rigidity.
manifestation manifestation
manifestation

of of
of

B.
C.
0 20 40 60 80 100

Age (years)

D.

manifestation

of

Figure 2

Copyright by The Berkeley Review

125

The Berkeley Review

Specializing in MCAT Preparation

Biology
Passage X (Questions 61-66)

Heart Muscle Action Potentials

Passage X

Figure 3 represents the membrane potentials of a cell


from the sinoatrial node, which is located in the right
atrium of the heart.

Contraction of cardiac muscle is triggered by

depolarization of the plasma membrane. A typical ventricular action potential from a contractile myocardial
cell is illustrated in Figure 1. In order to bring about this

action potential, there must be inherent changes in the permeabilities of ions flowing into and outof themuscle
cell.

61. In passing from the right atrium to the right


ventricle, blood will have passed through the:
A. B. mitral valve. semilunar valve.

C. D.

tricuspid valve. pulmonary valve.

0.15

0.30

Time (seconds)

Figure 1

62. Figure 2 depicts the permeabilities of ions during the action potential in a myocardial contractile cell. Graph C most likely depicts the permeability of
which of the following ions?
A. B. Na K

Figure 2 illustrates the membrane permeability changes which are depicted by the action potential.
10.0

J5 &
1.0
a u

1i

C.

Ca2
Cle

0.1

-)
0

/VB

^-^C

D.

63. The capacity for spontaneous and rhythmic self0.15 0.30

excitation is best manifested by which region of the action potential shown in Figure 3?
A. B. C. D. Region Region Region Region A. B. C. D.

Time (seconds)

Figure 2

Special cells located in the heart itself are responsible for the heart's spontaneous and rhythmic self-contraction. These specialized cells of the sinoatrial (SA) node have significant differences in their membranepotentials when compared to contractile myocardial cells described in
Figure 1.

64. A membrane potential plateau region, similar to the one shown in Figure 1, is best explained by:
A. inactivation of Na channels.

B.
C.

D.
Time (seconds)

a significant increase in the permeabilityof the membrane to Ca2. the flow of positive ions out of the cell equaling the flow of positive ions into the cell. both Na and K channels becoming
inactivated.

Figure 3

Copyright by The Berkeley Review

126

The Berkeley Review

Specializing in MCAT Preparation

Biology
65.

Heart Muscle Action Potentials

Passage X

The action potential initiated in the SA node spreads


throughout the right atrium from contractile cell to

contractile cell. The means by which this spread takes place is most likely through:
A.
B.

tight junctions between cells.


neurotransmitter communication between
cells.

C.

gap junctions between cells.

D.

desmosomes, which hold adjacent cardiac cells


together.

66.

The following graph depicts three different action potentials from a SA nodal cell. Which of the

following statements is most likely true of the


graph?
A = control

-50

Time (seconds)
A.

Graph B is the result of acetylcholine release


onto the heart muscle.

B. C.
D.

Graph B is the result of vagus nerve


stimulation.

Graph C is the result of norepinephrine release


onto the heart muscle.

Graph C is the result of vagus nerve


stimulation.

Copyright by The Berkeley Review

127

The Berkeley Review Specializing in MCAT Preparation

Biology
Passage XI (Questions 67-72)

Capillary Filtration
67.

Passage XI

An increase in intracapillary hydrostatic pressure


would:

The direction and magnitude of water movement

across a capillary wall are determined by the algebraic sum of the hydrostatic pressure and osmotic pressure that
exist across that membrane (Figure 1).

A. B.
C.
D.

increase the concentration of osmotically active particles within the vessel. result from a decrease in the arterial pressure.
favor movement of fluid from vessel to

interstitial space.
increase the venous resistance.
Filtration

jttMSSr
Reabsorption

68.

33 mmHg

13 mmHg
Lumen of Capillary
Interstitial Space
Venule End

Studies using plastic capsules implanted in the subcutaneous tissue indicate a Pj of -1 to -7 mmHg.
If such values are accurate, which of the following could most likely be concluded?

\
Arteriole End

Figure 1

A. B. C. D.

Pc-Pi>Pc. Pc-Pj<Pc. Pj-Pc>Pi. Pc-Pi<Pi-

The hydrostatic pressuregradientacross the wall of a capillary, Pc, depends on the arterial pressure (35 mmHg), the venous pressure (15 mmHg), and the interstitial fluid pressure, Pj, which is about 2 mmHg. Precapillary and post capillary resistances are also important. A reduction
in the arterial or venous pressure reduces capillary hydrostatic pressure. The interstitial fluid pressure

69.

A reduction in the diameter of a precapillary vessel


would:

opposes capillary filtration, and the difference between the two provides the driving force for filtration.
Important factors responsible for fluid retention in the capillariesare the osmotic pressureof the plasmaproteins,

A.

favor movement of fluid from the vessel to the

B.

interstitial space. result in a reduction of capillary hydrostatic


pressure.

7Cp, and the osmotic pressure ofthe interstitial proteins, 7tj.


The osmotic pressure exerted by these proteins is also called the colloid osmotic pressure or the oncotic
pressure.

C.

be equivalent to an increase in venous


resistance.

D.

significantly increase the concentration of osmotically active particles in the interstitial


space.

While this oncotic pressure is low compared to the total plasma osmotic pressure, it plays a major role in fluid reabsorption. This is due to the fact that the electrolytes that are responsible for the major fraction of plasma osmotic pressure are equal in concentration on
both sides of the endothelium.

70.

Fluid movement is most likely described by which

The protein albumin is dominant in determining plasma oncotic pressure. While albumin is relatively impermeable to thecapillary membrane, small amounts of the protein do leak into the interstitial fluid and create a
very small osmotic force.

of the following equations? Note: k is the filtration


constant for the capillary membrane.

A. B. C. D.

Fluid movement = k[(Pc + iz\) - (Pj + 7tp)]. Fluid movement = k[(Pc + Pj) - (7Cp - n\)]. Fluid movement = k[(Pc - Pi) + (rci + rcp)]. Fluid movement = k[(n\ + np) - (Pj + Pc)l-

Copyright by The Berkeley Review

128

The Berkeley Review

Specializing in MCAT Preparation

Biology

Capillary Filtration

Passage XI

71. Albumin exerts a greater osmotic force than can be accounted for solely on the basis of the number of molecules dissolved in plasma. Based on this
information, it can BEST be described that albumin:

A. B.

may be replaced by inert substances with no


effect.

dissolves into more than one protein molecule in plasma.

C.
D.

carries electrical charges at blood pH which


attracts various electrolytes. is permeable to the capillary endothelium.

72. Only a small percentage (2%) of the plasma flowing


through the vascular system is filtered, and of this, about 85% is reabsorbed in the capillaries and the
venules. The remaining 15% of fluid:

A.
B.

acts to increase intracapillary hydrostatic


pressure.

remains in the interstitial fluid.

C.
D.

returns to the arterial circulation via the

lymphatic system.
returns to the venous circulation via the

lymphatic system.

Copyright by The Berkeley Review

129

The Berkeley Review Specializing in MCAT Preparation

Biology
Passage XII (Questions 73-78)

Respiratory Calculations

Passage XII

74. A respiration physiologist measures a volume of air to be 1L at 0 degrees Celcius and 1 ATM. What will
be the volume of this air at 25 degrees Celcius and
1ATM?

The bronchial circulation is the nutritive supply to all

of the lung support structures. Such structures include airways, connective tissue, and pleura. The pulmonary
circulation is the nutritive supply to the alveolar walls.

The pulmonary capillaries form an extensive network


within the alveolar walls. The maximum capillary volume is about 200 ml, while the normal capillary volume at rest is about 70 ml. This capillary volume can

A. B.
C. D.

0.82 L 0.91 L 1.0 L 1.09 L

be increased by opening compressed or closed capillaries.


Such opening is callled recruitment.

For a given volume, the total gas pressure of all molecular species is the sum of the individual pressures. This law of partial pressures makes the assumption that gas molecules do not interact with each other. The dry room atmospheric pressure is given the value 760 mmHg.
This air contains 21% O2, 79% N2, and 0% CO2. Blood

75. The partial pressure of O2 in systemic arterial blood is normally 5-10 mmHg less than that in alveolar gas. The difference in partial pressure exists
because:

A. B.
C.

that leaves the. pulmonary capillaries has come into equilibrium with all of the alveolar gases. In addition, small quantities of venous blood from bronchial venules
and vessels from the heart join the pulmonary venous
outflow.

a fraction of O2 is directly dissolved into the pulmonary blood flow. a fraction of O2 is replaced by the partial pressure of water.
the venous blood from bronchial venules and

heart vessels contaminate the pulmonary


venous outflow.

D.

When air is inspired, it becomes saturated with water vapor at 37 degrees Celcius in the nose, throat, and
trachea. The source of the heat and water vapor are the
76.

the partial pressure of gases dissolved in liquid is not equal to the partial pressure of the gas phase at equilibrium.

pulmonary and bronchial blood flows. The water vapor exerts a mandatory partial pressure, with the PH2O = 47 mmHg. The total quantitiy of water in expired gas over a 24-hour period accounts for nearly one-half of the obligatory daily water loss from the body.
In the alveoli, some oxygen diffuses directly into the pulmonary capillary blood. In fact, 3.0 ml of O2 are dissolved in 1L of blood for a p02 = 90 mmHg. This reduces the fraction of O2 in alveolar gas to 0.143. Most oxygen binds to the protein hemoglobin, which is able to bind 1.34 ml 02/g, with the normal hemoglobin concentration in blood being 150 g/L of blood.

Pulmonary physiologists are often interested in only anhydrous gas volumes and partial pressures. Taking this into consideration, the partial pressure of oxygen in dry inspired air at the trachea is:
A.

B. C. D.

102 mmHg. 115 mmHg. 150 mmHg. 170 mmHg.

77.

Based on the information in the passage, the anhydrous partial pressure of oxygen in alveolar gas
is:
A.
B.

C. D.

102 mmHg. 115 mmHg. 150 mmHg. 170 mmHg.

73. Besides recruitment, capillary volume can be increased by enlarging open capillaries as their internal pressure rises. Such distension is most
likely caused by:
A. B. C. D. increased cardiac output. increased left atrial pressure. decreased left ventricular pressure. right heart failure.

78.

At an oxygen partial pressure of 90 mmHg, 97% of hemoglobin is fully saturated with oxygen. At 90 mmHg, which of the following is the BEST
calculation of the TOTAL concentration of oxygen

(ml O2/L) in the systemic arterial blood?


A. B. C. D. 145 ml 02/L. 195 ml O2/L. 197ml02/L. 200 ml O2/L.

Copyright by The Berkeley Review

130

The Berkeley Review

Specializing in MCAT Preparation

Biology
Passage XIII (Questions 79-85)

Aspirin

Passage XIII

Aspirin is a non-steroidal anti-inflammatory agent that works by inhibiting the initial enzyme in prostaglandin synthesis pathway, cyclooxygenase (CO). Aspirin
acetylates a serine residue in the active site of CO,

inactivating it. This makes aspirin a useful drug for modifying prostaglandin synthesis. The diagram in Figure 1 shows the pathways of
prostaglandin synthesis from the fatty acid, arachidonic
acid. 1
Leukotrienes

Days Following Aspirin Dose

Figure 2

Arachidonate

Cyclooxygenase

Prostaglandins

Thromboxanes
79.

Figure 1

Which of the following statements BEST describes what is happening in Figure 1? After aspirin treatment, platelets produce new
CO, but endothelial cells do not.
B.

Low-dose aspirin treatment is often used to reduce risk

of heart attack or stroke in people with a history of


cardiovascular disease. Aspirin lowers the chance of blood clots traveling through the vessels and obstructing blood flow through the coronary arteries of the heart or through the carotid arteries leading to the brain. Aspirin works by altering the ratio of various prostaglandins to
reduce clot formation.

D.

After aspirin treatment, endothelial cells produce new CO, but platelets do not. After aspirin treatment, both platelets and endothelial cells produce new CO. After aspirin treatment, neither platelets nor endothelial cells produce new CO.

Specifically, the important alteration is in the ratio of prostacyclin to thromboxane A2. Platelets, which have a life span of four days, are important in the clotting process. Platelets secrete thromboxane A2, which promotes platelet aggregation and vasoconstriction. The endothelial cells of the blood vessels secrete prostacyclin, which inhibits platelet aggregation and causes
vasodilation. The balance between the two causes

80.

How many degrees of unsaturation are present in the following structure, thromboxane B2?

appropriate clotting: a strong clot that does not cover an excessive area, and that permits unrestricted blood flow
around the clot.

Aspirin inactivates CO permanently in both platelets


and endothelial cells. Endothelial cells rapidly produce

more CO enzymatically, but platelets cannot manufacture


CO.

A. B.

C.

The model shown in Figure 2 indicates CO levels in platelets and vascular endothelial cells after the ingestion of 300 mg of aspirin:

D.

One degree of unsaturation. Two degrees of unsaturation. Three degrees of unsaturation. Four degrees of unsaturation.

Copyright by The Berkeley Review

131

The Berkeley Review Specializing in MCAT Preparation

Biology

Aspirin

Passage XIII

81. When aspirin modifies CO, inactivating it, which structure indicates the product amino acid residue?
B.
H

84.

Low-dose aspirin therapy is used every other day to prevent heart attacks in some people with heart disease. Why does this dosing regimen help?

A.
NH,-C-C-0I

A.
H O

The dosing regimen keeps platelet aggregation


low.

II

_ NH,

c-c-oCH,
I

B. C. D.

CH,
I O

'A CH, O

'A CH, 0

The dosing regimen keeps platelet aggregation high. The dosing regimen keeps endothelial cells proliferation low. The dosing regimen keeps endothelial cells proliferation high.

C.
H O I II NH,-C-C-0

D.

NH,-C-C-0
I

85.

Although the mechanisms behind the birth process


are not well understood, it is clear that

CH,
I

CH,
S

0 = P-0(
00

0 = P-0*
I

prostaglandins play an important role in initiating and promoting parturition. What does this indicate for late pregnancy (months 7-9)?

A.
B. C. D. 82. What symptoms would accompany an intake of aspirin that was too high?
I. Easy bruising. II. Prolonged bleeding time. HI. Rapid clotting.

Aspirin would probably stimulate an early


labor.

Aspirin would have no effect on pregnancy. Aspirin should not be used in late pregnancy. Aspirin would probably cause birth defects.

A. B. C.
D.

I only I and II only I and HI only


I, H, and HI

83.

The fatty acid molecule that is the precursor to prostaglandins and the substrate for cyclooxygenase
has the common name of arachidonic acid. What is the IUPAC name?

A. B. C. D.

5, 8,11, 6,9, 12, 6, 9, 12, 5, 8, 11,

14 arachidonic acid. 15-arachidonic acid. 15-eicosatetraenoic acid. 14 eicosatetraenoic acid.

Copyright by The Berkeley Review

132

The Berkeley Review Specializing in MCAT Preparation

Biology
Passage XTV (Questions 86-92)

Sickle Cell Anemia and HbF

Passage XIV

87.

In the blood cells of an adult heterozygote with the sickle cell trait, how are the hemoglobin types
distributed?
A. B.
C.

Sickle cell anemia is caused by a single substitution of

one amino acid on the P chain of the adult hemoglobin


(HbA) molecule. This change produces the form of hemoglobin called HbS. Under conditions of low blood partial pressure of oxygen (p02), HbS comes out of solution and forms a cross-linked crystalline structure
inside the red blood cell. This leads to the characteristic All cells contain both HbS and HbA.

D.

Some cells contain exclusively HbS, while others contain exclusively HbA. All cells contain HbA only. All cells contain HbS only.

sickle shape of the red blood cells. Complications can

arise because the sickled red blood cells cannot properly fold up enough to pass through small capillaries of the circulatory system. Tissues are deprived of some of their blood supply by blockages caused by sickled cells.
Homozygotes are severely affected by the sickling cells, but heterozygotes rarely experience sickling under normal oxygen levels. Heterozygotes may experience problems at high altitudes (such as in a depressurized plane) or under some types of anesthesia.
Certain drug treatments for sickle cell anemia focus on increasing levels of fetal hemoglobin (HbF), which does not sickle. The following chart shows types of Hb present at various ages:
a chain

88.

Treatment with hydroxyurea in homozygotes


promotes fewer crisis events. HbF is increased to

about 20% of the cell's hemoglobin content by


treatment with hydroxyurea. What beneficial role does HbF play inside the cell?
A. HbF binds and inactivates HbS inside the cell.

B. C.
D.

HbF promotes a lower oxygenation state inside


the cell.

HbF promotes a higher oxygenation state


inside the cell. HbF binds and activates HbA inside the cell.

89.

Sodium metabisulfate (Na20sS2) functions as an antioxidant in many medical preparations. It is transformed to sodium persulfate (Na20gS2) in its
role as an antioxidant. How could sodium metabisulfate be used to test for sickle cell trait

Birth

Gestation

Age
(months)

when combined with hemoglobin from red blood


cells?

(months)

Figure 1

A.

86.

Sickle cell anemia is usually diagnosed by 3 months of age, but not always at birth. This is most likely
due to:

B.

Sodium metabisulfate would promote reduction of the hemoglobin to a rusty color, which would identify affected individuals. Sodium metabisulfate would oxygenate hemoglobin from red blood cells, and the sickling pattern would be noted in affected
individuals.

A.

a decrease of HbF and an increase of HbA

C.

B.
C.

with age. persistent maternal hemoglobin until age 3


months.

D.

a decrease in HbF and an increase in HbS with


age.

Sodium metabisulfate would promote oxidation of the hemoglobin to a rusty color, which would identify affected individuals. Sodium metabisulfate would deoxygenate hemoglobin from red blood cells, and the sickling pattern would be noted in affected
individuals.

D.

persistent maternal antibodies to HbS until age


3 months.

Copyright by The Berkeley Review

133

The Berkeley Review Specializing in NCAT Preparation

Biology

Sickle Cell Anemia and HbF

Passage XIV

90. What would happen if an adult had too much HbF,


as is the case in some of the thalassemias, in which

too much of the y chain of hemoglobin is produced?


A. B.
C.
D.

Decreased delivery of oxygen to the tissues, compared to other adults. Increased delivery of oxygen to the tissues, compared to other adults.
The red blood cells would sickle, but at high p02 levels.
The red blood cells would sickle, but at low

p02 levels.

91. Which diagnostic technique would distinguish heterozygotes, homozygotes with sickle cell anemia, and wild-type homozygotes?
I. Restriction fragment length polymorphisms (RFLPs) on red blood cell DNA. II. Gel electrophoresis of hemoglobin samples
from the red blood cells.

in. Centrifugation of hemoglobin samples from


the red blood cells.

A. B. C.
D.

I only II only I and n only


I, n, and HI

92.

Which of the following statements is TRUE of HbF?


I. HbF does not bind 2,3-BPG.

II. HbF has a higher oxygen saturation at a given p02 than HbA. III. HbF contains two a and two P chains.
A. B. C.
D.

I only I and II only n and HI only


I, H, and m

Copyright by The Berkeley Review

134

The Berkeley Review

Specializing in MCAT Preparation

Biology
Passage XV (Questions 93-100)

Ventilation Regulation

Passage XV

A group of inspiratory neurons located in the medulla is responsible for controlling ventilation. Inputs to these neurons from both the peripheral and central chemoreceptors are important in regulating involuntary

Figure 2 shows the results of the same experiment with the variable being arterial pC02- The reflex involves the peripheral and central chemoreceptors and is closely tied to changes in the proton concentration.
An increased arterial pC02 will cause an increase in the arterial concentration of protons. The peripheral chemoreceptors are stimulated by this lowered pH. At the same time, the elevated arterial pC02 causes an increase
in the pC02 of extracellular brain fluid. This causes a rise

control of ventilation. The peripheral chemoreceptors


consist of two different bodies of receptors located in the carotid artery of the neck and in the arch of the aorta. The
nerve terminals are intimate with the arterial blood and

respond to changes in pC02 , p02, and H. Impulses


from these nerve terminals travel up afferent fibers toward the brainstem and eventually synapse with inspiratory neurons of the breathing center. The central chemoreceptors, located in the medulla, monitor the brain's extracellular fluid for changes in the concentration of protons. Fibers from this receptor synapse with inspiratory neurons.
~
c

in the proton concentration of the fluid, stimulating the


central chemoreceptors. It is the role of the central

chemoreceptor which is most important in mediating this


ventilation reflex.

30

I
S 20
c

93. To assess the effects of a changing arterial p02 on the ventilation rate accurately, which of the following conditions should exist?
A. The arterial pC02 should decrease at a
constant rate.

> B
s

10

B. C.
0 20 40 60 80 100

The arterial pC02 should increase at a constant


rate.

i
Arterialp02 (mmHg) Figure 1

D.

The arterial pC02 should be maintained at 40 mmHg. The arterial pC02 should be maintained at 46 mmHg.

Figure 1 shows the relationship between arterial p02 and the ventilation rate. The graph indicates that a lowering of the arterial p02 will stimulate the rate of ventilation. This reflex is mediated by the peripheral chemoreceptors which respond to a lower arterial p02 by increasing their rate of discharge to the breathing center.
16 r
c

94.

According to Figure 1, a drop of 30 mmHg from the normal resting value of p02 does NOT radically increase the ventilation rate. This occurs most likely
because the:

A. B.

6 12 -1

C. D.

pC02 is increasing. total amount of oxygen transported is relatively unaffected. body responds with a decreased use of oxygen. pC02 is decreasing.

i
c

8
95. The inspiration of carbon monoxide gas will result
4
Normal
in:

>

Resting
Level

A.

B.
40 44 48

Arterial pC02 (mmHg)


Figure 2

C.
D.

an unchanged arterial p02a lowered arterial p02an increased arterial p02an increased percentage of hemoglobin saturated with O2.

Copyright by The Berkeley Review

135

The Berkeley Review Specializing in MCAT Preparation

Biology
96.

Ventilation Regulation

Passage XV

According to Figure 1 and Figure 2, changes in


arterial pC02 are:

100. During times of sleep, the body's ventilation rate decreases more than the consumption of oxygen by
cells. The result of this situation is:

A.
B.

augmented by reflexes regulating ventilation to a greater degree than are equivalentchanges in


arterial p02augmented by reflexes regulating ventilation to a lesser degree than are equivalent changes in arterial p02-

A.
B. C.

C.

resisted by reflexes regulating ventilation to a greater degree than are equivalent changes in
arterial p02resisted by reflexes regulating ventilation to a lesser degree than are equivalent changes in arterial p02-

D.

D.

an increased arterial pC02 and a increased arterial p02an increased arterial pC02 and a decreased arterial p02a decreased arterial pC02 and an increased arterial p02a decreased arterial pC02 and a decreased arterial p02-

97.

Which of the following statements provides the BEST support for the type of ventilation found early
on in metabolic acidosis?

A.
B.

C.
D.

Hypoventilation is caused by increased neural output from the peripheral chemoreceptors. Hypoventilation is caused by increased neural output from the central chemoreceptors. Hyperventilation is caused by increased neural output from the peripheral chemoreceptors. Hyperventilation is caused by increased neural output from the central chemoreceptors.

98.

Which of the statements below is FALSE regarding pC02 during exercise?


A.
B.

The alveolar pC02 determines arterial pC02Alveolar ventilation increases.

C. D.

Venous pC02 increases. Arterial pC02 increases.

99.

In order to hold their breath for a longer duration, swimmers often hyperventilate immediately before competing in a swimming event. This activity can be dangerous because even though the:
A.

B.

low pC02 is permitting one to hold their breath, the exercise may lower the p02 to levels which may induce unconsciousness, low pC02 is permitting one to hold their breath, the exercise may raise the p02 to levels which may induce unconsciousness, high pC02 is permitting one to hold their breath, the exercise may lower the p02 to levels which may induce unconsciousness,

D.

high pC02 is permitting one to hold their


breath, the exercise may raise the p02 to levels which may induce unconsciousness.

Copyright by The Berkeley Review

136

The Berkeley Review

Specializing in MCAT Preparation

Biology
Passage 1(1 - 7) 1.

Heart St Lungs

Section II Answers

Mechanics of Blood Flow

A is correct, low average velocity because of a high total cross-sectional area. This is according to the continuity equation describing fluid How. This equation states that since blood flow must be equal throughout an entire closed system, the cross-sectional area times the average velocity equals the cross-sectional area times the velocity. The total cross-sectional area of all the capillaries is very large, the largest of all types of vessels seen in the cardiovascular system. Therefore, the blood flow through the capillaries will have a low average velocity because of
their high total cross-sectional area. The correct choice is A.

2.

D is correct, 16. The problem requires that we be familiar with the relationship between resistance of a vessel and

the radius. The resistance of the vessel varies inversely with the radius to the fourth power. Therefore, decreasing
the radius by a factor of 2 leads to an increase in the resistance by a factor of 16. The correct choice is D.

3.

C is correct, Q x TPR = (MAP - CVP). We are looking for an expression to represent blood How. The general
equation is given in the passage. Blood flow (Q) = Perfusion Pressure/Flow Resistance. We are asked about the

entire vascular unit. The perfusion pressure is therefore going to be the mean arterial pressure minus the central venous pressure. The resistance for the entire unit is given by the total peripheral resistance. Placing these values in
the formula and multiplying both sides by TPR will give the solution. The correct choice is C.

4.

D is correct, red blood cells. One can arrive at this answer by thinking about the composition of blood. Blood is made up of plasma and cells. The question is asking about the increase in viscosity seen in whole blood versus
plasma. Knowing the composition of blood leads one to believe that increase must come from the cellular

contribution. It then becomes a matter of WBCs or RBCs. At this point, one must be aware that there are millions of red blood cells in whole blood. In other words, there are many more red cell than white cells. Therefore, the
major contribution must come from the presence of red blood cells. The correct choice is D.

5.

C is correct, veins contain valves that favor one-way blood flow. We are required to draw on knowledge about the structure of blood vessels. Both arteries and veins have a layer of endothelial cells, a layer of elastic tissue, and a layer of smooth muscle. Therefore, we can eliminate answer choices A, B, and D. However, only veins contain one-way valves which prevent back flow of blood. One must understand that the pressure in the veins is not very large, and so there is not a tremendous amount of force to propel blood. When the blood is propelled (skeletal muscle contraction is one method of providing the force), we wish the blood to remain moving forward. The
presence of these one-way valves carry out this function. The correct choice is C.

6.

B is correct, venous pressure does not change significantly over the heart cycle. The MAP is an average figure because the pressure in the arterial system changes significantly over the heart cycle. It is very high right out of the aorta, but becomes smaller and smaller toward the capillaries. Therefore, we need to take an average value. The pressure in the veins does not change very dramatically over the entire heart cycle. The total pressure difference in the peripheral veins is usually 5 to 10 mmHg. Therefore, we do not need to lake an average value. The fact that the arterial pressure does change dramatically is true, but it is not proper rationale for trying to explain why we do not need to take an average for the veins. Furthermore the cross-sectional area and the pressure responses to increases in volume offer us no explanation as to why no average is taken. The average is not taken because the value remains
relatively constant. The correct choice is B.

7.

D is correct, arterial systolic pressure. We are told in the question that we need to use the pressure in the cuff to collapse the artery totally and prevent blood flow. Therefore, the pressure in the cuff must counter the maximum pressure offered by the artery. The maximum pressure in the artery will be represented by the arterial systolic pressure. Recall that systolic pressure is the maximum arterial blood pressure during cardiac cycle. To collapse the artery, the pressure in the cuff must exceed this value. The correct choice is D.

Passage II (8-14)
8.

Enzymes in Blood Clotting

D is correct, glycolysis. It is important to know which cells have organelles and which do not. It will also be important to know certain metabolic processes and where they take place. Even though we have not formally discussed metabolism yet, this question is designed to get you to think about cellular function. Mature red blood cells are enucleated eukaryotic cells. They do not have any organelles and they have only one membrane, the plasma membrane. As we will see in future discussions, the Krcbs cycle, electron transport, and oxidative phosphorylation are all associated (in eukaryotic cells) with mitochondria. Even though these three systems are responsible for the majority of energy generated in metabolism, they arc not found in red blood cells. However, red blood cells do require energy and they obtain that energy from glycolysis. The correct choice is D.
137

Copyright by The Berkeley Review

The Berkeley Review

Specializing in MCAT Preparation

Biology
9.

Heart & Lungs

Section II Answers

C is correct, reductase. The vitamin K dependent reaction, as written from glutamate to y-carboxyglutamate, is a carboxylation and would require a carboxylaseenzyme. If we wanted to go from y-carboxyglutamate to glutamate,
it would be a decarboxylation reaction, involving a decarboxylase enzyme. Proteases are proteolytic enzymes that catalyze the hydrolysis of peptide bonds.

Vitamin K

Vitamin K

Hydroquinone
H NH

Quinone
H 0
II

C-C-N CH,
I

:>

N-C-C-N
II I

r
C02
r>

CH,
I

CH,

CH
OOC COO

/-v

'
coo

Glutamate

y-Carboxyglutamate

In this reaction we are not degrading a protein. We are regenerating vitamin K hydroquinone, which is the reduced form of vitamin K. In order to go from the oxidized form (vitamin K quinone) to the reduced form, a reduction needs to occur. This reaction will be catalyzed by a reductase. The correct choice is C.

10.

B is correct, thromboplastin (Factor HI or Tissue factor). This question can be answered by looking directly at Table 1 and Figure 1 in the passage. In Table 1 the symbol "f" denotes dependency on vitamin K. All that is
needed is a correlation between the common names and the factor numbers. The correct choice is B.

11.

A is correct, consumption of leafy green vegetables. The major source of vitamin K is from one's diet, especially in leafy green vegetables. There are also intestinal bacteria that can synthesize vitamin K. Since vitamin K is being made available to be a cofactor in the conversion of glutamate to y-carboxyglutamate, we would expect an increase
in clot growth.

Since blood clots can break free from blood vessels and wreak havoc within the circulatory system (by causing heart attacks and strokes), their growth must be limited. Clot growth can be limited by inhibiting the serine proteases which act at various steps in the cascade. Since blood is dynamic (constantly moving), the areas near a clot are always being diluted with fresh blood. This helps to prevent a build up of cascade intermediates involved in the clotting process. The intermediates that are removed by the flow of bloodare transported to the liver where they are
removed and degraded. The correct choice is A.
12. C is correct, 0.50 (or 50%). Men have an X and a Y chromosome; women have two X chromosomes. We are told

in the question that the defect for both types of hemophilia (h) resides on the X chromosome and that it is recessive. The woman's father has hemophilia, which we can designatedas XnY. Since her mother is normal (XX) the woman must be a carrier (XXn) because she received one good X chromosome from her mother and one defective Xn chromosome from her father. This is shown in the Punnett square in Figure 1 below:
X"
X

X"

xxh

XXh

xxh

XX

XY

XY

XhY

XY

Figure 1

Figure 2

The woman (XXn) now marries a man who is normal (XY). The mode of inheritance of their offspring is indicated in Figure 2 above. There are two sons and two daughters. However, we care about only the sons. One will be normal, while the other will express the trait. Therefore, the probability that this woman will have an affected son is 0.50 or 50%. Even though we have not formally discussed simple Mendelian genetics yet, this question is designed to get you to think along those lines. The correct choice is C.
Copyright by The Berkeley Review
138

The Berkeley Review Specializing in MCAT Preparation

Biology
13.

Heart St Lungs

Section II Answers

A is correct, citrate. Calcium is an important cofactor in the cascade mechanism outlined in Figure 1 in the passage. This ion bears a +2 charge and can be chelated by certain compounds that bear negative charges. As stated in the passage the modified amino acid y-carboxyglutamate is an excellent chelatorof Ca2. When this amino acid is tied up in a protein, it presents two carboxyl groups to the calcium ion. Based on this we could assume that the more carboxyl groups on a molecule, the better the chelator. That is, the better the molecule is at sequestering calcium. If we follow this assumption, we can eliminate choice B first. Glutamate has two carboxylate groups. However, note the positively-charged amino group. This would tend to repel a positively-charged calcium ion. Not only that, but the positively-charged amino group will be tied up with the a-carboxyl group through electrostatic interactions. This will diminish the chelating ability of that molecule. We can eliminate choice D based on similar reasoning. The major difference between the last two choices is that choice C (succinate) has just two full carboxylate groups whereas choice A (citrate) has three full carboxylate groups. In this case, the more carboxyl groups, the better the
chelator, and the better the molecule is at preventing clotting in stored blood. The correct choice is A.

14.

B is correct, proconvertin (Factor VII). This question is designed to get you to think about a number of items at the same time. As stated in the passage, warfarin is an antagonist (i.e., a competitive inhibitor) of vitamin K. Now, you
need to ask yourself, "Which coagulation factors in the cascade are affected by inhibition of the use of vitamin K?" The factors which are affected (from Table 1) are those which arc dependent on vitamin K. Those factors turn out to be choices A-D in the answers (i.e., prothrombin, factor II; proconvertin, factor VII; Christmas factor, factor IX; Stuart-Prower factor, factor X). Since there is just one answer (and not four), we need to consider some more
information.

In the question it states that 3 days after oral administration, the PT (prothrombin time) is maintained at 1.2-1.5 limes the control. The PT screens the extrinsic and common pathways. Note that in the cascade shown in Figure 1, the extrinsic pathway includes factor VII and the common pathway includes factor X and factor II. The extrinsic pathway does not screen factor IX as it is part of the intrinsic pathway. This allows us to eliminate choice C
(Christmas factor, factor IX) as a possible solution. How do we distinguish between the remaining choices? Another way to look at this is to ask yourself, "Which single factor decreases so much after 3 days as to maintain a PT between 1.2-1.5 limes the control?" You should now be think about the half-life of these factors (see Table 1 in the passage).
The one factor which has the shortest half-life is factor VII (between 2-6 hours). Let's assume that its half-life is about 4 hours. In 3 days (72 hours) factor VII will have been reduced by 18 half-lives. The half-life of factor X is between 25-60 hours. Let's assume that its half-life is about 43 hours. Factor X will have been reduced by about 2 half-lives. Finally, factor II has a half-life between 50-120 hours. Let's assume that its half-life is about 85 hours. Factor II will have been reduced by about 1 half-life. [All values have been rounded up for simplicity.! Based on this analysis, we see that factor VII will have been decreased the most, because it has such a short life span. The
correct choice is B.

Passage III (15- 21)


15.

Cardiac Output & Venous Return

B is correct, arterioles. The answer to this queslion is not found in the passage but rather comes from our knowledge of the flow of blood through the circulatory system.

Roughly 64% of blood in the circulatory system is contained in the veins while about 15% is contained in the arteries. The veins include the large veins, small veins, and venules. The small veins and venules together contain about 25% of the total blood volume, leaving about 39% of the total blood volume found in the large veins. The arteries include the large arteries, small arteries, and arterioles. The large arteries and small arteries together contain about 13% of the total blood volume, leaving about 2% of the total blood volume found in the arterioles. About 5% of the blood is contained in the capillaries. The remaining 16% of the blood is contained in the heat and pulmonary vessels (associated with the lungs). This allows us to pick the arterioles as having the LEAST blood volume (at any given time) in the circulatory system. The correct choice is B.
16. C is correct, lower due to the accumulation of CCb- Recall that as blood passes through the tissues it gives up Cb

as a nutrient and picks up CO2 as a waste product. The blood that is leaving the pulmonary artery is coming from
the right ventricle, which in turn came from the right atrium, which in turn came from the venous system.
0

CCb + H<>0

f-hCO

HCO-,

+ H

Copyright by The Berkeley Review

139

The Berkeley Review Specializing in MCAT Preparation

Biology

Heart St Lungs

Section n Answers

Blood in the venous system is, for all practical purposes,deoxygenated blood. The oxygen had been given up at the level of the respiring tissues. The CO2 that was picked up by the venous system is either dissolved in the blood as
free CO2 (about 5%), bound to hemoglobin as the carbamate (about 5%), or in the form of the bicarbonate ion

(about 90%). When the bicarbonate ion is formed (Bohr reaction), hydrogen ions are produced which tend to lower
the pH of the blood. The correct choice is C.

17.

A is correct, (see the graph below). When there is a constriction of a blood vessel there is more resistance to blood flow. This leads to a greater pressure drop (i.e., a greater change in pressure, AP) between the veins and arteries. The result is a lower venous pressure. Therefore, the curve would be expected to drop along the y-axis.

The curve intersects at the samepoint on the x-axis because if the heart were to stop and the cardiacoutput reduced to zero, the pressures in the venous and arterial systems would soonequilibrate. In other words, the venous pressure
would not change. This makes choice A a good answer.
*3 10 i

-4

0-4

12

16

Venous pressure (mmHg)

Choice B indicates a decrease in the blood volume. When the blood volume decreases there is less blood to fill the

ventricles. This leads to less of a contractile force and a lower blood pressure. Choice C indicates decreased
arteriole resistance while choice D indicates and increase in blood volume. The correct choice is A.
18.

D is correct, vasopressin and aldosterone. When the blood volume is low, neural impulses form the atria and arterial baroreceptors aresent to the hypothalamus and vasopressin (antidiuretic hormone, ADH) is released into the blood. Also, if the blood volume is low, chances are that the blood osmolarity has increased. An increased
osmolarity is also detected at the level of the hypothalamus and this allows for the release of ADH as well. ADH

acts at the level of the late distal tubules and the collecting ducts in the kidney and directly promotes water
reabsorption (back into the blood). This increases the blood volume.

A low blood volume is usually associated with a low blood pressure. This is detected by the juxtaglomerular apparatus in the kidney and granular cells in that complex release renin. This enzyme converts angiotensinogen to angiotensin I, and then angiotensin I isconverted to angiotensin II by the angiotensin-converting enzyme. Not only does angiotensin II cause vasoconstriction of the arterioles (to help increase blood pressure), but it alsostimulates the release aldosterone from the cortex of the adrenal glands. Aldosterone stimulates the reabsorption of sodium at
the level of the kidney. Water follows down its concentration gradient into the blood and leads to an increase in
blood volume. The correct choice is D.
19.

C is correct, (see the graph below). The sympathetic nervous system is a component of the autonomic nervous system. Recall that the sympathetic nervous system is involved in thefight-or-flight response. Activation of the sympathetic nervous system will stimulate the heart and cause it to bea stronger pump. In other words, theactivity of theheart increases. This will cause the cardiac output (CO.) curve toshift to a higher level. Wesee this response
in choices A and C.

Oca

Oca

MCP

-4

12

16

12

16

Venous pressure (mmHg)


Choice A

Venous pressure (mmHg) Choice C

Copyright by The Berkeley Review

140

The Berkeley Review Specializing in MCAT Preparation

Biology

Heart St Lungs

Section II Answers

Activation of the sympathetic nervous system will also cause vasoconstriction of arteries, arterioles, and veins.

Contraction of venous smooth muscle will lead to a decrease in the diameter and increase in the pressure within the vessel. This will allow more blood to bereturned to the right heart (i.e., the right atrium and right ventricle) from the veins. The result isan increase invenous return and an increase inthe mean circulatory pressure.

Since cardiac output must equal venous return (see the first paragraph in the passage), we can equate the y-axis of our curve in the question not only tocardiac output but also to venous return. Therefore, we would expect the curve
to move upwards along the y-axis for venous return (V.R.) and outwards along the x-axis for an increase in mean
circulatory pressure (see above). The correct choice is C.

[Note: In the third paragraphof the passageit states that if the cardiac output andthevenous return is increased, the pressure in the arterial system will increase while the pressure in the venous system willdecrease. Be careful of what it means when it says "the pressure in the venous system will decrease." If we are at a venous return of 0 L/min, the venous pressureis about 8 mmHg (see Figure 1 in the passage, or below). If we now move to a venous return of 5 L/min, the venous pressure is about -4 mmHg. It is a decrease form the mean circulatory pressure
(MCP).

12

16

Venous pressure (mmHg)

In question 5 we have increased the MCP by constricting vessels. An increase in cardiac output due to sympathetic
stimulation will decrease the venous pressure relative to the new MCP.]

20.

A is correct, norepinephrine, because it opens Na and Ca^ channels in myocardial cells. This question ties ina
concept we examined when we discussed the nervous system. Recall that the preganglionic and postganglionic nerve fibers of the parasympathetic nervous system release the neurotransmitter acetylcholine (ACh). The preganglionic fibers of the sympathetic division release ACh while the postganglionic fibers release the neurotransmitter norepinephrine. We can now eliminate choices B and D.
Next, we need to recall that sympathetic fibers will stimulate the heart (think of the fight-orflight response). If the heart is to be stimulated, we would expect to see rapid depolarization of the myocardial cells. Depolarization of a

cell's membrane is due to an influx of Na into the cell. This allows a negative resting membrane potential to

become more positive. An influx of both Na and Ca^ into a cell will allow the resting membrane potential to become depolarized. Choice A looks likea good answer at the moment. What would happen if the K channels
were to open up? Potassium would leave the cell, making the resting membrane potential more negative. The cell would have a higher threshold potential and would not be depolarized as readily. We can eliminate choice C. The
correct choice is A.

21.

D is correct, decreased arterial pressure. The key word here is compensatory. How does the body handle a hemorrhage? The first thing that must be done is to prevent more loss of blood. Constriction of blood vessels would help, which means that an increased sympathetic discharge to the arterioles (choice A) is true. We still want the heart to pump blood. Therefore, we do not want to inhibit its action. We can remove inhibitory signals to the heart by decreasing the amount of parasympathetic discharge at the heart (choice B). After a blood loss the body still needs to maintain a reasonable cardiac output. Therefore, we would expect to see an increase in the cardiac output (choice C). What we would not expect to see is a decreased arterial pressure (choice D). Why? A decreased

arterial pressure is the RESULT of a hemorrhage. We want the COMPENSATION, which would be to increase the
arterial pressure. We can reason this out as outlined below.

When a hemorrhage takes place there is a blood loss and a decrease in the blood volume. The venous pressure,

venous return, and atrial pressure will all decrease accordingly. The volume of blood ejected by a ventricle in the heart (i.e., the stroke volume) during one heartbeat is decreasedas well. This leads to a decreased cardiac output and
a decreased arterial blood pressure.

Copyright by The Berkeley Review

141

The Berkeley Review


Specializing in MCAT Preparation

Biology

Heart St Lungs

Section II Answers

Located in the upper neck region are small arteries which branch off of the common carotid arteries. The common
carotid arteries stem from the aortic arch of the aorta, the main vessel that leaves the left ventricle of the heart. At

the fork where the small arteries leave the carotids is a region called the carotid sinus. The carotid sinus contains

nerve endings which are sensitive to the size of those small arteries. The carotid sinus acts as a baroreceptor, sensing the pressure differences within the small arteries. There are also baroreceptors in the aortic arch.
If there is a decrease in arterial blood pressure, the baroreceptors will sense this and compensate accordingly. As the blood pressure drops there is less expansion of the arterial walls and a decreased discharge of the nerve endings at the baroreceptors. The result is an increased sympathetic discharge to the arterioles (veins and heart) and a decreased parasympathetic discharge to the heart (because we do not want to inhibit its function). The parasympathetic and sympathetic divisions will act to increase heart rate and therefore increase cardiac output.
Sympathetic discharge to the arterioles and veins will constrict those vessels. Constriction of the arterioles will lead to an increase in the total peripheral resistance of that system, thereby increasing arterial pressure. This is the end result of the compensatory mechanism. Constriction of the veins will lead to an increase in the venous pressure, venous return, and cardiac output. An increased cardiac output leads to an increase in the arterial pressure. Again, this is the end result of the compensatory mechanism. The correct choice is D.

Passage IV (22 - 28)


22.

Circulatory Pressure, Area, Velocity, and Volume

C is correct, arterioles. Arterioles have strong muscular walls. When stimulated by the sympathetic nervous

system, they constrict. Metabolic products at the level of the arterioles, such as CCb, H, and Cb. regulate the
degree of constriction. The correct choice is C.
23.

C is correct, III. It is important to pay attention to the labels on the x-axis and y-axis. If we look at position I on the graph we note that the total blood volume is quite low, even though the pressure is quite high. Where is blood experiencing the highest pressure? As it is being ejected from the left ventricle. The pressure is going to decrease as the blood reaches the arteries and arterioles. Once the blood reaches the capillaries and the venules, the pressure of the blood will be approaching an even lower value as indicated by position II. As the blood begins to enter the veins, the pressure continues to decrease. As blood enters the right atrium, it is coming from the vena cava. This is represented by position III. Note that once the blood leaves the vena cava and enters the right atrium, it will then flow into the right ventricle. Contraction of the right ventricle increases the blood pressure and thus increases the velocity of the blood leaving the right ventricle as it flows into the pulmonary artery. Blood from the pulmonary artery will enter the lungs where the pressure drops to a rather low value. The correct choice is C.
D is correct. IV. Consider the x-axis (total blood volume) and the y-axis (area). The total area of the arteries will
be small (point I), and it will carry a small amount of blood volume. The total area of the venules and veins will be the largest (point II), and it will carry the largest blood volume. As we move from the veins to the vena cava and to

24.

the right atrium, right ventricle, and pulmonary artery, the total blood volume will again be minimal while the area will also be minimal (point III). However, as the blood leaves the pulmonary artery and enters the capillaries of the lungs, the total area of the vessels begins to increase as does the total blood volume (point IV). Note that the total blood volume in the lungs will be less than the total blood volume in the veins simply because (a) there are more veins in the systemic circulation than in the pulmonary circulation, and (b) the veins are quite distensible. The
correct choice is D.

25.

C is correct, III. If we look at position I on the graph, we note that the total blood volume is quite low even though
the velocity is quite high. Where is blood experiencing the highest velocity? As it is being ejected from the left
ventricle. The velocity is going to decrease as the blood reaches the arteries and arterioles. Once the blood reaches

the capillaries and the venules the velocity of the blood will be at its minimum value as indicated by position II. Blood then begins to enter the veins and the velocity begins to increase again due to muscular contraction squeezing the veins to help return the blood to the right atrium. As blood enters the right atrium it is coming from the vena cava. This is represented by position III. Note that once the blood leaves the vena cava and enters the right atrium, it will then flow into the right ventricle. Contraction of the right ventricle increases the blood pressure and thus increases the velocity of the blood leaving the right ventricle as it flows into the pulmonary artery. Thus position IV will be the velocity of the blood as it leaves the pulmonary artery and enters the capillary system of the lungs. The
correct choice is C.

26.

A is correct, viscosity of blood. Laminar blood flow is blood which flows at a steady rate through a blood vessel. Blood flow is said to be streamlined. Turbulent flow involves blood which is not flowing in a steady stream through
142

Copyright by The Berkeley Review

The Berkeley Review Specializing in MCAT Preparation

Biology

Heart St Lungs

Section II Answers

the blood vessel. Instead, turbulent flow creates eddy currents. These currents result from the blood passing over a rough surface in the vessel or encountering obstructions in the vessel such as a bifurcation in the path of blood flow (i.e., a splitting of the vessel itself). Turbulent flow, measured by the Reynold's number (Re), tends to increase as the velocity of the blood increases and as the diameter of the vessel increases. Turbulent flow is inversely proportional to the viscosity of blood. By using the equation Re = (v)(d)/(n/p), where v = velocity, d = diameter of
vessel, n = viscosity, and p = density, we find that as the Reynold's number increases, the turbulence of blood flow can increase. Usually turbulence will occur when the Reynold's number increases above 2000. The correct choice
is A.

27.

C is correct, systemic arteries. Arterial walls are stronger than the walls of veins. Blood vessel diameter increases
as the internal pressure within the blood vessel increases. This is because the vessels are distensible. Because arterial walls are much stronger than the walls of veins, the arterial walls are about 6 to 10 times less distensible than

the walls of veins. What this is saying is that for an increase in pressure there will be 6 to 10 times more blood in a vein as there will be in a comparable artery. Even though pulmonary arteries still have stronger walls than those of the pulmonary veins, pulmonary arteries are under less pressure (about 1/6 less) than systemic arteries. It turns out that pulmonary arteries have distensibilities about 1/2 that of veins. Even though the pulmonary arteries are still distensible, they are not as distensible as the pulmonary veins or systemic veins. The vascular distensibility would be least for the systemic arteries. The correct choice is C.
28. D is correct, The capillaries contain about 20% of the total blood volume. Capillaries are about 1 mm in length and, on the average, most cells are about 0.01 cm away from any given capillary. This allows for a highly efficient form of diffusion of 02 from a red blood cell within a capillary to the surrounding cells.
100
n.
U 1

~'i
60 40 20

"2.'

a'!

The value between the two

dashed lines represents the


total blood volume within

/ the capillaries.

:IlN
0 20

Oi K

y
40 60

" ,v
80
100

Total Blood Volume (%)

In order for this diffusion to be optimal, capillaries must have not only a very small diameter (about 8 urn, which is

just large enough for the passage of a red blood cell), but they must also have a very thin wall (about 0.5 urn). The thickness of the capillary wall is just one endothelial cell layer thick. Even though these values were not presented in the passage, it should be realized that gas exchange (for all practical purposes) occurs at the level of the capillary.
It would make sense for them to have the thinnest walls and the smallest diameter, to allow for maximal and

efficient diffusion. We can eliminate choices A and B, because they are characteristic of the systemic and pulmonary capillaries.

Capillaries in the systemic system contain about 5% of the total blood volume, while capillaries in the pulmonary system contain about 4% of the entire blood volume. This adds up to between 5% and 10% of the total blood volume, making choice C characteristic of the systemic and pulmonary capillaries and not a candidate for the best answer. Even though this value is not mentioned in the text of the passage, it can be obtained from the first graph in Figure 1 of the passage. In Figure 1 note that the dashed lines border the value of 20%. At first you might be
inclined to think that choice D, which states that total blood volume in the capillaries is about 20%, is a true statement, and therefore not the best answer. But think about what the x-axis is telling us. If we move out to 100%

total blood volume, does that mean that at that point in the curve a vessel contains 100% of the total blood volume? No, it does not. What the area between the borders of the dashed lines is telling us is that the total blood volume in

the capillaries is represented by what we see between those two dashed lines. In other words, the total blood volume in the capillaries is less than 10%of the total blood volume in the system. The correct choice is D.

Passage V (29 - 34)


29.

Thoracic Cavity

B is correct, pleural pressure will become more negative. The two lungs in the chest cavity are individually surrounded by a thin layer of cells called pleura. The visceral pleura attaches to the lung while the parietal pleura
143

Copyright by The Berkeley Review

The Berkeley Review

Specializing in MCAT Preparation

Biology

Heart St Lungs

Section II Answers

attaches to the diaphragm and interior region of the thoracic cage. Between the two pleural surfaces is a very thin layer of fluid called the intrapleural fluid. The volume of the intrapleural fluid is constant. Pressures in the respiratory system are listed relative to atmospheric pressure, which is 760 mmHg at sea level. Between inspiration and expiration the alveolar pressure is the same as atmospheric pressure. Since pressures are measured relative to atmospheric pressure, the alveolar pressure is said to be 0 mmHg. As stated in the passage the intrapleural pressure is negative relative to atmospheric pressure (about -4 mmHg). As an individual inspires the thoracic cavity begins to enlarge. The rib cage moves upward and outward. This is due to contraction of the external intercostal muscles. The diaphragm descends towards the abdomen. Both actions

move the wall of the thoracic cage away from the surface of the lungs. However, the parietal pleura is in contact with the intrapleural fluid which is in turn in contact with the visceral pleura. This intrapleural space increases in volume by a very small amount. As the volume of the intrapleural space increases (ever so slightly), the pressure within that space must decrease (Boyle's Law). In other words, the pressure within the intrapleural cavity will become more negative (relative to atmospheric pressure).

The same reasoning applies to the lungs. As the thoracic cage moves away from the walls of the lungs, the volume of the lungs increases. As the volume of the lungs increases, the pressure within the lungs must decrease. In other words, the alveolar pressure becomes subatmospheric and air flows from the outside into the lungs. The correct
choice is B.

30.

B is correct, parietal pleura. The parietal pleura and/or the visceral pleura are the most important layers of the
thoracic wall for maintaining the pressure component of the thoracic cage. A puncture of the intercostal muscles, ribs, or heart is not going to lead to a collapsed lung because these structures are outside the pleura. The correct
choice is B.

31.

B is correct, II only (diaphragm). The diaphragm is the primary muscle involved in normal respiration. The intercostal and abdominal muscles are execssory muscles and aid in respiration during times of forced expiration and
inhalation. The correct choice is B.

32.

A iscorrect, high pCCb in the capillaries; low Cb in the capillaries. As blood returns to the capillaries of the lungs from the systemic circulation via the pulmonary arteries, it will have a low pC>2 (deoxygenated blood) and a high pCCb. The partial pressures of these gases in the alveoli of the lungs will be reversed. This sets up a concentration
gradient that will allow oxygen to How from the alveoli to the capillaries and carbon dioxide to flow from the
capillaries to the alveoli. The correct choice is A.

33.

A is correct, a rise in pH requires a lower partial pressure of Cb to bind a given amount of O2. The graph shows that as the pH rises, the curve shifts to the left. This shift indicates that less of a partial pressure of oxygen is needed
in order to reach half saturation of hemoglobin with oxygen. As the blood becomes more alkaline (due to hyperventilation), hemoglobin shows a tendency to retain more oxygen (at lower partial pressures of oxygen). The
correct choice is A.

34.

B is correct, a rise in temperature requires a greater partial pressure ofO2 to bind a given amount ofO2. The graph
shows that as the temperature increases, the curve moves to the right. The shift to the right indicates that a higher
partial pressure of oxygen is needed to reach halfsaturation of hemoglobin with oxygen. The correct choice is B.

Passage VI (35 - 40)


35.

Electrocardiogram

C is correct, arrhythmias. Arrhythmia means an irregular heart beat or having an irregular rhythm. Blood pressure
is measured with a blood pressure cuff. Choice A is incorrect. Cardiac output is a measure of the volume of blood
pumped with each heartbeat. Choice B is incorrect. Hematocrit is a measure of the %RBCs in whole blood. It is used for checking on anemia. Choice D is incorrect. The correct choice is C.

36.

B is correct, QRS. P represents the depolarization of the atria. During depolarization, they arc contracting. Choice A is incorrect. QRS represents spread of depolarization through the ventricles. During this depolarization, they are contracting. Choice B is correct. During the T wave, the ventricles are relaxed and repolarizing. Choice C is
incorrect. There is no T-P phase, it is a fake answer. Choice D is incorrect. The correct choice is B.
D is correct, death, if a normal rhythm is not introduced. Ventricular fibrillation means the ventricles are not

37.

pumping together. The cells are contracting in their own unique patterns, producing an unproductive twitching.
Copyright by The Berkeley Review

144

The Berkeley Review

Specializing in MCAT Preparation

Biology

Heart St Lungs

Section II Answers

This can be fatal pretty quicklywithin minutes. The individual is definitely affected, therefore choice C is
incorrect. If there are no heartbeats at all, choices A and B arc incorrect. The correct choice is D.

38.

A is correct, sinoatrial node. All the choices are involved in conduction of action potentials in the heart, but only
the sinoatrial node is actually in the right atrium, as the question asks. The atrioventricular node is located on the septum between the atria and the ventricles. Choice B is incorrect. The bundle of His travels though the interventricular septum, and breaks into branches, forming the Purkinje fibers. Choices C and D are incorrect. The
correct choice is A.

39.

A is correct, tissue fluids contain ions that participate in conduction. Tissue fluids contain ions, such as sodium, potassium, bicarbonate, and chloride. Choices B and D are incorrect. Ions arc required for conduction. That is,
dcioni/.cd water does not conduct electricity. Choice C is incorrect. The correct choice is A.

40.

B is correct, a marathon runner. Exercise conditions the heart, so that greater volumes can be pumped per beat. This means the heart rate can slow and still maintain the same cardiac output as an untrained person. Infants have faster heart rales than adults. Choice A is incorrect. Weight lifting does not provide aerobic conditioning to strengthen the heart. Choice C is incorrect. A sedentary adult is also less conditioned than a trained athlete. Choice
D is incorrect. The correct choice is B.

Passage VII (41 -47)


41.

Folate Experiment

C is correct, plasma folate represents folate intake over the past day, while RBC lolate represent folate intake over the past few months. As stated in the passage, folate is a water-soluble vitamin. It is excreted by the kidney, so it is required every day. The serum contains folate eaten in the diet on that day. Choices A and D arc incorrect. RBCs live about 120 days, so their supply of folate represents the composite intake of folate over their lifetime of a few months, not a year. Choice B is incorrect. The correct choice is C.

42.

B is correct, ccntrifugation. Both extraction with an organic solvent and thin-layer chromatography would break the membrane of the RBC, mixing the plasma and the cellular contents. This would not be helpful. Choices A and C are incorrect. Dialysis tubing would contain the RBCs and probably the folate, too. This would not lead to a

separation. Choice D is incorrect. Finally, ccntrifugation is the perfect way to separate the heavy cells from the plasma. Following a short spin (10 minutes at 3000 rpm) the cells would sink to the bottom of the centrifugation tube and the plasma would move to the top, allowing separation for independent analysis of the two components.
The correct choice is B.

43.

A is correct, The subject's sample is added to a known amount of bacteria, incubated for a time period, and the

turbidity of the medium is measured. In this assay, the growth of the folate-dependent bacteria provide the
indication of the folate level present. The serum or RBC sample is added to a known amount of the bacteria in a

liquid medium, incubated, and the growth is measured by the turbidity (cloudiness) of the solution. The more growth, the more turbid the solution. No extra folate is added to confuse the results. This makes choice B incorrect. No further enzyme activity assay is required, the bacteria provide the enzymatic assay. And they do not produce folate. Choice C is incorrect on both these points. Choice D is trying to remind us of the Ames's test for mutagens.
It is also incorrect. The correct answer is A.

44.

Ais correct, they were consuming between 300 and 400 jig/day. Compare the initial levels with the final levels for each dosage group. Although all the women started with nearly identical plasma and RBC folate levels in each group, the 200 and 300 groups's levels fell. This means they were consuming less than their usual levels of folate.
This means choices C and D arc incorrect. However, the 400 group had a rise in both plasma and RBC folate levels.

This means the experiment provided more folate than their usual levels. Choice B is therefore incorrect. The
correct answer is A.

45.

D is correct, macrocytic anemia. The question tells you that the cells become loo large before they are released
from the bone marrow. The prefix "macro" refers lo large, while "micro" refers to small. Choice A is incorrect.
Choices B and C are made up words. The correct choice is D.

46.

B is correct, folate needs increase during pregnancy due to synthesis of many new cells. Since the experiment indicated 200 |ig/day was not adequate for nonpregnant women, and pregnant women are synthesizing more
nucleotides for the fetus, it makes sense that folate needs are increased during pregnancy. Choice A is incorrect.

The placenta docs not synthesize folate. Choice C is incorrect. Folate is a water-soluble vitamin, and is not stored
in the liver. Choice D is incorrect. The correct choice is B.

Copyright by The Berkeley Review

145

The Berkeley Review

Specializing in MCAT Preparation

Biology
47.

Heart St Lungs

Section II Answers

D is correct, thymine. First, make a distinction between purines and pyrimidines. Purines have 2 rings, while pyrimidines have 1. Choices A is the purine guanine while choice B is the purine adenine. This leaves us with the
structures shown in choice C and D. Is there any information the passage that will allow us to distinguish between these two nitrogenous bases? No. This question is designed to get us to eliminate as many of the possible choices as we can, and then to make an educated guess at the correct answer. The correct choice is D.

Passage VIII (48 - 54)


48.

Measurement of Blood Pressure

C is correct, the maximum pressure of the blood following a heartbeat. When the first sound is heard, the arterial

pressure is just overcoming the pressure of the inflated cuff. This is the maximum pressure the artery reaches. Choice A is incorrect. The laminar flow of blood is interrupted by the cuff. It does not return until the cuff is not compressing the arm at all. Choice B is incorrect. The heartbeat is not represented by taking blood pressure.
Pressure is measured. Choice D is incorrect. The correct choice is C.

49.

A is correct, the reading will be falsely high. If a cushion of fat absorbs someof the pressure from the cuff, then the cuff must be inflated more to compress the artery completely. This would mean that the reading of pressure needed to close the artery is falsely high. This would lead to an inflated blood pressure reading. The correct choice is A.

50.

D is correct, a fluid moves as a scries of individual layers, each of a different velocity. If you have notseen this in physics, try to figure out the answer based on vocabulary. A laminated countertop consists of a layer of plastic or other material glued onto a different material. Laminar refers to layers. Eliminate choice A. Since the question asks about layers (laminar) eliminate choice C as well, although atoms do have their own random activity. To get the
correct answer, think of water moving in a stream. It is slowest closest to the bank, where there is a lot of friction

with dirt and rocks. It is fastest in the center. Each layer moves with a different velocity. Choice B is incorrect.
The correct choice is D.

51.

men, 210 is your answer and is incorrect. Choice B is incorrect. If you read the diastolic column, 162 is your
answer. Choice A is incorrect. Choice D is just incorrect. The correct choice is C.

C is correct, 211. Read the table of blood pressures. The systolic pressure in a 46-ycar-old woman is 131 at the level of the heart. Add in the gravity factor (100 x0.8 = 80) and you get 131 + 80 = 211. If you read the column for

52.

C is correct, the location of the cuff does not matter. On Earth, gravity changes the reading from site to site on the
body. In space, since there is no gravity, this change will not occur. Choices A and B are incorrect. The cuff will function just fine, as will the heart, in an anti-gravity situation. Choice D is incorrect. The correct choice is C.

53.

B is correct, II only. Normally, arterial blood flow is laminar and smooth. It makes no sounds or vibrations.

Turbulent flow is noisy, just like in a stream. When the artery is slightly constricted by the cuff, the flow becomes turbulent and noisy. Before the systolic sound is heard, no blood is flowing through thecuff. Choice I is incorrect. After the diastolic sound, flow is silent and therefore laminar. Choice III is incorrect. Only during the interval
between the systolic and the diastolic sound can noises be detected, so that flow must be turbulent. Choice II is
correct. The correct choice is B.

54.

D is correct, 94. The calculation uses data from the table. A 27-year-old man. on average, would have a systolic pressure of 125 and a diastolic pressure of 78. The difference between the two readings is 125 - 78 = 47. The pulse
pressure is 47. MAP = 78 + (47/3) = 78 + 16 = 94. The correct choice is D.

Passage IX (55 - 60)


55.

Aortic Compliance

B is correct, AV/AP. Wc are looking for the best relationship for aortic compliance. Think about what "compliance" means. Compliance is the act of conforming. In this case, it is the conforming (stretch) of the aorta in response to successive injections of liquid. In the body blood is injected into the aorta from the heart which will cause a rise in pressure. The volume of the aorta will change, depending on the pressure. The compliance of the aorta is the ratio of the change in volume in response to a change in pressure. This relationship is denoted on the graph by the slopes of the lines, as the y-axis represents the volume and the x-axis represents pressure. Compliance
is the change in volume in response to a change in pressure. The correct choice is B.

56.

D is correct, inversely proportional to aortic compliance. Wc can arrive at this answer by the following logic: We
are told from the passage that the fractional change (AD/D) is a reflection of the change in volume of the aorta during left ventricular contraction. Therefore, one can say that (AD/D) can be replaced by AV. Making this

Copyright by The Berkeley Review

146

The Berkeley Review Specializing in MCAT Preparation

Biology

Heart St Lungs

Section II Answers

substitution in the equation for the elastic modulus, one sees that Ep = AP/AV. This is, of cours,c the inverse of compliance, so the elastic modulus is inversely related to the compliance. As the compliance increases, the elastic
modulus decreases. Looking at Figure 2, one sees that with age, the elastic modulus increases. This should clue

one to the idea that with age, compliance decreases. Nevertheless, the relationship is one of an inverse proportion.
The correct choice is D.

57.

D is correct, Curve E. In order to answer this question correctly, one must have made the conclusion that compliance decreases with age. From Figure 2 and the ideas in the passage, we can come to this conclusion. We have already established that in Figure 1; the compliance will be represented by the slopes of the lines. Each line represents a particular age group. Let us look at the normal pressure range of 80-120 mmHg. It is clear that Curve A has the highest slope, which represents the highest compliance. Wc arc looking for the oldest age group, which should have the lowest compliance (according to our conclusion). This is represented by the curve with the lowest slope which is best represented by curve E. The correct choice is D.
B is correct, later in systole. This question requires a careful reading of the passage. First, think about when the peak arterial pressure is going to occur. Is it going to occur during diastole or systole? The heart is contracting during systole, and that is when we shall see the most force and hence the peak arterial pressure. Therefore, we can eliminate the answers which claim that the peak arterial pressure will occur during diastole. Then the question becomes when will we sec the peak arterial pressure. The last sentence in the passage tells us that as compliance decreases, the heart is unable to eject its stroke volume as rapidly when compared to a more compliant system. If the heart is slowed at ejecting its stroke volume, it can be logically concluded that it will take progressively longer for the peak arterial pressure to occur. The correct choice is B. D is correct, least at very high and low pressures and greatest over the usual range of pressure variations. We can gather this information from looking at Figure 1. We should already be aware that Curve A represents the youngest age group. We simply need to look at slopes. It is clear that at certain points in the curve, the compliance of the youngest individuals is smaller than those of older individuals (this does not invalidate our general conclusion that

58.

59.

compliance decreases with age). On the other hand, it is clear that for the youngest individuals, their compliance is higher for most of the range of pressures. Therefore, when compared to the rest of the age groups, one cannot say that the compliance for the youngest individuals is either greater or less than for the entire range of pressures. One
can then eliminate choices A and B. We then look at choices C and D. Curve A is sigmoidal. The slopes are least

at the extreme pressures, and greatest during the normal range of pressures. This translates into the compliance for the youngest individuals is least at very high and low pressures, and greatest during the normal range of pressures.
The correct choice is D.

60.

C is correct, decreases with age, a manifestation of increased arterial rigidity. One should easily be able to
eliminate choices A and B. These claim that compliance increases with age, and Figure 1 clearly docs not support this claim. Therefore, we now look for a more likely manifestation of the decreased compliance. If the aorta does

not change its volume as well in response to a given change in pressure (less compliant), the arterial system is more likely to be rigid. The more rigid the aorta, the less it can conform, or comply with a given change in pressure. The increased rigidity is a result caused by progressive changes in the contents of collagen and elastin in the arterial walls. The increased rigidity with age results in a decreased compliance with age. The correct choice is C.

Passage X (61 - 66)


61.

Heart Muscle Action Potentials

C is correct, tricuspid valve. This question draws not on information from the passage, but from our own

knowledge of the anatomy of the heart. There are valves between the atria and the ventricles known as the
atrioventricular valves. The valve between the right atrium and the right ventricles has three cusps, making it the

tricuspid valve. When the right ventricles begins to contract, the valve between the two chambers closes and
prohibits the back flow of blood into the right atrium. The correct choice is C. 62. B is correct, K. We are looking at the permeability of potassium. As the cardiac action potential begins, the

permeability of potassium decreases as potassium channels close. This is the key to answering this question. Decreasing the permeability of potassium while increasing the permeability of sodium will result in depolarization of the membrane. During the plateau of the action potential, the potassium permeability stays below the resting value. Ultimately, repolarization does occur when the permeabilities of calcium, sodium, and potassium all return to their original state. As a note. Graph A represents the permeability of sodium, while Graph B represents the permeability of calcium. Because Graph C dips right as the depolarization begins, one can safely conclude that
Graph C represents the permeability of potassium. The correct choice is B.
The Berkeley Review

Copyright by The Berkeley Review

147

Specializing in MCAT Preparation

Biology
63.

Heart St Lungs

Section II Answers

A is correct, Region A. This region of the action potential is unlike a normal resting potential. Note that the resting potential of the SA node cell is not steady but instead manifests a slow depolarization. This gradual depolarization is known as a pacemaker potential. The pacemaker potential brings the membrane potential to threshold, at which point an action potential occurs. After the peak of the action potential, the potential repolarizes, and the gradual

repolarization begins over again. Thus, the capacity for spontaneous rhythmic self excitation is best manifested by
Region A of the action potential. The correct choice is A.
64.

C is correct, the flow of positive ionsout of the cell equaling the flow of positive ions into the cell. In myocardial cells, the original membrane depolarization causes voltage-gated calcium channels in the plasma membrane to open. This results in a flow of calcium ions down their electrochemical gradient into the cell. Because there is a delay in their opening, these are called slow channels. The flow of positive calcium ions into the cell, along with some sodium also entering through the slow channels just balances the flow of positive potassium charge out of the cell. This keeps the membrane depolarized at the plateau value. In other words, the flow of positive ions out of the cell equals the flow of positive ions into the cell. Do not be fooled by the other answers. While they may be true for certain aspects of the cardiac action potential, we are looking for the best reason to explain the fact that the membrane potential is essentially notchanging throughout the plateau region. The correct choice is C.

65.

C is correct, gapjunctions between cells. The wave of depolarization spreads from contractile cell to contractile

cell via gap junctions. Recall that gap junctions are channels which allow cells to share cytoplasm. In this way, the current responsible for depolarization in one cell can spread to another cell in a very quick manner. There are no
neurotransmitters involved, which greatly reduces the amount of time needed to cause contraction over the entire
chamber. The correct choice is C.
66.

sympathetic stimulation, while Graph C represents parasympathetic stimulation. Therefore, Graph C most likely
represents stimulation of the vagus nerve (a major parasympathetic fiber). The correct choice is D.

Graph C reaches threshold after quite some time. One could conclude that Graph Bwould most likely represent

Dis correct, Graph C is the result of vagus nerve stimulation. We know from the diagram that all three graphs are pacemaker potentials. The difference between the three graphs obviously lies in the slope at the beginning part of the potential. In Graph B, we see a large slope. In other words, we reach threshold very quickly. On The other hand,

Passage XI (67 - 72)


67.

Capillary Filtration

Cis correct, favor movement offluid from vessel to interstitial space. One needs to make the following conclusion
from the information in the passage. There are two forces which favor movement of fluid out of the vessel. These

forces are the intracapillary hydrostatic pressure and the interstitial fluid oncotic pressure. There are two forces which oppose movement of fluid out ofthe vessel. These are the plasma protein oncotic pressure and the interstitial
pressure would most likely favor movement of fluid from the vessel to the interstitial space. Such action will not affect the concentration of osmotically active molecules, and therefore we can eliminate choice A. Also, such action

fluid hydrostatic pressure. With this understanding, we can see that an increase in the intracapillary hydrostatic-

will not affect the venous resistance, and therefore we can eliminate choice D. From the passage we can eliminate choice B because we are told a decrease in the arterial pressure results in a decrease in the intracapillary pressure.
The correct choice is C.
68.

A is correct, Pc - Pj > Pc. According to the figure in the passage, filtration is the movement of fluid out of the
vessel. In addition, wc are told from the passage that the force of filtration comes from the difference between the

Pcand Pj. The force of filtration is thus Pc - Pj. According to the diagram, the Pc always has a positive value and is thus larger than the Pj. If Pj is a negative number, then the force of filtration will actually become larger than the capillary hydrostatic pressure. This is indicated by the equation in choice A. None of the other equations follows
this conclusion, and all of them can be eliminated. The correct choice is A.
69.

B is correct, result in a reduction of capillary hydrostatic pressure. We are reducing the diameter of a precapillary
vessel. This will increase the resistance of that vessel so blood will not flow through that vessel as well. The result of this increased resistance is that the capillary will not receive as much blood volume. In turn, this will result in a decrease in the capillary hydrostatic (blood) pressure. A reduction in blood volume will decrease the amount of force/ unit area placed on the walls of the capillary. A reduction in Pc will not favor movement of fluid from the vessel to the interstitial space, eliminating choice A. The result of this action will not be equivalent to an increasein venous resistance, but is exactly the opposite. Think about it! Finally, resistance changes will not affect the concentration of osmotically active particles in the interstitial space, so choice D can be eliminated. The correct
choice is B.

Copyright by The Berkeley Review

148

The Berkeley Review Specializing in MCAT Preparation

Biology
70.

Heart & Lungs

Section II Answers

A is correct, Fluid movement = k[(Pc + K\) - (Pj + np)]. This question requires us to put together knowledge gathered in the passage into an equation form. It brings us back to our discussion of forces. The forces moving water out of the vessel are the capillary hydrostatic pressure and the interstitial oncotic pressure. Therefore, we should see these two forces added together. This eliminates choices C and D. The two opposing forces, Pj and the
plasma protein oncotic pressure, should be added together because they move fluid in the same direction. This eliminates choice B and leaves us with choice A as the correct answer. We see the opposing forces subtracted from each other will result in fluid movement. If the algebraic sum is positive, filtration will occur, and if the sum is
negative, absorption will occur. The correct choice is A.

71.

C is correct, carries electrical charges at blood pH which attracts various electrolytes. The answer can be arrived at through a process of elimination. We are told that albumin exerts a greater osmotic force than can be accounted for solely on the basis of the number of molecules dissolved in the plasma. Therefore, there is something special about albumin. It is not very likely that it could be replaced by an inert molecule and have noeffect on the plasma protein oncotic pressure. Therefore, we can eliminate choice A. Consider choice B. How many proteins do you know of thatdissolve into more than one protein? Proteins are not simple ionic salts. We are talking about a large protein. It will not dissolve into more than one molecule, making choice B an unlikely choice (be aware that this would increase its osmotic activity, if it happened). Choice D provides no explanation for the claim in the question. In
fact, choice D would allow us to predict a smaller plasma protein oncotic force. Therefore, the best answer is choice C. Nothing in the passage tells us that it does carry a charge, but that is not the point. One should be able to reason that this is the best answer given our knowledge. This is indeed the case. The protein carries negative charges, which attract primarily sodium ions. In addition, chloride ions attach themselves to the protein, which attract even more sodium ions. The additional electrolytes provide the increased osmotic strength not accounted for by the
concentration of plasma albumin. The correct choice is C.

72.

D is correct, returns to the venous circulation via the lymphatic system. Since the remaining fluid is picked up by the lymphatic system, we can eliminate choices A and B. The fluid will not remain in the interstitial space. If it did, edema would result. Furthermore, the fluid in the interstitial space is certainly not going to increase the intracapillary hydrostatic pressure. Having decided that the lymphatic system is going to pick up the fluid, one has to ask where is the fluid being returned to. Nothing in the passage tells us this, so one must draw on their own knowledge. The lymphatic system returns fluid into the venous system, not the arterial system. In fact, the lymph capillaries drain into larger vessels that finally enter the right and left subclavian veins at their junctions with the
respective internal jugular veins. The correct choice is D.

Passage XII (73 - 78)


73.

Respiratory Calculations

B is correct, increased left atrial pressure. We are looking for a situation which causes an increased volume of blood in the pulmonary capillaries. The increase in volume will cause the distension of the vessels. Increased left atrial pressure indicates that blood is filling the left atrium, but it is not being injected into the left ventricle. This may be due to left heart failure. If blood is not being ejected from the pulmonary atrium, the pressure will increase and blood from the pulmonary capillaries will not be able to move into the atrium. This will cause congestion and will increase the volume in the pulmonary capillaries, causing distension. All of the other answers act to keep blood away from or out of the pulmonary capillaries. The correct choice is B.
D is corrrect, 1.09 L. According to the ideal gas law, which states that PV= nRT, we find that:
P1V1 P2VT^
and that:

74.

V2 =Vip.

Pj/P2 = 1 and T9/T1 (in Kelvin) > 1. The only possible answer choice is D, because we are looking for an answer greater than one. The answer then becomes (1L)(298/273) = 1.09 L. The correct choice is D.
75. C is correct, the venous blood from bronchial venules and heart vessels contaminate the pulmonary venous outflow.

We are looking for an explanation to the decrease in O2 partial pressure. From the passage, we know that the bronchial venous flow along with vessels from the heart join with pulmonary venules. In effect, by increasing the volume of blood without changing the amount of oxygen, we are decreasing the concentration of oxygen in blood.
The correct choice is C.

Copyright by The Berkeley Review

149

The Berkeley Review Specializing in MCAT Preparation

Biology
76.

Heart St Lungs

Section II Answers

C is correct, 150 mmHg. The question asks for the partial pressure of oxygen in dry inspired air at the trachea. First, we need to know the fraction of oxygen in the air. We get this from the passage, and it is 21%. Note that in the alveolus the fraction of oxygen dips to 14.3%, but in the trachea it still is 21%. Next, we must get the total pressure of the dry air. The total pressure of the air is 760 mmHg, but 47 mmHg is due to the vapor pressure of H2O. The total dry air presssure is 760 - 47 = 713 mmHg. The partial pressure of oxygen in anhydrous air becomes 0.21 x 713 mmHg 150 mmHg. The correct choice is C.
A is correct, 102 mmHg. The total anhydrous pressure is 760 - 47 mmHg. The only difference is the fraction of oxygen found in the air. In alveolar gas, the value is 0.143 (from the passage). To find the partial pressure of oxygen, we simply multiply 0.143 x 713 mmHg ~ 102 mmHg. The correct choice is A.

77.

78.

C is correct, 197 ml ChfL. We are looking for the total concentration of oxygen in the systemic arterial blood. This includes oxygen found in hemoglobin and oxygen dissolved in the blood itself. Let us start with the latter. We know this value straight from the passage. The value is 3 ml O2/L. Now, let us determine the oxygen content for

hemoglobin. From the passage, we see that hemoglobin binds 1.34 ml 02/g. The concentration of hemoglobin is 150 g/L. We need to multiply these values. We get 1.34ml 02/g x 150g/L = 201 ml O2/L. The question states that only 97% of hemoglobin is saturated with oxygen, so let us do an approximate calculation. We multiply 0.97 x 200 ml O2/L. This gives us 194 ml O2/L. Adding the 3.0 ml O2/L that are dissolved in the plasma, we obtain 197 ml
0->/L. The correct choice is C.

Passage XIII (79 - 85) 79.

Aspirin

B is correct, after aspirin treatment, endothelial cells produce new CO, but platelets do not. From the passage, we
learned that the turnovertime of platelets is four days. Since the platelets in Figure 1 have the most active CO after 4 days, this means they cannot produce more when aspirin blocks CO. This eliminates choices A and C. However,

endothelial cells show a rapid rise in CO after the aspirin dose. These cells simply produce more COenzyme since
they have the machinery to do so. Choice D is incorrect. The correct choice is B.

80.

D is correct, four degrees of unsaturation. Alkanes, which are saturated with hydrogen atoms, have the empirical
formula CnH2n+2, where n represents the number of carbon atoms in the molecule. This formula tells us that for

every carbon atom in an alkane there will be twice as many hydrogen atoms plus two more. Hydrocarbons that are composed of a single ring or a single carbon-carbon double bond (i.e., an alkene) are missing two hydrogen atoms and have the empirical formula CnH2n. These molecules are said to have one degree of unsaturation. If a hydrocarbon contains a single carbon-carbon triple bond (i.e., an alkyne), then that molecule is said to have two

degrees of unsaturation and is represented by the empirical formula CnH2n-2. Atoms which are not part of a hydrocarbon can be treated as follows: halides like F, CI, Br, and I can be treated as hydrogen atoms; oxygen can be
treated as a carbon atom.

In thromboxane B2 we see that there are two carbon-carbon double bonds and one ring. Thisgives three degrees of unsaturation, making choice C the apparent answer. However, the carboxylate group (COO") has a carbon-oxygen
double bond (C=0) which adds another unit of unsaturation, giving a total of four units of unsaturation. The correct
choice is D. 81.

A is correct, acetylated serine residue. The passage tells us that a serine is acetylated to inactivate CO. Choices B and D indicate a cysteine residue, and are incorrect. Choice C is a phosphorylated serine residue, and is incorrect.
The correct choice is A.

82.

B is correct, I and II only. Too much aspirin would lead to poor clotting. This means that minor accidents could more easily lead to bruising, and bleeding time would be prolonged. Choices I and II are correct. A prolonged bleeding time means that the wound is slow to clot, not rapid. Choice III is incorrect. The correct choice is B.

83.

D is correct, 5, 8 II, 14-eicosatetraenoic acid. Start counting from the carboxyl end. There are twenty carbons, which requires the prefix "eicosa". There are 4 double bonds and the carboxyl group, giving the "tetraenoic acid"
150

Copyright by The Berkeley Review

The Berkeley Review Specializing in MCAT Preparation

Biology

Heart St Lungs

Section II Answers

part. Choices A and B are incorrect. The carbons are counted from the carboxyl end, giving 5, 8, 11, and 14 as the
positions for the double bonds. The correct choice is D.
84.

A is correct, the dosing regimen keeps platelet aggregation low. The passage said nothing about the proliferation (growth) of endothelial cells. Eliminate choices C and D right away. To prevent heart attacks in people with narrowed arteries from cardiovascular disease, it is often desirable to keep the blood from excessive clotting. This
means that platelet aggregation (clumping together) should be low. Choice B is incorrect. The correct choice is A.

85.

C is correct, aspirin should not be used in late pregnancy. Since aspirin inhibits the synthesis of prostaglandins by inactivating the first enzymatic step, it could possibly interfere with the normal prostaglandin cycles involved in
labor and delivery. This would probably not stimulate labor. Choices A and B are incorrect. Birth defects are initiated in the first few weeks of pregnancy. Choice D is incorrect. The correct choice is C.

Passage XIV (86 - 92)


86.

Sickle Cell Anemia

C is correct, a decrease in HbF and an increase in HbS with age. HbA is normal, nonsickling hemoglobin. An increase in HbA would not promote sickling of cells. Choice A is incorrect. The maternal hemoglobin is separate
from the fetal hemoglobin, and they do not mix. Choice B is incorrect. There are no antibodies to HbS, or that would lead to an even bigger obstruction problem. Choice D is incorrect. The correct choice is C.

87.

A is correct, all cells contain both HbS and HbA. The hemoglobin in mixed in each blood cell. There are no exclusive HbS-containing cells or HbA-containing cells. Choice B is incorrect. In a heterozygote, both HbS and HbA are present. Choices C and D are incorrect. The correct choice is A.

88.

C is correct, HbF promotes a higher oxygenation state inside the cell. Adult (HbA) and fetal (HbF) hemoglobins do not bind to each other. This is seen in question 1, in which sickle cell anemia is diagnosed only after fetal Hb levels
fall. We also know that adults who have only HbA do not sickle. Choices A and D are incorrect. The cells sickle under lower oxygen conditions. This means that a beneficial action is to raise oxygen levels inside the cell, so that the HbS does not fall out of solution and cause sickling. Since HbF is beneficial, it must promote a higher oxygen state inside the cell. Also, we know from our reading that HbF binds oxygen more avidly then HbA. This is how the oxygen gets transferred from the mother to the fetus. Choice B is incorrect. The correct choice is C.

89.

D is correct, sodium metabisulfate would deoxygenate hemoglobin from red blood cells and the sickling pattern would be noted in affected individuals. Forget about the rusty color answers. We are interested in the sickling phenomenon itself. Choices A and C are incorrect. The conversion of sodium metabisulfate to sodium persulfate involves the removal of oxygen from the other substrate. Since HbS crystallizes and causes sickling under lowered oxygen tension, this removal of oxygen would allow cells to sickle if the individual has the sickle cell trait. Hemoglobin would not be oxygenated under these conditions. Choice B is incorrect. The correct choice is D. A is correct, decreased delivery of oxygen to the tissues compared to other adults. HbF binds oxygen more tightly and therefore releases less oxygen to the tissues. This would be a problem in disease states in which HbF is really high, such as the thalassemias. Choice B is incorrect. This question has nothing to do with HbS, but rather it asks about HbF. HbF, as is stated in the passage, does not sickle. Eliminate choices C and D. The correct choice is A. C is correct, I and II only. A single base pair is changed to produce the HbS product. Since the DNA differs at least in this spot, specific restriction enzymes can used to cleave the DNA to produce a unique pattern of fragments. When fragments are separated by gel electrophoresis, the pattern of homozygotes (both sickle cell affected and wildtype) and the heterozygotes can be easily distinguished. Choice I is correct. The hemoglobin proteins will move differently during gel electrophoresis. The pattern of bands can also be used to distinguish the groups. Choice II is correct. Centrifugation is used to separated things of different masses or viscosities. The hemoglobin proteins are too similar to be separated this way for diagnosis. Choice III is incorrect. The correct choice is C. B is correct, I and II only. HbF does not bind 2,3-BPG. This is one feature that increases its oxygen saturation capacity. Choice I is correct. HbF does have a higher oxygen saturation at a given p02 than HbA. This is a mechanism that ensures the fetus gets oxygen preferentially from the mother. Choice II is true. HbF contains two alpha and two gamma chains. Choice III is incorrect. The correct choice is B.

90.

91.

92.

Passage XV (93 - 100)


93.

Ventilation Regulation

C is correct, the arterial pC02 should be maintained at 40 mmHg. We want to assess the effects of a changing arterial pOo on the ventilation rate. To do this, we should keep the pC02 at a constant value. This eliminates

Copyright by The Berkeley Review

151

The Berkeley Review Specializing in MCAT Preparation

Biology

Heart & Lungs

Section II Answers

choices A and B. The question becomes whether the pC02 is maintained at 40 or at 46 mmHg. We are interested in the effects of the arterial p02, where the pC02 is 40 mmHg. Therefore, to obtain the most accurate results, we should keep constant the pC02 at the value which can be found in the arterial circulation. The correct choice is C.

94.

B is correct, total amount of oxygen transported is relatively unaffected. The question tells us that a drop of 30 mmHg from the normal resting value of arterial p02 does not change the rate of ventilation. Recall the oxygen dissociation curve, which tells of the percent saturation of hemoglobin with oxygen depending on the partial pressure of oxygen. Remember that the curve is sigmoidal, with the top reaching an asymptotic value. One can descend from 100 mmHg of oxygen to 60 mmHg of oxygen and the percent of hemoglobin saturated with oxygen does not change dramatically. In other words, the amount of oxygen delivered to the body is relatively unaffected.
The correct choice is B.

95.

A is correct, an unchanged arterial p02- Carbon monoxide has a very strong affinity for the iron on the heme of

hemoglobin which is responsible for binding oxygen. For this reason, the number of hemoglobins able to transport oxygen is decreased. Does this decrease the arterial partial pressure of oxygen? No. The reason is that only the oxygen dissolved in the plasma contributes to the partial pressure. The oxygen on hemoglobin is bound and is thus not soluble in the fluid. For that reason, carbon monoxide will not affect the partial pressure of oxygen. The
correct choice is A.

96.

C is correct, resisted by reflexes regulating ventilation to a greater degree than are equivalent changes in arterial p02- This question is asking us to look at the graphs and ask ourselves the following question: Is our body more sensitive to changes in the partial pressure of oxygen or to changes in carbon dioxide? Looking at the curves, the
answer is clearly changes in the partial pressure of carbon dioxide. We also know that the reflexes discussed in the

passage are involved in resisting changes that occur to the partial pressure, in hopes of restoring values to normal. For that reason, changes in the arterial partial pressure of carbon dioxide are resisted by reflexes regulating
ventilation to a greaterdegree than are equivalent changes in arterial p02- The correct choice is C.

97.

C is correct, hyperventilation is caused by increased neural output from the peripheral chemoreceptors. In a
metabolic acidosis, we have an overproduction of protons from some metabolic condition, like excessive exercise.

These extra protons will stimulate both the central chemoreceptors and the peripheral chemoreceptors to increase
their output, and cause a rise in the rate of ventilation. Therefore, we can eliminate choices A and B, which call for

a hypoventilation. Now, while the rise in protons will stimulate both central and peripheral receptors over time, ask
yourself which one will be the primary receptor. Can protons simply cross over the blood-brain barrier to have access to the central receptors? The answer is no, because they are a charged species. Therefore, we can deduce that it is the peripheral receptors which are primarily responsible for the rise in the rate of ventilation. The correct
choice is C.

98.

D is correct, arterial pC02 increases. During exercise, it is clear that the cells will be using more oxygen and thus
producing morecarbon dioxide. For that reason, the levels of venous pC02 will increase. Will this lead to a rise in
the arterial partial pressure of carbon dioxide? The answer is no, because the rate of ventilation will increase. We

know that we breathe "harder" when we are exercising. In other words, the excess carbon dioxide:is released as a

result of the higher rate of ventilation. The increase in the carbon dioxide production is equivalent to the increase in breathing rate. The result of this is that the arterial partial pressure of carbon dioxide does not change, making D a
false statement. The correct choice is D.

99.

A is correct, low pC02 is permitting one to hold their breath, the exercise may lower the p02 to levels which may induce unconsciousness. Theswimmer's hyperventilation will result in a lowered partial pressure of carbon dioxide,
because they are eliminating a good deal of gas. During the race, that lowered level of carbon dioxide will allow them to hold their breath. However, the exercise will lower the level of oxygen. Recall the body is more sensitive to levels of carbon dioxide relative to oxygen. Forthat reason, the body does not know to breathe, but oxygen levels may be reaching dangerous lows. The level may become low enough to induce unconsciousness, as not enough
oxygen is reaching the brain. The correct choice is A.

100. B is correct, an increased arterial pC02 and a decreased arterial p02- This question is very straightforward. During
times of sleep, the body is not breathing as often. The end result of this is that we are bringing in less amount of oxygen, and eliminating lesser amounts of carbon dioxide. Thequestion tells us that the body's usage of oxygen per production of carbon dioxide is unchanged during this dormant period. Therefore, we should expect to see an
increased pC02 and a decreased p02- The correct choice is B.

Copyright byThe Berkeley Review

152

The Berkeley Review


Specializing in MCAT Preparation

Biology
Section III
Gastrointestinal
Tract and
A. The Gastrointestinal Tract

1. Nutrients and Digestion

B. The Kidney
1. Renal Function

2. Renal Physiology
3. Homeostatic Mechanisms

Kidney

Practice Passages and Answers

fEEKKELEY
Specializing in MCAT Preparation

Gastrointestinal Tract and Kidney


Top 10 Section Goals
Be familiar with the anatomy of the gastrointestinal tract.

Know how food passes from the mouth, down the esophagus, into the stomach, andthrough the
intestinalsystem, and how it is eliminated as feces.
Understand the interactions between the various gastrointestinal secretions.

Thefourgastrointestinal peptidehormones ofimportance aregastrin, cholecystokinin, secretin, and glucose-dependent insulinotropic peptide. Be aware oftheir functions.
Be familiar with the process of digestion and the areas of nutrient absorption.

intestine. Understand how peristalsis is relatedto the movementof chymem the system.

Know what types ofnutrients are broken down inthe stomach andwhattypes are degraded in the

Ofm Be familiar with the anatomyof the kidney.


^

jfir Understand how molecules are filtered at the glomerulus and which ones are reabsorbed from the
different parts ofthe nephron tubular system.
Be aware of the effect of the autonomic nervous system on the kidney. between vasoconstriction and vasodilation at the level of the glomerular arterioles.
Understand how the renin-angiotensin-aldosterone system functions.

Know how the kidney responds tosympathetic andparasympathetic stimulation. Know theeffects

Thissystemstimulates the reabsorption of Na in the distal convoluted tubule and in the collecting duct. Understand why this is important to the body

^|ft* Be aware of the different types of transport systems that line the tubules.

^Br Understand the differences between passive reabsorption (no energy required) and active reabsorption
^ (energy required) and generally what isreabsorbed atwhich points along the tubules.
how the kidney a arterial pressure. |m Understand there is a short-termresponds toanddecrease inadjustmentblooddecrease in arterial Be aware that adjustment a long-term for a

blood pressure. Understand how they work interms ofthe glomerular filtration rate.

()<!** Be familiar with diabetes mellitus and diabetes insipidus.

Akey feature ofdiabetes mellitus iselevated blood glucose levels due toinsulin problems, while a
key feature of diabetes insipidusis a vasopressin deficiency. Both lead to a largeurine loss.
Be familiar with some of the conditions brought about by renal failure.

The twomost life-threatening consequences ofrenal failure involve the retention ofK, due to poor tubular secretion of K, and metabolic acidosis, due to impropertubular secretion of H.

Biology

Gastrointestinal Tract St Kidney

Nutrients & Digestion

Gastrointestinal Tract
Nutrients & Digestion
Let's examine the gastrointestinal system. Digestion is the processby which food

that is eaten is broken down into progressively smaller particles and ultimately
absorbed by the intestinal tract. The gastrointestinal system includes the mouth

and associated salivary glands, esophagus, stomach, small intestine, large


intestine, and certain aspects of the liver and pancreas. In the average adult this

tract, running from mouth to anus, is about 30 feet in length and is actually an
extension of the external environment.

The food that we eat has a diverse composition. A typical meal might include macromolecules such as starch in the form of bread or potatoes, cellulose in the
form of a salad or other greens, protein in the form of meat or cheese, and fat in the form butter or ice cream. Each of these dietary categories needs to be degraded before they can be absorbed into the body. Starch and glycogen are both important polysaccharides in nature. Starch, characteristic of plant cells, and glycogen, characteristic of animal cells, are hydrolyzed within the digestive tract by enzymes called amylases. Upon hydrolysis both polymers release the monosaccharide glucose. Cellulose is

found within the cell walls of plants and is a polysaccharide consisting of glucose
residues linked together. The enzyme cellulase can hydrolyze cellulose into its constituent glucose residues. See Figure 3-1.
Amylase
Cellulose

Starch r.

=> Glucose

Cellulose

^ Glucose
An example of bacterial symbiosis

Figure 3-1 Enzymatic hydrolysis of starch and cellulose.

It is important to note that even though starch, glycogen, and cellulose all contain glucose residues linked together to form long polymers, we as humans cannot digest cellulose. The reason that we cannot digest cellulose is because the glucose residues in cellulose are linked together in a different configuration than the glucose residues in starch or glycogen. This special linkage can only be hydrolyzed only by the enzyme cellulasean enzyme that we do not have. Why, then, can cows eat grass? The only vertebrates that can utilize cellulose are the ruminant animals (like the cow and goat). The rumen of these vertebrates contains microorganisms which manufacture and secrete the cellulase that can breakdown the cellulose in the vegetation being consumed. This is an example of a symbiotic relationship between the host ruminant and the microorganisms living within the rumen. The microorganisms get a warm place to live and the ruminant gets energy in the form of digested cellulose. As we have mentioned in previous sections, proteins are composed of amino acids. Enzymes called proteases can hydrolyze proteins to their constituent amino acid residues. Fat cells or adipocytes store triacylglycerols (also called a
Copyright by The Berkeley Review
155

The Berkeley Review Specializing in MCAT Preparation

Biology

Gastrointestinal Tract St Kidney

Nutrients &Digestion

triglyceride but better known as just plain fat). Fats are hydrolyzed into fatty
acids and glycerol by the enzyme lipase. See Figure 3-2.

Proteins I

Protease _

[> Amino Acids

Fats I

Lipase

[> Glycerol

Fnrtv arirls

Figure 3-2 Enzymatic hydrolysis of proteins and fats.

Vitamins are needed in only small amounts in the diet yet they are essential in order to prevent certain nutritional deficiencies. [For example, the disease beriberi is due to a deficiency of vitamin B\ or thiamine.] As a group, vitamins participate in many different chemical reactions in the body. Inorganic minerals such as iron, potassium, calcium, and zinc are also important for proper development. Iron is the atom in the center of the heme group that carries oxygen in both hemoglobin and myoglobin, two proteins that transport oxygen within the body. All of the proteins that we are concerned with in the biological sciences are composed of a basic set of 20 different amino acids joined together in peptide linkages. Out of these 20 different amino acids we need to obtain 9 of them in our diet. The rest we can synthesize. In other words, we cannot synthesize certain
amino acids from our metabolic reactions and must therefore obtain them from

the food we eat. The amino acids that we cannot synthesize are referred to as being essential amino acids. Thus, the five general components to a complete diet are carbohydrates in the form of sugars like glucose, proteins, fats, vitamins,
and minerals.

The Gastrointestinal Tract

The gastrointestinal tract includes the mouth, esophagus, stomach, small intestines and large intestines. Let's consider a cross section of the small intestine and examine the tissue layers as we work outward from the lumen. The first barrier we encounter is a convoluted layer of epithelial cells (Figure 3-3a).
The convolution of this layer helps to increase the surface area of the gastrointestinal tract for absorption of nutrients. Scattered throughout this epithelial layer are ducts from external exocrine glands like the pancreas and liver (and the salivary glands in the oral cavity). Juxtaposed to many of the epithelial cells are endocrine cells which contain hormones that can be released into the blood. These hormones influence other cells in the gastrointestinal system. Both the parasympathetic and sympathetic nerves (of the autonomic nervous system) innervate the gastrointestinal system. The more important nerves stem from the parasympathetic system. Recall that the parasympathetic nerves are more active during times of relaxation and digestion while the sympathetic nerves are more active during times of "fight or flight."

As food is passed from the mouth to the lower portions of the gastrointestinal tract the smooth musculature surrounding the epithelial cells begin to contract in peristaltic waves. These waves (peristalsis) are not controlled by conscious thought but rather by the action of the parasympathetic division and by the action of hormones. Peristaltic action is rather quick because of the electrical

Copyright by The Berkeley Review

156

The Berkeley Review Specializing in MCAT Preparation

Biology
of the gastrointestinal tract.

Gastrointestinal Tract St Kidney

Nutrients & Digestion

continuityimposed by the gap junctionsin the circular and longitudinal muscles

(a)
Gastric Pits Blood Vessels

and/or Lymphatics
Epithelial
Cells

Circular

Muscle

Longitudinal
Muscle

Nerves to and Gland outside the

gastrointestinal tract
Figure 3-3 Cross section through the small intestines.

from the CNS

Smooth Muscle

Let's follow a bolus of food as it passes its way through the gastrointestinal tract. In the mouth the muscles of mastication move the jaw and the food is ground between the teeth. Salivary amylase is secreted into the oralcavity and begins to digest the starch and glycogen. This secretion is controlled by the
parasympathetic nerves.

Asthe food is swallowed and passed into the pharynx access to the nasal cavity is closed. A flap of tissue called the epiglottis covers the opening to the larynx and prevents food from entering into this passageway (to protect the airway).
Instead, the food passes into the esophagus and then down into the stomach. This swallowing reflex is controlled by centers in the medulla. Once the food reaches the stomach a sphincter (circular muscle) called the gastroesophageal sphincter contracts and prevents regurgitation of the food back into the esophagus. If this sphincter were not closed off, stomach acid would enter the esophagus and irritate the nerve endings in the smooth muscle.
This burning sensation is referred to as heartburn.

As the food enters into the stomach a littlemore digestive action takesplace. Two functions of the stomach is to break the food down into smaller particles and to detoxify it (by acidic secretions). There are four major types of secretion within the stomach. Mucus is secretedby surface cells and acts to protect the lining of
the stomach and lubricate the food. Gastrin, located in endocrine cells in the
Copyright by The Berkeley Review
157

The Berkeley Review

Specializing in MCAT Preparation

Biology

Gastrointestinal Tract fir: Kidney

Nutrients et Digestion

lower portion of the stomach, is secreted in response to protein enteringinto the


stomach. This hormone stimulates the secretion of HCl and pepsinogen. HCl, at

a pH of about 1,is secreted by parietal cells. Pepsinogen is secreted by thechief


cells. Parietal cells and chief cells are located in the gastric pits the line the

epithelium of the stomach (see Figure 3-3b). Pepsinogen is the inactive form of the peptidase enzyme pepsin. Peptidases, like pepsin, cleave peptide bonds. Since proteinis composed of amino acids and amino acidsare linked via peptide bonds, pepsin hydrolyzes proteins (at specific places in their amino acid sequence). Pepsinogen is converted to pepsin by the action of HCl. However, once pepsin is formed it can autocatalytically act on pepsinogen to form more
pepsin.

Besides secretingHCl the parietal cells also secrete a glycoprotein called intrinsic factor. This glycoprotein is important because it complexes with vitamin B12 and is then absorbed by the intestinal epithelial cells and transported by the
bloodstream. If there is a defect in the synthesis of intrinsic factor, vitamin B12 cannot be bound and therefore cannot be absorbed by the epithelial cells in the intestine. Vitamin B12 is important in erythrocyte (red blood cell) formation.
If too much acid is secreted into the stomach, ulcers can occur in the stomach and

in the small intestine (which is directly connected to the stomach). Ulcers are simply erosions of the walls of these two organs, and if they are extensive enough can cause bleeding. One of the most powerful stimulants that causes HCl
to be released into the lumen of the stomach is histamine. It turns out that a

compound called cimetidine (trade name is Tagamet) inhibits the binding of histamine to its receptor on the parietal cells. This reduces the amount of HCl
secreted in the lumen of the stomach. Cimetidine and its analogs are quite

commonly prescribed by doctors as a non-surgical treatment for ulcers.


As the dissolved food (referred to as chyme) passes from the lower stomach and into the small intestine more enzymatic activity takes place. This is partly due to distension of the stomach and small intestine and the generation of nerve impulses that stimulate enzymatic secretions. Roughly 90% of the digestion and

absorption that takes place in the gastrointestinal tract occurs in the small intestine. Not only do fluids enter the small intestine from the stomach but the pancreas and liver also add secretions as well. The small intestine acts to neutralize the acid which has been secreted by the stomach and to further digest
and absorb food particles.
Distension of the small intestine also causes the hormone cholecystokinin

(abbreviated as CCK) to be released from the intestinal mucosa. CCK diffuses by the way of the bloodstream to the pancreas where it causes the pancreas to

release digestive enzymes. The hormone secretin is released from the small intestine in response to the entering chyme from the stomach. Secretin is also absorbed by the blood and is transported to the pancreas where it causes the
release of bicarbonate ion and other fluids.

Thepancreas not only hasendocrine cells which secrete insulin and glucagon but it alsocontains secreting structurescalledacini that secretes a fluid into the small intestine which has a high bicarbonate content that is rather alkaline. The bicarbonate ions combine with the protons from HCl to produce carbonic acid
which is then converted into carbon dioxide and water. Carbon dioxide is

absorbed into the blood and transported to the lungs where it is expired. This

Copyright by The Berkeley Review

158

Specializing in MCAT Preparation

The Berkeley Review

Biology

Gastrointestinal Tract St Kidney

Nutrients & Digestion

mechanism increases the pH in the small intestine to a more alkaline value. See Figure 3-4.

Na+ + HC(V + H+ + CI"


H2C03 c
Figure 3-4
Removal of the bicarbonate ion.

HCO3" (bicarbonate ion) H2CO3 (carbonic acid)

=> COo + H90

Theliver has a diverseset of functions, one of which is to synthesize a compound called bile which it concentrates and stores in the gallbladder. The major pigment in bile is a compound calledbilirubin, which is a breakdown product of hemoglobin. Bile also contains bile salts which are important in the digestion and absorption of fats. Whenbileis released from the gallbladder it passesdown a duct that joins with the pancreatic duct, through a constriction called the sphincter of Oddi, and emptiesinto the smallintestine. Bile acts to emulsify fats (i.e., decrease their surface tension in order to break them up into smallersizes)
and it also helps the epithelial cells of the small intestine absorb those fats. The
presence of fats in the small intestine releases CCK which then acts on the

gallbladder and causes contraction, and on the sphincter of Oddi and causes relaxation, so the bile can pass into the lumen of the small intestine. [The greenish colorof the gallbladder is due to the variousbreakdownproducts found
in the concentrated bile.]
Apical

Basolateral
siae

side
Na+
Glucose
^

V +

Na+ ^- Na/Kpump
Glucose

Lumen

Blood

Figure 3-5
Transport systems.

Aswe have mentioned, the smallintestine is where the majority of absorption of fluid and nutrients takes place. Located on the apical and basolateral regions of the epithelial cells are specialized transport proteins ("carriers") responsible for
the absorption of sodium, chloride, sugars, amino acids, vitamins, and other such compounds. For example, consider the absorption of glucose into the epithelial cell. Glucose is only able to enter the cellif sodium is cotransported along with it.. As sodium enters the cell down its concentration gradient, it "drags" glucose into

the cytoplasm of the cell. Because the intracellular concentration of glucose increases it is able to diffuse out the basolateral side and eventually into the blood. As the concentration of sodium increases in the cell it is also pumped out
Copyright by The Berkeley Review
159

The Berkeley Review

Specializing in MCAT Preparation

Biology

Gastrointestinal Tract St Kidney

Nutrients 8t Digestion

on the basolateral side in exchange for potassium via the sodium-potassium pump. See Figure 3-5.

As the ions and nutrients are being absorbed by the epithelial cells of the small intestine, water is diffusing through the membrane. Water is trying to equilibrate
on both sides of the membrane by osmosis. Recall that cholera toxin causes

massive loss of fluid by diarrhea. In the majority of cases this can be treated by giving the individual large quantities of glucose and saline solutions.
How much fluid is being absorbed by the small intestine? Roughly 1.2 liters of water per day are taken in from the foods we eat. Another 7.0 liters are secreted by the stomach, pancreas, liver, small intestines and so on. Roughly 8.1 liters of
the total 8.2 liters is reabsorbed in the small intestine. Similarly, out of the

roughly 800 grams of solid food consumed, roughly 570 grams are absorbed in the small intestine. The remaining fluid and solid material is excreted as waste.
Recall that fats are degraded to fatty acids and glycerol by the action of the enzyme lipase. These compounds can diffuse into the intestinal epithelial cells where they are resynthesized into triglycerides (fats) and aggregate into
structures called chylomicrons. These aggregates are released at the basolateral membrane and into the extracellular space (Figure 3-6). The chylomicrons enter

into the lymph and are transported to the veins and eventually the tissues.

Fatty
acids

^
^

Fatty
acids

r-FATS
Glycerol

^^ Glycerol

To the lymph
and tissues

Figure 3-6 Release of chylomicrons.

The large intestine absorbs most of the water and ions that are left in the chyme as it passes from the small intestine. As the sodium and chloride ions are absorbed an osmotic gradient is established that allows for the absorption of
water. What is not absorbed is passed out the body in the feces.

Copyright by The Berkeley Review

160

The Berkeley Review

Specializing in MCAT Preparation

Biology

Gastrointestinal Tract St Kidneys

Renal Function

The Kidneys
Renal Function
Roughly 60% (a little less than 2/3) of an average 70-kg (154-lb) man's body
weight is water. Most of this fluid is located inside the cells (called intracellular

fluid) and about 1/3 of it is located in the interstitial spaces outside the cells (called the extracellular fluid). If you think about the organ systems which we
have previously discussed, you will realize that water can be lost from the

integument (the skin) by evaporation, the gastrointestinal tract, and the lungs.
Water can also be lost from the urinary system. However, the water that is lost

from the body by these four avenues exactly matches the water gained by the body through consumption of fluids and regulation at the level of the kidneys.
Recall that we have mentioned that osmolarity refers to the total solute

concentration of a solution. Remember, the higher the osmolarity of a given


solution, the lower will be the concentration of water in that solution. If an

organism can change the internal ionic concentration of its body fluids to meet that of the surrounding environment, they are referred to as osmoconformers.
Many marine invertebrates are osmoconformers. However, most vertebrates do

not change the internal ionic concentration of their body fluids to meet that of a surrounding environment. Organisms like this are referred to as osmoregulators. The difference between osmoconformers and osmoregulators can be seen on the
graph in Figure 3-7.

>>

Osmoregulator
Limited

Osmoregulator

Environmental osmolarity

Figure 3-7 The difference between osmoconformers and osmoregulators.

The vertebrate kidney presumably arose from freshwater teleosts (fishes). The blood osmolarity of a freshwater teleost is about 300 milliosmols/liter, which is greater than that of the surrounding aqueous environment. This means that the
concentration of water is less in the freshwater fish than it is in the water in

which the fish is swimming. Water from the environment will diffuse down its concentration gradient and into the fish. The excess water that is continually diffusing into the fish is filtered and removed by the kidneys and excreted in the
Copyright by The Berkeley Review
161

The Berkeley Review Specializing in MCAT Preparation

Biology

Gastrointestinal Tract &Kidneys

Renal Function

form of a dilute urine. Since copiousamounts of urine are excreted on an hourly basis the freshwater fish also loses important ions like sodium and chloride ions.
These ions are in rather low concentration in freshwater habitats and so the

freshwater teleost has developed (via evolution) gills for active transport of
sodium and chloride into its circulatory system. See Figure 3-8.
Water enters

by osmosis
Sallt loss

though gills

Copious
urine

Active transport of salt through gills

(salt and water loss)

Figure 3-8
Salt regulation.

Generalized Kidney Function


The kidneys in vertebrates like the freshwater teleost have three main functions. The first function is that of filtration. The functional unit in the kidney is the nephron which consists of a glomerulus (Latin, meaning "little ball"), Bowman's capsule, and a tubular system. The glomerulus is a collection of capillaries that receives blood from an artery terminating in the renal system. Blood is pumped into the glomerulus by the hydrostatic pressure of the heart and that pressure forces the blood through the capillary walls and into Bowman's capsule. The cellfree ultrafiltrate found in Bowman's capsule lacks many of the plasma proteins found in the blood. Blood plasma is simply a solution which is about 90% water that is composed of organic (e.g., proteins, sugars, amino acids, etc.) and inorganic substances (e.g., various ions like sodium and chloride). Within the blood plasma one also finds red blood cells, white blood cells, and platelets. The filtrate in Bowman's capsule is essentially the plasma minus the (large molecular weight) proteins. The second function of the kidney is reabsorption of important organic and

inorganic compounds from the filtrate in Bowman's capsule. Reabsorption occurs through many of the epithelial cells which line the tubular lumen of the nephron. The cilia of these epithelial cells move in such a way as to help propel the filtrate through these renal tubes. Glucose, small proteins, amino acids, salts, bicarbonate ions, and water are reabsorbed along this tubular system and transported back into the blood. The epithelial cells also have the ability to secrete protons, potassium, urea, uric acid, and ammonia. The third function of the kidney is
excretion of waste, salts, and excess water.

Copyright by The Berkeley Review

162

The Berkeley Review

Specializing in MCAT Preparation

Biology

Gastrointestinal Tract 6* Kidney

Renal Physiology

Renal Physiology
Both human kidneys sit on the dorsal side of the abdominal cavity and are beanshaped organs about the size of a fist. Blood from the descending aorta enters into the renal artery and eventually into the glomeruli of the nephrons. Blood leaves the kidney by way of the renal vein which itselfempties into the inferior
vena cava. Leaving each kidney is a ureter which transports the urine to the bladder. Urine exits the bladder by way of the urethra. These anatomical considerations are shown in Figure 3-9.
Inferior Vena Cava' Adrenal Gland ^

Abdominal Aorta

Renal Artery

Renal Vein

Left

Kidney

Bladder
Urethra

Figure 3-9 Anatomical position of the kidneys.

Each kidney contains more than a million nephrons. The glomerulus and Bowman's capsule of each nephron is located in the cortex and gives this portion of the kidney a granular appearance. In contrast, the striated appearance of the medulla is primarily due to a portion of the tubular system of the nephron called the loop of Henle and the collecting duct. The collecting ducts, which collect the urine, empty into the renal pelvis of the kidney and then into the ureter. This is
shown in Figure 3-10.

Copyright by The Berkeley Review

163

The Berkeley Review Specializing in MCAT Preparation

Biology

Gastrointestinal Tract St Kidney

Renal Physiology

Renal Pelvis

Nephron
Ureter

Figure 3-10
Kidney anatomy.

The Nephron
We mentioned that the functional unit of the kidney is the nephron. As the renal artery branches into smaller divisions and enters the kidney it becomes the
afferent arteriole. It is the afferent arteriole which enters into Bowman's capsule

and forms the capillary bed called the glomerulus. An efferent arteriole leaves the glomerulus and forms a capillary network which surrounds the renal tubules. The blood that leaves this capillary network does so by a venule which later empties into the renal vein that leaves the kidney. Extending from Bowman's capsule is a long tubular structure which is divided into the proximal convoluted tubule (PCT), loop of Henle, distal convoluted tubule (DCT), and the collecting duct. Many different nephrons can attach to a single collecting duct. These structures are shown in Figure 3-11.

The cardiac output of the heart is roughly 5 liters of blood every minute. Out of this 5 liters of blood roughly 20% of it (about 1/5) passes through the kidneys each minute. Thus, the volume of filtrate passing into Bowman's capsule will be rather large. Roughly 180 liters (47 gallons) of filtrate per day passes into all these capsules and of that about 1.0 to 1.5 liters enters into the urine. Depending on how much fluid you drank, the upper end of urine production might be about
3 liters per day.

The osmolarity of the filtrate in Bowman's capsule and in the proximal


convoluted tubule is about 300 milliosmols per liter. Recall that this is essentially the same osmolarity we found in the plasma. The concentration of the urine that is collected varies depending on the circumstance. If you wanted to excrete a dilute urine, then the concentration of the fluid leaving the collecting ducts should be lower than 300 milliosmols per liter. For example, a urine

concentration which is low might have an osmolarity which is 0.7 times that of
Copyright by The Berkeley Review
164

The Berkeley Review Specializing in MCAT Preparation

Biology

Gastrointestinal Tract & Kidney

Renal Physiology

theosmolarity of the plasma. In otherwords, theosmolarity of this "dilute" urine


might be 200milliosmols per liter. In contrast, a concentrated urine would have a

high osmolarity, something in the range of 4.2 times that of the osmolarity of
plasma (or about 1200 milliosmols per liter).
Bowman's

Glomerulus

Proximal Convoluted
Tubule

Dustal
Convoluted
Tubule

V Cortex

>
Capillary .
Network

Medulla

k
*

x-_ Collecting

J
Figure 3-11 The nephron and its related capillary system.

Consider the proximal convoluted tubule. The PCT is the obligatory section of the nephron because roughly 65% of all reabsorption and secretion occurs here. The epithelial cells in the PCT are quite metabolically active and support a variety of active transport processes. Glucose, small molecular weight proteins,
amino acids, and vitamins are completely (100%) reabsorbed in the PCT.

Roughly 80% of the Na, CI, and water are also reabsorbed in the PCT. Many
of the mechanisms for reabsorption involve sodium. For example, glucose and
sodium ions are reabsorbed together as are the amino acids and sodium ions.

Sodium ions are also reabsorbed with chloride ions. Little regulation occurs in
the PCT.

Copyright by The Berkeley Review

165

The Berkeley Review Specializing in MCAT Preparation

Biology

Gastrointestinal Tract St Kidney

Renal Physiology

As the PCTbegins to descend from the cortexinto the medulla it forms the loop of Henle. The loop of Henle has a thin portion and a thick portion (see Figure 311).The descending thin portion of the loop of Henle is very permeable to water but only relatively permeable to ions like sodium and molecules like urea. The ascending thin portion of the loop of Henle is much more permeable to urea but much less permeable to water than the thin descending portion of the loop.
As the loop of Henle ascendsit becomes thicker. The epithelial cells in this region of the loop actively transport ions like sodium and potassium from the lumen of the loop into the interstitial fluid. However, this region is impermeable to urea and water. This means that as the filtrate is passing up the thick portion of the loop of Henle it is becoming more dilute because of the fact that ions are being transported out into the interstitial fluid. Remember, the lower the osmolarity, the higher will be the concentrationof water (i.e., a more dilute fluid).
The filtrate from the thick portion of the loop of Henle passes into the distal convoluted tubule. The segment of the DCT closer to the loop of Henle is also quite impermeable to urea and water but rather permeable to ions like sodium. Therefore, as the filtrate passes through this segment of the DCT it will become a
bit more dilute.

The epithelial cells of the segment of the DCT closest to the collecting duct and that portion of the collecting duct located in the cortical region of the kidney are still rather impermeable to urea. However, these two areas of the nephron are quite sensitive to the hormone aldosterone (secreted by the cortex of the adrenal glands which sit on top of the kidneys) which regulates sodium absorption. An
increase in the concentration of aldosterone causes sodium to be reabsorbed by

the epithelial cells. When sodium ions are pumped out into the interstitial space, potassium ions are simultaneously transported into the lumen of this region of the nephron. Epithelial cells in this region of the nephron also respond to antidiuretic hormone (ADH, produced by hypothalamus and released by the posterior pituitary) which regulates water absorption. If the concentration of ADH increases, then water will be reabsorbed from the epithelial cells in the collecting duct and passed to the interstitial space. Removing water from the lumen of the collecting duct acts to concentrate the urine. The cells of the collecting duct can also secrete hydrogen ions (as can the cells of PCT and DCT).

Copyright byTheBerkeley Review

166

The Berkeley Review


Specializing in MCAT Preparation

Biology

Gastrointestinal Tract & Kidney

Homeostatic Mechanisms

Homeostatic Mechanisms

Suppose we want to maintain the levels of sodium in the body. Certain cells in
the cortex of the adrenal gland are sensitive to the levels of sodium in the blood.
When the concentration of sodium decreases in the blood these cells release the

hormone aldosterone. Aldosterone actsat the level of the DCT and the collecting
ducts and stimulates the epithelial cells in those regions to reabsorb sodium. As a result the blood concentration of sodium begins to rise. Thisacts as a signal and feeds back on the cortical cells of the adrenal gland telling it to reduce the
secretion of aldosterone.

Suppose we want to maintain the proper levels of water in the body. If there has been a decrease in the plasma volume of the body, there will be a tendency to have a lower than normal blood pressure (detected by baroreceptors) but a higher than normal osmotic pressure (detected by osmoreceptors). Receptors receiving this information stimulate specific cells in the hypothalamus to synthesize and transmit ADH to the posterior pituitary where it is released into the blood. ADH stimulates the epithelial cells in the latter portion of the DCT and the collecting ducts to reabsorb water. This leads to less water excretion and a higher plasma volume. If you were to drink too much liquid (e.g., pitchers of beer), then the opposite process would happen and you would excrete copious amount of urine. It turns out that the release of ADH is inhibited by alcohol.

Excretion of Nitrogenous Waste


Proteins and nucleic acids are the two primary metabolic sources of nitrogenous wastes. There are essentially three ways to get rid of the nitrogen that is produced during the metabolism of these compounds. Nitrogen can be removed as ammonia, as urea, or as uric acid. The structures of these compounds are shown in Figure 3-12.
O
ii

H-N

I JU0=0
H H
Uric acid

NH3
Ammonia

H2N-C-NH2
Urea

(toxic, soluble)

(less-toxic, soluble)

(toxic, insoluble)

Figure 3-12
Nitrogenous waste compounds.

As glutamine is metabolized near the gills of fish, the nitrogen in this amino acid can be converted to ammonia which can then combine with a proton and, in exchange for a sodium ion, be carried away with the passing water. The ammonium ion can also be excreted by mammals. One of the by-products of the amino acid glutamine is urea which is carried by the blood to the kidney where it is excreted in the urine. Urea is excreted by most mammals, amphibians and some reptiles and birds. The ammonia that is produced from the metabolism from the amino acids glycine, aspartate, and glutamate can be converted to uric acid which is the excretory product of birds and reptiles.
Copyright by The Berkeley Review
167

The Berkeley Review Specializing in MCAT Preparation

Biology

Gastrointestinal Tract St Kidney Homeostatic Mechanisms

Control of ptl and tl


The hydrogen ion concentration of the extracellular fluid is closely monitored. The normal pH of the body is 7.4. This value can fluctuate between a pH of 7.35 and 7.45. If the pH of the body decreases to values below 7.35 for an extended period of time, the individual becomes lethargic and death can result. If the pH is above 7.45 for extended periods of time, the individual becomes hyper and death can result. The regulation of blood pH is by means of a buffering system in which hydrogen ions are reversibly bound to a buffer like the bicarbonate ion (HCC>3e), the hemoglobin molecule, or even certain plasma proteins. For example, if there are hydrogen ions in the plasma, they can be taken up by the bicarbonate ion as shown in equation (3-1) to form carbonic acid. This is a weak acid and decarboxylates to form carbon dioxide and water. Carbon dioxide is eliminated by respiration. In fact, the PCO2 levels in the alveoli of the lungs are quite important in regulating the plasma pH. This can be seen if you examine the (Henderson-Hasselbalch) equation shown in (3-2). The levels of bicarbonate and hydrogen ion are controlled by the kidney whereas the levels of carbon dioxide levels are controlled by respiration.

H+ + HCO,

H2C03 ^=^ C02 + H20

(3-D

pH = pKa + log [HCO3]


[C02]
Cholera Toxin

(3-2)

Suppose you had a severe case of diarrhea caused by the bacterial cholera toxin (Vibrio cholera) that we have previously discussed. This would result in a decrease in the pH and an increase in the PCO2. Why? Recall that the gastrointestinal tract secretes a large amount of sodium bicarbonate. What happens if you lose a lot of sodium bicarbonate by way of the gastrointestinal tract? Metabolic acidosis results. Look at equation (3-1). If you decrease the

concentration of HC03e, then by LeChateller's principle (from general


chemistry) CO2 will combine with H2O to make H2CO3. This will dissociate into

HC03, and H. In other words, the hydrogen ion concentration has increasedor,
if you like, the pH has decreased. This same conclusion can be derived from equation (3-2). A decrease in pH will stimulate the DCT to secrete hydrogen ions and reabsorb as much bicarbonate as possible. By a negative feedback loop this tends to counter the initial decrease in pH. Simultaneously, an increase in PCO2 stimulates the central chemoreceptors in the medulla and pons to increase the contraction of the diaphragm and chest muscles in order to increase respiration. An increase in the respiratory rate will eliminate more CO2 from the lungs.

Copyright by The Berkeley Review

168

The Berkeley Review Specializing in MCAT Preparation

Gastrointestinal Tract

and Kidney
15 Passages 100 Questions

Passage Titles
I. II. Gastrointestinal Smooth Muscle Protein Turnover

Questions
1 -5 6- 12 13-20 21 -27 28-33 34-40 41 -48 49-53 54 - 60 61 -66 67-72 73 - 79 80-86

III.

Liver, Pancreas, and Intestines

IV. V. VI. VII. VIII. IX.


X. XI. XII.
XIII. XIV. XV.

Digestion Safeguards Digestion of Fats Diabetic Diet Experiment The Kidney Kidney and pH Dialysis and Ultrafiltration
Caffeine. Kidney and Calcium Lymph
Renal Clearance Gastrointestinal Junction Cholera Toxin

87-93 94- 100

BERKELEY Ur-e-v-i^e-w
Specializing in MCAT Preparation

Suggestions
The passages that follow are designed to get you to think in a conceptual manner about the processes of physiology at the organismal level. If you have a solid foundation in physiology, many of these answers will be straightforward. If you have not had a pleasant experience with the topic, some of these answers might appear to come from the void past the Oort field of the solar system.
Pick a few passage topics at random. For these initial few passages, do not worry about the time. Just focus on what is expected of you. First, read the passage. Second, look at any diagrams, charts, or graphs. Third, read each question and the accompanying answers carefully. Fourth, answer the questions the best you can. Checkthe solutionsand see how you did. Whether you got the answers right or wrong, it is important to read the explanations and see if you understand (and agree with) what is being explained. Keep a record of your results.
After you feel comfortable with the format of those initial few passages, pick another block of passages and try them. Be aware that time is going to become important. Generally, you will have about 1 minute and 15 seconds to complete a question. Be a little more creative in how you approach this next group. If you feel comfortable with the outline presented above, fine. If not, then try different approaches to a passage. For example, you might feel well versed enough to read the questions first and then try to answer some of them, without ever having read the passage. Maybe you can answer some of the questions by just looking at the diagrams, charts, or graphs that are presented in a particular passage. Remember, we are not clones of one another. You need to begin to develop a format that works best for you. Keeping a record of your results may be helpful.

The last block of passages might contain topics that are unfamiliar to you. Find a place where the level of distraction is at a minimum. Get out your watch and time yourself on these passages, either individually or as a group. It is important to have a feel for time, and how much is passing as you try to answer each question. Never let a question get you flustered. If you cannot figure out what the answer is from information given to you in the passage, or from your own knowledge-base, dump it and move on to the next question. As you do this, make a note of that pesky question and come back to it at the end, when you have more time. When you are finished, check your answers and make sure you understand the solutions. Be inquisitive. If you do not know the answer to something, look it up. The solution tends to stay with you longer. (For example, what is the Oort field?) The estimated score conversions for 100 questions are shown below. At best, these are rough approximations and should be used only to give one a feel for which ballpark they are sitting in.

Section ID

Estimated Score Conversions


Scaled Score >12
10-11

Raw Score

86-100
79-85

8-9
7 6 5
4

65-78

59-64
54-58
48-53 0-47

Biology
Passage I (Questions 1-5)

Gastrointestinal Smooth Muscle

Passage I

3.

According to the diagram shown below, the force of

smooth muscle contraction is most likely dependent


The smooth muscle cells of the gastrointestinal (GI) tract are typically of the single unit type, roughly 50 to 100 microns long and 2-5 microns wide. The action potentials found in smooth muscle are smaller in amplitude, but longer lasting than those in neurons. The muscle cells can be arranged in sheets and can be excited by action potentials from neighboring cells. These sheets of cells also exhibit an intrinsic pattern of periodic depolarization known as pacemaker activity. Slow waves of depolarization followed by repolarization make up the basic electrical rhythm. In the presence of excitatory neural or hormonal input, a slow wave may exceed the threshold for action potential generation and initiate stronger contractions.
Motility in the GI tract falls into three major categories. The first is peristalsis, which involves
coordinated waves of contraction and relaxation. The
A. B.

upon the:

Action Potentials

I
Time

jv_

second is segmentation, which involves alternating contraction and relaxation of the circular muscle layer. The third is the mixing motions of villi and microvilli which reduce the effect of the unstirred layer of fluid that lies adjacent to the apical surface of cells.

size of the action potential.


interval between successive sets of action

C. D.

potentials. number of action potentials within a set. time duration of individual action potentials.

Segmentation, in contrast to peristalsis, is primarily responsible for:


1.

Two types of smooth muscle, single-unit and multiunit, are distinguishable based on their electrical activity. Which of the following statements about
this distinction is most likely true?

A.
B.

absorption of nutrients from the lumen of the


GI tract.

movement of intestinal contents through the GI


tract.

A. B.
C. D.

Single-unit fibers are coupled, with many


individual cells containing gap junctions.

C.
D.

Single-unit fibers are uncoupled, with few


individual cells containing gap junctions. Multi-unit fibers are coupled, with few individual cells containing gap junctions. Multi-unit fibers are uncoupled, with many individual cells containing gap junctions.

mixing of intestinal contents, breaking larger particles into smaller ones. stimulating the secretion of digestive enzymes from the intestinal exocrine glands.

The mixing of intestinal contents most aids in digestion because:

A.
2.

smaller particles move faster through the GI


tract.

The conduction velocity of action potentials along


smooth muscle fiber is low because activation of:

B.
C.

smaller particles are able to cross the epithelial


layer with increased efficiency. smaller particles carry less charge, enabling more efficient passage of particles over biological membranes. breaking larger particles into smaller particles increases the surface area available to digestive
enzymes.

A.
B. C.

potassium channels is slow.


sodium channels is slow. calcium channels is slow.

D.

D.

magnesium channels is slow.

Copyright by The Berkeley Review

171

The Berkeley Review

Specializing in MCAT Preparation

Biology
Passage II (Questions 6-12)

Protein Turnover

Passage n

8.

The diagram shown in Figure 1 is a summary of daily protein intake and outputturnoverin a 70 kg human. [Note: The gastrointestinal (GI) system is equivalent to
the gut.]
Protein Intake

The urine carries 80 gm of protein equivalents per day. The term protein equivalent is used because protein is degraded and another molecule is used as the excretory route for nitrogen. Which of the following is the chemical formula for this excretory product?

(100 gm)

Body Protein (10,000 gm)


Muscle

(50 gm)
Turnover of

Liver

A.

(25 gm)

Body Protein "V* (250 gm)

if Cells ^White
(20 gm)
(8gm)

B.

NH3

Free

"^i Hh Hb
o n

C.

H2N C- NH2

Amino Acids I

(100 gm)

Skin

(2gm)

Fecal

(10 gm protein)

Urinary (80 gm protein equivalent)

+
9.

D.

H2N- NH2

Rank in increasing order the following organs or tissues according to their amount of daily protein
turnover.

Figure 1

A.

White cells, gut, liver, muscle.


Gut, muscle, liver, white cells.

6.

You can see from the diagram that the gut secretes more protein than the person ingests. Which of the following gut secretions contain recyclable protein?
I.
II. m.

B.

C. D.

Muscle, liver, gut, white cells. White cells, liver, muscle, gut.

Digestive enzymes
Mucus Bicarbonate 10.

A. B. C.
D.

I only I and II only II and m only


I, n, and in

Albumin is the most abundant plasma protein. What type of experiment could be used to quantify the
albumin turnover rate?

A.

Give a known dose of albumin and sample the blood periodically to measure albumin
concentration.

7.

Given the following equation, what is the efficiency of protein digestion and absorption in the gut?

B.

Give isotopically labeled albumin, sample the blood periodically, and calculate the decay rate
of labeled albumin.

Efficiency =
A. B. C. D. 63% 77% 90% 94%

Protein Absorbed Total Protein Available

C.

Give a known dose of albumin and sample the blood periodically to measure albumin decay
rate.

D.

Combine a blood sample with isotopically


labeled albumin in a test tube and calculate a

decay rate.

Copyright by The Berkeley Review

172

The Berkeley Review Specializing in MCAT Preparation

Biology
11.

Protein Turnover

Passage U

During a fast, which tissue is the first to break down its protein to provide amino acids for other tissues?
A. B. Smooth muscle. Cardiac muscle.

C.
D.

Enzymes of energy metabolism.


Skeletal muscle.

12. The following experiment, a nitrogen balance study, was performed to determine nitrogen needs in human subjects. To be in nutrient balance means that intake - output = zero for that nutrient. For nitrogen, this means that the nitrogen contained in protein in food eaten and secreted by the gut equals the amount of nitrogen excreted in urine, feces, and through skin losses.
Subjects were placed on 3 different diets sequentially. The results of their nitrogen balance
are noted in Table 1.

Protein Intake

(gm/kg)
0.1 0.6 1.5

Nitrogen Balance (mg/kg)


-42 -21

+ 12 Table 1

What is the amount of protein needed by these subjects to maintain a nitrogen balance? A. B. C. D. Less than 0.1 gm/kg. Between 0.1 and 0.6 gm/kg. Between 0.6 and 1.5 gm/kg. Greater than 1.5 gm/kg.

Copyright by The Berkeley Review

173

The Berkeley Review Specializing in MCAT Preparation

Biology
Passage HI (Questions 13- 20)

Liver, Pancreas, and Small Intestines

Passage in

13.

Which secretion from the small intestine would be

In controlling the processes of digestion, the pancreas and liver work in tandem to prepare food that exits the stomach for absorption in the small intestine. Secretionof
bile salts from the liver (sodium and potassium salts

most appropriate for processing a meal containing a high fat content?


A. B. CCK Bile Salts

conjugated to glycine and taurine) is important in the formation of micelles, water soluble complexes from which lipids can be more easily absorbed by the intestinal lining. Secretions from the pancreas contain a highly
alkaline bicarbonate solution that is important in the neutralization of the hydrochloric acid (HCl) secreted by the parietal cells of the stomach. Secretion of HCl can be

C.
D.

Secretin
Pancreatic Fluids

inhibited by gastrin inhibitory peptide (GIP), an enterogastrone released by the duodenum. HCl can leave
the stomach and enter into the duodenum through the pyloric sphincter.

14. Which of the pancreatic enzymes activated in the small intestine is important for the initiation of the cascade shown in the diagram below?

Proenzyme

Enzyme

The pancreas also secretes enzymes needed to breakdown proteins entering the duodenum. All of these pancreatic enzymes are secreted as inactive proenzymes. Trypsin inhibitors are secreted by the same pancreatic cells to prevent activation of these enzymes. These proenzymes travel through the ductal system to the duodenum were the enzyme enteropeptidase
(enterokinase) is secreted from the brush border of the small intestine. Enteropeptidase initiates the conversion of the pancreatic enzymes to their active state.

_,

^~

, Enteropeptidase ' , Trypsinogen ^> Trypsin

Chymotrypsinogens ryps'"^ Chymotrypsins


Proelastase
_ , Trypsin^

^ Elastase

Procarboxypeptidases ryPsm^.

Carboxypeptidases

Bile salts and exocrine secretions of the pancreas are controlled by secretin and cholecystokinin (CCK), two enterogastrones released from endocrine cells in the upper portion of the small intestine.
Secretin is released from the S cells in the intestinal

A.
B. C. D.

Bile salts
Secretin CCK Enterokinase

mucosa when the pH of the duodenum falls below 4.5. Secretin release is increased by gastric acid bathing the lining of the small intestine and products of protein digestion. Secretin stimulates the ductal cells of the pancreas to release a fluid containing a high concentration
of bicarbonate ion but a low concentration of chloride ion.

15.

Deficiency of the enzyme enteropeptidase results in a disorder displaying which of the symptoms listed
below?

Secretin, also increases the production of pancreatic


enzymes, decreases acid secretion in the stomach, and

may cause contraction of the pyloric sphincter.


CCK secretion is stimulated by peptide, amino acid, and fatty acid products coming in contact with the intestinal lining. CCK stimulates the contraction of the gall bladder, a storage organ that concentrates bile salts produced in the liver. Furthermore, CCK increases the secretion of pancreatic fluids rich in enzymes and alkaline phosphates, inhibits gastric emptying, increases
enteropeptidase production, and increases motility of the
small intestine and colon.

A. B. C. D.

Fat deficiency Increases in fat absorption Protein deficiency Vitamin B \2 deficiency

16.

Which hormone is important in initiating the neutralization of gastric secretions in the small
intestine? A. B. Secretin Enterokinase

C.
D.

Proenzymes
CCK

Copyright by The Berkeley Review

174

The Berkeley Review Specializing in MCAT Preparation

Biology
17.

Liver, Pancreas, and Small Intestines

Passage III

The enzyme phospholipase A can remove a fatty acid residue from lecithin (a component of bile) to form lysolecithin, a compound that damages cell membranes. Activation of phospholipase A in the pancreatic ducts causes damage to the surrounding tissue and can lead to acute pancreatitis, a severe and often fatal disease. Which of the following catalytic enzyme is most likely involved with this disorder?
A. B.
C.
D.

20. Antacid tablets, which are mixtures of magnesium hydroxide and aluminum hydroxide, have been
shown to heal duodenal ulcerations.

Mg(OH)2 + 2 HCl -> MgCl2 + 2 H20 Al(OH)3 + 3 HCl -> AICI3 + 3 H20
Based on the diagram shown below, antacids should

Trypsin Endopeptidase
Bile salts
Secretin

be taken 1 and 3 hours after a meal because they:

'3

18.

As acidic chyme enters the small intestine, secretin


stimulates the release of the bicarbonate ion from the

pancreas. The following reaction will result in the


duodenum:
1 3 5

HCl + NaHC03 -> NaCl + H2C03 Carbonic acid (H2CO3) will immediately:

Hours After Eating a Meal


A = No Antacid

A.

begin to increase the acidity of the duodenum


and activate CCK in order to decrease motility of the small intestine and increase absorption.

B = Antacid taken 1 hour after meal C = Antacid taken 1 and 3 hours after meal

B.

decrease the pH of the intestinal lumen so the entering chyme can easily be hydrolyzed by
the acidic medium.

A.
B.

will allow for additional release of acid to aid

C.

dissociate into CO2 and H2O in order to allow


the NaCl solution in the small intestine to
remain neutral.

C.

in food digestion. delay the appearance of acid in the stomach. combine with either magnesium hydroxide or aluminum hydroxide to form an acid which is
more concentrated.

D.

dissociate into H and HCC3e in order to


increase the acidity of the intestinal lumen and prepare it for nutrient absorption.

D.

produce water as a by-product of the reaction


and reduce the acidity of the stomach.

19.

Severe duodenal ulcers, leading to the removal of


the duodenum, will cause an increase in all of the

following EXCEPT:

A. B. C.

gallbladder relaxation. parietal cell activity. pancreatic ductal cell activity.

D.

passage of chyme into the small intestine.

Copyright by The Berkeley Review

175

The Berkeley Review Specializing in NCAT Preparation

Biology
Passage TV (Questions 21-27)

Digestion Safeguards
23.

Passage IV

Which parts of the digestive tract are lined with


mucus?

The body is about 95-98% efficient in digesting protein of animal origin. Since 20% of the human body is protein of animal origin, we require some sophisticated safeguards to avoid digesting our own tissue.
First, mucopolysaccharides line the epithelial tissue of the digestive tract. Mucus is indigestible and protects the tissues from the actions of acids and enzymes of digestion. Second, the pancreas contains proteolytic enzymes in an inactive form (zymogens) to avoid digestion of the pancreatic cells. These zymogens are released unchanged through the pancreatic duct into the
duodenum.

I. II. III.

Mouth Colon Small intestine

A.
B. C.

I only
I and II II and HI

D.

I, H, and in

24. What is the approximate pH of the stomach?


A. B. C. D.

Trypsin, a pancreatic protease, is activated in the duodenum by the enzyme enterokinase. Trypsin, in turn, activates the other zymogens by cleaving a portion of the molecule so that the active site is exposed. A further

6.8
0.2 9.0 2.0

mechanism for protection of the GI tract is the rapid turnover of cells. The epithelial lining of the GI tract is replaced every 5-7 days. Finally, although the stomach acidifies the bolus of food considerably, the pancreas secretes bicarbonate to neutralize the pH of the duodenal
contents.

25.

What could happen if enterokinase entered the pancreatic duct and went to the pancreas? I. The zymogens would be activated.

II. Pancreatic enzymes would appear in the


bloodstream.

III. Insulin secretion would be impaired.


21. Raw soybeans contain a protein that acts as a trypsin inhibitor. What would be the digestive effect of eating a large quantity of raw soybeans?
A. Trypsin would be more active, but the other enzymes would remain as zymogens. Trypsin would be inactive, and the other pancreatic enzymes would function normally. Trypsin would be inactive, and the other pancreatic enzymes would remain as
zymogens.

A. B. C.
D.

I only I and II only II and HI only


I, II, and ffl

B.

26.

What type of acid(s) do the parietal cells of the


stomach secrete?

C.

I.
II. III.

Hydrochloric acid
Sulfuric acid Carbonic acid

D.

Trypsin and other pancreatic enzymes would function normally.

A. B. C. 22. Chemotherapeutic agents target rapidly growing and dividing cells. This is beneficial for attacking tumor cells, but may be detrimental to other body cells. Which cells would be most adversely affected?
A. B. neurons cardiac muscle cells

I only I and II only II and m only


I, n, and ffl

D.

27.

To what major class of mucopolysaccharides belong?


A. Protein

molecules

do

C.
D.

gastric epithelium
skeletal muscle cells

B. C.
D.

Lipid Carbohydrate
None of the above

Copyright by The Berkeley Review

176

The Berkeley Review Specializing in MCAT Preparation

Biology
Passage V (Questions 28-33)

Digestion of Fats

Passage V

29.

The digestion of fats and cholesterol by the body is primarily carried out in the duodenum of the small intestine. Triglycerides are the most common fat products in the diet. The initial process in the digestion of fats is the emulsification of fat droplets by bile salts released from the liver and gall bladder. Bile salt release is stimulated by increased levels of long-chained fatty acids (containing more than 10 carbon atoms) within the
duodenum. Bile salt emulsification of fat is not essential

Which of the components of fat digestion is most closely associated with principles utilized by household detergents to remove grease?
A. Emulsification of fats.

B. C. D.

Micelle endocytosis. Exocytosis of fatty acids. Packaging of fat products in the endoplasmic
reticulum.

for digestion and absorption since as much as 60% of all triglycerides can be absorbed without the aid of bile salts. Cholesterol processing, however, is dependent on the
actions of bile salt.

30.

Sprue, a disorder of malabsorption in the small intestine, is characterized initially by the increased soap-like appearance of fat in the stools. Which of the following reasons could lead to this symptom?

Cells within the duodenum sensitive to fatty acids will

A.
B.

Decrease in chylomicron production.


Decrease in bile salt secretion.

release cholecystokinin (CCK), a hormone which will stimulate contraction of the gall bladder and increase fluid secretion from the pancreas. Once the fats have been emulsified, pancreatic lipase, an enzyme produced from the exocrine pancreas and activated by the enzyme trypsin, will hydrolyze the fats into monoglycerides, free fatty acids, and glycerol. The hydrolyzed products of fat droplets are then packaged by bile salts into water soluble micelles and then transported to the cellular membrane of
the intestinal microvilli. The micelles cross the cell

C.

Decrease in intestinal cell microvilli.

D.

Increase in pancreatic lipase production.

31.

What will occur if bile salt secretion is decreased?

A.

Increases in cholesterol absorption in the small


intestine.

B.
C.
D.

Complete blockage of fatty acid digestion in


the small intestine.

membrane and are incorporated into vesicular packages

(chylomicrons) in the endoplasmic reticulum. The chylomicrons are then exocytosed into the intercellular spaceand taken up by the lactealsof the villi. Chylomicrons also contain triglycerides, cholesterol, and phospholipids. They are transported by way of the lymphatics and the capillary system to the liver. In route to the liver, lipoprotein lipase, an enzyme located in the capillaries, strips triglycerides from the chylomicrons for storage in adipose tissue. The remaining components of the chylomicrons are degraded in the liver and stored for later production of steroid hormones, VLDLs, or excreted
with bile salts into the intestine.

Inhibition of cholesterol absorption in the


small intestine.

Decreased levels of CCK secretion from the


small intestine.

32. Chylomicrons are degraded in the liver. Which of the following compounds is a component of a chylomicron as it enters into the liver to be
degraded?

A.
B.

Fatty acids.
Bile salts.

C.
D.

Lipoprotein lipase.
Cholesterol.

28. A trypsin antagonist is present during fat


metabolism. Which of the following processes will
NOT occur?

33. Disorders characterized by increases in fat

deposition in the liver may be produced by one of


the abnormalities below.

A. B.

Emulsification of fats. Increase in fat content in the small intestine.

A.
B.

Decreased levels of pancreatic lipase.


Decreased levels of CCK.

C.
D.

Hydrolyzation of fat droplets in the small


intestine.

Decrease in CCK production.


177

C. D.

Decreased levels of lipoprotein lipase. Decreased levels of chylomicrons. The Berkeley Review

Copyright by The Berkeley Review

Specializing in MCAT Preparation

Biology
Passage VI (Questions 34-40)

Diabetic Diet Experiment

Passage VI

Table 2: Plasma lipid and lipoproteinlevels


during the study.
Measurement
Base line ADA HF

Diet control is important in non-insulin-dependent diabetes mellitus (NIDDM), not only to improve control of hyperglycemia but also to reduce the risk of coronary heart disease by improving plasma lipid levels. The American Diabetes Association (ADA) currently recommends the following diet for patients with NIDDM:
30% of energy as fat (with 10% as saturated fat) 40-50% complex carbohydrates (starches) less than 10% simple carbohydrates (sugars) Lastly, protein makes up 10-20% of the diet's energy.
Some studies indicate that diets like the ADA diet, that

(mg/dL)
Total Cholesterol

Diet
205 218
43

Diet
196

225*
285

Triglycerides
VLDL Cholesterol

163f
28
134

58
135
32

LDL Cholesterol HDL Cholesterol

131
30

34

$ P < 0.02 for baseline versus ADA and HF values, t P < 0.01 for ADA versus HF values. P < 0.005 for ADA versus HF values.

are high in carbohydrates, lead to increased triglyceride levels, increased very low density lipoprotein levels, and reduced high density lipoprotein levels: These would not be beneficial in a diabetic population concerned with maintaining optimal plasma lipid levels.

34. Which of the following statements are supported by


the data in Table 1?

The following experiment was designed to test the effect of the ADA diet versus a higher monounsaturated fat diet (HF) on plasma glucose and lipid levels.

I.
II.

The HF diet lowered plasma glucose in these


subjects. Patients required more insulin on the HF diet
than at the baseline measurement.

HI. Less glucose was excreted in the urine during the HF diet compared to the baseline
Experiment J
After a baseline measurement of 1 week, 10 NIDDM patients ate the ADA diet and the HF diet for 5 weeks in
measurements.

random order during 2 hospital admissions. Metabolic measurements regarding glucose (Glc) are outlined in

A. B. C.
D.

I and III only I and II only II and ffl only


I, II, and ffl

Table 1. [Note: Hb refers to hemoglobin.]

Table 1: Metabolic variables during the study.


Base Variable line
129

35. In Table 1, glycosylated hemoglobin (G-Hb) is


reported as an indicator of glucose metabolism in the

ADA Diet
117

HF

Diet

body. Glycosylatedmeans that glucose residues are


attached. G-Hb decreases on both the HF and the

Plasma Glc (mg/dL) Urinary Glc (mg/day) Insulin Requirements (units/day)


Glycosylated Hb (%)

101$ ot 70*
8.1

550
84

142

ADA diets during the study.


beneficial or detrimental?

Is this decrease

81

A.
11.3
7.8

This decrease

is

detrimental,

because

glycosylated hemoglobin indicates persistent


levels of low plasma glucose.
B. This decrease is detrimental, because

t P < 0.02 for HF versus ADA and baseline. t P < 0.01 for baseline versus HF values. P < 0.005 for baseline versus HF and ADA values.

glycosylated hemoglobin indicates persistent levels of high plasma glucose.


C. This decrease is beneficial, because

Metabolic measurements on plasma lipids and lipoproteins are outlined in Table 2.

glycosylated hemoglobin indicates persistent levels of low plasma glucose.


D. This decrease is beneficial, because

glycosylated hemoglobin indicates persistent levels of high plasma glucose.


Copyright by The Berkeley Review
178

The Berkeley Review Specializing in MCAT Preparation

Biology
36.

Diabetic Diet Experiment

Passage VI

The table shown below indicates the fatty acid composition of both ADA diet and HF diet in % energy intake/day.
Fatty
Acid
12:0
14:0

39. Some patients with NIDDM maintain good glucose control using drugs that increase insulin secretion. What is the target of these drugs that affect insulin
secretion?

ADA

HF Diet
0.2
0.2

Diet
0.2 0.3

A. B. C.
D.

The alpha cells of the pancreas. The exocrine cells of the pancreas. The beta cells of the pancreas.
Both the exocrine and beta cells of the pancreas.

16:0 16:1

6.8
0.2

7.6
0.4

18:0 18:1
18:2

2.4
7.4 7.2

2.2
29.4

8.0
0.2 0.2 1.5

40.

18:3
20:4 22:0

0.0
0.2

Insulin is secreted from the pancreas into the hepatic portal vein. Which is the first major organ insulin encounters after entering this portal system?
A. B. C. D. Small intestines Stomach Brain Liver

0.3

Total:

25.0

49.9

Table 3

Which of the fatty acids was used to supplement the monounsaturated portion of the HF diet ?
A. B. C. D. Palmitoleic acid Stearic acid Oleic acid Palmitic acid

37.

Which of the following oils or fats could be used to


increase the amount of monounsaturated fat in a
diet?

A. B. C. D.

Olive oil Lard Butter Beef tallow

38.

Which of the following statements is/are supported by the data in Table 2?


I. Both VLDL and LDL were lower in the HF

diet compared to the ADA diet.

II. Triglycerides were significantly lowered by


the HF diet compared to baseline.
III. HDL cholesterol was increased by the HF diet compared to the ADA diet.

A. B. C. D.

I and ffl only I and II only n and III only


I, II, and ffl

Copyright by The Berkeley Review

179

The Berkeley Review

Specializing in MCAT Preparation

Biology
Passage VOL (Questions 41-48)

The Kidney

Passage VII

Antidiuretic hormone (ADH) and aldosterone are both

The functional unit of the human kidney is the nephron

(Figure 1), and each kidney contains approximately 1


million of these tubular structures.

regulators of urine composition. Both hormones promote NaCl reabsorption in the ascending limb of the loop of Henle and in the DCT and collecting ducts. ADH promotes water reabsorption in the DCT and collecting

ducts while aldosterone promotes K secretion in the


DCT.

arteriole

Efferent

Glomerulus
41. Constriction of the efferent arterial will lead to:

_,

A.
Afferent arteriole

B. C. D.

a decrease in glomerular pressure, followed first by a decrease and then by an increase in glomerular filtration rate. an increase in glomerular pressure, followed first by an increase and then by a decrease in glomerular filtration rate. a decrease in glomerular pressure, followed first by an increase and then by a decrease in glomerular filtration rate. an increase in glomerular pressure, followed first by a decrease and then by an increase in glomerular filtration rate.

Figure 1
42.

The reabsorption of sodium across the epithelial


cells of the proximal convoluted tubule and into the

Blood is supplied to each kidney by a renal artery. The smallest branch of this artery is the afferent arteriole, a vessel influenced by sympathetic stimulation. A capillary
bed (glomerulus) is formed from the afferent arteriole and

peritubular capillary would be significantly reduced


by:

is contained within Bowman's capsule. The efferent arteriole that leaves the glomerulus is further divided into a capillary system that surrounds different portions of the nephron. The plasma ultrafiltrate that passes through the glomerulus and into Bowman's capsule is essentially
protein-free and devoid of cellular structure. This filtrate

A.
B.

reduced secretion of ADH from the posterior


pituitary.
an increased rate of secretion of aldosterone
from the adrenal cortex.

C.
D.

reduced plasma glucose concentrations.


an increase in the tubular colloid osmotic
pressure.

flows into the proximal convoluted tubule (PCT), through


the loop of Henle and into the distal convoluted tubule (DCT), through the collecting ducts and into the renal

pelvis before making its way to the ureter and urinary


bladder.

43. Sympathetic stimulation of the kidneys primarily


affects the:

The PCT reabsorbs essentially all of the glucose and amino acids filtered by the glomerulus and roughly 70%

A. B. C. D.

of the filtered K, Na, Cle, and water. Approximately 20% of the K, Na, Cle, and water thatpasses from the
PCT into the loop of Henle is reabsorbed at the loop. The

remaining 10% of the K, Na, and Cle is reabsorbed at


the DCT. Variable amounts of water also reabsorbed at the DCT.

afferent arterioles urine flow. afferent arterioles urine flow. efferent arterioles urine flow. efferent arterioles urine flow.

and decreases the volume of and increases the volume of and decreases the volume of and increases the volume of

Copyright by The Berkeley Review

180

The Berkeley Review Specializing in MCAT Preparation

Biology

The Kidney

Passage VH

44. Which of the following graphs BEST represents the plasma osmolarity (X-axis) of the blood and the plasma concentration of antidiuretic hormone (Yaxis)?
A.

46. All of the following statements about aldosterone are


true EXCEPT it:

A.
B.

promotes sodium reabsorption in the distal


convoluted tubule.

B.

promotes the secretion of potassium into the


lumen of the distal convoluted tubule.

C.
D.

is a steroid hormone secreted by the adrenal


medulla.

is the major mineralocorticoid in humans.

X-Axis

X-Axis

47.

Excessive aldosterone secretion will result in all of

the following EXCEPT:


C.
D.

A.
B.

hyperpolarization of nerve and muscle


membranes. increased muscle contraction.

C.
D.

an excessive loss of potassium ions from the


extracellular fluid.

hypokalemia.

X-Axis

X-Axis

48.

Which of the following substances is filtered by the glomerulus and into Bowman's capsule?
A. B. Platelets Proteins

45. Which of the following graphs best represents the plasma concentration of glucose (X-axis) and the rate of glucose excretion in the urine (Y-axis)?

C. D.

Electrolytes Erythrocytes

A.

B.

X-Axis

X-Axis

C.

D.

X-Axis

X-Axis

Copyright by The Berkeley Review

181

The Berkeley Review

Specializing in MCAT Preparation

Biology
Passage VHI (Questions 49-53)

Kidney & pH

Passage VHI

Tubular

Most metabolic reactions are highly sensitive to the pH of the fluid in which they occur. Due to this sensitivity,

Interstitial

Lumen

Epithelial Cells

Fluid

the hydrogen ion concentration of body fluids is closely


regulated.

Buffering systems, both intracellular and extracellular, act to minimize changes in the hydrogen ion concentration. However, these buffering systems do not have the ability to eliminate or retain hydrogen ions from the body. This role is left for the kidneys, which accomplish this regulation in two ways. The first is by altering the secretion of hydrogen ions and the second is by altering the reabsorption of bicarbonate. Hydrogen ion excretion and bicarbonate reabsorption are both achieved by the secretion of hydrogen ions.
CA = Carbonic Anhydrase
Tubular
Lumen

Interstitial

Epithelial Cells

Fluid

Figure 2

In the body, the events shown in Figure 1 and Figure 2 occur together, producing a situation where a high rate of hydrogen ion secretion achieves complete reabsorption of
filtered bicarbonate.

49.

Which of the following situations will NOT produce


a gain of hydrogen ions?

Urine

CA = Carbonic Anhydrase

Figure 1

A. B.
C.
D.

An increase in the concentration of CO2. Production of lactic acid during severe


exercise.

According to Figure 1, the hydrogen ion to be secreted into the lumen is generated in the tubular cell rather than coming from the blood. Once in the lumen, the hydrogen
combines with a buffer and is secreted in that form.
fluid.

Excessive vomiting.
Loss of bicarbonate due to diarrhea.

In

Figure 1, the bicarbonate is transported to the interstitial


The hydrogen ion in the lumen can also bind with filtered bicarbonate to form CO2 and H2O according to Figure 2. This is in essence bicarbonate reabsorption. The net result is that filtered bicarbonate disappears while bicarbonate (the result of intracellular events) appears in the blood. This situation is equivalent to one where
filtered bicarbonate is reabsorbed back into the blood.
50.

The events shown in Figure 1 and Figure 2 MOST likely illustrate renal compensation for:

A. a decreased blood hydrogen ion concentration. B. an increased blood hydrogen ion concentration.
C. D. a decreased sodium ion concentration. an increased sodium ion concentration.

Copyright by The Berkeley Review

182

The Berkeley Review Specializing in MCAT Preparation

Biology
51.

Kidney & pH

Passage VIII

The transport of a hydrogen ion from a tubular cell to the tubular lumen shown in Figure 1 represents:

A.
B.

simple diffusion.
facilitated diffusion.

C. D.

primary active transport. secondary active transport.

52. Respiratory alkalosis is best characterized by :


A. an increase in the hydrogen ion concentration, resulting from an increase in the level of CO2. a decrease in the hydrogen ion concentration, resulting from a decrease in the level of CO2. an increase in the hydrogen ion concentration, resulting from a decrease in the level of CO2. a decrease in the hydrogen ion concentration, resulting from an increase in the level of CO2.

B.
C.
D.

53. During metabolic acidosis, the partial pressure of CO2 can be expected to:
A. B. C. D. increase, due to increased ventilation. increase, due to decreased ventilation. decrease, due to increased ventilation. decrease, due to decreased ventilation.

Copyright by The Berkeley Review

183

The Berkeley Review

Specializing in MCAT Preparation

Biology
Passage IX (Questions 54-60)

Dialysis and Ultrafiltration


55. Which structure of the

Passage IX

kidney

performs

ultrafiltration?

Dialysis is a process used to separate molecules in


solution based on their size. Semipermeable membranes

that have pores smaller than the macromoleculeof interest are used to contain one or more large molecules. Dialysis tubing is filled with the solution of interest, and the ends of tubing are tied. The package is immersed in a large quantity of solvent. Diffusion of molecules smaller than
the pores of the tubing allows exchange of solvents, salts, and small metabolites between the solution in the tubing and the surrounding solution. This can be used to remove excess salts, for example, from a protein to be purified.
Ultrafiltration is a related technique that is used to
concentrate macromolecules. Pressure is used to force the
56.

A. B.
C.

Peritubular capillaries Loop of Henle


Afferent arterioles

D.

Glomerular capillaries

solution through the semipermeable membrane. The solvent and small molecules pass through the membrane,
while a more concentrated solution of macromolecules is

A student is purifying an enzyme in the laboratory by overnight dialysis. The student tests for successful dialysis using an enzyme activity assay. What would the student find if the dialysis tubing was punctured with a pin before the experiment?
A. B.
C.

left behind. Normal kidneys perform ultrafiltration constantly in the removal of waste products from the blood. The entire blood supply of 5-6 liters is filtered approximately every 45 minutes.

Diseased kidneys require exterior dialysis support to prevent dangerous increases in the plasma solute load, including, salts, electrolytes, and urea. Uremia is the condition of having an increased urea plasma
concentration.

No enzyme activity in the bag. Increased enzyme activity in the bag compared to the activity measured before the experiment. There is no effect caused by the pinhole, and the enzyme activity in the bag is the same as
before.

D.

Slightly decreased enzyme activity in the bag


compared to the activity measured before the experiment.

Dialysis support is conventionally performed in one of two ways. The first involves passing the blood through a machine containing semipermeable membranes that allow waste products to diffuse out. An in vivo technique uses the patient's peritoneal membranes (in the abdominal cavity) to form an intraabdominal dialysis system. A sterile fluid is instilled through an abdominal catheter,
allowed to remain from 15 minutes to 1 hour, and drained

57.

Which of the following hormones would most likely be supplemented during chronic kidney disease?
A. B. Erythropoetin Antidiuretic hormone (ADH) Epinephrine and norepinephrine
Insulin

through a second abdominal catheter. This process is repeated many times. Both methods are time-consuming, but they allow the necessary removal of waste products
from the blood.

C.
D.

54. What complications could result from the peritoneal dialysis procedure?
I. Increased risk of abdominal cavity infections. II. Changes in electrolyte balance. III. Kidney damage by ultrafiltration.
A. B.

58.

What would be a physiological effect of uremia?


A.
B.

C. D.

I only I and II only II and HI only


I, H, and HI

Increased blood pH
Edema

C. D.

Increased urination Alkalosis

Copyright by The Berkeley Review

184

The Berkeley Review Specializing in MCAT Preparation

Biology
59.

Dialysis and Ultrafiltration

Passage IX

When a dialysis machine is used to clean blood,

what precautions are taken to avoid loss of glucose


and salt?

A.
B.

Use of a dialysis membrane that retains glucose and salts. Use a dialysis solution that is isotonic to the blood in glucose and salts.

C.
D.

The patients drinks an electrolyte and glucose


drink during dialysis. Nothing can be done to avoid loss, so the patient has an IV of saline and glucose at the
same time.

60. If a person has 5 liters of blood and it is passed


completely through the kidneys every 45 minutes, what is the glomerular filtration rate?
A.

225 ml/min 111 ml/min 66 ml/min 9 ml/min

B.

C. D.

Copyright by The Berkeley Review

185

The Berkeley Review Specializing in MCAT Preparation

Biology
Passage X (Questions 61-66)

Caffeine

Passage X

62. Which of the following hormones would most increase plasma FFA concentration?
A. CCK

Caffeine is a commonly used beverage in the U.S.

The average individual consumes 2-3 cups per day, with some people drinking 10-15 cups. Coffee contains about 150mg caffeine/cup for drip-style makers. Caffeine is the physiologically active agent in coffee. The following experiments were carried out to examine the effects of
caffeine in coffee on free fatty acid (FFA) metabolism: All subjects followed on overnight fast. At 8:00 AM, each subject drank one of the following beverages:
Experiment1: 500 mL hot water containing 5 g instant coffee (250

B. C.
D.

Thyroid hormone Epinephrine


Insulin

63. In Experiment 4, how did the sucrose lead to lower


FFA concentrations?

A. B.

Sucrose was digested to glucose and fructose;


fructose mediated the effects on FFAs.

mg caffeine) and 3 tablets of sodium saccharin (15


mg).

C.
D.

Sucrose stimulated glucagon, which decreased release of fatty acids from adipocytes. Sucrose acted as a competitive inhibitor of
caffeine.

Experiment2: 500 mL hot water containing 5 g decaffeinated coffee

Sucrose stimulated insulin, which decreased

release of fatty acids from adipocytes.

(20 mg caffeine) and 3 tablets of sodium saccharin


(15 mg).

Experiment 3:
500 mL hot water containing 3 tablets of sodium saccharin (15 mg) Experiment 4:

64. The action of caffeine is mediated through increased cAMP levels in target cells. What is the enzyme that is activated by second messengers to make cAMP? A. B. C. D. Epinephrine G protein Adenylate cyclase Protein phosphorylase

500 mL hot water containing 5 g instant coffee (250 mg caffeine) and 25 g sucrose

Blood samples for all experiments were taken before the start of the experiment and at 1, 2, and 3 hours following ingestion of the test beverage. The following table follows the FFA concentrations throughout the study:
Expt
1

65. From Experiment 2, what would you conclude about the effects of the caffeine present in decaffeinated coffee on plasma FFA concentration?

A.
0 hours 0 1 hour 2 hours 3 hours

+182*
-16* -19

+245*
+18* -59

+384$
+93*
+17

B.

2 3 4

C.
D.

0
0

There are slight increases in FFA con centration due to the 20 mg of caffeine present The effects are the same as drinking coffee with 25 g sucrose. There are slight decreases in FFA con centration due to the 20 mg of caffeine present. The effects are the same as drinking water.

-96t

-90t

+120t

* Not significant compared to Experiment 3

66.

What was the point of Experiment 3?


I. n. III.
A. B. C. D.

XSignificant compared to Experiment 2 and Experiment 3 t Significant compared to Experiment 1

Table 1: Mean Changes in Plasma FFAs (^EQ/L)

To determine the effects of hot water To determine the effects of sodium saccharin To establish control data

61.

Which of the following mechanisms is the most important in raising plasma FFA concentrations?
A. B.
C.

I only Ilonly I and II only


I, II, and m

Lipolysis Lipogenesis
Increased fat oxidation

D.

Increased dietary fat


186

Copyright by The Berkeley Review

The Berkeley Review Specializing in MCAT Preparation

Biology
Passage XI (Questions 67-72)

Kidney St Calcium

Passage XI

68.

According to the diagrams in Figure 1, all of the following statements concerning substances X, Y,
and Z are false EXCEPT:

Kidney function can be characterized by three processes. The first involves movement of an essentially protein free plasma from renal glomerular capillaries into Bowman's capsule. This is referred to as glomerular filtration. Tubular reabsorption refers to the movement of substances from the renal tubular lumen to the peritubular capillary plasma. Movement in the opposite direction is
termed tubular secretion.

Afferent Renal Arteriole

Efferent Arteriole

Artery

The kidney, among its many other functions, is involved with regulating levels of calcium ion. The extracellular calcium concentration is normally held relatively constant. However, a low calcium ion concentration increases the excitability of nerve and
muscle cell membranes.
Urine

Peritubular

Capillary

Calcium homeostasis is achieved through the functioning of three major effector sites. These sites are the kidney, bone, and gastrointestinal (GI) tract. Almost 99% of total-body calcium is contained in bone, which is a framework of organic molecules upon which calcium phosphate crystals are deposited. Bone is a dynamic tissue which can either remove or deposit calcium into the
extracellular fluid. In the GI tract, control of active transport systems that move calcium into the blood can

result in large fluctuations in the amount of calcium

absorbed. The kidney modulates calcium levels by reabsorption of both calcium and phosphate ions.
Urine

All three effector sites are under control of parathyroid hormone, whose plasma concentration fluctuates directly in response to changes in plasma calcium levels.

Figure 1

A. B.
C. D.

Z is filtered and partially reabsorbed. Y is totally reabsorbed.


X is filtered and secreted, but not reabsorbed. X, Y, and Z are all secreted.

67.

All of the following statements about glomerular


filtration are true EXCEPT: 69.

A.
B.

fluid pressure in Bowman's capsule opposes


filtration.

In response to a decrease in plasma calcium levels, one would most likely expect to see: A. increased plasma parathyroid levels, with increased urinary calcium excretion. increased plasma parathyroid levels, with
increased removal of bone.

osmotic force due to protein in plasma favors


filtration.

C. D.

glomerular capillary blood pressure favors


filtration.

B. C. D.

forces favoring filtration are larger than those opposing filtration.

decreased plasma parathyroid levels, with decreased urinary calcium excretion. decreased plasma parathyroid levels, with
decreased removal of bone.

Copyright by The Berkeley Review

187

The Berkeley Review

Specializing in MCAT Preparation

Biology
concluded that:

Kidney St Calcium

Passage XI

70. According to information in the passage, it can be


A. the concentration of extracellular calcium in no

B.

wayeffects neuromuscular excitability. persons with low extracellular calcium


concentrations may suffer total skeletal muscle
spasms.

C.
D.

persons with increased extracellular calcium


concentrations may suffer from frequent
muscle spasms.
the effect of extracellular calcium concentration on membranes is distinct from its role as a mediator of muscle contraction.

71.

A decrease in calcium will most likely cause:


A. an increase in the extracellular concentration

of phosphate.

B.
C. D.

no change in the extracellularconcentration of


phosphate. a decrease in the urinary excretion of phosphate. a reduction in the tubular reabsorption of phosphate.

72. Parathyroid hormone is secreted by the:


A. B. C. D. anterior pituitary. posterior pituitary. parathyroid gland. thyroid gland.

Copyright by The Berkeley Review

188

The Berkeley Review

Specializing in MCAT Preparation

Biology
Passage XII (Questions 73-79)

Lymph

Passage XII

75.

Fat-soluble vitamins are carried from the cells of the

The lymphatic system has 3 functions: (1) its lymphocytes provide immunological defenses, (2) it
transports absorbed fat from the small intestine to the blood, and (3) it transports interstitial fluid back to the
blood.

small intestine into the blood system via the lymph. They are packaged into chylomicrons and with fatty acids. Which of the following vitamins are fatsoluble vitamins?

Both lymph nodes and lymphocytes of the immune system play important roles in immunological defense. Lymph nodes contain phagocytic cells through which lymph is filtered before returning to the blood supply.

I. n. m.
A. B.

Vitamin A Vitamin K Vitamin E

Lymph nodes also contains germinal centers that produce lymphocytes, which are present in the lymph and in the
plasma.

C.
D.

I only II only II and III only


I, n, and IE

76. What is the function of the lymph node?


The lymphatic system also carries dietary fat from the small intestine to the blood supply. Absorbed fat, including long chain fatty acids and cholesterol, along with fat-soluble vitamins, are packaged into chylomicrons by the cells of the small intestinal mucosa. These chylomicrons transport fat from the intestine to the general blood circulation. Other nutrients from digestive processes in the small intestine travel first to the liver via the hepatic portal vein. A. To destroy microorganisms.

B.
C. D.

To filter chylomicrons out of the lymph.


To produce chylomicrons. To add intestinal nutrients to the lymph.

77. What force moves the lymph inside the lymphatic


vessels?

The liquid part of lymph is an ultrafiltrate of plasma, called interstitial fluid. This is the fluid that is squeezed out of capillaries under pressure. Interstitial fluid enters the lymph capillaries through its endothelial cells which have porous junctions. This fluid, along with chylomicrons, microorganisms, and lymphocytes, is then called lymph. From the lymph capillaries, the lymph moves into the lymphatic vessels, which have a 3-layer structure similar to veins. The lymphatic system ultimately returns the fluid to the cardiovascular system
via the right and left subclavian veins.

I. n.

Squeezing by the muscles. Contraction of the muscular layer of the lymph


vessels.

III. Blood pressure.


A. B. C.
D.

I only I and II only II and m only


I, II, and HI

78. The term ultrafiltrate means fluid formed by the hydrostatic pressure of the blood acting against a semi-permeable membrane. Interstitial fluid is one example of an ultrafiltrate. Which of the following organs also produces another ultrafiltrate?
A. Liver

73. What is the consequence of the chylomicron system of fatty acid packaging?
A. Muscle and adipose tissue have first access to dietary fatty acids from chylomicrons. Liver tissue has first access to dietary fatty acids from chylomicrons. Microorganisms are packaged into chylomicrons. Interstitial fluid carries chylomicrons.

B.
C.

Kidney
Pancreas

D.

Salivary gland

B. C.
D.

79. Which of the following statements are TRUE of both lymph and plasma?
I. Both contain chylomicrons after a meal. II. Both contain lymphocytes. m. Both contain erythrocytes. A. B. C.
D.

74. What is the major component of lymph?


A. Fat-soluble vitamins

B.
C.

Chylomicrons
Water

I only I and II only II and HI only


I, n, and HI

D.

Triglycerides
189

Copyright by The Berkeley Review

The Berkeley Review Specializing in MCAT Preparation

Biology
Passage XHI (Questions 80-86)

Renal Clearance

Passage xm

81.

What is the volume of urine that could be collected

in 30 seconds if the [ ]u of substance Y is 50 mg/ml,

The concept of renal clearance, based on the principle


of mass balance, is a means to quantify the excretory

the [ ]p ofsubstance Y is 1mg/ml, and the clearance


for substance Y is 50 ml/min?
A. B. C. D. 0.00025 L 0.0005 L 0.001 L 0.0015 L

function of the kidney. The relationship of clearance (C)


to the concentration of a substance in the urine ([ ]u), the

urine flow rate (Fu), and the plasma concentration of a

substance ([ ]p)is
C = ([]U)X(FU)/ []p
The glomerular filtration rate (GFR) is the volume per unit time that enters the kidney from the plasma. Under
certain conditions, the amount of a substance filtered is

equal to the amount excreted in the urine. To calculate


the amount filtered, one can multiply the GFR by the

[substance] p.
Similar to the plasma, not all of a substance traveling through the renal artery is filtered at the glomerulus. The fraction of plasma filtered can be calculated as the ratio of GFR to renal plasma flow (RPF). The fraction normally
takes on values of 15% to 20%.

82.

Which of the following equations represents the glomerular filtration rate (GFR)?

A. B. C. D.

GFR = [ substance ]p x []u x Fu (GFR) x (Fu) = ([ substance ]p) x ([]u) GFR = ([ ]u x Fu)/ ([ substance ]p) (GFR) x ([]u) x ([]p) x (Fu)=l

The renal handling of metabolites may be instrumental in diagnosing certain clinical disorders such as diabetes. When determining the amount of glucose reabsorbed in the nephron, one must subtract the amount excreted from the amount filtered at the glomerulus. The amount of glucose reabsorbed by epithelial cells of the tubule can be thought of as having a maximum transport rate, known as the tubular transport maximum. The [plasma glucose] will ultimately determine whether glucose is excreted or is 100% reabsorbed. The level of plasma glucose which first gives rise to glucose in the urine is termed the plasma
threshold.

83.

According to the passage, the amount of a substance filtered is equal to the amount excreted in the urine only under certain conditions. Which of the following conditions CANNOT exist for this
statement to be TRUE?

A.

The substance has no barrier to filtration at the

glomerulus.

80. Mass balance describes the principle that the mass of a substance entering the kidneys must equal the mass of that substance leaving the kidneys. Using the following key, and information from the passage, which relationship BEST describes this principle?

B.

The substance must not be reabsorbed and

must be secreted by the nephron.


C. The substance must not be metabolized or

D.

produced by the kidney. The rate of glomerular filtration cannot be altered by the substance.

Key
[ ]A = concentration of substance in renal artery
[ ]v = concentration of substance in renal vein

PFA = plasma flow rate in renal artery PFV = plasma flow rate in renal vein
A. B. C. D. []A x PFV = (Hu x Fu) + ([]v x PFA) ([]A x PFV) + ([]u x Fu) = []v x PFA ([]A x PFA) + Q]ux Fu) = []v xPFv []A x PFA = ([]u x Fu) + (Uv x PFV)
190

84.

Glucose is reabsorbed through the epithelial cells of


the:

A. B.
C.

proximal convoluted tubule. loop of Henle.


distal convoluted tubule.

D.

collecting duct.

Copyright by The Berkeley Review

The Berkeley Review

Specializing in MCAT Preparation

Biology
85. is:

According to the graph below, the plasma threshold

LOEFxeiclvtrfsodn,AR(emabgrp/tion
3
-J c

Renal Clearance

Passage XIII

1000 800
600

400

f
200
0
1 3

Reabsorption

10

Plasma Glucose (mg/ml)


A.

B. C. D.

3.5 mg /ml 7.6 mg /ml 380 mg / ml 400 mg / ml

86.

The following graph illustrates the influence of size and charge on a substance's filterability at the glomerulus. Which of the following statements is most likely true based on this graph?

22

26

30

34

38

42

Effective Molecular Radius (A)


A.

The cationic species has the highest


filterability, because cationic atoms are smaller
than anionic atoms.

B.

C.

The cationic species has the highest filterability, because of the presence of anionic glycoproteins on the surface of all glomerular filtration components. The anionic species has the highest filterability, because cationic atoms are larger
than anionic atoms.

D.

The anionic species has the highest filterability, because of the presence of cationic glycoproteins on the surface of all glomerular filtration components.

Copyright by The Berkeley Review

191

The Berkeley Review Specializing in MCAT Preparation

Biology
Passage XIV (Questions 87-93)

Gastrointestinal Junction

Passage XIV

89.

It is known that a hypertonic solution in the duodenum decreases the rate of gastric emptying.
This mechanism prevents a: A. B.
C. D.

The pyloric sphincter separates the terminal portion of


the stomach (gastric antrum) from the beginning segment
of the small intestine (duodenum). This sphincter, which consists of a ring of connective tissue preceded by two

rise in the blood pH. decrease in the blood pH.


rise in the blood volume. decrease in the blood volume.

distinct rings of circular smooth muscle, carries out two

important functions. First, it helps to regulate the rate of gastric emptying into the duodenum. The transfer of the
stomach's contents to the duodenum is dependent on the

duodenum's ability to process chyme. Second, it prevents


the reflux of small intestinal contents into the antrum.
90.

These two functions are controlled by both hormones and

100 mM HCl was injected into the duodenum of a dog while the contractile activities of both the
antrum (AC) and duodenum (DC) were measured. Which of the following graphs BESTS represents the expected results of the experiment?
A.

neural input. Norepinephrine and cholecystokinin (CCK), for example, both cause a constriction of this sphincter.
It has been discovered that several distinct mechanisms

can lead to an alteration in the rate of gastric emptying. A

lowering of the pH in the duodenum has been shown to slow gastric emptying. This decrease in pH has been shown to not only affect motility, but also increase the
release of bicarbonate ion into the small intestine via the

iiiiiiiiiiiiiii inn minimi


iniiiiiiiiiiiiiiiiiiiiiiimi
t
HCL Infusion Starts

AC

hormone secretin. In addition to pH, the presence of fat results in a retarded gastric emptying. This response is believed to be carried out mainly by the hormone CCK. The hormone gastric inhibitory peptide (GIP) is also believed to be a component of this mechanism, as GIP has been shown to decrease the rate of gastric emptying. Finally, the secretion of gastrin lowers the rate of gastric emptying. Gastrin is secreted in response to the presence of amino acids and peptides in the stomach.

DC

t
HCL Infusion Ends

B.

lllllllllllllllllllll

AC

lllllllllllllllllllll
87. Based on information in the passage, which of the following statements is TRUE? A. B. A low rate of gastric emptying may lead to
duodenal ulcers.

DC

t
HCL Infusion Starts

t
HCL Infusion Ends

A high rate of gastric emptying may lead to


stomach ulcers.

IIIIIIIIIIIIIII! iiiiiiiiiiiiiii mil minimi


t
HCL Infusion
Starts

AC

C.

Regurgitation of duodenal contents may lead


to duodenal ulcers.

DC

D.

Regurgitation of duodenal contents may lead


to stomach ulcers.

t
HCL Infusion
Ends

D.

88. Norepinephrine is released in response to

sympathetic stimulation and acts to increase pyloric sphinctercontraction. Norepinephrine is an example


of a:

imiiimiiiiimiii

AC

iiiiiiiiiiiiiiiiiiiiiiiiiiiiii
mineralocorticoid. catecholamine.
steroid.

DC

A. B. C.

t
HCL Infusion
Starts

t
HCL Infusion Ends

D.

peptide.

Copyright by The Berkeley Review

192

The Berkeley Review

Specializing in MCAT Preparation

Biology

Gastrointestinal Junction

Passage XIV

91. As stated in the passage, GIP is involved in decreasing the rate of gastric emptying in response
to fats in the small intestine. This hormone is most

likely to increase the secretion of which of the following hormones?

A.
B.

Glucagon
Bile

C.
D.

Insulin
Cortisol

92. Cholecystokinin is known to stimulate contraction of the antrum and promote constriction of the pyloric sphincter. The overall effect of CCK is to:

A.
B.

C.
D.

increase the rate of gastric emptying, and contribute to the mixing of stomach contents. increase the rate of gastric emptying and inhibit the mixing of stomach contents. decrease the rate of gastric emptying and contribute to the mixing of stomach contents. decrease the rate of gastric emptying and inhibit the mixing of stomach contents.

93.

Which of the following statements is FALSE regarding parietal cell secretion of protons and pancreatic release of bicarbonate ion?

A. B. C.
D.

The pH of the stomach decreases. The pH of the blood increases. The acid entering the small intestine is
neutralized.

The bicarbonate ion is derived from carbon dioxide.

Copyright by The Berkeley Review

193

The Berkeley Review Specializing in MCAT Preparation

Biology
Passage XV (Questions 94-100)

Cholera Toxin

Passage XV

96. Cholera toxin affects a second-messenger signal


cascade. The toxin would NOT affect the actions of:

Cholera is a potentially lethal disease caused by the


bacterium Vibrio cholerae. Individuals afflicted with cholera suffer from severe dehydration due to diarrhea,
which left untreated can be fatal.

A. B. C.
D.

epinephrine. gastrin. norepinephrine.


testosterone.

V. cholerae bacteria replicate in the small intestine of the host after infection (usually from contaminated

97. Why is rehydration therapy alone not sufficient to


cure severe cholera?

drinking water), producing a protein known as cholera


toxin (CT). CT then binds to the surface of intestinal

epithelial cells, and through a complex series of events irreversibly alters membrane-associated G-proteins such that they are constitutively active, allowing for the constant overproduction of the second messenger cyclicAMP. This triggers a sodium and chloride efflux that is responsible for the water loss associated with diarrhea. Rehydration therapy for cholera sufferers takes advantage
of an inwardly-pumping sodium/glucose symport present in the apical membranes of intestinal cells.

A.

B.
C.
D.

The body cannot establish normal blood pressure after severe dehydration. Pores in the plasma membrane will allow water to eventually leak back into the intestinal
lumen.

Chloride ion must be ingested to replace that


which is secreted.

The underlying bacterial infection must be treated so that dehydrating diarrhea ceases.

CT is frequently used in research to help elucidate the functions of G-protein-related signal transductional
mechanisms. Recently, lines of mice have been

98.

Humans are one of the few mammals affected by


cholera. Unaffected mammals:

engineered which have a transgene encoding the CT protein linked to a promoter which allows protein expression only in cells of the pituitary gland which produce growth hormone. These mice exhibit gigantism,
an effect of excessive growth hormone.
99.

A. B. C. D.

lack G-protein mediated signaling. always produce large amounts of cyclic-AMP. lack membrane receptors for cholera toxin. normally have low cyclic-AMP levels.

Assuming they could be applied specifically to the pituitary, which of the following would be an effective inhibitor of the gigantism seen in the transgenic mice described above? I. II. An enzyme which degrades cyclic-AMP Excessive normal G-proteins HI. Excessive cyclic-AMP
I only II only III only II and III only

94.

Cholera toxin causes intestinal cells to secrete large amounts of Na/Cle into the intestinal lumen. How does this cause dehydration?

A.
B.

Salt loss triggers the opening of waterpermeable pores in the plasma membrane. Salt loss creates an osmotic gradient, which
water follows out of the cell.

A. B.

C.
D.

C.
D.

Salt loss triggers the active transport of water


out of the cell.

The distal convoluted tubules of the kidney


reabsorb more water.

100. A recent hypothesis suggests that individuals who are heterozygous for the recessive gene that causes cystic fibrosis (CF) may exhibit some resistance to cholera (i.e., the CF gene confers some selective advantage). Which of the following statements provides evidence in support of this theory?

95. What should be given to help rehydrate a cholera patient?


A. Distilled water.

A.

B.
B. C. Water and Na ions. Water and glucose.

C.
D.

The frequency of the CF gene is low in populations chronically exposed to cholera. The frequency of the CF gene is high in populations chronically exposed to cholera. The frequency of the CF gene is high in populations rarely exposed to cholera.
Individuals with cholera are often

D.

Water, glucose, andNa ions.

heterozygous for the CF gene.

Copyright by The Berkeley Review

194

The Berkeley Review Specializing in MCAT Preparation

Biology
Passage 1(1 - 5)

Gastrointestinal St Renal

Section III Answers

Gastrointestinal Smooth Muscle

1.

A is correct, single-unit fibers are coupled, with many individual cells containing gap junctions. This question draws on our knowledge ofsmooth muscle types. Even ifwe did not know the answer right from the question, the answer can be arrived at through some common sense. If something is acting as a single unit, then it is likely that
the muscle fibers which make up the unit will be coupled to each other. Therefore we can eliminate choice B.

Furthermore, in order for the muscle fibers to be electrically coupled to each other, many individual cells should have gap junctions. Remember, gap junctions offer connections between two different cellular cytoplasms, which
will allow current to move from cell to cell in a rapid fashion. The correct choice is A.

2.

C is correct, calcium channels is slow. As in cardiac muscle, an inward calcium current is an important component of the action potential in smooth muscle. We are told that the conduction velocity is low along smooth muscle fiber.
It is logical to assume that the conduction velocity of action potentials along GI smooth muscle fibers is slow because activation of the calcium channels is slow. This question wasjust designed to makeone think aboutthe role
of calcium in the conduction velocity of an action potential of smooth muscle. The correct choice is C.

3.

C is correct, number of action potentials within a set. We are looking at the diagrams to getan understanding of the relationship between force and action potentials. In the graphs we see three action potentials giving rise to a larger force of contraction when compared to two action potentials within the set. Therefore, we can say that the size of the force depends on the number of action potentials within a burst. There is simply noevidence or support from the
graph to consider the other answers as viable possibilities. We cannot compare amplitudes as the action potentials are all of the same size. Thus, the force of contraction depends on the number of action potentials. The correct
choice is C.

4.

C is correct, mixing of intestinal contents, breaking larger particles into smaller ones. As stated in the passage, segmentation allows alternating contraction and relaxation within the circular muscle. This type of movement is similar to opening and closing your hand. If you had a tomato in your hand, it would soon turn to mush. The same happens to food within the intestines. It is continually being broken down into smaller and smaller pieces. There is no indication that absorption of nutrients occurs because of segmentation, nor is there any direct evidence that segmentation causes movement of the intestinal contents through the GI tract. Even though peristalsis and
segmentation may stimulate (indirectly) the secretion of digestive enzymes, this is not the best answer choice. The
correct choice is C.

5.

D is correct, breaking larger particles into smaller particles increases the surface area available to digestive enzymes. Wc are told from the question that mixing aids in digestion. We know that mixing will break larger particles into smaller ones. This aids in digestion because the surface area exposed to digestive enzymes will have
increased. These enzymes will break down complex molecules into those molecules that can be absorbed across the layer and eventually be used by the body. While the other answers may seem attractive, one has to ask themselves,

what would this do to aid in digestion. Moving through the gut faster does not seem to aid in digestion, thereby eliminating choice A. Furthermore, there is simply no evidence from the passage that smaller molecules will carry less charge or cross over the epithelial layer of the lumen any easier. Absorption will most likely be occurring through channels in the cell layer. Therefore, we can say that increasing the surface area available to digestive
enzymes will aid in digestion. The correct choice is D.

Passage II (6 - 12)
6.

Protein Turnover

A is correct, I only. Enzymes are proteins, and their amino acids, can themselves be digested into amino acids and taken back up to feed the body. Choice I is correct. Mucus, a protective coating on the GI tract, is made of polysaccharides. Mucus is not digestible. How could it protect tissue if it were digested by our enzymes? Choice II is incorrect. Bicarbonate is not protein. Choice III is incorrect. The correct choice is A.
D is correct, 94%. If we merely looked at food intake and fecal output, wc would have calculated 90/100 = 90%. This did not consider secretion of proteins from the gut that were recycled. Choice C is incorrect. If we did not
include food intake, we would have calculated 63%. Choice A is incorrect. Choice B is also incorrect. The total

7.

protein available was 100 gm from the diet and 70 gm from gut secretions to make a total of 170 gm of protein available. 10 gm was excreted in feces, so 160 gm was digested and absorbed. 160/170 = 94%. Notice that the skin
is not included in this calculation The correct choice is D.

8.

C is correct, urea. Choice A is the chemical structure for uric acid. Choice B is ammonia. Choice C is urea.

Choice D is hydrazine. The correct choice is C.

Copyright by The Berkeley Review

195

The Berkeley Review Specializing in MCAT Preparation

Biology
9.
day: The correct choice is D.

Gastrointestinal St Renal

Section III Answers

D is correct, white cells, liver, muscle, gut. From Figure 1, look at the amount of protein each tissue turns over each

10.

B is correct,give isotopically labeled albumin, sample the blood periodically, and calculate the decay rate of labeled
albumin. The concentration of albumin will not give information about the turnover rate. Choice A is incorrect. An

isotopic label will allow quantification of the decay rate of the label is the blood as the albumin is recycled. A nonlabeled albumin will not help. Choice C is incorrect. Since a turnover rate in the body involves many different interactions, studying this is a test-tube would not work. Choice D is incorrect. The correct choice is B.
11. D is correct, skeletal muscle. The first tissue to break down would be the most expendable one. Smooth muscle is the muscle of the diaphragm and many other internal organs. This is not expendable. Choice A is incorrect.
Cardiac muscle is heart tissue. It is not the first to be broken down either. Choice B is incorrect. Enzymes of

energy metabolism are required even during a fast, especially the enzymes controlling gluconeogcnesis. They are
not used first, cither. Choice C is incorrect. Skeletal muscle serves as a reserve of amino acids for use during a fast.
It is the first tissue to break down. D is correct.

12.

C is correct, between 0.6 and 1.5 gm/kg. The nitrogen balance will be zero mg/kg at the appropriate protein intake. The chart moves from negative to positive between 0.6 and 1.5 gm/kg. This means zero is contained somewhere in
that intake range. The correct choice is C.

Passage III (13-20)


13.

Liver, Pancreas, St Intestines

A is correct, CCK. CCK from the small intestine is stimulated by free fatty acids entering the duodenum. CCK secretion will stimulate bile salt and pancreatic juice secretion. Secretin will be stimulated by gastric acids and high protein content in the small intestine. Hie correct choice is A.

14.

D is correct, enterokinase. Enterokinase (enteropeptidase) is the enzyme small intestine. It catalyzes the conversion of trypsinogen to trypsin. conversions. Secretin and CCK are both hormones that will stimulate pancreas, while bile salts, which are not enzymes, are produced in the liver.

located within the mucosal layer of the Trypsin will catalyze the remaining release of these proenzymes from the The correct choice is D.

15.

C is correct, protein deficiency. The lack of enteropeptidase will result in decreases in the breakdown of protein products entering the small intestine. Breakdown of these proteins is essential for absorption by the small intestine. Enteropeptidase does not affect fat or B12 metabolism. The correct choice is C. A is correct, secretin. Secretin is stimulated by gastric acids entering the small intestine. It will stimulate secretion of bicarbonate from the pancreas to neutralize the acidity of the gastric juices. Enterokinase and proenzymes are enzymes that do not affect gastric acids. CCK will stimulate the secretion of bile salts needed for the emulsification
of fats in the small intestine. The correct choice is A.

16.

17.

A is correct, trypsin. Acute pancreatitis is produced by abnormal levels of trypsin, which is produced as a proenzyme (trypsinogen) in the pancreas. Trypsin is a powerful enzyme that is not normally active in the pancreas. Activation of this enzyme within the pancreas will cause the breakdown/digestion of the pancreas itself. One type of damage to the pancreas involves lysolecithin (as outlined in the question). Secretin is a hormone in the small intestine. Bile salts are not found in the pancreas. Endopeptidase is a fictitious enzyme. It sounds like enteropeptidase (also known as enterokinase), which is the enzyme responsible for converting trypsinogen to trypsin. The correct choice is A.
C is correct, dissociate into CCb and H2O in order to allow the NaCl solution in the small intestine to remain

18.

neutral. The whole point behind the function of the small intestine is to neutralize the acidic chyme that enters from

the stomach. The bicarbonate ion (HCC>3e) that is released from the pancreas neutralizes the HCl from the stomach
by forming the neutral salt, NaCl. If we want to form more of this neutral salt, then we must pull the reaction given in the question to the right (via LeChatelier's principle). We can do that by removing carbonic acid (H2CO3). How? By allowing it to dissociate to CO2 and H2O. The CO2 is absorbed into the blood and carried back to the lungs where it is blown off as a gas. The H2O becomes part of the intestinal fluid. One reason that we do not want to increase the acidity of the intestinal lumen is because peptic ulcers can result. The correct choice is C.

19.

C is correct, pancreatic ductal cells. Severe duodenal ulcers, leading to the removal of the duodenum, will result in the removal of cell-types associated with the upper small intestine. Two important cell types are those that secrete
CCK and secretin. CCK stimulates gallbladder contraction. The gallbladder stores and concentrates bile. If the

Copyright by The Berkeley Review

196

The Berkeley Review Specializing in MCAT Preparation

Biology

Gastrointestinal St Renal

Section III Answers

cells that secrete CCK are removed, then contraction of the gallbladder will not increase. The gallbladder tends to stay in the relaxed state for longer periods of time. Secretion of HCl from the parietal cells in the stomach is inhibited by GIP, an enterogasterone released by cells in the duodenum. Removal of GIP-secreting cells will remove the inhibition on the parietal cells and lead to an increase in parietal cell secretion (of HCl). Both CCK and
secretin act to inhibit gastric emptying into the small intestine. Removal of the cells that secrete these two hormones will result in an increase in the passage of chyme into the small intestine. Secretin also acts to increase the

production of bicarbonate form the pancreatic ductal cells. Removal of the cells which secrete secretin will lead to a

DECREASE in the production of pancreatic bicarbonate and a DECREASE in pancreatic ductal cell activity. The
correct choice is C.
20.

B is correct, delay the appearance of acid in the stomach. The name antacid tells us what it does. The antacid

tablets act to neutralize the increase of gastrointestinal acid. This is exactly what the two equations in the question
are addressing. We do not see acid being formed in either equation. Instead, we see the neutralization of HCl. We can eliminate choices A and C. Even though it is true that water is a by-product of the reaction with the antacid and

HCl, it is not addressing the question as to why antacids should be taken 1 to 3 hours after a meal. The delay in taking antacids after a meal is simply to delay the appearance of acid in the stomach. This is seen from the graph in the question. If the antacid is taken 1 hours after a meal, the appearance of acid in the stomach is delayed for about
2 hours. We can eliminate choice D. The correct choice is B.

Passage IV (21 -27)


21.

Digestion Safeguards

C is correct, trypsin would be inactive, and the otherpancreatic enzymes would remain as zymogens. This is to test our understanding of the passage. Trypsin is required to activate all other secreted pancreatic digestive enzymes.
Raw soybeans contain a trypsin inhibitor, so trypsin would be inactivated. Choices A and D are incorrect. Since the other enzymes require cleaving by trypsin to achieve activity, they will remain inactive as zymogens. Choice B is
incorrect. The correct choice is C.

22. 23.

C is correct, gastric epithelium. The cells that are most rapidly turning over of those listed arc in the gastric
epithelium. Neurons and muscle cells turnover minimally, if at all, in adults. The correct choice is C.

D is correct, I, II, and III. Since the passage tells you mucus lines the digestive tract, wc could just pick choice D
and be done. However, mucus is present in the mouth to protect the tissue and to lubricate food. Choice I is correct.

Mucus is present in the colon. Choice II is correct. Mucus is very important in protecting the small intestine,
particularly the duodenum, from auto-digestion. Choice III is correct. The correct choice is D.

24.

D is correct, 2.0. The stomach is acidic, choice C, a basic pH, is incorrect. A pH of 6.8 is almost neutral, so choice A is incorrect. A pH of 0.2 is 10 times more acidic than a pH of 2.0, and is much too acidic. A pH of 2.0 is correct.
The correct choice is D.

25.

D is correct, I, II, and III. Enterokinase activates trypsin. If enterokinase entered the pancreas via the pancreatic duct, then trypsin would be activated, and it would activate the other zymogens. They would all go to work on the pancreatic tissue, causing damage so that pancreatic enzymes leak into the blood. Extensive damage could damage the islet cells that secrete insulin, leading to impaired insulin secretion. Actually, a person would probably be very
sick or have some sort of surgery before the damage reached this level. It is important that we think of both the exocrine and the endocrine parts of the pancreas. The correct choice is D.

26. 27.

A is correct, I only. The stomach's parietal cells arc the acid-secreting cells. HCl, hydrochloric acid, is the acid in
stomach contents. The correct choice is A.

C is correct, carbohydrate. The give-away is in the name polysaccharide. This term refers to sugar units, so it is a
carbohydrate derivative. The correct choice is C.

Passage V (28 - 33)


28.

Digestion Of Fats

C is correct, hydrolyzation of fat droplets in the small intestine. The hydrolyzation of fat droplets is dependent on the enzyme pancreatic lipase. This enzyme is activated by the pancreatic enzyme trypsin. Emulsification of fats is dependent on bile salts secreted by the gall bladder. Stimulation of bile secretion is induced by increase in fat
content in the small intestine and CCK stimulation. The correct choice is C.

29.

A is correct, emulsification of fats. The removal of household grease docs not involve micelle endocytosis, exocytosis of fatty acids, or packing of fat products in the endoplasmic reticulum. The correct choice is A.
197

Copyright by The Berkeley Review

The Berkeley Review Specializing in MCAT Preparation

Biology
30.

Gastrointestinal St Renal

Section III Answers

C is correct, decrease in intestinal cell microvilli. Because of the soap-like appearance of the stools, bile salt

secretion is probably normal. Therefore, the problem most likely lies in the absorption of the fat droplets.
Decreases in the surface area of the small intestine due to decreases in the microvilli will initiate this response.

Increases in pancreatic lipase alone will notcause this problem because any increase in free fatty acids and micelles will usually be absorbed by the intestines. Chylomicron production is not involved with this process. The correct
choice is C.

31.

C is correct, inhibition of cholesterol absorption in the small intestine. Cholesterol is dependent on bile salts for

absorption in the small intestine. Fatty acids, even though aided by bile salts, are still able to be absorbed. With the
increase in fatty acid and cholesterol content in the small intestine, CCK secretion will be elevated. The correct
choice is C.

32.

D is correct, cholesterol. By the time chylomicrons reach the liver, lipoprotein lipase will have cleaved all the fatty acids from the molecule. Lipoprotein lipase is found in the arterial system. The bile salts remaining in the circulatory system are not attached to the chylomicrons. The correct choice is D.
C is correct, decreased levels of lipoprotein lipase. If levels of lipoprotein lipase are decreased, chylomicron deposition of fats in the liver will increase. Lipoprotein lipase, while in the peripheral circulation, will cleave triglycerides from the chylomicrons for deposition in fat stores in the periphery. Without lipase the liver will cleave and store these same fats. Decreased levels of CCK and pancreatic lipase will only allow fatty acids to be lost in the stools due to malabsorption. The correct choice is C.

33.

Passage VI (34 - 40)


34.

Diabetic Diet Experiment

A is correct, I and III only. Table 1 indicates a drop in the plasma glucose levels from the baseline to the HF group. Choice I is correct. Insulin requirements were lower in the HF diet phase compared to baseline. Choice II is incorrect. Less glucose was excreted in the urine in the HF group compared to baseline. Choice III is correct. The
correct choice is A.

35.

D is correct, this decrease is beneficial because glycosylated hemoglobin indicates persistent levels of high plasma glucose. When blood glucose is elevated persistently, then plasma proteins become glycosylated (glucose residues are attached to the proteins). A decrease in glycosylated hemoglobin means that, on the average, blood glucose levels were lower during the past few months. (The life span for a RBC is about 120 days). This would be a beneficial change. Choices A and B are incorrect. Glycosylation occurs when plasma glucose is high. Choice C is
incorrect. The correct choice is D.

36.

C is correct, oleic acid. The notation for the fatty acids is (# of carbons: # of double bonds). In the HF diet, the amount of 18:1 is dramatically increased. In the passage, you are told that the HF diet is specifically higher in monounsaturated fatty acids. Monounsaturated means having one double bond. Palmitic acid (16:0) nor stearic acid (18:0) have double bonds. Choices B and D are incorrect. Palmitoleic acid is 16:1, and oleic acid is 18:1. Choice A
is incorrect. The correct choice is C.

37.

A is correct, olive oil. Even if you have no clue about nutrition, use your test-taking skills to eliminate wrong answers or see patterns. One of these things is not like the others. Olive oil is from plants and the others are from animal fats. Anyway, olive oil is a good source of monounsaturated fatty acids (oleic acid to be specific). The other fats are more saturated (they are solid at room temperature). The correct choice is A.

38.

C is correct, II and III only.

Although VLDL was lower in Table 2, LDL was not. Choice I is incorrect.

Triglycerides are lower on the HF diet. (Use the significance symbols to help interpret the data. Researchers must apply statistical methods to see if a finding is real or not.) Choice II is correct. HDL did increase on the HF diet.
Choice IQ is correct. The correct choice is C.

39.

C is correct, the beta cells of the pancreas. The alpha cells of the pancreas secrete glucagon. Choice A is incorrect.

The exocrine portion of the pancreas secretes digestive enzymes, while the endocrine portion secretes hormones.
Choice B is incorrect, as is choice D. The correct choice is C.

40.

D is correct, the liver. The word hepatic refers to the liver. The hepatic portal vein drains the blood supply of most

of the organs of digestion and passes it to the liver first for processing. Insulin released from the pancreas first
reaches the liver. Choices A, B, and C are incorrect. The correct choice is D.

Copyright by The Berkeley Review

198

The Berkeley Review Specializing in MCAT Preparation

Biology
Passage VII (41 -48)
41.

Gastrointestinal St Renal

Section III Answers

The Kidney

B is correct, an increase in glomerular pressure, followed first by an increase and then by a decrease in glomerular
filtration rate. If the efferent arteriole is constricted, blood cannot flow pass the point of constriction. This leads to an increase in the glomerular pressure, and allows us to immediately pick choice B or D. Constriction of the efferent arteriole also means that the blood flow in the glomerulus will decrease, and more plasma will begin to filter out into Bowman's capsule. With this information we can pick choice B. The more the plasma filtered out into Bowman's capsule, the more the solute concentration begins to increase in the glomerulus. Eventually this will decrease the glomerular filtration rate. The filtrate simply will not be able to flow out of the glomerulus and into the capsule because of the change in the concentration gradient. The correct choice is B.

42.

C is correct, reduced plasma glucose concentrations. Recall that in the proximal convoluted tubule the transport of sodium across the apical membrane (from the lumen of the proximal convoluted tubule to the cytoplasm of the lumenal epithelial cell) is mediated by proteins called symports (a cotransportcr). For example, these symports allow for the passage of both sodium and glucose together into the cell. Another example is the symport that allows for the passage of both sodium and various amino acids into the cell. If the concentration of glucose is decreased in the filtrate, the amount of sodium that can be reabsorbed at the level of the proximal convoluted tubule is reduced. The
correct choice is C.

43.

A is correct, afferent arterioles and decreases the volume of urine flow. Sympathetic innervation of the kidneys primarily affects the afferent arterioles. This innervation constricts the afferent arterioles and reduces the glomerular filtration rate. The urine output will be dramatically reduced. The correct choice is A. A is correct, (see the diagram below). Notice that along the X-axis the plasma osmolarity is steadily increasing. At some particular point the solute concentration in the plasma will be so great that osmoreceptors, located in the supraoptic nuclei of the anterior hypothalamus quickly respond to a change in the osmolarity of the extracellular fluid (especially to the sodium ion).

44.

X-Axis

Since the osmolarity of the plasma is becoming higher (hyper-osmotic), water will flow down its concentration gradient (osmosis) and out of the osmoreceptors and cause them to shrink. As the receptors shrink their rate of discharge increases. If the rate of discharge increases, the posterior pituitary gland releases more ADH into the plasma. This is exactly what we see as the plasma ADH concentration along the Y-axis begins to increase. If there is
more ADH in the system, then there is more ADH to act at the baso-lateral membrane of the late distal tubules,

collecting tubules, and collecting ducts. In other words, the cells in those locations are more permeable to water. Water is reabsorbed into the blood and the urine becomes more concentrated. Notice that as ADH is being released,
the osmolarity does not change that much. The correct choice is A.
45.

D is correct, (see the diagram below). Again, along the X-axis the plasma concentration of glucose begins to increase. As soon as the concentration of glucose reaches a particular point where it can no longer be reabsorbed from the lumen of the proximal convoluted tubules, it is passed through the lumen of the kidney tubules in an ever increasing rate.

X-Axis

Copyright by The Berkeley Review

199

The Berkeley Review Specializing in MCAT Preparation

Biology

Gastrointestinal St Renal

Section III Answers

This is shown along the Y-axis where we observe a continual increase in the rate of excretion of glucose in the urine of the kidney. We do not see this type of curve in the other three answers. The correct choice is D.
46. C is correct, is a steroid hormone secreted by the adrenal medulla. Aldosterone is a cholesterol-derived steroid which is synthesized in the zona glomerulosa (the outermost region) of the adrenal cortex. Aldosterone is referred to

as a mineralo-corticoid because of its affect on electrolytes such as Na and K. This hormone has a half-life of about 30 minutes. Aldosterone promotes Na reabsorption to a great extent at the late distal tubular region, the cortical collecting tubules, and the collecting ducts. Aldosterone also controls the secretion of K ions into the
lumen of the distal convoluted tubule. The correct choice is C.

47.

B is correct, increased muscle contraction. Excessive aldosterone secretion results in an excessive loss of K from

the extracellular fluid and into the urine. This lowers the plasma K concentration, a condition referred to as

hypokalemia. Loss of K ions from a cell tends to decrease the K concentration in that cell, making the resting membrane potential for K more negative that it would have been had aldosterone levels been normal. Recall that as K rushes out of the nerve cell, thatcell tends to hyperpolarize. Hyperpolarization leads to a decrease in the muscle
contraction, not an increase. The correct choice is B.

48.

C is correct, electrolytes. If we were to centrifuge down a sample of blood, we would find "formed elements" such

as red blood cells (erythrocytes), white blood cells (leukocytes), platelets, and proteins (high molecular weight) at the bottom of the testtube. In the plasma wc would find such substances as sugars, amino acids, and the electrolytes, to name but a few. Electrolytes are rather small molecules that can easily be filtered by the glomerulus. The correct
choice is C.

Passage VIII (49 - 53)


49.

Kidney & pli

C is correct, excessive vomiting. We are looking for a situation which will not result in the gain of hydrogen ions. One must realize that a potential source of net bodily gain or loss of hydrogen ions is gastrointestinal secretions
leaving the body. Recall that vomiting has its origins in the stomach, which contains acidic secretions. This acidic vomitus leaving the body will result in a net loss, not a net gain of hydrogen ions. The correct choice is C.

50.

B is correct, a increased blood hydrogen ion concentration. It is clear from figures one and two that two events are occurring. One event is the secretion of hydrogen ions from the tubular cells of the kidney into the renal tubule. These hydrogen ions, after reacting with a buffering system, will leave the body in the urine. The body is removing hydrogen ions. This tells us that the hydrogen ion concentration must have been elevated, and the kidney is compensating for this elevation. Second, Figure 2 is indicating the reabsorption of bicarbonate. One must understand that reabsorption of a bicarbonate ion is equivalent to the loss of a hydrogen ion. Therefore, this reabsorption is also acting as a compensatory mechanism to relieve an elevated hydrogen ion concentration. The correct choice is B. D is correct, secondary active transport. This question draws on your previous knowledge of transport systems. As one can see, a hydrogen ion is transported out of the tubular cell while a sodium ion is moved from the renal tubule

51.

lumen to the cell. Sodium moves into the cell because it is falling down its concentration gradient. Even though this is not stated in the passage, one should know that the sodium ion concentration is always higher cxtracellularly when compared to intracellularly. The reason sodium can continue to fall down its concentration gradient is because of a sodium/potassium ATPase pump which uses the energy of ATP to pump sodium out of the cell. One can refer to sodium moving down its gradient as primary active transport. To transport the hydrogen ion across the membrane, one can harness the energy found in the sodium concentration gradient (really a form of potential energy). Since the hydrogen ion is cotransported along with the sodium ion, it is given the term secondary active transport. The correct
choice is D.

52.

B is correct, a decrease in the hydrogen ion concentration, with a decrease in the level of CO2. It should be clear

that an alkalosis is a situation where the pH is above that of neutral, and there is a deficiency of hydrogen ions. In this case, we have an alkalosis that is brought on by respiratory troubles. Recall the equation that is shown in Figure 1. In order to produce a situation where there is a deficiency in the hydrogen ion, we must first create a situation where there is a deficiency in C02- A loss of carbon dioxide will pull the equation towards CO2 to compensate for the loss. The result of that shift is that there is a decrease in the amount of hydrogen ions. A reduction in the level of hydrogen ions clearly results in alkalosis. The correct choice is B.
53. C is correct, decrease, due to increased ventilation. We have a situation here where there is simply too much hydrogen ion. We want to restore the original level of hydrogen ion. Again, refer to the equation shown in Figure 1.
200

Copyright by The Berkeley Review

The Berkeley Review Specializing in MCAT Preparation

Biology

Gastrointestinal St Renal

Section III Answers

One way we can reduce the hydrogen ion concentration is to lower the level of carbon dioxide. This should sound

very familiar, like the previous question. How do we lower the partial pressure of carbon dioxide? We breathe air
out with an increased frequency. In other words, we increase the ventilation rate. This will cause a decrease in the

partial pressure of carbon dioxide, and shift the equation in such a manner so we experience a decrease in the level
of hydrogen ion. This will help restore the normal level of hydrogen ion from its current elevated level. The correct
choice is C.

Passage IX (54 - 60)


54.

Dialysis St Ultrafiltration

B is correct, I and II only. Since the solution is introduced into the abdominal cavity through catheters, there is always a possibility of infection. Choice I is correct. Depending on the person's state of hydration, dialysis can lead to electrolyte imbalance. Choice II is correct. Since there is no pressure involved in this technique, there is no
ultrafiltration occurring. Choice III is incorrect. The correct choice is B. D is correct, glomerular capillaries. The glomerular capillaries are fenestrated, that is, they contain large pores. Although large proteins cannot pass, small molecules and water are free to pass. The pressure of the blood acts to force the molecules through the semipermeable capillaries. Choice D is correct. The ultrafiltrate collects in the glomerular capsule. The afferent arterioles are not permeable to these molecules. Choice C is incorrect. The

55.

peritubular capillaries are involved in resorption. Choice A is incorrect. The loop of Henle is involved in concentrating the urine. Choice B is incorrect. The correct choice is D. 56. A is correct, no enzyme activity in the bag. Although enzymes are large compared to atoms, they are small

compared to a pinhole. This is why you test your dialysis tubing for holes before you begin enzyme purification. Since there is no way the amount of enzyme can increase, choice B is incorrect. Since there is an effect caused by the pinhole, choice C is incorrect. Choice D is incorrect since the enzyme would all (or almost all) move out during
an overnight dialysis. The correct choice is A.

57.

A is correct, erythropoetin. The kidney produces erythropoetin in response to low blood volume. This hormone

signals the bone marrow to make more red blood cells. If a kidney were diseased, this hormone may not be produced. Choice A iscorrect. Choices B, C and Dare incorrect because the kidney does not make these hormones.
ADH is released from the posterior pituitary where it is stored after its manufacture in the hypothalamus. The

adrenal gland produces epinephrine and norepinephrine. Insulin is produced by the beta cells of the pancreas. The
correct choice is A.

58.

B is correct, edema. You are given 2 choices that give the same answer, choices A and D. Eliminate them. If

uremia is present, the kidney is probably diseased, so urination is probably decreased. Choice C is incorrect. Edema
means an accumulation of fluid in the intracellular spaces, leading to tissue swelling. Uremia indicates kidney

function is declining. This would lead to an increased solute load in the blood and retention of fluid. The fluid increase would mainly move to the extracellular spaces, since cells have only a little room for expansion. The
correct choice is B.

59.

B is correct, use of a dialysis solution that is isotonic to the blood in glucose and salts. The pores of dialysis membrane are not small enough to retain the small molecules of glucose and salts. Also, if glucose and salts were retained in the dialysis membrane, urea would be, as well. Choice A is incorrect. Choices C and D are incorrect, because even through glucose and salts will enter the blood, they would be lost to a hypotonic dialysis solution.
these molecules. This will prevent loss. The correct choice is B.

Finally, to prevent movement ofglucose and salts, the dialysis solution should be isotonic in its concentration of
B iscorrect, 111 ml/min. 5 liters = 5000 ml. 5000 ml/45 min = 111 ml/min. Thecorrectchoice is B.
Caffeine

60.

Passage X (61 - 66) 61.

Aiscorrect, lipolysis. Lipolysis is the mechanism ofbreakdown of stored triglyceride to release FFA and glycerol
into the blood. This would raise FFA concentrations. Lipogenesis is the synthesis of fat. This would not raise FFA
levels. Choice B is incorrect. Increased fat oxidation would decrease plasma FFA levels. Choice C is incorrect.

Increasing dietary fat would increase chylomicrons, but not FFAs. Choice Dis incorrect. The correct choice is A. 62. C is correct, epinephrine. CCK is a hormone secreted from the duodenum in response to dietary fat. It causes gall
bladder contraction and release of pancreatic enzymes. Choice A is incorrect. Thyroid hormone does not increase
Copyright by The Berkeley Review
201

The Berkeley Review

Specializing in MCAT Preparation

Biology

Gastrointestinal St Renal

Section III Answers

plasma FFA concentration. Choice B is incorrect. Insulin decreases lipolysis, and increases fat storage. Choice D is
incorrect. Finally, epinephrinepromotes lipolysis to increase FFA concentration in the blood in response to stresses. Think of the fight or flight mechanism. The muscles will need fuel and the elevated FFAs will provide that. The
correct choice is C.

63.

D is correct, sucrose stimulated insulin, which decreased release of FFAs from adipocytes. Insulin turns down

lipolysis. If you havejust eaten, and insulin is secreted, your body does not need the extra fuel thatcirculating FFAs provide. You haveswitched from withdrawal from stores to depositing into yourstores. Sucrose docs not stimulate
glucagon, so choice B is incorrect. Fructose does not mediate the effect on FFAs, so choice A is incorrect. Sucrose
is not an inhibitor of the action of caffeine, so choice C is incorrect. The correct choice is D.

64.

C is correct, adenylate cyclase. The wrong answers in this question are all part of the cAMP activation pathway. Be careful! A hormone such as epinephrine stimulates a receptor on the cell. A second messenger, the G protein, communicates the message internally to the enzyme adenylate cyclase. Adenylate cyclase produces cAMP. cAMP activates protein phosphorylases. The question is for the enzyme, so adenylate cyclase is your answer. The correct
choice is C.

65.

D is correct, the effects are the same as drinking water. The answer to this question in the table. The *, t, and t indicate if differences between groups are significant. This is a very common way for data to be summarized. Experiment 2 has the symbol "*", which indicates that the results were not significantly different from Experiment 3, water. Although the numbers look like a slight increase, the * indicates there is actually no difference. The
correct choice is D.

66.

D is correct, I, II, and III only. This is an experiment to determine the control data for plasma FFA concentrations over time. To simulate the real experiment as much as possible, hot water and sodium saccharin were both included.
This means I, II, and III are all valid choices. The correct choice is D.

Passage XI (67 - 72)


67.

Kidney & Calcium

B is correct, osmotic force due to protein in plasma favors filtration. This is a false statement. Filtration is the

movement of substances from the glomerular capillaries into Bowman's capsule. The protein that is unable to be filtered creates an osmotic force which pulls the movement of fluid away from Bowman's capsule and into the glomerular capillaries. Therefore, the protein in the plasma creates an osmotic force which opposes, not favors, filtration. We are looking for a false statement, and choiceB certainly fits the description. The correct choice is B.

68.

C is correct, X is filtered and secreted, but notreabsorbed. This question is requiring one to interpret Figure 1. It is clear that substance X leaves the glomerular capillaries and enters the renal tubule at Bowman's capsule. This classifies as filtration. Also, it is apparent that substance X leaves the peritubular capillaries and enters the renal tubule after Bowman's capsule. This classifies as secretion. The only event that does not occur is movement of substance X back into the peritubularcapillaries. In other words, there is no reabsorption. Therefore, X is filtered
and secreted, but not reabsorbed is a true statement. The correct choice is C.

69.

B is correct, increased levels of parathyroid hormone, with increased removal of bone. This question is requiring
one to think about the role of parathyroid hormone. Wc have a decrease in the level of calcium, and so we will

therefore want to act to increase the levels of calcium. This results in an increase in the levels of parathyroid
hormone. Why? One of the functions of the parathyroid hormone is to remove bone. Removal of bone releases

calcium and phosphate from the organic molecules. The release of calcium acts to counteract the original decrease.
Therefore, we will see an increase in the level of parathyroid hormone, and an increase in the removal of bone. All
of the other choices are either false, or lead to a further decrease in calcium levels. The correct choice is B.
70. D is correct, the effect of the extracellular calcium concentration on membrane is distinct from its role as a mediator

of muscle contraction. Wc are told from the passage that a low extracellular calcium concentration leads to an increase in nerve and muscle cell excitability. In fact, a person with a low calcium concentration may suffer from

muscle spasms. We often think of calcium as being involved with the troponin-tropomyosin complex, where
calcium is needed to initiate contraction. In that situation, a low calcium concentration would lead to decreased

excitability or contraction. We are told that in this case, a low extracellular concentration leads to increased nerve and muscle excitability. Based on this apparent contradiction, we can conclude that the effect of calcium concentration on the membrane is distinct from its role as a mediator of contraction. In fact, these effects reflect

calcium's ability to bind to plasma membrane proteins that function as ion channels. The binding alters the state of
the these channels. The correct choice is D.

Copyright by The Berkeley Review

202

The Berkeley Review Specializing in MCAT Preparation

Biology
71.

Gastrointestinal St Renal

Section III Answers

Dis correct, a reduction in the tubular reabsorption ofphosphate. This question is not easy. The following logic
can be used to arrive at the answer: We have a low calcium concentration. We will want to counteract that decrease

with an increase. We will see an increase in the removal of bone due to increased levels of parathyroid hormone. This not only releases calcium, but from the passage, one should beable toconclude that it releases phosphate ion as
well. Now, if the extracellular concentration of phosphate were to increase as a result of this release, further

movement of calcium from bone would be hampered because a high extracellular concentration of phosphate would bind with calcium and cause the saltto be re-deposited on bone and other tissues. The point is that we want to get rid of the phosphate ion. One way to do this is to decrease the amount reabsorbed in the kidney. In that way, phosphate ion is released with the urine and calcium levels can return to normal. Again, we will most likely see a
reduction in the tubular reabsorption of phosphate ions. The correct choice is D.

72.

C is correct, the parathyroid gland. This question is simply asking us to recall the function of our endocrine glands. Parathyroid hormone, a protein hormone, is released by the parathyroid gland. While the parathyroid gland is
embedded in the thyroid gland, the glands arc distinct. The correct choice is C.

Passage XII (73 - 79)


73.

Lymph

A is correct, muscle and adipose tissue have first access to dietary fatty acids from chylomicrons. The chylomicron packaging system avoids direct delivery of fatty acids to the liver. This is stated in the text of the passage. Choice B is incorrect. Although microorganisms may be present in the lymph, they are not packaged into the chylomicrons. Choice C is incorrect. Lymph, not interstitial fluid, carries chylomicrons. Choice D is incorrect. The correct choice
is A.

74.

C is correct, water. Water is the main component of our body fluids. Since lymph is a derivative of interstitial fluid, it is mostly water. Fat soluble vitamins are usually present in very small amounts (jag), triglycerides may be present in larger amounts (grams), but water is the most abundant. There must be more water than chylomicrons so that they remain soluble. Although chylomicrons carry fat, neither they nor the fat they carry is the major component of lymph. Choices A , B, and D are incorrect. The correct choice is C.
D is correct, I, II, and III. All three vitamins are fat-soluble vitamins. The correct choice is D.

75.

76.

A is correct, to destroy microorganisms. Chylomicrons are produced and added by the intestinal cells. Choices B and C are incorrect. Intestinal nutrients are either packaged in chylomicrons and put into the lymph at the small intestine, or nutrients from the small intestine directly enter the hepatic portal vein and travel to the liver. The lymph node does not add intestinal nutrients. Choice D is incorrect. Lymph nodes filter microorganisms through the phagocytic cells, as mentioned in the passage. The correct choice is A.

77.

A is correct, I only. In the passage, the structure of the lymph vessel is described as similar to the vein. The vein does not have a muscular layer like the artery. Choice II is incorrect. Blood in the vein is moved by squeezing by the skeletal muscles. Choice I is correct. Blood pressure leads to the creation of interstitial fluid, it does not move lymph. Choice III is incorrect. The correct choice is A.

78.

B is correct, kidney. Urine is formed as an ultrafiltrate of blood in the glomerular capsule of the kidney. Choice B is correct. Neither the liver, the pancreas, nor the salivary gland produces an ultrafiltrate. Choices A, C, and D are
incorrect. The correct choice is B.

79.

B is correct, I and II only. After a meal, chylomicrons first enter the lymphatic vessels, then the blood vessels. Choice I is correct. Lymph does not contain erythrocytes (red blood cells). Choice III is incorrect. Lymphocytesare the second most abundant type of white blood cells in the plasma. They are also present in the lymph. Choice II is
correct. The correct choice is B.

Passage XIII (80 - 86)


80.

Renal Clearance

D is correct, [ U X PFa = ([ ]u x fu) + ([ Iv x pfv)- The principle of mass balance is the idea of what enters the

kidney must leave the kidney. That which enters the kidney will be from only the plasma of the renal artery. This can be quantified as [ ]A X PFA. There are two ways substances can leave the kidney. One is through the urine, quantified as Fu X (' ]u. The other is through the renal vein, quantified as PFU X [ |v. Therefore, mass balance is best represented as [ ]A X PFA = ( [ ]u X Fu) + ([ ]v X PFV). The correct choice is D.

Copyright by The Berkeley Review

203

The Berkeley Review

Specializing in MCAT Preparation

Biology
81.

Gastrointestinal St Renal

Section III Answers

B is correct, 0.0005 L. We are interested in knowing the volume of urine collected in 0.5 minutes. We need to know the rate of urine flow to figure out the volume. The rate of urine flow can be calculated using the relationship

established in the passage. In other words, Fu = (C) X ([ ]p)/[ ]u. Putting in the values, weget Fu = 50 ml/min X 1
mg/ml divided by 50 mg/ml, which turns out to be 1 ml/min. We arc asked for 0.5 minutes, so we have 0.5 ml as
our volume. 0.5 ml X 1 L/1000 ml = 0.0005 L. The correct choice is B.
82.

C is correct, GFR = [ ]u X Fu/[ substance]p. From the passage, we know that under certain conditions, the amount
of a substance filtered is equal to the amount excreted in the urine. The amount of a substance excreted in the urine will be [ ]u X Fu. To calculate the amount filtered, we can multiply the GFR by the concentration of the substance

in the plasma. Equating the two, we have GFR X [ ]p = [ ]u X Fu. Therefore, the GFR = [ ]u XFU = [ ]. The
correct choice is C.
83.

B is correct, the substance must not be reabsorbed, and must be secreted by the nephron. From the passage, we know that only under certain conditions can the amount filtered at the glomerulus equal the amount excreted in the

urine. It should make logical sense that one of thoseconditions should be that nothing is added or taken away from
the filtrate as it passes through the nephron. Statement B claims that the substance must be secreted into the

nephron. If this was the case, the amount excreted will be more than likely greater than the amount filtered.
Therefore, condition B cannot exist if our claim is to be valid. The correct choice is B.
84.

A is correct, Proximal Convoluted Tubule. This question is simply calling on our previous knowledge of kidney function. Recall that most reabsorption occurs in the proximal convoluted tubule. This is the first part of the nephron. The substances reabsorbed are returned to the venous system through the peritubular capillaries. The reabsorption of glucose takes place via a secondary active transport, and under normal conditions, 100% of glucose
is reabsorbed. The correct choice is A.

85.

A is correct, 3.5 mg/ml. The plasma threshold is defined as the level of plasma glucose which first gives rise to
glucose in the urine. Looking at the curve, we are interested in the line labeled excretion. When this line first

becomes positive, we need to look at the value of the blood glucose which gave rise to this first appearance of glucose in the urine. This line becomes positive between the values of 2 and 4. Looking carefully, the value of the
blood glucose is around 3.5 mg/ml. The correct choice is A.
86.

B is correct, the cationic species has the highest filterability because of the presence of anionic glycoproteins on the surface of all glomerular filtration components. Let us read the curve given in the question. Draw an imaginary vertical line in the graph. With this line, we can say that if a molecule has the same radius, the cationic species will have the higher filterability. Therefore, any choice that uses the argument that cationic or anionic species have
different radii is invalid. In other words, we can compare a positive and negative molecule that has the same radii, and know from the graph that the positive cationic species will have the greater filterability. That means there must

be some other reason for the enhanced filterability. The most likely reason is the presence of anionic glycoproteins
on the surface of all filtration components. The opposite signs will create an electrical attraction that can lead to an
enhanced filterability. The correct choice is B.

Passage XIV (87 - 93)

Gastrointestinal Junction

87.

D is correct, regurgitation of duodenal contents may lead to stomach ulcers. We know from the passage that the
pyloric sphincter carries out two essential functions. One of these functions is toprevent the reflux of small intestinal contents into the antrum. This is the case because the lining of the stomach is not protected from the bile that is secreted into the small intestine. The reflux of this bile containing content may contribute to the development of
stomach ulcers. So again, while the stomach is protected from the acid it secretes, it is vulnerable to the bile secreted

by the small intestine. The opposite holds true for the small intestine. The small intestine is protected from the bile, but is vulnerable to the acid from the stomach. Therefore, regurgitation of duodenal contents may lead to stomach
ulcers. The correct choice is D.

88.

B is correct, catecholamine. This question cannot be answered from any of the information in the passage, but must be answered solely on our own knowledge of hormones and hormone classification. Norepinephrine, along with
epinephrine and dopamine, are catecholamines which fall under the major category of amine hormones. The correct
choice is B.

89.

D is correct, a decrease in blood volume. If a hypertonic solution is introduced into the lumen of the small intestine,

this creates a situation where water will enter into the lumen from the vascular system. In other words, the water
Copyright by The Berkeley Review
204

The Berkeley Review

Specializing in MCAT Preparation

Biology

Gastrointestinal & Renal

Section III Answers

from the vascular system will cross the intestinal wall and diffuse to where there is a higher concentration of solute.
This will certainly impair the vascular system. In particular, it will cause a decrease in blood volume which could lead to a dangerous drop in arterial blood pressure. The correct choice is D. 90. A is correct. We are told from the passage that a decrease in the pH of the duodenum will cause a decrease in the

rateof gastric emptying. One way to achieve a decrease in gastric emptying would be to decrease thecontractility of the antrum and to increase the contractility of the duodenum. In this way, wc could achieveour goal of a decreased flow of material from the antrum into the duodenum. Looking at Graph A, the injection of HCl into the duodenum certainly does cause a decrease in thecontractility rate of the antrum. In addition, the injection of the HCl certainly does cause a significant increase in the rate of contraction for the duodenum. Therefore, Graph A BEST represents
our expected results. The correct choice is A.

91.

C is correct, insulin. We are looking for a hormone that is most likely to be released in response to a rise in GIP.
Since we are looking for a hormone, bile can automatically be eliminated. We know that GIP is itself released as a

result of fat in the small intestine. The fat will soon find its way into the vascular system by first going through the
lymphatic system. The fat will want to enter into the adipose tissue. Insulin is the hormone responsible for taking fat from the vascular system and facilitating its uptake into adipose tissue. Think in very general terms. We are in an absorbing state for the body, as food is being absorbed from the small intestine. For this reason, insulin is the hormone that is necessary, and GIP is stimulating the release of insulin in anticipation of the arrival of fat into the
vascular system. The correct choice is C.

92.

C is correct, decrease the rate of gastric emptying and contribute to the mixing of stomach contents. Wc know from the passage that CCK does retard the rate of gastric emptying. For that reason alone, we can eliminate choices A and B. We now must concentrate our efforts into understanding whether the actions of CCK contribute or inhibit the mixing of stomach contents. The question tells us that CCK causes a constriction of the pyloric sphincter. However, it also causes an increase in antrum contractility. Think about this. If the antrum has an increased contractility and the contents of the stomach cannot enter into the small intestine, they will simply be pushed back into the body of the stomach. The contents of the stomach have no where to go but backwards. Therefore, there will be an increase in the mixing of stomach contents and an overall decrease in the rate of gastric emptying (remember, the pyloric sphincter is closed). The correct choice is C. B is correct, the pH of the blood increases. Wc are looking for afalse statement surrounding the notion of parietal and pancreatic cell secretion. We know that a parietal cell secretes protons into the lumen of the stomach. First, how is this proton obtained. The parietal cell converts a carbon dioxide molecule into a bicarbonate ion and a proton. The cell secretes the proton into the lumen. What does it do with the bicarbonate ion? It transports the ion into the blood. In that way, the parietal cell itself does not become basic. However, that does present a problem for the pH of the blood, right? Well, not exactly. A pancreatic cell does just the opposite. The cell secretes a bicarbonate ion into the lumen of the small intestine and is left with a proton. The proton is released into the blood. If the secretions of the

93.

parietal cell and pancreatic cells are equivalent, the pH of the blood does not change throughout the digestive process. The correct choice is B.

Passage XV (94 - 100)


94.

Cholera Toxin

B is correct, salt loss creates an osmotic gradient, which water follows out of the cell. The epithelial cells lining the intestine form a selectively permeable boundary between the blood and the lumen, or cavity, of the intestines. In cholera, these epithelial cells secrete sodium and chlorine ions into the lumen. This changes the relative concentrations of solutes, meaning that there is a lower concentration of solutes inside the cell in comparison to outside the cell. Water diffuses across membranes by osmosis, following a gradient from low dissolved solutes to high dissolved solutes (i.e., high water concentrations to low water concentrations; the concentration of water is less in a solution with a lot of ions in it.). Therefore, water leaves the epithelial cells and enters the lumen of the intestine, following the sodium and chloride ions. This causes diarrhea. Choice A is wrong because the membrane is already highly permeable to water, and extra pores won't make a difference. Choice C is wrong because water cannot be actively transported. Choice D involves reabsorption of water and is completely incorrect. The correct choice is B.

95.

D is correct, Water, glucose, and Na ions. From the passage, we learn that rehydration therapy involves an inwardly-pumping glucose/sodium symport. A symport is a protein which transports two ions at a time across a

membrane, in this case, into the cell. If a cholera patient is given both glucose and Na with water, the
glucose/sodium symport will transport both of these ions into the intestinal cells, increasing the solute concentration
inside. This will cause water to follow osmotically and reenter the blood (via the epithelial cells). All of the other
Copyright by The Berkeley Review
205

The Berkeley Review

Specializing in MCAT Preparation

Biology
balance. The correct choice is D.

Gastrointestinal St Renal

Section ffl Answers

answer choices are wrong because none of them allows solute to enter the cell, favorably changing the osmotic
96. D is correct, testosterone. This problem requires that we know that mechanisms of the hormones that are mentioned. Epinephrine, gastrin, and norepinephrine are peptide hormones which operate by binding to receptors on the outer surface of cell membranes. This then triggers a signal cascade involving second-messengers like cyclic-AMP. As
such, the cholera toxin would affect their actions by increasing cyclic-AMP levels. Testosterone, on the other hand, is a steroid hormone. Steroid hormones can pass through the plasma membranes of cells, directly (or through a

bound protein complex) affecting DNA transcription. Steroid hormones therefore do not use second-messenger
systems. The correct choice is D.

97.

D is correct, the underlying bacterial infection must be treated so that dehydrating diarrhea ceases. Rehydration
therapy is just that, a therapy, not a cure. It does not address the underlying problem, which is the bacterial infection that produces the cholera toxin in the first place. In severe infections, antimicrobial agents must be administered to

destroy the bacteria. Otherwise, dehydration will simply reoccur after each rehydration attempt due to the

persistence of thecholera toxin which causes saltsecretion and water loss. Choice A is wrong because the body can reestablish a normal blood volume and pressure. Choice B is incorrect because pores won't affect the existing osmotic gradient(see Question 1). Choice C is not right because more chloride willjust make the osmotic gradient
steeper and more water will leave the body. The correct choice is D.

98.

C is correct, lack membrane receptors for cholera toxin. The question is basically asking why some mammals (a class of animal closely related to humans, relatively speaking) aren'taffected adversely by cholera in the way that humans are. Choices A and B are wrong because mammals, like humans, useG-protein mediated signaling (in fact, most studies are done in mice) and generally only have high cellular cyclic-AMP levels when cells are stimulated (i.e., by hormones). Choice D is wrong because if cyclic-AMP levels are normally low, then cholera toxin would increase them. Choice C is correct because thecholera toxin binds to specific receptors on the surface of intestinal cells. If these receptors are absent, as in many mammal species, the toxin can't attach and exert its effects. The
correct choice is C.

99.

A is correct, I only. The transgenic mice described in the passage are engineered to have the gene which encodes
thecholera toxin. This gene is attached to a promoter (a DNA sequence that allows DNA to be transcribed) which is

only active in the growth-hormone producing cells of the pituitary gland. Therefore, these mice produce cholera
toxin specifically in theirpituitary cells, causing a sustained increase in cyclic-AMP levels in these cells.Thesemice subsequently exhibit gigantism, which results from excess growth hormone. Therefore, we can assume that the

increased cyclic-AMP levels are somehow signaling these cells to produce more growth hormone. The question asks what would inhibit this effect. Excessive normal G-proteins wouldn't because they would simply bealtered by the cholera toxin and would become constantly active. Excessive cyclic-AMP would add to the gigantism because it seems to behigh levels ofcyclic-AMP which iscausing it inthe first place. An enzyme which degrades cyclic-AMP
would reduce the effect. The correct choice is A.

100. B is correct, the frequency of the CF gene is high in populations chronically exposed to cholera. The theory mentioned in the question states that the cystic fibrosis gene, when present in one copy (i.e., heterozygously) may
confer a selectiveadvantage becauseit provides a degree of resistance to cholera. This is a similar case to sickle-cell

anemia heterozygotes, which are somewhat resistant to malaria. If the CFgene did confer a selective advantage on heterozygotes, then we would expect to seea high frequency of this gene in areas of high cholera incidence. This is because people with the gene would bemore likely tosurvive epidemics and would evolutionarily be more "fit" than people without thegene. All of theother answers are wrong because they do notprovide evidence thatsupports this
theory, even though they may not contradict it. The correct choice is B.

Copyright by The BerkeleyReview

206

The Berkeley Review Specializing in MCAT Preparation

Biology
Section IV
Reproduction
and
A. Reproduction
1. Male Reproduction 2. Female Reproduction Development 1. Developmental Stages

B.

2.
3.

Developmental Mechanisms
Human Embryo Development

Development

Practice Passages and Answers

JJr.e.v-ke-w
Specializing in MCAT Preparation

Berkeley

Reproduction and Development


Top 10 Section Goals

0||Bcfiamiliar with the male reproductive anatomy.


*Sr Know howtotrace thepassage ofsperm from thetestis, through theepididymis, intothevasdeferens, Kno\
*l throi through

theejaculatory ductand prostate, andout ofthebodythrough the urethra.

@^f^ Know how sperm cells are produced during spermatogenesis.

^Sr Be familiar with the actions of LH and FSH on developing sperm. Know the different cell types
^ involved in sperm production. Make sure you understand both mitosis and meiosis.
#* Be familiar with the female reproductive anatomy.

Jjr Know the general steps ofhow anovum develops within the ovary. Be aware ofthe different types
* of cellswithin the ovary and their relationship to the developing egg.

QIHI Know how eggs are produced during oogenesis.

jSr Be familiar with the actions of LH and FSH on developing ova. Know the different cell types involved
J in egg production. Make sure you understand both mitosis and meiosis.

)<* Be able to compare and contrast male and female gamete production.
jar Understand why the male Leydig cells are analogous to the female theca cells, andwhy the male
* Sertoli cells areanalogous to thefemale follicle cells.

(D'fm Understand how to read the hormonal curves for a woman's monthly cycle.

jlf Understand the relationship between LH, FSH, estrogen, and progesterone during both the follicular
* phase and the luteal phase of ova production.

Olfe Be familiar with the different hormonal feedback mechanisms in the brain. jSf In the male and female, gonadal hormones circulate to the central nervous system and exhibit
feedback control onhypothalamic-pituitary secretions. Understand these general feedback mechanisms.

#* Understand what happens after a sperm fertilizes an egg.

jBr Be familiar with the process of fertilization, where itis most likely to occur, and where the fertilized
* egg is most likely to implant itself in the uterus.

#* Understand the hormonal relationships associated with fetal development.


j5r Be familiar with how placental hCG affects the corpus luteum andhow estrogen andprogesterone
fromthe corpusluteum affects the anterior pituitaryand the mammary glands.

([l)lrm Be familiar with the different stages of embryonic development.

Developmentbegins with fertilization and proceeds through cleavage, gastrulation, neurulation, neural crest formation, organogenesis, and eventually parturition. Be familiar with.these stages.

Biology

Reproduction St Development

Male Reproduction

Reproduction
Male Reproduction
General Reproductive Endocrinology
During the endocrinology of reproduction a hormone called gonadotropin releasing hormone (GnRH) is released from the hypothalamus. GnRH signals the anterior pituitary to release the gonadotropins luteinizing hormone (LH)

and follicle stimulating hormone (FSH). Both LH and FSH travel to the gonads. In the male the gonads are the testes while in the female they are the ovaries. Each of these gonadotropins elicitsa number of different responses at their target tissues. There are two general types of responses that we will be considering. The
first is the production of steroid hormones. The second is the production of germ cells. In the male the germ cells are called spermatozoa while in the female they
are tlie ova.

Male Reproductive Anatomy


The organs which carry out the reproductive functions in the male are the testes, epididymis, vas deferens, ejaculatory ducts, seminal vesicles, prostate, bulbourethral glands, and penis. These structures are shown in Figure 4-1.

Prostate "land

Urethra

Seminal vesicle

Corpus
cavcrnosum

Ejaculatory
Corpus spongiosum
duct Bulbourethral

gland
Testis

Figure 4-1 Male reproductive anatomy.

All of these structures are important in the synthesis and delivery of sperm to the female vagina. The actual production of sperm is referred to as spermatogenesis. Because the formation of sperm requires a temperature lower than that of body temperature, they are produced in the testes which lie in the scrotum outside of the body cavity. Within each testis are a series of convoluted tubules called the
seminiferous tubules. It is within these tubules that we find the spermatogenic
Copyright by The Berkeley Review
209

The Berkeley Review

Specializing in MCAT Preparation

Biology

Reproduction &Development

Male Reproduction

cells. Not only are sperm produced within the testes but the hormone testosterone is synthesized within specialized interstitial cells called Leydig cells that lie outside the seminiferous tubules. A third cell type found within the area
of the seminiferous tubule is the Sertoli cell. As we will see, these cells act to

promote spermatogenesis and they also produce the protein hormone inhibin.

Sperm Production
The target tissue for hormones like FSH and LH are the gonads. In the male the gonads are the testes. Within the testes are a series of convoluted tubules referred to as the seminiferous tubules. There are several different cell types that interact within the seminiferous tubules to allow for the development of spermatozoa. Between adjacent seminiferous tubules are the Leydig cells (also
called the interstitial cells). If we were to take a cross-section of a seminiferous

tubule, we would note a lumenal space in the center. Adjacent to the lumenal space are fairly large cells called Sertoli cells. In direct contact with the Sertoli cells are a series of spermatogenic cells. As you proceed from the basement
membrane of the seminiferous tubules toward the lumen, the cells become more

and more differentiated. In other words, those cells are becoming more and more developed. An important aspect of this development is that the Sertoli cells are in constant contact with the spermatogenic cells. You could view this as the Sertoli cells regulating the development of the spermatogenic cells.

Let's consider the development of the spermatogenic cells. If we consider a

portion of the seminiferous tubule, we willseea basement membrane separating


the Sertoli cells from the interstitial cells. The Sertoli cells are in contact with one

another and essentially form a ring around the inside of the basement membrane. Different developmental events take place on different sides of the Sertoli cells. For example, spermatogenic cells closer to the basement membrane

turn out to dividein a completely different way than they do further away from
the basement membrane.The spermatogenic cells that are closer to the basement membrane are calledspermatogonia. They have 46 chromosomes.

The way that spermatogonia divide is by mitosis. The resulting "daughter" cells
are exactly the same as the parental cell. In the area closest to the basement membrane the spermatogonia are undergoing constant division. Recall that the first thing that happens in mitosis is duplication of the chromosomes. These

chromosomes eventually migrate towards the cell's equator and the cell undergoes cytokinesis to produce two daughter cells which are exactly like the parental cells. The spermatogonia that undergo mitosis by the basement membrane are referred to as primary spermatocytes. As the primary
spermatocytes form they begin to squeeze by the juxtaposed Sertoli cells and
move towards the lumen of the seminiferous tubule. In the area near the lumen a

completely different set of events takes place. The primary spermatocytes


undergo meiosis and will reducetheir genetic complement to 23chromosomes.

Meiosis is simply one duplication with two divisions. Recall that during the
meiosis the first thing that happens is duplication of the chromosomes. The

chromosomes lineup at the cell's equatorand the cell undergoes cytokinesis. The cells that are formed after the first meiotic division are called secondary spermatocytes. At this point a second meiotic division takes place. Each secondary spermatocyte will again divide to give two daughter cells. The daughter cells of the second meiotic division are referred to as spermatids and
they have half the chromosomal complement (i.e., 23 chromosomes) as the

Copyright byThe Berkeley Review

210

The Berkeley Review


Specializing in MCAT Preparation

Biology

Reproduction St Development

Male Reproduction

original spermatogonia. Therefore, every primary spermatocyte that undergoes


mitosis and meiosis ends up producing four spermatids.
The very next event that takes place is a transformation event. Each of the

spermatids is transformed into a spermatozoa. The transformation process has an interesting characteristic to it. When the spermatids are initially formed, they are all connected to each other by cytoplasmic bridges. During the transformation process the spermatozoa essentiallybud out from the spermatids.
At the very tip of a spermatozoan there is a structure called the acrosome which

contains a lot of digestive enzymes. These digestive enzymes help the


spermatozoa gain access to the interior of the egg once fertilization has taken place. Inside the head of the spermatozoa is a nucleus which contains DNA. In the midsection are mitochondria, providing energy for the whipping movement

of the tail. This enables the sperm to swim towards their destination (the egg).

Hormonal Control of Sperm Production


It is important to remember that the spermatogenic cells are in constant contact with the Sertoli cells. The Leydig cells have a rather special function. They convert cholesterol to testosterone (which we can abbreviate as T). Testosterone has several roles. Testosterone can diffuse out of the Leydig cells and move to other target tissues of the body. It can also leave the Leydig cells and diffuse into the Sertoli cells. Within the Sertoli cell testosterone binds to a specific receptor and is converted to a compound called dihydrotestosterone (which we can abbreviate as dHT). This complex diffuses into the nucleus of the Sertoli cell and instructs the DNA to synthesize RNA. It is the products of the RNA synthesis that affect the spermatogenic cells. See Figure 4-2.
Recall that we mentioned that the hypothalamus releases GnRH which then acts on the anterior pituitary and causes it to release FSH and LH. These two gonadotropins have a direct effect on the Sertoli cells and Leydig cells. LH binds to a specific receptor on the membrane of the Leydig cell. The secondary

messenger produced from this binding interaction increases the conversion of


cholesterol into testosterone. FSH has a different function. FSH will bind to a

surface receptor on the Sertoli cell and induce a secondary response within the Sertoli cell. This secondary response helps convert testosterone into

dihydrotestosterone. It also induces the synthesis of the receptors. [A thought question. If you had to make a male contraceptive, what events would you disrupt in this pathway? You would disrupt the production of dihydrotestosterone or FSH. One of the problems with producing a male contraceptiveis that it is rather difficult to stop the synthesis of FSH.]

As with any hormonal system there is always the consideration of a feedback mechanism. In males there is a constant production of LH and FSH. The reason is due to a feedback mechanism. One of the consequences of testosterone diffusing out of the Leydig cells is that it has a negative feedback at the anterior pituitary

and at the hypothalamus. Testosterone is preventing the synthesis of GnRH and


LH. The Sertoli cells are involved in a different feedback mechanism. The Sertoli

cells also secrete a compound called inhibin. Inhibin acts as a negative modulator of the anterior pituitary. The reason that the gonadotropin levels are relatively constant is that you have a constant synthesis of the spermatogenic
cells. If testosterone levels are too high, the feedback mechanism will decrease the levels of LH. If the levels of dihydrotestosterone are too high, there is an

increase in inhibin synthesis which results in a decrease in the levels of FSH.


Copyright by The Berkeley Review
211

The Berkeley Review

Specializing in MCAT Preparation

Biology

Reproduction & Development

Male Reproduction

(-)

Hypothalamus
i
GnRH

I
..().>
Anterior Pituitary <
(-)

Testosterone

LH

FSH

Inhibin

i
dHT

Cholesterol

Testosterone

T*i>
nucleus

Basement

Receptor

Leydig Cell

membrane

Sertoli Cell

Testes
Figure 4-2
Hormonal regulation in the testes.

Spermatogenic
cell

Secondary Sex Characteristics in the Male


Almost all of the secondary sex characteristics in the male are due to testosterone.

Some of those characteristics are development of a beard, pubic hair, deepening


of the voice, texture of the skin, muscle distribution, bone development, sexual
drive, and, if you are genetically predispositioned to it, baldness.

Sperm Delivery
Sperm leave the seminiferous tubules and enter into the epididymis and then the vas deferens. Sperm are stored in the epididymis and vas deferens for about 14 days before ejaculation. Contraction of smooth muscle lining the walls of these structures ejects the sperm down the vas deferens and into tlie ejaculatory duct. The seminal vesicles, prostate gland, and bulbourethral gland secrete
components that make of the remainder of the fluid that is ejaculated (now called semen). Some of the components secreted by these glands are fructose vitamins, bicarbonate zinc, prostaglandins, and mucus. The total volume of the fluid ejaculated with the sperm is about 3 to 5 milliliters.

Copyright by The Berkeley Review

212

The Berkeley Review Specializing in MCAT Preparation

Biology

Reproduction St Development

Female Reproduction

Female Reproduction
Female Reproductive Anatomy The essentialanatomical features of the female reproductive system includes the vagina, uterus, Fallopian tubes, and ovaries. Once sperm have been ejaculated
into the vagina they can live for about 48 hours. However, about a half an hour after ejaculation the leading sperm arrive at the site of fertilization which is

usually the oviduct (also called the Fallopian tubes). The fertilized egg continues to move down the oviduct towards the uterus where it will implant in the uterine lining. About a week after ovulation the fertilized egg, now called a blastocyst, implants in the lining of the uterus where it will continue to grow and develop
until parturition (delivery). A few of the anatomical structures mentioned here are shown in Figure 4-3.

Fallopian
tube

0vary

Bladde

Urethra

Vagina

Figure 4-3 Female reproductive anatomy.


Ova Production

The production of the female germ cells occurs via a process called oogenesis. In certain aspects this process is similar to that of spermatogenesis in males. The important similarity in the female is that when cell division is complete the ovum will have half the DNA as the original parent cell. The regulation of oogenesis, however, is quite different. First, there are a series of events in which the oogonia mitoticallydivide to form primary oocytes. The primary oocytes,which have 46 chromosomes, are analogous to the primary spermatocytes in the male. In the female all of these divisions occur within the first two to three months of fetal development. About the third month of gestation all the mitotic divisions cease. In other words, all the oogonia that a female will produce will be produced within the first three months of fetal development. In fact, at birth a female will have about 400,000 primary oocytes in both of her ovaries. However, by the time she has finished her reproductive life only about 400 will have ever matured. The remaining primary oocytes will have degenerated at various stages during the developmental process. This process of degeneration is referred to as atresia.
Copyright by The Berkeley Review
213

The Berkeley Review Specializing in MCAT Preparation

Biology

Reproduction & Development

Female Reproduction

The primary oocytes will eventually undergo the first meiotic division, caused by a surge in the gonadotropin LH. The first meiotic division happens in monthly cycles. Some of these primary oocytes could take as long as 50 years to have their first meiotic division. Once the first meiotic division is complete the secondary oocyte, which has 23 chromosomes, is formed. During this division one of the two daughter cells obtains all the cytoplasm while the other cell is just a small sphere of DNA called the first polar body. The secondary oocyte undergoes the second meiotic division only after fertilization has taken place. The product of that division is the ovum and a second polar body. [The first polar body also divides to give two second polar bodies as well.] The ovum now has 23 chromosomes. When sperm and ovum unite in fertilization, the complement of 46 chromosomes will be restored (23 from the male and 23 from the female).

If we take a cross-section of the ovary, we will see a series of specific cell types called primary follicles which are in different developmental states. The primary follicle is actually a primary oocyte surrounded by a layer of follicle cells. These follicle cells are in constant contact with the primary oocyte. Eventually one of these primary follicles will start to develop.[An exception arises with fraternal twins in which case two follicles will form and develop at the same time.]

Theca
CeIls
Ovum

Growing
Follicle
Primary
Follicle

Zona Pellucida

Granulosa Cells

Regressing Corpus

Corpus
Ovulation Luteum

Luteum

Figure 4-4 The ovarian cycle.

In order to mature the primary oocyte estrogen, LH, and FSH are needed. This developing time period is referred to as the follicular phase and lasts up to about the 14th day of the woman's monthly cycle. During this period the primary

follicle gradually develops. Surrounding the primary oocyte will be a membrane called the zona pellucida. The zona pellucida is surrounded by more follicle cells called granulosa cells and finally by theca cells. See Figure 4-4. These cell types are a direct result of the estrogen, LH, and FSH that are present. The theca cells are analogous to the Leydig cells in the male while the granulosa cells are analogous to the Sertoli cells in the male.
Copyright by The Berkeley Review
214

The Berkeley Review Specializing in MCAT Preparation

Biology

Reproduction St Development

Female Reproduction

ill d

|ll Follicular Phase c^


14

28 Days

Figure 4-5
The hormones of the ovarian cycle.

Within the primary follicle a fluid starts to build up forming the antrum. The system is now primed for ovulation. The next event that takes place is the LH surge. This surge causes the primary oocyte to undergo the first meiotic division, forming a polar body and the secondary oocyte. The LH surge also causes the production of a series of enzymes which break down the membrane in the primary follicle. Once the secondary oocyte is released, ovulation has occurred. Ovulation usually comes at about the 14th day in the woman's monthly cycle. As we have mentioned, the process leading up to ovulation is referred to as the follicular phase. The left over follicle (after ovulation) is transformed into a

gland-like structure called the corpus luteum. One of the main functions of the corpus luteum is to produce estrogen and progesterone. If fertilization does not occur and there is no pregnancy, the corpus luteum will degenerate and the whole cycle will start again. From the point of ovulation, at about the 14th day, until the beginning of the menstrual flow is the luteal phase. See Figure 4-5.
Hormonal Control of Ova Production

During a woman's monthly cycle her gonadotropic hormones fluctuate rather dramatically. LH remains relatively constant until just before ovulation at which time there is a sudden surge. FSH will also have a surge, but not as great as the LH surge. There will be a surge of estrogen before ovulation and then a smaller
one after ovulation. The increase in estrogen before ovulation actually leads to the increase in LH. Another hormone called progesterone will surge after
ovulation as well. How are these hormones regulated?

Copyright by The Berkeley Review

215

The Berkeley Review

Specializing in MCAT Preparation

Biology

Reproduction St Development

Female Reproduction

....> Hypothalamus
i
GnRH

(-) ">

Anterior Pituitary

LH

FSH

|::;::::.,; ; j ^

Testosterone

c
O

o
'

Cholesterol

o to
PJ P

I
Testosterone
Basement

Estrogen *""

^^ Low level

>
Follicle cell

High
level

Theca cell

membrane

Primary
oocyte

Corpus
Luteum
Figure 4-6 The hormones of the ovarian cycle.

Ovaries

In the female the theca cells convert cholesterol into testosterone. The

testosterone diffuses into the follicle cells where it is converted into estrogen. One of the effects of estrogen is to help in the development of the primary follicle (i.e., the primary oocyte). We had mentioned that the primary oocyte reaches maturity
by undergoing reactions with estrogen, LH, and FSH. Recall that the anterior pituitary secretes LH and FSH. LH affects the theca cells while FSH affects the follicle cells. As estrogen is being synthesized, the primary oocyte is developing. At the same time we have a proliferation of the follicle cells. If we have more follicle cells, then we will be able to synthesize more estrogen. This is where the first estrogen surge comes from. It turns out that at low concentrations of estrogen there is a negative feedback on FSH production. See Figure 4-6. However, the follicle cells are growing and at a certain point in time there is a threshold level in estrogen. This means that relative to the system we no longer have a low concentration of estrogen but instead have a high concentration of estrogen. At high concentrations we see that estrogen has a positive feedback on
Copyright by The Berkeley Review
216

The Berkeley Review Specializing in MCAT Preparation

Biology

Reproduction & Development

Female Reproduction

the anterior pituitary and LH production. Thus, with a high level of estrogen concentration of estrogen is having a positive feedback and therefore causing more LH production-giving the LH surge. The reason that one sees these surges is because of the rapid growth of the follicle cells during development of the
primary follicle.

production we now have the LH surge. This is important. The increased

Immediately following the LH surge the levels ofLH drop down to very low levels. What causes the drop in LH and the increase inestrogen and progesterone during the luteal phase? If we no longer have a high concentration ofestrogen,
we will lose the positive feedback on the anterior pituitary. Recall that at ovulation all the otherfollicle cells become transformed into the corpus luteum. They are no longer follicle cells. During that transformation they transiently lose

the ability to produce estrogen. This ability to produce estrogen is lost just long enough so the LH levels decrease. However, the levels of both estrogen and
progesterone begin to increase. Whereis this comingfrom?

The corpus luteum becomes an endocrine gland and begins to synthesize


estrogen and progesterone. Estrogen and progesterone, in combination, have a

negative feedback on LHand FSHproduction in the anterior pituitaryas wellas a negative feedback on the hypothalamus and the synthesis of GnRH. What is the result of having a negative feedback on LH and FSH? Essentially what is happening is that the primary follicle is prevented from developing. As long as the corpus luteum is producing estrogen and progesterone, the primary follicle is unable to develop. But this is what wewant to happen. At this point we do not need
another primary follicle being made. [With this information, what is the basis for

female oral contraception? Birth control pills are essentially a combination of estrogen and progesterone in concentrations that represent the needs of the woman.] Remember, estrogen alone has a positive feedback. Estrogen and
progesterone in combination have a negative feedback.

A very different set of events takes place during pregnancy. What happens when
the spermatozoan comes in contact with the ovum? The acrosome of the spermatozoan allows for the digestion of the membrane of the secondary oocyte. Within the membrane of the ovum (zona pellucida) are receptor sites for the sperm. These receptor sites prevent cross-species fertilization. The only thing that enters the secondary oocyte is the nucleus of the sperm. After the nucleus of the sperm enters the secondary oocyte the zona pellucida changes and prevents any other spermatozoa from entering. This process is referred to as fertilization. At this point the secondary oocyte undergoes the second meiotic division to form the ovum and the polar body. The nucleus of the sperm and the nucleus of the egg fuse together to form the zygote, which now has a complement of 46
chromosomes.

The zygote rapidly begins to develop and in about 7 days attaches itself (as a blastocyst) to the uterine lining. A blastocyst is essentially a small ball of cells with a central fluid-filled cavity. When the implantation takes place the placenta, made up of maternal and fetal cell types, begins to form. What one does not want to happen during pregnancy is the development of another primary follicle. Throughout pregnancy there are high levels of estrogen and progesterone. The reasons for maintaining high levels of estrogen and progesterone are different during the first three months of pregnancy than they
Copyright by The Berkeley Review
217

The Berkeley Review Specializing in MCAT Preparation

Biology

Reproduction fie Development

Female Reproduction

areduring the last sixmonths ofpregnancy. Shown in Figure 4-7 is a diagram of


the hormonal considerations during the first three months of pregnancy. The

placenta itself is an endocrine gland. During the first three months the corpus
luteum is still a viable gland and it secretes estrogen and progesterone. The placenta synthesizes chorionic gonadotropin (CG) which stimulates the corpus luteum to make estrogen and progesterone. One of the best pregnancy tests is to
detect CG in the blood-stream. The only time CG is made is during the first three
months of pregnancy.
Estrogen & Progesterone (-)

Corpus
Luteum
/V

Anterior

Pituitary
r*e (+) (+)
V
Prolactin

CG

(+)

cs

Placenta
(+)

Mammary
Glands

Figure 4-7
The placenta and its relationships.

Chorionic

Gonadotropin

What are the functions of estrogen and progesterone? Estrogen and progesterone have a negative feed-back on the anterior pituitary, thus inhibiting LH and FSH production. This prevents the formation of the primary follicles during pregnancy. It also means that ovulation and menstrual cycles will be eliminated for the duration of the pregnancy. Another important gland during pregnancy is the mammary gland. There are a series of hormones that positively feedback on the mammary glands. One hormone, prolactin, comes from the anterior pituitary. The placenta forms another hormone called chorionic somato mammotropin (CS) which also acts on the mammary glands. They help the mammary glands to grow. Estrogen and progesterone have positive feedback on
the mammary glands as well.

12

345

67

89

Months after start of last

menstrual period

During the last six months of pregnancy there in only a slight variation on this theme. CG is no longer made. This results in a loss of feedback to the corpus luteum. After three months the corpus luteum breaks down and the supply of estrogen and progesterone from the corpus luteum comes to a halt. However, at that point in time the placenta itself starts to synthesize estrogen and progesterone. In fact, the placenta makes much more estrogen and progesterone that the corpus luteum did. This can be seen in Figure 4-8.

Figure 4-8
Placental synthesis of estrogen and progesterone.

Secondary Sex Characteristics in the Female


Some of the effects of the female sex steroid estrogen involve development of female body configuration, growth of breasts, growth of external genitalia, pattern of pubic hair, fluid retention, influence on the cardiovascular system, and it may be involved in inhibiting atherosclerosis.

Copyright by The Berkeley Review

218

The Berkeley Review Specializing in MCAT Preparation

Biology

Reproduction St Development

Developmental Stages

Development
Developmental Stages
Recall that when we discussed meiosis we mentioned that the haploid sperm and the haploid egg can unite to form a zygote. The zygote will undergo a series of mitotic cell divisions which will form a clone of cells. All of those mitotically dividing cells were derived from a single precursor cell and eventually they will give rise to an embryo. The embryo will continue to develop and later forms a sexually immature juvenile. The juvenile undergoes a maturation process and
becomes the adult.

Brain

Thoracic Cavity

Spinal Cor

Abdominal Cavity

Figure 4-9 The body outline.

In our discussion on development we will focus on the vertebrate. In particular, we will want to examine the arrangement of the body plan in vertebrates. As we will see, human beings, as all other vertebrates, have essentially the same body plan. For example, the skeletal system in vertebrates helps to support the body. One aspect of the skeletal system is a dorsally located vertebral column. Within this vertebral column is the spinal cord. Anterior to the spinal cord is the brain. The brain and the spinal cord comprise the central nervous system. Nervous tissue outside the central nervous system is referred to as being part of the peripheral nervous system. Ventral to the vertebral column is the body cavity, or coelom. The coelom is divided into the upper thoracic cavity and the lower
abdominal cavity. The thoracic cavity contains the heart and lungs while the abdominal cavity contains a variety of organs like the stomach and the intestinal system. These basic landmarks are shown in Figure 4-9.

The body is composed of a variety of highly differentiated cell types. For example, there are stratified epithelial cells found in the epidermis. Cuboidal
Copyright by The Berkeley Review The Berkeley Review Specializing in MCAT Preparation

219

Biology

Reproduction & Development

Developmental Stages

epithelium can be found in thekidney. Smooth muscle cells canbe found in the
walls of the intestine. Skeletal muscle cells can be found in the voluntary muscles and cardiac muscle cells can be found in the heart. This is just a sample of some

of the cell types fond in the body. Cells of similar type form tissuesand tissues can come together to form organs like the heart and liver. During development the cells, which initially all start out alike, begin to grow and differentiate. This happens even though those cells all contain the same genetic constitution. They all have the samegenes. Not onlyis there differentiation but some of these cells undergo morphogenesis (i.e., theybegin to take on differentshapes).
There are a number of stages that one can use to characterize development (see Table 4-1). The first developmental stage involves the union of the male and female gametes. This process is called fertilization. The gamete producing organs are the gonads, m the male these are the testes and in the female they are the ovaries. When the gametes unite a zygote is formed. The next developmental stage involves cleavage of the zygote. During cleavage the zygote rapidly divided into many smaller cells without an overall increase in size. Gastrulation is the third developmental stage. Many embryologists consider this to be the most important part of an organism's development. During this stage the cells of the zygote move to form the three primary germ layers (ectoderm, mesoderm, and endoderm) of the organism. The next developmental stage is neurulation. During neurulation we begin to see the formation of the nervous system. This is the first organ system to begin differentiation. Neural crest formation is the fifth developmental stage. The neural crest cells help to form parts of the nervous system, skull, and sensory organs. The last stage of development is organogenesis. It is during this stage that the different organs of the body are
formed.

1. 2.

Fertilization

Cleavage
Gastrulation Neurulation Neural Crest Formation

3.

4.
5. 6.

Organogenesis

Table 4-1

Development In The Frog


We can use the frog as an example of vertebrate development because of the

large size of its eggs (about 1.5 millimeters in diameter). The life cycle of the frog is shown in Figure 4-10. As we have mentioned, males give rise to haploid sperm
(IN) while females eventually give rise to haploid eggs (IN). Initially the female frog will give rise to a primary oocyte which then undergoes the first meiotic division. The secondary oocyte is formed but it is arrested at metaphase of the second meiotic division. This second meiotic division is not completed until the sperm from the male frog fertilize the secondary oocyte. Fertilization initiates a

process called egg activation. This is a signal for development to begin. The
second meiotic division in the secondary oocyte is completed and the zygote nucleus is formed. This nucleus is now diploid or 2N. The zygote undergoes a

series of cleavages and an embryo is formed. Further cell division leads to the
Copyright by The Berkeley Review
220

The Berkeley Review

Specializing in MCAT Preparation

Biology

Reproduction & Development

Developmental Stages

is a process of metamorphosis.

formation of the tadpole. The tadpole, being an aquatic organism, eventually loses its tail and forms legs so itcan begin its terrestrial life as an adult frog. This

Female 2n

Vs^-^// expelled /
meiotic divis ion '

y// bodies
1/

/ Nj^^
/

Secondary

Egg's second >nd

| /

i/

//

*'

y oocyte

Sperm n

Zygote 2

S,Fertilization

Figure 4-10 The life cycle of the frog.

In the unfertilized egg there is a large amount of yolk that resides in the vegetal pole (i.e., the lower hemisphere). This yolk will act as food for the developing embryo.The animalpole (i.e., the upper hemisphere) contains mainly cytoplasm. In a fertilized egg an interesting phenomenon takes place. On the side of the egg opposite to where the sperm penetrates the egg's membrane, a structure called the gray crescent forms. The gray crescent is located on the dorsal aspect (the future back) of the animal. This is where we will eventually find the spinal cord and brain. The side opposite the dorsal aspect is the ventralaspect. This is where
Copyright by The Berkeley Review
221

The Berkeley Review

Specializing in MCAT Preparation

Biology

Reproduction St Development

Developmental Stages

the belly ofthe organism will form. An imaginary line which connects the two poles is called the meridian. Imaginary lines which run around the circumference of the cell are equatorial in nature. The middle of the gray crescent defines the

body midline of the future organism. By the time the gray crescent has appeared
the mainaxes ofthebodyof the organism has beenestablished.

Cleavage

Once the zygote isformed it undergoes a special cell division to increase itsmass which bisects the gray crescent. This division gives two cells. The next division also occurs along the meridian, but this time at right angles to the last division.
We now have four cells. The third cellular division occurs equatorially and gives

but not its overall size. The first cellular division occur along the body midline

four cells in the animal pole and four cells in the vegetal pole. The individual cells involved in this growth arecalled blastomeres. Eventually a small, solid ball
of cells will be formed called a morula. Further cellular division results in the formation of a hollow ball of cells called the blastula. Within the blastula is a

fluid-filled cavity called the blastocoel. These structures are outlinedin Figure411.

Animal Pole

Vegetal Pole
Unfertilized

Grey
Crescent

Cleavage Down Meridian Body Midline Cleavage


(2 cells) (4 cells)

Equatorial Cleavage
(8 cells)

Meridian

Cleavage (16 cells)

Egg
Blastocoel

Morula

Blastula

Cross Section of Blastula

(32 cells) Figure 4-11 Different stages in embryonic development.

Gastrulation

Duringgastrulation rearrangement of cells occurs. Not far from the gray crescent

an open develops intheblastula called theblastopore. Cells from the animal pole begin to migrate inwards through the dorsal lip of the blastopore. As this outer layer of cells migrates inward they form a second layer of cells immediately below that outer layer. The blastocoel is reduced in size and is eventually eliminated. In its place a new cavity is formed called the archenteron. At this stage the embryo is referred to as a gastrula (because gastrulation has taken place). Invagination of this outer cell layer produces two cell layers. The outer cell layer is called the ectoderm. The inner cell layer is called the endoderm. A layer of mesoderm will laterform between these twocell layers. These structures
are shown from different viewpoints in Figure 4-12.

Copyright by The Berkeley Review

222

The Berkeley Review

Specializing in MCAT Preparation

Biology

Reproduction St Development

Developmental Stages

The events that lead up to gastrulation differ widely in the animal world. However, gastrulation is common to all ofthem. It seems that the generation of
the threecelllayers (ectoderm, mesoderm, and endoderm) has beenconserved in

throughout the evolutionary process. An animal cannot be made if gastrulation


does not occur.

Blastocoel

Dorsal Lip of Blastopore Blastopore

Dorsal Lip of Blastopore

Archenteron

Blastocoel

Dorsal Lip of Blastopore

Lateral Margin
of Endoderm
Archenteron

Ectoderm
Mesoderm

Dorsal Lip of Blastopore Yolk Plug

Dorsal Lip of Blastopore Yolk Plug

Remains of

Blastocoel

Figure 4-12
The process of gastrulation.

The ectoderm, mesoderm, and endoderm cell layers play different roles during the developmental process. They have different developmental fates. For example, the ectoderm will eventually differentiate into structures like the skin, the lens of the eye, and the brain and nervous system. Mesoderm will
differentiate into structures like the notochord, heart, skeleton, muscle, the outer

coverings of internal organs, and the reproductive organs. Endoderm will differentiate into the inner lining of the digestive tract and the respiratory tract, and major glands of the body like the liver and pancreas. These are just a few of the developmental fates of these cell layers.
Neurulation

During this stage of development the ectoderm, mesoderm, and endoderm begin to form the structures that will eventually define the embryo and later the adult.
Copyright by The Berkeley Review
223

The Berkeley Review Specializing in MCAT Preparation

Biology

Reproduction & Development

Developmental Stages

During neurulation the formation of the notochord takes place along the body
midline. This structure is derived from mesoderm. Superior to the notochord is a mass of ectodermal cells called the neural plate. The neural plate will begin to fold in on itself and form the neural groove. As the edges of this folding fuse with
one another the neural tube is formed. Within the neural tube will form the

spinal cord (encased in the spinal column) and anterior to the spinal cord will
form the brain. In other words, the neural plate, which is composed of ectodermal cell, gives rise to the nervous system. This tissue is referred to as primordium (from the Latin primus, first, + ordior, to begin) and it is the earliest stage of development of a structure. During neurulation the embryo is sometimes
called a neurula.

As the neural plate begins to fold in on itself it will form the neural groove. Two views are shown in Figure 4-13.Below the neural groove is the notochord and on either side of the notochord is mesoderm. As the neural groove begins to form the mesoderm is split and forms a coelom (i.e., a body cavity). The lungs will eventually develop within this coelom. When the edges of the neural groove fuse together the neural tube will be formed. Within the neural tube forms the spinal cord and anterior to the spinal cord forms the brain.
Neural Crest

Neural Groove
Neural Crest Cells

Cells

Ectoderm

Neural

Plate

|
Notochord

Neural Tube

Neural
Plate Notochord Ectoderm

Neural
Groove

Neural
Tube

Mesoderm SomitS
Coelom

Figure 4-13
Neural tube formation.

Neural Crest Formation

As the edges of the neural groove fused together and became the neural tube,

specialized ectodermal cells were left in a more dorsal position on the neural tube. These specialized cells are called the neural crest cells (Figure 4-13). As these cells begin to move to the sides of the developing embryo, they begin to
functionalize. For example, ectodermal cells in the anterior portion of the developing embryo associate with the neural crest cells and forms placodes. These structures will eventually form the sense organs located in the head. Some neural crest cells will help form sensory cells (e.g., olfaction and touch). Other neural crest cells will form the adrenal medulla. During times of stress the animal
Copyright by The Berkeley Review
224

The Berkeley Review

Specializing in MCAT Preparation

Biology

Reproduction St Development

Developmental Stages

will experience the "fight or flight" syndrome. In order to respond to this stress the adrenal medulla releases adrenaline. This hormone prepares the animal to respond byincreasing blood sugarlevels, heart rate, andblood pressure.
Organogenesis During the initial stages of organogenesis there is an interaction between ectoderm and mesoderm. The neural tube becomes longer and thinner and has

an anterior to posterior developmental gradient. Neural crest cells begin to


migrate and take up positions in the vicinityof the neural tube. Mesodermal cells migrate towards the neural tube. Eventually they will form the vertebral column.

The brain begins to form at theanterior portion oftheneural tube. Optic vesicles begin to form. As the neural tube continues to form segments of mesodermal tissue called somites beginto appear. Thesomites will eventually give rise to the vertebrae, connective tissue, and the muscles of the body.

In the frog embryogenesis is complete with the appearance of the sexually


immature tadpole. Thesecreatures have gillsand live in an aquaticenvironment. They have no limbs. Instead, they have a tail which is used to propel them through the water. The last critical phase in the life of a tadpole is one of morphogenesis. Thetadpolewillchange from a sexually immature organism into a sexually active organism, which is the frog. During this process of morphogenesis the tadpole grows limbs, develops lungs, loses its tail and its gills. This change is initiated by a hormone which is released during a specific time of development. The hormone which is released is thyroxin and it is
released from the thyroidgland.

Many organisms pass through a larval stage (sexually immature) like the tadpole. When the frog eggs hatch the tadpoles are left to fend for themselves. If the post-embryonic organism is left on its own, then the development is termed indirect. In contrast, direct development involves care being given to the postembryonic organism by the mother. For example, a human fetus developswithin
the female of the species. Even after birth the female (and male) care for the young until they are able to take care of themselves.

Copyright by The Berkeley Review

225

The Berkeley Review Specializing in MCAT Preparation

Biology

Reproduction fie Development

Developmental Mechanisms

Developmental Mechanisms
If the cells of the body all contain the same genetic information, how can they become so differentiated?There are two general classes of interaction associated
with the differentiation of cells that we need to consider. There are intracellular

interactionswhich involve interactions between the components within the cells


themselves. There are also intercellular interactions in which the interactions are

between cells. The intracellular interactions usually result in the setting up of a

prepattern. The intercellular interactions usually undergo developmental


induction.

Intracellular Interactions

We have mentioned that the unfertilized egg has an animal pole, which contains the cytoplasm, and a vegetal pole, which contains the yolk. The egg is not homogeneous. However, at this point in the egg's existence a longitudinal axis is already present. In the frog the animal pole will give rise to the head while the vegetal pole will give rise to the tail. This is true for the frog but it is not a truth set in stone. Remember, there are always variations on a given theme.
Recall that after fertilization and induction of development the gray crescent forms at a point opposite to where the sperm penetrated the egg. The formation of the gray crescent is due to an intracellular interaction. The formation of the gray crescent is not a prepatterned phenomenon that took place before the sperm interacted with the egg. This is clearly shown by the fact that the gray crescent developed after the penetration of the egg by the sperm. The entry of the sperm
in the animal half of the cell is random. However, one entry is made and the gray
crescent forms, the dorsal midline is established. This allows us to define

directions. In other words, prior to the first cellular division the axes of the organism is established. In this case a prepattern is laid down which is adhered to during the rest of development.

Hans Spemann
During the 1920's Hans Spemann was able to demonstrate that the gray crescent is an important landmark in the future development of the embryo. In his experiment, Spemann took a newly fertilized frog egg and tied a string around it such that the string bisected the gray crescent (see Figure 4-14a). He slowly tied the string tighter and separated the egg into two halves, each with half a gray crescent. Two separate blastomeres developed and two separate embryos were formed. Since these embryos were twins, this procedure was called twinning. This experiment demonstrated that each blastomere was equivalent in its potential to form a complete embryo. Even though two complete embryos were formed, they were smaller than the normal frog embryos. Why? Because when the fertilized egg was cut in half, each blastomere received half of the original amount of yolk.

There is one other important point to make note of in this experiment. During the last lecture we said that the body midline bisected the gray crescent and that this was what eventually formed the neural tube. In the Spemann experiment the body midline was itself bisected. In other words, instead of the midline being in the middle it was to the side of the bisected gray crescent. However, the embryos developed normally. What this means is that the midline had to migrate to a position which was in the middle of the bisected gray crescent. Hence, there was

Copyright by The Berkeley Review

226

The Berkeley Review


Specializing in MCAT Preparation

Biology

Reproduction St Development

Developmental Mechanisms

a developmental regulation inside the blastomere which allowed for the movement of the body midline.

Spemann did the same experiment on another fertilized frog eggbut this time he cleaved the egg at right angles to the cleavage in the previous experiment (see Figure 4-14b). Two blastomeres were formed. One blastomere had thecomplete gray crescent while the other blastomere did not contain any piece of the gray crescent. As development proceeded a complete embryo was from the
blastomere with the complete gray crescent. The other blastomere formed a mass

ofcells thatdid not differentiate into an embryo. It did not gastrulate. Again, the embryo thatwas formed was smaller due to the reduction in theamount ofyolk
received. These two experiments proved for the first time that there was a clear prepattern during the development of an egg.
(a)
Gray
Crescent.

Bisection

Bisection

(b)
Gray
Crescent

e?

<*

Complete Embryos will Develop


Figure 4-14 Hans Spemann's gray crescent experiment.

Undifferentiated Mass of Cells

Complete Embryo

Hilde Mangold and Hans Spemann


In 1924Hilde Mangold performed an experiment in which she removed a section of the dorsal lip from one species of salamander embryo (before gastrulation started) and transplanted it into the belly of another species of salamander embryo. These two species differed in their pigmentation (the donor being of lighter pigmentation that the recipient). Recall that the dorsal lip is that area where cells from the animal pole begin to invaginate into the blastula. This process leads to the formation of the blastopore and the development of the archenteron. The blastopore, and hence the dorsal lip of the blastopore, form close to the boundary of the vegetal hemisphere and the gray crescent. [In Figure 4-14b one blastula did not receive the gray crescent. Therefore, it could not develop the dorsal lip and ectodermal cells could not invaginate. This resulted in the blastula never developing into an embryo.]

Copyright by The Berkeley Review

227

The Berkeley Review Specializing in MCAT Preparation

Biology

Reproduction St Development

Developmental Mechanisms

The recipient salamander embryo now has two dorsal lips. When it developed it formed a second embryo (of a lighter pigmentation) at the location of the
transplant (Figure 4-15).

The transplanted dorsal lip developed into a notochord. In turn the notochord
induced the formation of a neural plate and eventually a neural tube. What this

experiment is saying is that the ectoderm of the recipient salamander was induced by the dorsal lip of the donor to take on a fate that it was not normally designed to fulfill. In other words, the dorsal lip acts as an organizer and organizes the cells of the recipient such that a second embryo is formed. This process later became known as embryonic (or developmental) induction.
The mesoderm is removed

opposite the dorsal lip of recepient embryo.

Mesoderm is removed from a donor embryo near the dorsal lip and transplanted into the recepient embryo.

c=>
Recepient Embryo Recepient Embryo

Donor Embryo

Primary
Neural Fold

Primary Neural Development

Secondary Neural Development

Secondary
Neural Fold

Development of a double embryo.

Figure 4-15 Transplant of the dorsal lip.

Hilde Mangold was a student of Hans Spemann when they did this experiment. She later died while cooking in her kitchen when her gas stove blew up. A number of years later Spemann got the Nobel Prize for his participation in this
work.

In another experiment Spemann examined a chain of successive inductions that leads to the formation of the eye. In the anterior most portion of the neural tube (the forebrain) there are two symmetrical protrusions from the developing brain. These protrusions extend to the ectoderm. Upon contact the ectoderm begins to invaginate and pushes into the growing optic stalk. This creates the optic cup.

The optic stalk will eventually become the optic nerve. The lens (placode)of the eye develops from ectoderm which is in contact with the edges of the optic cup. As the developing lens pulls away, the remaining ectoderm develops into the
Copyright by The Berkeley Review
228

The Berkeley Review Specializing in MCAT Preparation

Biology

Reproduction St Development

Developmental Mechanisms

cornea. The remainder of the optic cup develops into the retina, which contains the photoreceptors necessaryfor vision (Figure4-16).
Ectoderm

Retina

Invagination Optic Stalk


Figure 4-16 Development of the eye.

Experiments like these indicate that there is a hierarchy of developmental stages. During the 1930'sthis gave rise to the field of chemicalembryology. Researchers at the time thought that induction was due to some kind of biochemicalagent. It turns out that this line of thought was a waste of time and effort. Why? Molecular biology had not yet been invented. The selfish gene needed to be explored.

Copyright by The Berkeley Review

229

The Berkeley Review Specializing in MCAT Preparation

BlOlOgy

Reproduction & Development


Iiuinan EmDii^o D

Human Embryo Development


.tii&ti'.

Recall that fertilization of the egg by the sperm usually takes place in the oviduct

(Fallopian tube). Within the first 36 hours following fertilization the zygote will undergo its first mitotic division. It will continue to divide as it travels down the oviduct toward the uterus. By the fifth or sixth day the embryo will reach the uterus. At this point the embryo is called a blastocyst. The blastocyst is a hollow
ball of cells with a mass of cells on one side.

The surrounding cells which form the ball are called trophoblast; the others are called the inner cell mass. If the lining of the uterus is prepared to receive the embryo, then the embryo will implant. [Otherwise, the embryo is rejected and sloughed off during the next menstrual period.] After implantation the trophoblastic cells grow into the lining of the uterus. In fact, the cells of the trophoblast grow little finger-like projections into the uterus. As the fetus gets bigger it will need to get nourishment from the mother's blood. The blood vessels from the umbilical cord will grow into the projections of trophoblast. There is a layer of cells between the trophoblast and the fetal blood vessels called the chorion. The chorionic villi are the projections of

fetal/umbilical blood vesselsand the chorion that covers them. The trophoblast abuts the uterine cells of the mother, but allows arteries in the uterine lining to drain into sinuses around the chorionic villi. The chorion preserves the barrier
between the mother's and fetus's blood (but diffusion of nutrients and waste

products may still occur). This whole exchange apparatus is called the placenta. The inner cell mass will undergo changes similar to those in the frog such that the three basic cell layers are formed. First, the inner cell mass forms a cavity within itself. This is called the amniotic cavity and the cells which line it are the ectoderm. Below the amniotic cavity and its ectoderm, on the hollow side of the blastocyst, a layer of cells will form along the ectoderm and will eventually cover the entire hollow blastocyst. This layer is the endoderm. So what we have at this stage is a hollow ball of cells, the blastocyst, which is surrounded by trophoblast.

Inside this ball there are two compartments; an amniotic cavity defined by ectoderm and yolk cavity defined by endoderm. It is the two-layered sheet of cells formed by the ectoderm and endoderm that will develop into the fetus. So unlike the frog which developed from the whole ball of cells, the human only develops from the sheet of cells suspended in middle of the blastocyst. The rest of the cells of the blastocyst will be important in forming the placenta and chorion which protect the fetus inside the womb.
The primitive streak, equivalent to the neural plate in frogs, forms in the ectoderm above the endoderm. Cells from the primitive streak migrate down between the ectoderm and endoderm to become mesoderm. Further folding of the primitive streak gives rise to a neural groove and then a neural tube, etc. So

the formation of the primitive streak in mammals marks the beginning of gastrulation. It is quickly followed by neurulation. The fate of cells from the three basic cell layers will be the same as in frogs: endoderm will become the GI lining;
mesoderm will become connective tissue, bones, muscle, blood; ectoderm will

become skin and nervous tissue (from the neural grove and neural crest cells),
etc.

Copyright by The Berkeley Review

230

The Berkeley Review


Specializing in MCAT Preparation

BlOlOgy

Reproduction fir: Development

Human Embryo Development

Hormones and Pregnancy The embryo cannot implant if the uterus is not receptive, that is, if it is not quiescent. Furthermore, pregnancy will not be maintained by the uterus even afterimplantation unless it remains quiescent. So to maintain the uterine lining through the first part of pregnancy the trophoblastic cells secrete chorionic
gonadotropin. Chorionic gonadotropin (CG) is a hormone which causes the

corpus luteum to continue to produce estrogen and progesterone. As long as the levels of estrogen and progesterone, particularly progesterone, are sufficient, they keep the uterine lining quiescent and pregnancy continues. Eventually, the placenta is able to take over the production of estrogen and progesterone itself
and the corpus luteum is not needed.

The placenta secretes increasing amounts of estrogen and progesterone. However, the levels of estrogen increase faster than the levels in progesterone. And at some point near the end of pregnancy the levels of progesterone plateau. At a crucial ratio of [progesterone]:[estrogen] the uterus is no longer quiescent; it begins to have contractions.The smooth muscle of the uterus contracts,putting it under tension. Also the size of the fetus at this stage can put the smooth muscle in the walls of the uterus under tension. When the walls of the uterus are put under tension they send nervous impulses to the hypothalamus. The hypothalamus sends signals to the posterior pituitary (via nerves) to release
oxytocin. Oxytocin is released into the blood and is a strong inducer of more contractions of the smooth muscle of the uterus. Oxytocin also stimulates the production/secretion of prostaglandins which further induce contractions.

All of these hormones and nerves form a positive feedback system. Once started (i.e., contraction) each step stimulates the next and eventually stimulates the first process more (stronger contractions). Eventually the contractions force the fetus through the vagina and into the hard, cold world. We call the birth process
parturition.
Lactation

There are two different processes that go on in lactation. First, there must be milk production, and second, there must be milk ejection. Breast milk is a fairly

complex fluid containing proteins, fats, vitamins, and other goodies (antibodies, etc.). There are epithelial cells within the breast that make up the glands that produce the milk. Around these epithelial cells are myoepithelial cells which can contract around the milk glands, ejecting the milk that has been produced.
Production of milk is stimulated by the suckling of the infant on the mother's nipple. The nipple sends a nervous impulse to the hypothalamus. The hypothalamus releases PRH, prolactin releasing hormone, which acts on the anterior pituitary. PRH stimulates the anterior pituitary to release prolactin. Prolactin goes into the bloodstream and stimulates the epithelial cells in the breasts which comprise the milk glands to produce more milk. Milk ejection uses a similar process, but different hormones. The suckling on the breast sends a nervous signal to the hypothalamus. The hypothalamus send a nervous signal to the posterior pituitary. The posterior pituitary responds by releasing oxytocin. Oxytocin causes contraction of the myoepithelial cells in the breast. Milk gets squeezed out.

Copyright by The Berkeley Review

231

The Berkeley Review Specializing in MCAT Preparation

Reproduction
and Development
15 Passages
100 Questions

Passage Titles
I. II. III. IV. V. VI. VII. VIII. IX. X. XL XII. XIII. XIV. XV.

Questions
1-8 9- 15 16-21 22-29 30-35 36-43 44-49 50-56 57-63 64-69 70-76 77-83 84-90 91 -95 96- 100

Spermatogenesis, Oogenesis, and RU486


Male Contraception Fertilization of the Ovum Pregnancy Fuel Utilization Female Reproductive Hormones

Endocrine Control of the Ovarian Cycle


Gestation Female Birth Control Vaccine

Estrogen lsoflavone Experiment


Ovulation Vertebrate Gastrulation

Oxytocin and Labor


Puberty
Testicular Cancer

The

BERKELEY
J 1

X-TR-E-V-I-E-W Speciali zing in MCAT Preparation

db

Suggestions
The passages that follow are designed to get you to think in a conceptual manner about the processes of physiology at the organismal level. If you have a solid foundation in physiology, many of these answers will be straightforward. If you have not had a pleasant experience with the topic, some of these answers might appear to come from the void past the Oort field of the solar system.
Pick a few passage topics at random. For these initial few passages, do not worry about the time. Just focus on what is expected of you. First, read the passage. Second, look at any diagrams, charts, or graphs. Third, read each question and the accompanying answers carefully. Fourth, answer the questions the best you can. Check the solutions and see how you did. Whether you got the answers right or wrong, it is important to read the explanations and see if you understand (and agree with) what is being explained. Keep a record of your results.
After you feel comfortable with the format of those initial few passages, pick another block of passages and try them. Be aware that time is going to become important. Generally, you will have about 1 minute and 15 seconds to complete a question. Be a little more creative in how you approach this next group. If you feel comfortable with the outline presented above, fine. If not, then try different approaches to a passage. For example, you might feel well versed enough to read the questions first and then try to answer some of them, without ever having read the passage. Maybe you can answer some of

the questionsby just lookingat the diagrams, charts,or graphs mat are presented in a particular passage. Remember, we are not clones of one another. You need to begin to develop a format that works best for you. Keeping a record of your results may be helpful.

The last block of passages might contain topics that are unfamiliar to you. Find a place where the level of distraction is at a minimum. Get out your watch and time yourself on these passages, either individually or as a group. It is important to have a feelfor time, and how much is passing as you try to answer each question. Never let a question get you flustered. If you cannot figure out what the answer is from information given to you in the passage, or from your own knowledge-base, dump it and move on to the next question. As you do this, make a note of that pesky question and come back to it at the end, when you have more time. When you are finished, check your answers and make sure you understand the solutions. Beinquisitive. If you do not know the answer to something,look it up. The solution tends
to stay with you longer. (For example, what is the Oort field?)

The estimated score conversions for 100 questions are shown below. At best, these are rough approximations and should be used only to give one a feel for which ballpark they are sitting in.

Section IV

Estimated Score Conversions


Scaled Score
>12 10-11

Raw Score
86 -100
79-85

8-9
7

65-78

59-64 54-58
48-53 0-47

6 5
<4

Biology
Passage I (Questions 1-8)

Spermatogenesis, Oogenesis, & RU 486

Passage I

Cellular division does NOT occur in which of the

following cell types?


Human development begins with the union of the sperm (male gamete) and the ovum (female gamete) during fertilization. The unicellular organism which is formed is called the zygote.
The formation of the male and female gametes occurs during gametogenesis, a process in which there are two
successive meiotic divisions to reduce the chromosome

A. B. C. D.

Spermatogonia Oogonia Spermatids Primary oocytes

Before fertilization the secondary oocyte is arrested


in meiosis II at:

number from a diploid state to a haploid state. During spermatogenesis the male germ cells (called spermatogonia) grow and are transformed into primary spermatocytes. Each primary spermatocyte forms a haploid secondary spermatocyte after the first meiotic division. After the second meiotic division, secondary spermatocytes form spermatids which are then gradually transformed by differentiation (spermiogenesis) into mature sperm. Spermatogenesis takes about 64 days.

A. B. C. D.

interphase. prophase. metaphase. anaphase.

Roughly 3 days after fertilization in the ampulla of


the human fallopian tube, the morula:

During oogenesis the female germ cells (called oogonia) grow and are transformed into primary oocytes. These oocytes begin their first meiotic division before parturition but are arrested in prophase until after puberty. This reductive division is not completed until shortly before ovulation. As the primary oocyte grows during puberty, follicular epithelial cells begin to surround the oocyte, forming a primary follicle. As soon as a second layer of epithelial cells is formed the primary follicle becomes a secondary (or mature) follicle. The second meiotic division begins at ovulation but is arrested midway through the cycle unless the secondary oocyte is fertilized by a sperm. Once fertilization takes place the mature oocyte is referred to as the ovum.

I.
II.

develops a blastocoel.
contains about 16 blastomeres.

III. passes into the uterus. IV. forms an outer cell layer called the tropoblast.
A. B.

C.
D.

I and II only II and III only HI and IV only I and IV only

As the mature ovarian follicle begins to swell, a

small oval protrusion called a stigma forms on the side of the follicle juxtaposed to the peritoneal cavity. Rupture of the stigma:
I. releases progesterone. II. is due to increased follicular pressure. m. releases a secondary oocyte.

The monthly reproductive cycles of the female is regulated by the gonadotropins, follicle-stimulating
hormone (FSH) and luteinizing hormone (LH). FSH induces the development of an ovarian follicle. However, both FSH and LH are required for follicular maturation. As the mature follicle grows, it begins to produce

IV.
A. B. C. D.

results from stimulation by prostaglandins.


I only II and HI only IV only I, n, m, and IV

estrogen. High estrogen levels produce a surge in the synthesis of LH and this surge triggers ovulation of the secondary oocyte. The mature follicle develops into a glandular structure called the corpus luteum and begins to secrete both progesterone and estrogen. These hormones begin to prepare the endometrium of the uterus for implantation of the blastocyst.
If fertilization occurs, the corpus luteum becomes the

Dispermy is an abnormal fertilization process in which two sperm fertilize an ovum. The resulting
embryo will: A. show trisomy 21.

corpus luteum of pregnancy and is maintained for about 20 weeks until the placenta can assume production of progesterone and estrogen. If fertilization does not occur, the corpus luteum begins to degenerate about 12 days after ovulation and becomes the corpus luteum of
menstruation.

B.
C. D.

be triploid and therefore exhibit aneuploidy.


show monosomy. will not exhibit polyploidy.

Copyright by The Berkeley Review

235

The Berkeley Review

Specializing in MCAT Preparation

Biology

Spermatogenesis, Oogenesis, & RU 486


8.

Passage I

The molecular structure that BEST represents oxytocin is:


A.
CH,

The molecular structure of the synthetic steroid RU 486 is shown in Figure 1.

HO

-P*
HO

- CH, NH2

H,C

B.
H 0

:"/ c= c- CH,
I II
H I O II

RU486

H3N-C-C-N-C-C-O-CH3 CH2

coo(

""NQ>
shown below.

Figure 1

c.

The molecular structures of four naturally


occurring steroids derived from cholesterol are

D.
ss-

h3c

oh

HjN- Cys- Tyr- He Gin- Asn- Cys


H,N- C Gly - Leu- Pro
II

Progesterone

Testosterone

7.

RU 486, used widely in France to terminate unwanted pregnancies, was originally tested as an antagonist for glucocorticoid receptors. During testing this compound was also shown to have a high affinity for other steroid receptors as well.
Administered alone, RU 486 has about an 80%

success rate in expulsion of the embryo from the endometrial lining of the uterus. However, when a drug which causes uterine contractions is administered about 36 to 48 hours after receiving RU 486, the success rate increases by about 26%. RU 486 has a high success rate because it blocks steroid receptors from binding:
A. glucocorticoids and allows administered progesterone to cause uterine contractions. testosterone and allows administered oxytocin
to cause uterine contractions.
Based on the structures of the four steroids shown

B.

above, which receptors will RU 486 bind to with the strongest affinity? A. B. C D. Progesterone and testosterone receptors. Estradiol and Cortisol receptors. Testosterone and estradiol receptors. Cortisol and progesterone receptors.

C
D.

progesterone and allows administered prosta glandins to cause uterine contractions. estradiol and allows administered glucocor
ticoids to cause uterine contraction.

Copyright by The Berkeley Review

236

The Berkeley Review Specializing in MCAT Preparation

Biology
Passage II (Questions 9-15)

Male Contraception

Passage D

12. What changes does a vasectomy cause in the hypothalamus, particularly involving the secretion of
FSH and LH?

Sterilization is a contraceptive option for people who are positive they wish to have no children. About 500,000 men in the U.S. choose to have vasectomies per year. Vasectomy is the surgical ligation of the vas deferens, resulting in sterilization. Although sperm are produced normally in men with vasectomies, the sperm cells are not able to exit the testis. In a local phenomenon, sperm cells are phagocytosed in the testis by macrophages and other immune system cells in men with vasectomies. As a result of this immune attack, antibodies to sperm are elevated in
men with vasectomies.

A.

Both FSH and LH are increased.

B. C.

FSH is increased, and LH is decreased. FSH is decreased, and LH is increased.

D.

Both FSH and LH are unchanged.

13.

What is the anatomical location of sperm production?


A. B. Vas deferens Seminiferous tubules

9.

What is the effect of vasectomy on sperm production


and secretion after several months?

C.
D.

Prostate gland
Testicle interstitium

A. B.

Both sperm production and sperm secretion


increase.

Both sperm production and sperm secretion


decrease.

C.
D.

Neither sperm production nor sperm secretion


change.

14.

Which of the following statements are TRUE? I. The sperm count ultimately drops to zero following a vasectomy. II. The female counterpart of vasectomy hysterectomy, the removal of the uterus. in. Vasectomy decreases testosterone levels.
A. B. C.
D.

Sperm production is unaffected and sperm secretion stops.

is

10. What is a potential consequence of reversing a


vasectomy?

I only I and II only II and HI only


I, II, and III

A. B. C.
D.

Antibodies to sperm may reduce fertility. Antibodies to sperm may increase fertility. Birth defects are more likely in subsequent
children.

Birth defects are less likely in subsequent


children.

15. In an experiment on dogs, researchers used an injection of a stable polymer to block the vas deferens. What results could be expected from this experiment?
11. If a man were diagnosed with oligospermia, what implications would this have for his fertility status?
A. He could not ever father a child.

A.
B. C. D.

The dogs undergoing this treatment would


remain fertile.

B. C.
D.

He has reduced fertility status compared to the population average. He has increased fertility status compared to the population average. He has the same fertility status as the population average.

The dogs undergoing this treatment would develop enlarged testicles. The dogs undergoing this treatment would
become sterile.

The dogs undergoing this treatment would develop autoimmune disease.

Copyright by The Berkeley Review

237

The Berkeley Review Specializing in MCAT Preparation

Biology
Passage HI (Questions 16-21)

Fertilization Of The Ovum

Passage m

18. During pregnancy the levels of progesterone and


estrogen:

In fertile women, ovulation occurs approximately 12-

16 days after the onset of the previous menses. The ovum


must be fertilized within 24-48 hours if conception is to
result.

A.
B. C. D.

Fertilization of the ovum by the sperm usually occurs in the mid portion (ampulla) of the uterine tube. The
fertilized ovum, now called a blastocyst, moves down the
uterine tube into the uterus. Once in contact with the

increase during the first trimester, but decrease during the remaining six months. increase during the first six months, but decrease during the last trimester. increase steadily until parturition. mimic the levels found during the menstrual cycle.

endometrium, the blastocyst becomes surrounded by an outer layer of syncytiotrophoblast and an inner layer of cytotrophoblast. The syncytiotrophoblast erodes the endometrium, and the blastocyst burrows into it, a process called implantation. After this process, the development of the placenta then proceeds. Human chorionic gonadotropin (hCG) is secreted by the placenta, and during the early stages of pregnancy, acts to maintain the corpus luteum, an endocrine organ that was maintained by luteinizing hormone (LH) during the luteal phase of the menstrual cycle. The corpus luteum, now termed the corpus luteum of pregnancy, primarily secretes estrogens and progesterone. In addition to secreting hCG, the placenta, after about the 6th week of pregnany, takes over the function of the corpus luteum and produces sufficient estrogen and progesterone to continue suppression of gonadotropin releasing hormone (GnRH), follicle stimulating hormone (FSH), and LH.

19. According to the passage, it can be inferred that during pregnancy leutenizing hormone and follicle stimulating hormone levels would:

A.
B.

remain relatively low, thereby eliminating further follicle development and ovulation.
be undetectable in either fetal or maternal blood.

C.

increase in the maternal blood but decrease in the fetal blood.

D.

progressively increase until ovulation has taken place.

16. Removal of hCG during the first 6 weeks of pregnancy would result in: I. n. HI.
A. B.

20. Ovariectomy (removal of the ovaries) or damage to the ovaries during the last six months of fetal development would:

continuation of the pregnancy. regression of the corpus luteum. termination of the pregnancy.
I only Ilonly I and II only II and HI only

A. B.
C.

have no effect at all on the pregnancy. lead to termination of the pregnancy.


result in a decrease in the levels of hCG.

D.

result in an increase in follicular development.

C. D.

17. The protein hormone hCG is very similar but NOT


identical in structure to:

21.. If implantation of the blastocyst does not occur in the uterine lining, then the proper hormonal levels needed to maintain the lining are removed and menses normally follows. Which of the following structures is sloughed off during menses?
A.

Endometrium

A.

LH

B.

B. C. D.

FSH GnRH

C. D.

Myometrium Primary oocyte Primary follicle

Estrogen

Copyright by The Berkeley Review

238

The Berkeley Review Specializing in MCAT Preparation

Biology
Passage TV (Questions 22-29)

Pregnancy Fuel Utilization

Passage IV

23.

Human gestation is 40 weeks. Along with the changes apparent to the eye, fuel metabolism changes considerably from early pregnancy (0-25 weeks) to late pregnancy (2540 weeks). In early pregnancy, the main goal is to build up maternal stores of adipose tissue. The fetus is quite small and has low energy needs during this time. The mother experiences increased insulin sensitivity which increases glucose and fatty acid uptake by adipose tissue, especially in the lower body depots.

Insulin is important for energy storage during early pregnancy. Which of the following statements is FALSE regarding the effects of insulin on adipose
tissue?

A. B.

C.
D.

Insulin increases synthesis of triglyceride. Insulin increases production of oc-glycerol phosphate. Insulin increases uptake of lipid from circulating lipoproteins. Insulin increases synthesis of glucose.

However, in late pregnancy, the fuel needs of the fetus


are increased. The mother becomes insulin resistant, so

that glucose is diverted to the fetus. The growing fetus feeds continuously. Glucose, received through the

24.

placenta via facilitated diffusion, is its preferred fuel. The mother mobilizes fatty acids from her adipose tissue to meet her energy needs. Maternal gluconeogenic enzymes are increased, as well, during late pregnancy.
22. When triglycerides are mobilized from adipose tissue, which of the following compounds are
released into the blood?
I. H - C- O - C (CH2),4 CHj
o
II

Some women experience gestational diabetes during late pregnancy. This is partially due to insulin resistance. Therefore, many obstetricians screen all their patients for abnormal glucose tolerance using an oral glucose tolerance test. An oral load of 50 grams (200 kcals) of glucose is given to a fasting person. A blood sample is taken 1 hour later. What changes in glucose and insulin compared to pretest values would be seen in a woman with gestational
diabetes?

A. B. C. D.

Increased glucose, increased insulin. Increased glucose, decreased insulin. Decreased glucose, decreased insulin. Decreased glucose, increased insulin.

H- C- O-C (CH,)I4 CH,


o

II

H - CO-C (CH2)I4- CH3


H

25.

Which organ produces insulin?


A. B. Pancreas

Thyroid gland
Liver

II.
H
I

C. D.

Spleen

H-C-OH

I
H-C-OH

I
H-C-OH
H

26.

Which of the following statements are TRUE for late pregnancy?

I.
III.

CH3-(CH2)14COO

Gluconeogenic enzymes provide glucose for the fetus when the mother is not eating. II. Fatty acids are a required fetal energy source. HI. Maternal fat stores are rapidly filled during
late pregnancy.

A.
B.

C.
D.

I only II only II and HI only I and IQ only

A. B. C.
D.

I only I and II only II and HI only


I, n, and m

Copyright by The Berkeley Review

239

The Berkeley Review Specializing in MCAT Preparation

Biology

Pregnancy Fuel Utilization

Passage IV

27. How would untreated gestational diabetes most likely affect the size of the fetus?
A. B. C. D. Lower birthweight. Higher birthweight. No change in birthweight. During the first trimester fetal weight is slightly above normal, but at parturition the birthweight is much lower.

28.

Which of the following is TRUE regarding glucose transport across the placenta? I. It is an energy requiring process. EL It occurs by facilitated difussion. EOT. It is higher during early pregnancy. A. B. C. D. I only II only I and ffl only I and IV only.

29. What type of cells contain gluconeogenic enzymes?


A.
B.

Placental cells Neurons

C.
D.

Hepatocytes
Intestinal mucosal cells

Copyright by The Berkeley Review

240

The Berkeley Review

Specializing in MCAT Preparation

Biology
Passage V (Questions 30-35)

Female Reproductive Hormones

Passage V

32.

According to the passage, the preovulatory decline


of FSH is due to:

The female reproductive system undergoes a series of regular cyclic changes termed the menstrual cycle. The most obvious of these changes is periodic vaginal bleeding resulting from shedding of the endometrial lining of the uterus. It results primarily from the interaction of hormones derived from the hypothalamus, pituitary gland, and ovaries. In most women in the middle reproductive years, menstrual bleeding recurs 25-35 days, with a median cycle length of 28 days. The interval from the
onset of menses to ovulation is termed the follicular or

A. B. C. D.

positive feedback of estradiol. negative feedback of FSH. negative feedback of estradiol. positive feedback of FSH secretion.

proliferative phase. The time proceeding ovulation to the


onset of menstrual bleeding is termed the luteal or secretory phase. Ovulation normally occurs at about the 14th day of the cycle.
33.

In the absence of pregnancy, menses normally occurs. This may be due to the decline of the hormonal requirement needed by the endometrium. The primary hormones required by the
endometrium, so that menses does not occur, are:

In normal menstrual cycle, serum concentrations of both leutenizing hormone (LH) and follicle stimulating hormone (FSH) begin to increase prior to menses. FSH
concentrations attain maximum levels during the first half

I.
II. m. IV.
A. B.

progesterone.
estradiol. hCG. FSH and LH.

of the follicular phase and, with the exception of a brief peak at midcyle, continue to fall until the lowest
concentration in the cycle are reached during the second half of the luteal phase. The preovulatory decline of FSH is due to the increasing concentration of estradiol. LH levels increase gradually throughout the follicular phase and at midcyle, there is a large peak in serum concentration of LH. Subsequently, LH levels gradually decline reaching their lowest concentration late in the
luteal phase.

C.
D.

I and II only I and ffl only II and IV only ffl and IV only

30. Women treated over a long period of time with

34.

All of the following structures will either secrete

relatively large doses of progesterone and estrogen


do not ovulate. This is most likely due to:
I. inhibition of FSH.

estrogen alone, progesterone alone, or estrogen and progesteronetogether EXCEPT the:


A. B. C.
D.

II.
A. B. C. D.

direct inhibition of progesterone by estrogen.


I only II only I and II only II and HI only
35.

DI. over stimulation of FSH by estrogen.

anterior pituitary. placenta. granulosa cells.


adrenal cortex.

The stimulus for FSH and LH production and

31. Ovulation is marked by a maximal peak of LH


concentration. This is primarily due to:

secretion is governed by the pulsatile release of gonadotropin releasing hormone (GnRH). Which one of the following structures produces GnRH?
A. B. C. D. Anterior pituitary Posterior pituitary Hypothalamus Pineal gland

A. B. C. D.

positive feedback by progesterone. positive feedback by estradiol.


negative feedback by progesterone. negative feedback by estradiol.

Copyright by The Berkeley Review

241

The Berkeley Review

Specializing in MCAT Preparation

Biology
Passage VI (Questions 36-43)

Endocrine Control Of Ovarian Cycle

Passage VI

36.

What is meant by negative feedback?

Endocrine control of the ovarian cycle is complex, with cases of both positive and negative feedback control. The following diagram illustrates interactions between the hypothalamus, the anterior pituitary, and the ovaries. The following abbreviations are used in Figure 1: GnRH (gonadotropin releasing hormone), FSH (folliclestimulating hormone), and LH (luteinizing hormone).

A.

B.

The rate-limiting enzyme in a pathway is activated by the product(s) of the pathway. The rate-limiting enzyme in a pathway can only be inhibited.

C.
D.

The rate-limiting enzyme in a pathway is inhibited by the product(s) of the pathway. The rate-limiting enzyme in a pathway can
only be activated.

Hypothalamus ^
GnRH

"\

37. Where is the ovum released upon ovulation?


A. B. C. Uterus Uterine tube Cervix

Anterior

Decreased

>3
p
aB3

D.

Abdominal cavity

Pituitary

negative feedback
inhibition

38. Thecorpus luteum produces which of the following


hormones? A. GnRH

r- FSH & LH

(Day 1) i Estradiol j
Ovaries

(Day 28) i Progesterone *\


Corpus luteum
regresses

B.
C. D.

Progesterone
LH FSH

T Estradiol

Increased sensitivity
of follicles to FSH
LH

ft
Ovaries

39. Oral contraceptives contain both estradiol and progesterone. Usually, the pills are taken for 3 weeks and stopped for 1 week to allow menstrual

only'jj
iLH

flow. What happens to the ovarian cycle when oral


contraceptives are taken?
I. n. Inhibition of ovulation Decreased FSH and LH

Growth of
Follicles

I FSH

17
T Estradiol

Anterior

III.
I "a

Increased growth of follicles


II only II and ffl only I and II only I, H, and ffl only

pituitary

Jl (+) Feedback
Anterior

ft
4 GnRH

Pituitary

ft
Hypothalamus

A. B. C. D.

LH Surge

(-) Feedback4^
T Estradiol

40. Menopause is the cessation of the menstrual cycle. New follicles cease to develop in the ovary, although the anterior pituitary continues to function normally.
What levels of FSH and LH would be seen in the

Ovaries

T Progesterone

ft ^
Ovulation

blood of a menopausal women who takes no exogenous hormones?


.

> Empty follicle will


become the

(Day 14)

corpus luteum J
Figure 1

A. B. C. D.

Increased FSH, decreased LH. Decreased FSH, increased LH. Decreased FSH, decreased LH. Increased FSH, increased LH.

Copyright by The Berkeley Review

242

The Berkeley Review Specializing in MCAT Preparation

Biology
41.

Endocrine Control Of Ovarian Cycle

Passage VI

To what class of hormones do estradiol and

progesterone belong?
A. B. Steroid hormones. Growth hormones.

C. D.

Peptide hormones. Glucoregulatory hormones.

42. A hysterectomy is the surgical removal of the uterus.


What effect would this have on the cycle depicted in Figure 1?

A.
B.

No changes in ovarian cycle, but no


menstruation.

C. D.

Anovulation due to lack of negative feedback from estradiol and progesterone. Decreased development of corpus luteum.

Prolonged menstrual flow.

43. What is the trigger for ovulation?


A.
B.

Regression of corpus luteum.


Follicle sensitization to FSH.

C.
D.

LH surge.
Increased estradiol.

Copyright by The Berkeley Review

243

The Berkeley Review

Specializing in MCAT Preparation

Biology
Passage VII (Questions 44-49)

Gestation

Passage VII

45.

One of the factors involved in the differentiation of

By the 42nd day of gestation, the embryonic gonads are distinguishable. Under the influence of the genes that
code for male sex determination, the gonad will begin testicular differentiation by 43-50 days of gestation. In the

the Wolffian ducts is testosterone. The production and secretion of testosterone, during this stage of development, is under the influence of:
A. B. C. D.
46.

gonad destined to be an ovary, the lack of differentiation persists. At 77-84 days, a significant number of germ cells
enter meiotic prophase to characterize the transition of oogonia into oocytes, which marks the onset of ovarian differentiation from undifferentiated gonads.
In the 7th week of gestation, the embryo has both male and female primordial genital ducts. In a normal female fetus, the Miillerian duct system develops into the uterine tubes, uterus, cervix, and upper one-third of the vagina. In normal male fetus, the Wolffian duct system on each side develops into the epididymis, vas deferens, seminal vesicles, and ejaculatory ducts. In the presence of a functional testes, the Miillerian

follicle stimulating hormone (FSH). human placental lactogen (hPL). human chorionic gonadotropin (hCG). gonadotropin releasing hormone (GnRH).

Testosterone is produced by which of the following


structures?

I. II.
m.
A. B.

Corpus luteum. Interstitial cells of Leydig.


Sertoli cells.

ducts involute under the influence of "Miillerian inhibiting factor" secreted by Sertoli cells. The differentiation of the Wolffian duct is stimulated by testosterone secretion from the testes. In the presence of an ovary or in the absence of
a functional fetal testis, Miillerian duct differentiation
occurs and the Wolffian ducts involute.

C. D.

I only Ilonly I and II only II and HI only

47.

Treatment of an XY embryo at gestational day 77 with anit-Mullerian inhibitingfactor antibody would


result in development of:
A. Wolffian duct structures

Experimental evidence has accumulated demonstrating


that a small portion of the distal short arm of the Y chromosome, termed H-Y antigen, is critical for testicular organogenesis of the bipotential gonads. The following experiments are a summary that supports this hypothesis.
Experiment J

B.

Miillerian duct structures and male external

genitalia.
C.
D.

Wolffian duct structures and male external genitalia.


Miillerian duct structures and female external

genitalia.
48.

Using the "moscona" technique, dissociated cells derived


from either mouse or rat newborn testes in culture

Administration of anti-H-Y antigen antibody at gestational day 80 to an XY embryo would result in:
A. B. undifferentiated testes. formation of "follicular-like" structures.

reorganized to form seminiferous tubules. Another group


treated with anti-H-Y antibody resulted in the reorganization of cells into "follicular-like" structures
rather than seminiferous tubules.

C.
D.

no change.
undifferentiated testes and formation of
"follicular-like" structures.

Experiment 2

In a similar experiment, bovine or human fetal XX undifferentiated gonads formed testicular-like structures' when incubated with H-Y antigen.

49. Treatment of an XX embryo with H-Y antigen during gestational day 90 would result in
development of:
I. Wolffian duct structures.

44. The Sertoli cells, which produce "Miillerian


inhibiting factor" are located in the:
A. seminiferous tubules.

II.

Miillerian duct'structures.

in. female external genitalia. IV. male external genitalia.


A. B.

B.
C.

epididymis.
vas deferens.

D.

corpus luteum.

C. D.

I only Ilonly I and HI only II and III only

Copyright by The Berkeley Review

244

The Berkeley Review Specializing in MCAT Preparation

Biology
Passage VHI (Questions 50-56)
A

Female Birth Control Vaccine

Passage vm

52. If the study were continued by giving a repeat


injection of the same concentration of vaccine at

birth control vaccine for women has been

week 6, what would happen to the anti-hCG


antibody titer?

researched by the World Health Organization since 1974. This vaccine promotes antibodies against the hormone human chorionic gonadotropin (hCG) which is produced by the embryo. hCG maintains a functional corpus luteum, allowing it to continue to produce estrogen and progesterone during pregnancy, and promotes normal implantation of the embryo into the uterine lining. Normal, nonpregnant levels of hCG are 0.000 nmol/L in plasma, and levels increase during the first trimester of
pregnancy.

A. B. C.
D.

Increase and stay constant Decrease and stay constant No change in titer
Increase and then decrease

53.

Why is diphtheria toxin used?


A.

The vaccine incorporates a synthetic peptide representing the amino acid sequence 109-145 of the Cterminal region of the B subunit of hCG. This particular region was chosen to avoid homology between hCG and luteinizing hormone (LH). The hCG peptide is conjugated to diphtheria toxin to form a hapten-protein carrier complex. Antibodies to hCG are raised with this complex in female baboons and in women, following one injection. The vaccine was tested in 30 sterilized, human subjects, with the following results:
Anti-hCG antibody concentration

To make a larger complex for antibodies to recognize, allowing more effective antibody
formation

B. C.
D.

To protect LH against antibody formation via cross-reactivity The toxin prevents implantation of the embryo To provide simultaneous diphtheria immunity
and birth control.

54. At-home pregnancy tests contain monoclonal


antibodies in a test kit to which a small volume of

urine is added. Which of these compounds are present in urine only during pregnancy?
A.
B.

Group
1

Dall* Day 7 Week 5 (nmol/L) (nmol/L) (nmol/L) dose (^g)

Vaccine

Estrogen
Human chorionic villi

50 100
200

0.000
0.000

0.033

0.226
0.268 0.229

C. D.
55.

Human chorionic gonadotropin Progesterone

0.025
0.022 0.044 0.058

3
4

0.000 0.000 0.000

How would this anti-hCG vaccine affect a woman's

normal menstrual cycles?

500 1000

0.950
0.783

A. B.

Injection

The presence of hCG would mimic pregnancy, halting menstruation. The anti-hCG antibody would also bind LH, preventing ovulation, and causing irregular
menstruation.

50.

How could a researcher determine the concentration

C.
D.

of an antibody in a blood sample?

Although the vaccine would raise hCG levels to pregnancy levels, it would allow normal
menstruation.

A. B. C. D.
51.

Affinity chromatography using LH. Ion exchange chromatography. Affinity chromatography using hCG. Bradford protein assay.

The vaccine would not change normal menstrual cycles.

56. Which of these compounds can act as an antigen in


the vaccine?

Researchers estimated that a concentration of 0.52

nmol/L were required to provide a contraceptive effect. Which group achieved this effect by Week 5?
A. B. C. D. Group 1 Group 2 Group 3 Group 4

I. The hCG synthetic peptide II. Diphtheria toxin


ra.
A.

LH

B. C. D.

I only Ilonly I and II only II and III only

Copyright by The Berkeley Review

245

The Berkeley Review

Specializing in MCAT Preparation

Biology
Passage IX (Questions 57-63)

Estrogen

Passage IX

59.

Throughout a women's life, estrogen is produced primarily by the:


A. B. uterus. ovaries.

Estrogen, a steroid hormone, is not limited in its role as being one of the major hormones present during a woman's reproductive cycle. Estrogen receptors can be
found in liver cells, melanocytes, neurons, vascular
endothelial and vascular smooth muscle cells. After

C. D.

anterior pituitary. posterior pituitary.

menopause, a prolonged estrogen deficiency results. This prolonged deficiency has been shown to promote osteoporosis and atherosclerotic disease. Studies indicate that women using estrogens after menopause have half the risk of dying of a myocardial infarction when compared to matched women not using estrogens. Furthermore, women with established coronary artery disease show a
70% decrease in mortality over a 10 year period when compared to similar women who do not receive estrogen. In the liver, estrogen stimulates enzyme production
that affects cholesterol catabolism. The overall effect is to

60.

Increasing the diameter of a blood vessel by a factor


of 2 results in a resistance which is:

A. ^/2 of the original resistance. B. ' I4 ofthe original resistance. C. J/8 of the original resistance. D. Vi6 of the original resistance.

impede the development of a lipid profile consistent with atherosclerosis. In addition, estrogens have direct effects on arterial walls. Recent cell culture studies suggest that estrogen may inhibit platelet aggregation and adhesion seen in early atherosclerosis.
Endothelial derived relaxing factor (EDRF) is the most important molecule produced by the endothelium. EDRF

61.

Estrogen has been shown to induce arteriolar relaxation in arteries lacking endothelial cells. The BEST explanation for this is that:
A. EDRF is not a vasodilator.

inhibits myofibril contraction in smooth muscle, leading


to vasodilation. Estrogen appears to stimulate EDRF. EDRF is opposite in function to endothelin, a potent vasoconstrictor released by the vascular endothelium. Estrogens have been shown to induce arteriolar relaxation in which the endothelium has been removed. Subsequent studies have shown that estrogens block voltage-gated
calcium channels.

B. C.

EDRF is not produced by the endothelial cells. estrogen directly acts on vascular smooth
muscle to cause relaxation.

D.

estrogen stimulates the production of EDRF.

62.

Which of the following effects of estrogen is not


consistent with the idea that estrogens benefit the circulatory system?

57.

An estrogen receptor complex within a cell will most likely:


A. be carried through the blood in search of a
target organ.

I.
II.

Introduction of estrogen leads to a rapid


decrease in vascular resistance.

Introduction of estrogen leads to decreased


catabolism of LDL-cholesterol.

III. Estrogens inhibit endothelial cell expression of


adhesive molecules.

B. C.

D.

result in the production of an intracellular secondary messenger. create mutations within estrogen regulated elements of the genome. bind to DNA, resulting in changes in transcriptional rates.

A. B. C. D.

I only II only I and II only I and III only

58.

EDRF most probably inhibits myofibril contraction


by:

63.
permitting calcium entry into the muscle cell. inhibiting calcium entry into the muscle cell. permitting potassium entry into the muscle
cell.

Based on information in the passage, it could be


concluded that estrogen:

A. B. C. D.

A. B.
C.

moderately stimulates release of endothelin. significantly stimulates release of endothelin.


inhibits release of endothelin.

inhibiting sodium entry into the muscle cell.


246

D.

plays no role in regulating levels of endothelin.

Copyright by The Berkeley Review

The Berkeley Review Specializing in MCAT Preparation

Biology
Passage X (Questions 64-69)

Isoflavone Experiment

Passage X

64. Isoflavones are antagonists to estrogen. What is the


role of an antagonist? A.
B.

The rate of breast cancer is about 75% lower in Far

Eastern countries compared to Western countries. Epidemiological studies from migrant populations suggests that an environmental rather than a genetic explanation is plausible. When people immigrate to other
countries, their rates of breast cancer come to resemble

An antagonist mimics a hormone, interacts

with the hormone's receptor, and leads to very


similar intracellular effects.

An agonist mimics a hormone, interacts with

the country where they are living rather than their home
country.

the hormone's receptor, and leads to very


similar intracellular effects.

C.

In the Far East, a significant quantity of soybean protein is consumed in the diet in many different forms, including beans, miso, tofu, and soy milk. Soy provides a rich supply of isoflavones, nonsteroidal compounds with estrogenic properties. Isoflavones are structurally similar to estrogens. They bind to the estrogen receptor and act to partially antagonize estrogen.
In a clinical trial, 6 women were studied during several menstrual cycles while living in a research center. After recording usual changes during several menstrual cycles, they began the clinical trial. First they followed a control diet for 1 menstrual cycle, then they followed another 1month diet with similar composition of macronutrients,

An antagonist mimics a hormone, interacts with the hormone's receptor, and leads to a
block of the intracellular effects of the
hormone.

D.

An agonist mimics a hormone, interacts with the hormone's receptor, and leads to a block of
the intracellular effects of the hormone.

65. What effect does the soy-protein diet have on the


concentration of FSH and LH at ovulation?

A.
B.

The soy diet group has suppressed levels of


FSH and LH compared to the controls. The control diet group has suppressed levels of FSH and LH compared to the soy group. FSH is suppressed by the soy diet, but LH is unchanged. LH is suppressed by the soy diet, but FSH is unchanged.

except the protein was largely from soy isolate (60 g of a


98 g protein diet).

C.

The effects of a soy diet on the concentrations of LH and FSH as compared to the control diet is shown in Figure 1.
201" o Control Soy

D.

66. If the amount of soy protein could be increased to

completely suppress LH and FSH, what would be


the theoretical effect on the subject?

A.
~-4 -3 -2-1 0 1 2 3 4
-4-3-2-101234

B. C. D.

Day of Ovulation

Day of Ovulation

The subject could become pregnant at any phase of her menstrual cycle. The normal menstrual cycle would continue without any change. Menstruation would begin sooner than

Figure 1
Table 1 indicates the results of several blood parameters that were followed during the study.

predicted by the past menstrual cycle. The subject would not ovulate.

[Plasma Hormone]
Testosterone (nmol/L)
Cholesterol (mmol/L)

Control Diet
1.25 0.38
4.27 1.08

Soy Diet
1.46 0.52
3.86 1.01

67. Which of the following statements could you infer


from the passage?
A. The sole risk factor for breast cancer is genetic background.

B.
Table 1

Japanese women and European women have


comparable rates of breast cancer.

The soy protein for this dietary intervention provided 45 mg of isoflavones. For comparison, a Japanese diet
contains about 150-200 mg of isoflavones/day.

C.
D.

Exposureto estrogen is a probable determinant


of developing breast cancer.
Breast cancer is not related to diet.

Copyright by The Berkeley Review

247

The Berkeley Review

Specializing in MCAT Preparation

Biology
provided by the soy isolate?
A. B. C. D. 98% 25% 33% 61%

Isoflavone Experiment

Passage X

68. What percentage of the protein in the soy diet was

69. What was the effect of the soy protein diet on


cholesterol levels?

A. B. C. D.

The soy diet was hypercholesterolemic. The soy diet was hypocholesterolemic. The soy diet was isocholesterolemic. The soy diet was microcholesterolemic.

Copyright by The Berkeley Review

248

The Berkeley Review

Specializing in MCAT Preparation

Biology
Passage XI (Questions 70-76)

Ovulation

Passage XI

70.

What structure is the source of the rise in progesterone after ovulation?

By studying physiological changes during the menstrual cycle, a woman may plan a pregnancy based on the time interval during which she is fertile. One change that is frequently tracked by hopeful parents is the basal body temperature, that is the temperature taken just after awakening in the morning. The hormone progesterone
leads to a rise in basal body temperature of about 0.5F. This rise signals ovulation has occurred. The temperature remains high until menstruation. By developing a temperature log over several months, a woman may use this information to predict times when she is most likely
to conceive.

A. B. C.
D.

Corpus luteum Egg Corpus albicans


Cervix

71. Suppose a woman is planning a pregnancy and is


plotting her basal body temperature daily. What
conditions should she observe to make an accurate
measurement?

A. B. C.
n

Eating or drinking nothing before arising. Lying still during the measurement. Taking measurement at same time daily.
All of the above.

Is.
E

D.

a
c

72. Which hormonal changes precede ovulation?


I. Rise in luteinizing hormone. II. Rise in progesterone. III. Rise in estrogen.
A. B. C.
D.

II
.5'2

I only I and III only n and in only


I, II, and m

"5b

73.

Contraceptive methods either inhibit ovulation, act as barriers to prevent conception, or prevent implantation of a fertilized egg. Which of the follow
methods inhibits ovulation?

15 7

14

21

28

Days of the Ovarian Cycle Figure 1

A. B.

Condoms

RU-486

C. D.

Birth control pills Diaphragm

Another predictor for ovulation is a changes in the mucus produced by the cervix. Cervical mucus, under the influence of estrogen, becomes more clear and watery
immediately prior to ovulation, to allow penetration by sperm. The mucus may be placed on a microscope slide, dried, and examined for a pattern the resembles a fern plant, a phenomenon called "ferning". This phenomenon is present at ovulation and signals fertility.
74. If a clinician tells a woman planning a pregnancy that she is exhibiting ferning, what does this mean?
A. B.
C.

She is pregnant. She is anovulatory.


She is infertile.

D.

She is ovulating.

Copyright by The Berkeley Review

249

The Berkeley Review Specializing in MCAT Preparation

Biology
ovulation?

Ovulation

Passage XI

75. Which of the following hormone(s) directly triggers


I. Luteinizing hormone II. Follicle stimulating hormone m. Estrogen

A. B. C.
D.

I only I and II only H and m only


I, H, and D3

76. What molecule is the precursor for the hormones estrogen and progesterone?
A.
B. C.

Sphingomyelin
Cholesterol Arachidonic acid

D.

Phosphatidyl choline

Copyright by The BerkeleyReview

250

The Berkeley Review

Specializing in MCAT Preparation

Biology
Passage XII (Questions 77-83)

Vertebrate Gastrulation

Passage XII

79.

During invagination, sheets of cells around the

Gastrulation is one of the most dramatic and crucial

blastopore increase in length and extend themselves into the interior of the embryo. Which of the

stages of vertebrate embryogenesis. The embryo, at first simply a hollow ball of cells, undergoes a transformation into a multilayered structure with a central gut tube and bilateral symmetry. The outer layer of cells formed during
gastrulation is considered ectoderm, while the middle and inner layers are considered mesoderm and endoderm, respectively. Since interactions between these three cell layers will determine the further developmental fate of the embryo, events occuring during gastrulation are vital for the proper development of the organism.

following could be responsible for this process of


cell sheet extension?

I.
II.

Cell shape changes within the cell sheet.


Mitotic division within the cell sheet.
sheet.

in. Unidirectional migration of every cell in the


A. B. C.
D.

I only II only I and H only


I, H, and HI

Gastrulation begins when cells around the embryonic blastopore begin to invaginate, or move towards the inside of the embryo. The site where this invagination initiates is referred to as the dorsal lip of the blastopore. In the first half of this century, it was learned that the dorsal lip acts as an organizer, inducing the tissue directly around it to begin invaginating while triggering tissue further away to adopt other specific cell fates.
It was later found that the cells of the dorsal lip secrete a diffusible signalling substance that slowly degrades after its secretion. The result is a concentration gradient of the substance, with higher concentrations existing closer to
the source of the secretion. Cells at different distances

80. In an embryo that is about to undergo gastrulation,


the outermost cells protect more interior cells from the external environment. This is partially accomplished by tight junctions which form near the outer surface of these cells. This outer cell layer is
most similar in structure and function to:

A.
B. C.

epithelial cells.
neuronal cells. smooth muscle cells. connective tissue cells.

from the dorsal lip are exposed to different concentrations of the substance, leading to the signalling of different behaviors. Due to its effect on morphogenesis (the "shaping" of the embryo), this substance was referred to as a morphogen.

D.

81.

The early stages of human embryogenesis look very similar to comparable stages in the formation of other vertebrate embryos. For example, human embryos have gill slits and a tail at a certain stage in development. This observation does NOT support which of the following statements?
A. Humans evolved from lower vertebrates.

77.

If the mesodermal cell layer is improperly formed during gastrulation, which of the following later structures would most likely be affected?
A.
B.

Nervous system
Heart

B. C. D.

Primate embryos should not have gill slits. Vertebrates may share a common ancestor. There is little selective advantage in altering these stages of embryogenesis.

C.
D.

Stomach lining
Liver

82.

If the morphogen mentioned in the passage were not

slowly degraded after being secreted, which of the


78. Which of the following could NOT act as a morphogen? A.
B.

following would result?

A.
B.

A very steep concentration gradient of


morphogen would form. The concentration gradient of morphogen would persist indefinitely. A concentration gradient of morphogen could
never be formed.

A small, inorganic compound


A steroid hormone

C. D.

A secreted protein A transmembrane protein

C.
D.

There would be no effect on gastrulation.

Copyright by The Berkeley Review

251

The Berkeley Review Specializing in MCAT Preparation

Biology

Vertebrate Gastrulation

Passage XII

83. The dorsal lip from a frog embryo is removed and

grafted opposite the dorsal lip of a second frog embryo. Assuming this was done prior to gastrulation, which of the following would belikely
to result?

A.

Gastrulation would never occur and the

embryo would die.

B.
C. D.

The grafted cells would be rejected by the


immune system of the host embryo. The embryo would develop into Siamesetwin tadpoles, the result of two independent invaginations during gastrulation. The embryo would gastrulate and develop
normally.

Copyright by The BerkeleyReview

252

The Berkeley Review

Specializing in NCAT Preparation

Biology
Passage XIII (Questions 84-90)

Oxytocin and Labor

Passage xm

86. Which of the following would NOT increase


contractions in a female in labor?

Human pregnancy lasts on average 270 days. During the last month of pregnancy, irregular uterine contractions increase in frequency. At the time of labor and delivery, the cervix dilates and softens, and the muscular body of
the uterus contracts to push out the fetus.
Oxytocin plays both a direct and an indirect role in labor. It acts directly on uterine smooth muscle cells to

A. Intravenous injection of oxytocin. B. Intravaginal administration of prostaglandins.


C. Intravenous injection of human chorionic
gonadotropin.

D.

Oral administration of prostaglandins.

make them contract. It alse^stimulates the production of


prostaglandins in the endometrium of the uterus, which

enhance the contractions produced by oxytocin.


During early labor, the uterus contracts without

87. What strategy could be used to prevent a pre-term


birth?

significantly higher levels of oxytocin compared to prepregnancy. Once labor is initiated, a positive feedback
loop is set up between the cervix and secretion of oxytocin. Signals of dilation from the cervix lead to signals in afferent nerves that promote increased secretion of oxytocin. In late labor, the plasma oxytocin level rises, leading to higher concentrations of oxytocin presented to
the uterus.

A. B.

Administration of an oxytocin secretagogue. Administration of prostaglandins such as RU486.

C.
D.

Administration of inhibitors of prostaglandin


production.

Administration of an oxytocin agonist.

Cys- Tyr He Gin- Asn- Cys- Pro Leu- Gly- NH2

88. A prostaglandin is a twenty-carbon long fatty acid that contains a five-membered carbon ring. Pictured below is the prostaglandin thromboxane B2.

Oxytocin

84. Where is oxytocin produced?


A. B. C.
D.

Posterior pituitary Hypothalamus Anterior pituitary


Placenta

Which fatty acid is the precursor for prostaglandins?


A. Stearic acid

B.
C. D.

Myristic acid
Linolenic acid Arachidonic acid

85.

During early labor, plasma oxytocin levels are not higher than prepregnancy levels of 25 pg/ml. What mechanism(s) could explain an increase in uterine contractions without an increased plasma level of oxytocin?
I. An increase in the number of uterine receptors for oxytocin during late pregnancy. Paracrine release of oxytocin that does not
communicate with the bloodstream.

89.

Oxytocin is a polypeptide hormone. What does this imply about its method of interaction with uterine
cells?

II.

I. II. III.

It works through a second messenger system. It stimulates mRNA synthesis directly. It is binds to a cellular membrane receptor.

IH. Increased conjugation of oxytocin in the liver.


A. B. C.
D.

I only I and II only II and HI only


I, n, and ffl

A. B. C.
D.

I only I and ffl only II and III only


I, n, and ffl

Copyright by The Berkeley Review

253

The Berkeley Review Specializing in MCAT Preparation

Biology

Oxytocin and Labor

Passage xm

90. Studies indicate that paraplegic women can go into


labor and deliver with few difficulties. What would this finding suggest?

A. Delivery can occur without stretch signals


from cervix.

B. C.
D.

Delivery requires voluntary contractions of the


abdominal muscles.

Prostaglandins play only a minor role in the delivery of children to paraplegic mothers.
Smooth muscle contraction does not occur in
these women.

Copyright by The Berkeley Review

254

The Berkeley Review

Specializing in MCAT Preparation

Biology
Passage XIV (Questions 91-95)

Puberty

Passage XIV

92.

Which of the following events does NOT occur


within the seminiferous tubules of the male

After an interval of childhood quiescence, hypothalamic-pituitary-gonadal activity intensifies in the


peripubertal period leading to increased secretion of

reproductive system?

gonadal sex steroids that cause secondary sexual development, the pubertal growth spurt, and fertility. This stage of development is often termed puberty or
adolescence.

A. B.

C.
D.

Spermatogenesis. Testosterone synthesis. Secretion of androgen-binding protein by the


Sertoli cells.

Enhancement of sperm production by follicle


stimulating hormone (FSH).

The first sign of puberty in the female is an increase in growth. This is accompanied by breast development, a process stimulated by increase in estrogen levels. Other developmental changes influence by estrogen include: enlargement of the labia minora and majora, dulling of the vaginal mucosa, production of a clear whitish vaginal secretion prior to menarche, and changes in uterine size and shape. The development of pubic hair is primarily influenced by adrenal and ovarian androgen secretion. In males, the first sign of normal puberty is an increase in the size of the testes, primarily due to seminiferous
tubular development. The stimulation of the interstitial cells of Leydig also plays a small component in the increase in testicular size. Pubic hair development is under the influence of the adrenal gland, as well as the testes, through secretion androgens.

93.

The development of the seminiferous tubules is


under the influence of:

I. leutenizing hormone (LH). II. follicle stimulating hormone (FSH). III. gonadotropin releasing hormone (GnRH).
A. B. C.
D.

I only II only I and II only


I, II, and III

The dramatic increase in overall body growth during the growth spurt at the time of puberty is under the influence of complex endocrine control. In addition to sex steroids, growth hormone (GH) plays an important role in this phenomenon.

94.

The stimulation of the interstitial cells of Leydig is


under the direct influence of:

A. B. C. D.

follicle stimulating hormone (FSH). leutenizing hormone (LH). gonadotropin releasing hormone (GnRH). Miillerian inhibiting hormone (MIH).

95.

In addition to growth hormone's direct effects on muscle and bone tissue growth, it also acts indirectly on these tissues by stimulating the production and
secretion of another hormone or factor which then

91.

Breast development in females is stimulated by an increase in estrogen secretion. Which of the


following hormones must increase in order for this phenomenon to occur?

acts on these tissues. Which of the following hormones mediate the indirect effects of growth
hormone?

A.

Insulin

A. B.
C.

Thyroid stimulating hormone (TSH) Gonadotropin releasing hormone (GnRH)


Prolactin

B. C. D.

Thyroxine Epidermal growth factor Insulin-like growth factor I (IGF-I)

D.

Progesterone
255

Copyright by The Berkeley Review

The Berkeley Review Specializing in MCAT Preparation

Biology
Passage XV (Questions 96-100)

Testicular Cancer

Passage XV

97.

Which of the following statements is TRUE regarding testicular cancer?


A. At least 90% of testicular cancers are found in
the vas deferens.

Cancer of the testes if found in only 1% of malignant tumors of male internal organs. This cancer, whichshows

little basis in genetics, is ten times more likely to be found


in males afflicted with cryptorchidism, a condition where
the testes have failed to descend down into the scrotum

B.
C.
D.

At least 90% of testicular cancers are found in


the seminiferous tubules.

during development. Only 5% of testicular tumors do not originate from germ cells. Because germ cells undergo spermatogenesis, it is difficult to determine the exact origin of tumor cells. There is some evidence that spermatogonia are the likely candidates giving rise to
testicular cancer. However, researches have found that

At least 90% of testicular cancers are found in

the epididymis.
At least 90% of testicular cancers are found in
the rete testes.

early carcinoma cells bear more of a similarity to fetal


germ cells than spermatogonia. It has been discovered, through studying teste tissue
98.

Based on information in the passage, one could

samples from infertile men, that certain morphologically


distinct cells were precursors to invasive cancer. These malignant cells were termed carcinoma in situ (CIS).
Most CIS cells will evolve into invasive cancer. Looking
for differences between invasive and CIS cells, researches

argue that germ cell tumors that come from fetal germ cells have a retarded developmental fate.
Which of the following DOES NOT support this
hypothesis?

noted that CIS cells have a tetraploid complement of


chromosomes.

A.
B.

Cryptorchidism is a risk factor.


Evidence shows that tumor development occurs in gestation. In mouse studies, cancer genes act in fetal germ cells. Infertile men run a greater risk for carcinoma
in situ.

The initial events transforming germ cells into tumor


cells is not well understood, but has been likened to

C.
D.

parthenogenesis in the female. Tumors arising from parthenogenesis, known as teratomas, contain multiple tissue types with little organization. It is postulated that
testicular cancers may have similar origins.

99.

How many chromosomes are found in a carcinoma


in situ cell? A. B. C. D. 23 46 69 92

96. Which of the following statements supports the theory that spermatogonia give rise to testicular
cancer?

100.

Which statement is TRUE regarding the genetic


basis of testicular cancer? A. Testicular cancer is an autosomal recessive
trait.

A.

B.

Sertoli cells give rise to tumors which differ significantly from tumors originating from germ cells. Cryptorchidism is a risk factor for testicular
cancer.

B. C. D.

Testicular cancer is found commonly within families prone to testicular cancer. Testicular cancer is rare among identical twins. Testicular cancer is largely solved by surgical
resection.

C.
D.

Testicular cancer is a rare cancer.

Tumor cells

do

not

have

haploid

chromosome number.

Copyright by The Berkeley Review

256

The Berkeley Review

Specializing in MCAT Preparation

Biology
Passage 1(1 - 8)

Reproduction St Development

Section IV Answers

Spermatogenesis, Oogenesis, & RU486

called spermatogonia are converted into mature spermatozoa. Asingle diploid (2N) spermatogonia is transformed process called spermiogenesis to form the mature spermatozoa. The spermatids do not undergo a cellular differentiation to form a spermatozoa. They simply differentiate. The whole process of spermatogenesis takes about
spermatocytes undergo meiosis II to form four haploid spermatids. Each spermatid next undergoes a transformation

Cis correct, spermatids. Cellular division does not occur in spermatids. During spermatogenesis male germ cells

into a 1 spermatocyte which then undergo meiosis I to form two haploid (N) V spermatocytes The two 2

64 days.

During oogenesis female germ cells called oogonia are transformed into mature ova. This process involves the cellular divisions ofmeiosis Iand meiosis II. Early in the development of the fetus oogonia undergo mitosis and are transformed into 1 oocytes. The diploid (2N) 1 oocyte enters into meiosis I and is arrested at prophase until puberty. Ashort time before ovulation meiosis I is completed and a haploid (N) 2 oocyte is formed along with the first polar body (which may divide into two more polar bodies). At ovulation the 2 oocyte undergoes meiosis II but
is arrested at metaphase. At fertilization the 2 oocyte completes meiosis II and forms the mature ovum and a second

polar body. Union ofspermatozoa and ovum gives rise to the zygote which is diploid. The correct choice is C.
2.

C is correct, metaphase. See the explanation for oogenesis given above. The correct choice is C.

3.

Biscorrect, II and III only. Fertilization leads to the formation of a zygote. The zygote undergoes numerous mitotic divisions to increase the number of cells but not the cytoplasmic mass. When the zygote divides into two cells (about 30 hours after fertilization) those two cells are referred to as blastomeres. About 3 days after fertilization the dividing zygote has formed about 16 blastomeres (a specialized cell-type). At this stage the cell mass is referred to
as a morula and it begins to enter into the uterus.

Roughly four days after fertilization a fluid-filled cavity, called the blastocyst cavity or a blastocoel, begins to form in the central portion of the morula. As this cavity forms the cells within the morula arrange themselves into two parts. One group of cells forms the outer layer of cells called the tropoblast. The other group of cells forms an inner cell mass called the embryoblast. The tropoblast gives rise to the placenta while the embryoblast gives rise to the embryo. By this stage in development the morula is referred to as a blastocyst. Around the sixth day after
fertilization the blastocyst (normally) attaches to the endometrium of the posterior wall of the uterus. The correct
choice is B.

D is correct, I, II, III, and IV. As the primary follicle begins to develop, an antrum is formed and the primary oocyte is moved to one side. The formation of the antrum signals the development of the secondary follicle. Towards the end of the follicular phase the granulosa cells (i.e., follicle cells) begin to secrete large amounts of progesterone (17oc-hydroxyprogesterone) into the antrum. As the fluid in the antrum increases the pressure builds up and places a
stress on the stigma. Prostaglandins, which are synthesized and released from almost all cells in the body, can act to stimulate the smooth muscle of the uterus and the smooth muscle surrounding the outer layer of theca cells (called the theca externa). Contraction of this muscle can cause an increase in pressure in the antrum and rupture the stigma. The contents of the antrum, including the secondary oocyte and the progesterone, are released into the peritoneal
cavity (the space outside the ovary). The correct choice is D.

B is correct, be triploid and therefore exhibit aneuploidy. If two sperm fertilize an ovum, then each sperm will contribute a chromosomal complement of 23. Since the ovum also has a chromosomal complement of 23, this
results in an embryo that is triploid and contains 69 chromosomes. If there is a deviation from the normal

chromosomal complement of 46, then that deviation is referred to as aneuploidy.

Trisomy 21 involves the presence of three chromosomes instead of the usual pair of chromosomes. This type of chromosomal abnormality is referred to as Down's syndrome and usually results in a germ cell with 24 instead of 23 chromosomes. Monosomy involves embryos that are missing a chromosome. Cells which are polyploid contain
multiples of the haploid number of chromosomes. The haploid number of chromosomes is 46. If a cell contained 69

chromosomes, it would be triploid. A triploid cell is a cell which exhibits polyploidy. The correct choice is B.
6.

D is correct. Oxytocin is considered to be a peptide hormone, as are the other hormones of the hypothalamus and
pituitary gland. In general, peptide hormones contain from 3 to over 200 amino acid residues. Choice A represents adrenaline (epinephrine), a water-soluble amine hormone. It stems from a class of hormones

called catecholamines. In the brain catecholamines act as neurotransmitters. Choice B is the methyl ester of a
Copyright by The Berkeley Review
257

The Berkeley Review

Specializing in MCAT Preparation

Biology

Reproduction St Development

Section IV Answers

dipeptide called aspartame (trade name is NutraSweet), an artificial sweetener which is commercially synthesized
in huge quantities. This is not a hormone. Choice C is estradiol, a lipid soluble steroid hormone that is synthesized in
the ovaries. The correct choice is D.
7.

C iscorrect, progesterone and allows administered prosta-glandins to cause uterine contractions. RU 486 resembles both progesterone (a progestin) and Cortisol (a glucocorticoid) and will bind to both of their receptors with high affinity. RU 486 does not resemble testosterone or estradiol as well as it resembles progesterone and Cortisol.
Therefore, binding to the receptors of these two steroids is minimal (if at all). The reasoning and structures
associated with question #7 would help you with this answer, but it is not necessary.

RU 486 is a progesterone antagonist and binds to progesterone receptors. Recall that progesterone aids in the conversion of the growing endometrial lining of the uterus into a tissue that canaccept a developing embryo (i.e., the blastocyst). Progesterone allows the cells of the endometrium to synthesize and store glycogen, increases the secretory activity of the endometrial lining, and causes an increase in the vascularization of the tissue. Progesterone
also acts to inhibit expulsion of the implanted embryo by reducing uterine contractions and constricting the opening of the cervix. If RU 486 binds to progesterone receptors, then the developing embryo will be facing a hostile
environment. Furthermore, 36 to 48 hours after RU 486 has been taken by the pregnant woman, prostaglandins are administered. Recall that prostaglandins (released by most cells of the body) cause contraction of smooth muscle, especially the smooth muscle of the uterus. Glucocorticoids, such as Cortisol, have a variety of effects on metabolism. In the liver Cortisol increases

gluconeogenesis and glycogen synthesis. In skeletal muscle it decreases glucose uptake, increases protein degradation, and decreases protein synthesis. In adipose tissue it decreases glucose utilization and increases lipid
metabolism. Glucocorticoids tend to impair wound healing. An antagonist to glucocorticoid receptors might actually increase wound healing. Glucocorticoids do not cause uterine contractions. The correct choice is C. D is correct, Cortisol and progesterone receptors. Glucocorticoid and progestin receptors bind RU 486 with the greatest affinity. Look for similarities in the molecules. First, note the three substituents at the C-3, C-11, and C-17
carbons of RU 486 (see below). Estradiol has a hydroxyl function at the C-3 position; RU 486 does not. Estradiol does not have a substituent at the C-l 1 position; RU 486 has a bulky dimethylaminophenyl group. Estradiol only has a hydroxyl group at the C-17 position. RU 486 has both a hydroxyl and a bulky alkyne residue. A similar analysis
can be done for testosterone. We will find that the substituents on RU 486 do not match well with those on both

estradiol and testosterone. If we now compare RU 486 to both progesterone and Cortisol, we will find more similarities. For example, both progesterone and Cortisol have a carbonyl function at the C-3 carbon. They also have a (somewhat) bulky substituent at the C-17 position. The correct choice is D.

">C= C-CH3

Passage II (9 - 15)
9.

Male Contraception

D is correct, sperm production is unaffected, and sperm secretion stops. This would be a pretty poor form of birth control if sperm secretion was unchanged. Immediately eliminate choices A and C as incorrect answers. Since the vas deferens is cut and closed, no sperm will be present in the semen several months following a vasectomy. Usually the patient is checked for aspermia before the procedure is considered a success.Sperm production is not affected by a vasectomy. Choice B is incorrect. The correct choice is D.
A is correct, antibodies to sperm may reduce fertility. Antibodies to sperm are produced when macrophages enter the testis to catabolize the extra sperm. These would hamper fertility if they immobilized sperm. Choice B is incorrect. There is no effect of vasectomy on birth defects, since the sperm turnover cycle is about 10 weeks. The man simply makes new sperm. Choices C and D are incorrect. The correct choice is A.

10.

Copyright by The Berkeley Review

258

The Berkeley Review Specializing in MCAT Preparation

Biology
11.

Reproduction St Development

Section IV Answers

Oligospermia is the condition of having few sperm. (This is compared to aspermia, the condition of having no sperm). Although this man could impregnate a woman, his chances are less than that ofthe population average.
However, his chances are not zero. Choice A is incorrect. Choices C and D are incorrect. The correct choice is B.
12.

B is correct, he has reduced fertility status compared to the population average. The prefix "oligo" means few.

only interruption is in the transport of sperm outside of the body. This has no effect on the hormones controlling
sperm production. Choices A, B, and C are incorrect. The correct choice is D.
13.

Dis correct, both FSH and LH are unchanged. Since sperm production continues along its normal course, really the

B is correct, seminiferous tubules. The testis is the site of sperm production. It is compartmentalized into the seminiferous tubules and the interstitium. The interstitium contains androgen-secreting Leydig cells.
Spermatogenesis occurs in the seminiferous tubules, not the interstitium. Choice D is incorrect. The vas deferens

drains sperm from the testis. Choice A is incorrect. The prostate gland is not involved in spermatogenesis. Choice C
is incorrect. The correct choice is B.
14.

A is correct, I only. Following a vasectomy, the sperm count will eventually drop to zero, since the sperm cannot exit the testis. Choice I iscorrect. The female counterpart of vasectomy is the ligation of the fallopian tubes (a tubal ligation). Choice II is incorrect. Since sperm production continues normally, there is no reason to suspect decreased
testosterone levels. Choice III is incorrect. The correct choice is A.

15.

C is correct, the dogs undergoing this treatment would become sterile. This treatment achieves the same effect as a vasectomy. The vas deferens is blocked, so that sperm cannot exit the testis. The effects would be the same as expected for vasectomy. The dogs would eventually have a sperm count of zero and be sterile. Choice A is incorrect. Normal sperm production continues, but the cells are destroyed by phagocytes in the testis. There would be no

enlargement of the testicles. Choice B is incorrect. There is no evidence that an autoimmune disease would develop,
since the phagocytic cells are localized in their attack on the sperm cells, not the testis itself. Choice D is incorrect.
The correct choice is C.

Passage III (16 - 21)


16.

Fertilization of the Ovum

D is correct, II and III only (regression of the corpus luteum and termination of pregnancy). The corpus luteum is quite dependent on hCG during the first 8 weeks of pregnancy. During the luteal phase of the uterine cycle the corpus luteum was maintained by LH, even though the LH levels were relatively low. The corpus luteum is responsible for the secretion of large amounts of estrogen and progesterone. These two steroids, in concert, inhibit release of GnRH. The end result is that FSH and LH levels are kept low and follicular development and ovulation are suppressed. Increased levels of estrogen and progesterone lead to further endometrial development. After fertilization the outer layer of cells (trophoblast) of the blastocyst begin to secrete hCG. This peptide stimulates and the corpus luteum to continue its synthesis of estrogen and progesterone. Why? Because the developing placenta will not be able to secrete adequate amounts of estrogen or progesterone until about the 3rd month of pregnancy. High levels of these two steroids allow the endometrial lining to proliferate and implantation of the blsatocyst is maintained. In other words, pregnancy will continue. What happens if hCG is removed?
Removal of hCG during the first 6 weeks of pregnancy means that the corpus luteum is not being stimulated. The corpus luteum begins to regress and the output of estrogen and progesterone decreases. Uterine blood vessels begin to constrict, depriving cells of nutrients and oxygen. The uterine lining begins to slough. Pregnancy is terminated. In turn, GnRH, LH, and FSH levels begin to rise and the cycle begins again. The correct choice is D.

17.

A is correct, LH. We can reason out this answer based on information in the first sentence of the second paragraph of the passage. We are told that during the luteal phase of the uterine cycle LH maintains the corpus luteum. After fertilization we are told that hCG maintains the corpus luteum. We could assume that if both hormones maintain the corpus luteum, then they must have similar structures if not similar receptors. It turns out that hCG is a glycoprotein that consists of an alpha and a beta chain. The alpha chain is almost identical in sequence to the alpha chains of TSH, FSH, and LH. The beta chain, which determines biologic effect, has a 67% homology with the beta chain of LH. Consequently, hCG and LH have very similar biologic actions. The correct choice is A.

18.

C is correct, increase steadily until parturition. Although FSH and LH levels are low during pregnancy, human chorionic gonadotropin takes over the functions of LH to stimulate estrogen and progesterone secretion. In addition to the placenta, the fetal and maternal adrenal cortices may also produce estrogen and progesterone. The correct
choice is C.

Copyright by The Berkeley Review

259

The Berkeley Review Specializing in MCAT Preparation

Biology
19.

Reproduction St Development

Section IV Answers

A is correct, remain relatively low, thereby eliminating further follicle development and ovulation. LH and FSH are inhibited by high levels of estrogen and progesterone during pregnancy. Therefore, they remain relatively low throughout the duration of pregnancy. The correct choice is A. A is correct, have no effect at all on the pregnancy. The ovaries produce estrogen and progesterone during the first 6 weeks of pregnancy, after which, other tissues such as the placenta take over this role. Therefore, pregnancy can
occur in the second term without the ovaries. The correct choice is A.

20.

21.

A is correct, endometrium. Unless implantation of the blastocyst occurs, the continuing secretion of progesterone and estrogen by the corpus luteum is stops. The result is that the glandular epithelial cells that make up the endometrium begin to slough. Immediately below the endometrium is the myometrium, which is the smooth muscle tissue of the uterus. A primary oocyte will become surrounded by a layer of granulosa cells to become a primary follicle. The primary follicle will be converted to a secondary oocyte, and it is the secondary oocyte that will become the ovum. If an ovum is not fertilized, it will be removed in the monthly menstrual flow. Thus, the only structure
mentioned that is sloughed during menses is the endometrium. The correct choice is A.

Passage IV (22 - 29)


22.

Pregnancy Fuel Utilization

C is correct, II and III only. Triglycerides in the adipose tissue are hydrolyzed by the enzyme triglyceride lipase. This yields 3 fatty acids and 1 glycerol molecule per triglyceride. All the glycerol escapes to the blood. Some fatty acids may remain in the adipocyte for re-esterification, and the remainder will escape to the blood. The correct
choice is C.

23.

D is correct, insulin increases synthesis of glucose. We are looking for the FALSE choice. We have learned from the passage that the goal of early pregnancy is storage of energy in adipose tissue. If a person eats, insulin levels increase, and some of the energy from the food will be stored as fat. Insulin does increasesynthesis of triglyceridein

adipose tissue. It activates triglyceride synthase. Choice A is correct. Glucose that is taken up by the adipocyte is used to make a glycerol phosphate, an activated glycerol backbone for triglyceride synthesis. Choice B is correct.
Food lipids are packed into chylomicrons and are taken up by cells, including adipocytes. Insulin increases the activity of lipoprotein lipase. Choice C is correct. Finally, the false choice is D. The synthesis of glucose is gluconeogenesis, and that occurs in the hepatocytes. Insulin does not increase gluconeogenesis after a meal. The
correct choice is D.

24.

A is correct, increased glucose, decreased insulin. Insulin resistance means that the insulin in less effective in

producing its effects compared to normal insulin sensitivity. The effects of insulin are to lower blood glucose by activating glucose transporters and moving glucose into cells for processing. A feed-back loop is set up so that insulin will decrease as glucose decreases. Since the woman with gestational diabetes is quite insulin resistant, glucose will remain higherlonger after the test load. Insulin will also remain high because the feed-back loop to turn
it down is not completed. The correct choice is A.

25.

A is correct, pancreas. Insulin is produced by the betacells in the pancreas. The correct choice is A.

26. 27.

A is correct, I only. Glucose is the required fetal fuel source. The. mother builds fat stores during early pregnancy.
This makes choices B, C, and D incorrect. The correct choice is A.

B is correct, higher birthweight. Since the fetal preferred source of energy is glucose and glucose is higher in
women with gestational diabetes, the fetus will be able to "overeat". This condition is called macrosomy. Choice B is correct. This makes choices A and C incorrect. As for choice D, we have no way of inferring from the passage (or from our own knowledge) that during the first trimester fetal weight will be slightly above normal. Not only that, but
the second part of choice D states that the birthweight is lower when we know it should be higher. The correct
choice is B.

28.

B is correct, II only. Glucose transport is by facilitated diffusion, which is passive transport. It occurs throughout pregnancy, not just following a meal. It is lower in early pregnancy, when the fetal needs are lower, and mother is
insulin sensitive. The correct choice is B.

29.

C is correct, hepatocytes. The liver's cells are called hepatocytes. The hepatocyte is the only location of gluconeogenic enzymes listed here. There is some evidence that the kidney can also do gluconeogenesis, but that is not an issue in this question.The correct choice is C.

Copyright by The Berkeley Review

260

The Berkeley Review Specializing in MCAT Preparation

Biology
Passage V (30 - 35)
30.

Reproduction St Development

Section IV Answers

Female Reproductive Hormones

the secretion GnRH from the hypothalamus and FSH and LH from the anterior pituitary. Suppression of FSH prevents follicular growth and suppression of LH prevents ovulation. There is no mention inthe passage that
estrogen inhibits progesterone or that estrogen stimulates the synthesis of FSH. The correct choice is A.
31.

A is correct, I only (inhibition of FSH). Together, estrogen and progesterone act at the level of the brain to inhibit

Bis correct, positive feedback by estradiol. Estradiol is normally an inhibitor of LH secretion, but at increasing levels it becomes a stimulus for LH secretion. The LH peak is due to the rise in estrogen levels during the
preovulatory period. The correct choice is B.

32.
33.

C is correct, negative feedback of estradiol. Estradiol inhibits FSH secretion. The correct choice is C.
A is correct, I and II only (progesterone and estradiol). Progesterone and estradiol are the main hormones that act on the endometrium. FSH, LH, and hCG primarily act on the ovaries. The correct choice is A.

34.

A is correct, anterior pituitary. The anterior pituitary is responsible for the release of FSH, LH, growth hormone (GH), thyroid stimulating hormone (TSH), prolactin, and adrenocorticotropic hormone (ACTH). All of these

hormones are protein hormones. This is information that is important to remember. Notice that the steroids estrogen
and progesterone are not produced at the level of the brain. Right away this allows us to select choice A as the correct answer. The placenta produces both estrogen and progesterone. Granulosa cells (as well as thecal cells) can produce estrogen. Recall that the granulosa cells are those cells which immediately surround the oocyte while the thecal cells are those cells that surround the granulosa cells. Together, the granulosa and thecal cells are called the

follicle cells. The adrenal cortex helps to define the outer region of the adrenal glands. This tissue secetes a variety
of steroid hormones, including androgens (male sex hormones) and estrogens (female sex hormones). The correct
choice is A.
35.

C is correct, hypothalamus. We have learned that both FSH and LH are synthesized and released from the anterior pituitary. The release of these two hormones (and the other hormones of the anterior pituitary) are regulated by hypothalamic hypophysiotropic hormones, such as GnRH. GnRH is released from the hypothalamus once every two to three hours in a pulsatile fashion. In turn, the release of both FSH and LH from the anterior pituitary is seen in a

pulsatile fashion as well. The poterior pituitary releases vasopressin (ADH) and oxytocin. These two peptide hormones are synthesized within the hypothalamus, packaged into secretory granules, and transported to the posterior pituitary for release. The pineal gland, derived from the roof of the diencephalon and posterior to the thalamus, secretes melatonin, an amino acid derivative. The function of this hormone has not been entirely worked
out yet. The correct choice is C.

Passage VI (36 - 43)


36.

Endocrine Control of the Ovarian Cycle

C is correct, the rate-limiting enzyme in a pathway is inhibited by the product(s) of the pathway. First of all, eliminate choices B and D. Enzymes may be both activated or inhibited, turned up or down. Negative control means that the product of a pathway acts to decrease its own synthesis. This means the pathway is inhibited, not activated.
Choice A is wrong. The correct choice is C.

37.

D is correct, abdominal cavity. The ovum is released from the mature follicle directly into the abdominal cavity. It is picked up and directed into the uterine tube by thefimbriated ends of the tube. Finally it passes into the uterus and
leaves by the cervix if it is not fertilized. The correct choice is D.

38.

B is correct, progesterone. From the diagram, we can see that the hypothalamus produces GnRH, so choice A is
incorrect. We can also see that the anterior pituitary produces LH and FSH, so choices C and D are incorrect. This leaves B, progesterone, which is indeed made by the corpus luteum. The correct choice is B.

39.

C is correct, I and II only. Look at the diagram. At around Day 1, the levels of estradiol and progesterone'are falling. This decreases the negative inhibition on the hypothalamus. Then the normal cycle of follicle development and ovulation follows. Therefore, if estradiol and progesterone are kept artificially high with oral contraceptives, the negative feedback control is not removed. The brief time between cycles of pills does not allow the normal time for
all the ovulatory steps to get in sync. So there is no ovulation. Choice I is correct. There is also decreased FSH and

LH because the fall in progesterone and estradiol does not occur in pill users. Choice II is correct. Choice III is incorrect, because we have already established that normal follicle development and ovulation does not occur. The
correct choice is C.

Copyright by The Berkeley Review

261

The Berkeley Review Specializing in MCAT Preparation

Biology
40.

Reproduction St Development

Section IV Answers

D is correct, increased FSH, increased LH. This is another question regarding feedback control. During the

secretory phase, high levels of estradiol and progesterone feed back on the hypothalamus and decrease GnRH. This,
in turn, decreases FSH and LH. After menopause, levels of estradiol and progesterone are low because normal follicular development has ceased. This means FSH and LH would remain high due to lack of feedback inhibition.
This means choices A, B, and C are incorrect. The correct choice is D.

41.

A is correct, steroid hormones. The precursor of estradiol and progesterone is cholesterol, which is a steroid. This means choice A is correct. Growth hormone is a specific peptide hormone. So choice B is incorrect. Peptide hormones are made of chains of amino acids, so choice C is incorrect. Finally, glucorcgulatory hormones control plasma glucose levels. Estradiol and progesterone do not do this. The correct choice is A.

42.

A is correct, no changes in ovarian cycle, but no menstruation. Removal of the uterus alone will not affect the cycle involving the ovaries, hypothalamus, and anterior pituitary. Therefore, choices B and C are incorrect. There should be no changes in the ovarian cycle. Choice D is incorrect because there can be no menstrual flow without the uterus,
which is the source. The correct choice is A.

43.

C is correct, LH surge. Although the follicle is prepared by FSH and LH, the final stimulus for ovulation is a surge of LH about 16 hours before ovulation. Choices B and D precede ovulation, but are preparatory steps, not the trigger. Choice A is about 10days following ovulation, so it could not be a trigger. The correct choice is C.
Gestation

Passage VII (44 - 49)

44. A is correct, seminiferous tubules. Sertoli cells are located within theseminiferous tubules. Theircell body extends from the base of the tubules to the cytoplasm. In addition to producing Miillerian inhibiting factor, they also provide
nutrients to the developing sperm. The correct choice is A.

45.
46.

C is correct, human chorionic gonadotropin (hCG). The production of steroid during the early part of gestation is
dependent on hCG derived from the placenta. The correct choiceis C.

B is correct, II only (interstitial cells of Leydig). The interstitial cells of Leydig are the testosterone producing cells
located in the testes, outside of the seminiferous tubules. The correct choice is B.

47.

C is correct, Wolffian duct structures, and male external genitalia. According to the passage, testicular differentiation occurs at43-50 days of gestation and MIF has already taken effect. Therefore, the fetus will develop
male genitalia. The correct choice is C.

48.

C is correct, no change. According to experiment 1, treatment of anti-H-Y antigen to XY, newborn testes would not

differentiate into seminiferous tubules. In the passage, testicular differentiation occurs at 43-50 days of gestation.
Theoretically, anti-H-Y antigen should not have an effect at gestational day 80. The correct choice is C.

49.

D is correct, II and III only (Miillerian duct structures and female external genitalia). Similarly, there should not be an effect since treatment occurs beyond the critical period of ovarian differentiation (day 77-84 of gestation). The
correct choice is D.

Passage VIII (50 - 56)


50.

Female Birth Control Vaccine

C is correct, affinity chromatography using hCG. Affinity chromatography consists of an inert column which supports compounds that bind your molecule of interest. Since you want anti-hCG, you attach hCG to the support. This will allow binding of your antibody. LH is not the hormone of interest, so choice A is incorrect. Ion exchange chromatography is for separatingcharged molecules, so choice B is incorrect. The Bradford protein assay only gives
total protein, so choice D is incorrect. The correct choice is C.

51.

D is correct. Group 4. We can read this answer off the table. Only group 4 had antibody concentrations above the 0.52 level. Only group 4 could be considered protected against pregnancy by the anti-hCG antibody. The correct
choice is D.

52.

D is correct, increase and then decrease. Compare this to what we know about being immunized. We get a tetanus series when we are young, and then we get booster shots to elevate our antibody level as it decreases over time. With a booster, as in this question, wc seen an increase and then a decrease over time. The correct choice is D.

Copyright by The Berkeley Review

262

The Berkeley Review Specializing in MCAT Preparation

BlOlOgy
53.

Reproduction St Development

Section IV Answers

C is incorrect, the toxin has no effect on reproductive status. Choice D is incorrect, the goal is not to provide
simultaneous immunity, but contraception. The correct choice is A.
54.

immunologically robust carriers. This elicits the strongest antibody response and allows your small molecule of interest to have antibodies produced against it. Choice Bis incorrect, because the toxin does not protect LH. Choice

is called a hapten-carrier complex. Small molecules that an immunologically inert alone are bound to larger,

Ais correct, to make a larger complex for antibodies to recognize, allowing more effective antibody formation. This

Cis correct, human chorionic gonadotropin. hCG is present only during pregnancy. Estrogen and progesterone are
present throughout pregnancy and also during the normal menstrual cycle. Thiseliminates choices A and D. Human chorionic villi are partof the developing tissues accompanying the fetus. The correct choice is C.

55.

D is correct, the vaccine would not change normal menstrual cycles. The vaccine causes the body to produce
antibodies to hCG, so that any hCG produced by an embryo is trapped by the antibodies. hCG is critical for the progression of pregnancy, so the pregnancy is terminated at this stage. Choice A is incorrect, the vaccine does not

contain the hCG molecule, merely a piece of it, made synthetically. Choice B is incorrect, the peptide is chosen to avoid cross-reactivity with LH. Choice C is incorrect, the hCG levels will be reduced to nonpregnant levels, 0.000
nmol/L. The correct choice is D.

56.

C is correct, I and II only. The intended antigen is the hCG synthetic peptide. Choice I is correct. However, since the diphtheria toxin is present, a small antibody response occurs to it as well, so choice II is correct. LH is a
hormone and is not an antigen. Choice III is incorrect. The correct choice is C.

Passage IX (57 - 63)

Estrogen

57. D is correct, bind to DNA, resulting in changes in transcriptional rates. As stated in the passage, estrogen is a steroid hormone. Therefore, we must think about how steroid hormones bring about changes on a cellular level. Since steroids are very hydrophobic in nature, they can cross the cell membrane and bind to receptors within the cell'scytoplasm. These steroid/receptor complexes have the ability to bind to DNAand alter transcriptional levels of certain proteins. This is very different from protein hormones which bind to receptors found on the cell surface.
These types of interactions lead to the formation of secondary messengers that bring about change within the cell. Eliminate choice B as a possible answer. Answer choices A and C are a bit non-sensible and do not fit the general
scheme of how hormones bring about change. The correct choice is D.

58.

B is correct, inhibiting calcium entry into the muscle cell. Entrance of calcium into a muscle cell is what triggers contraction. We have seen this in skeletal muscle cell. The question asks what inhibits myofibril (there are many myofibrils in a muscle cell, and each myofibril contains many sarcomeres) contraction. Well, the only real possibility should involved the calcium ion. As for sodium and potassium, we have no evidence they are involved
with the contraction of a myofibril. They will be involved in the nerve transmission to stimulate contraction, but we cannot assume they take part in the actual contraction of the muscle. If we want to inhibit the myofibril contraction which will lead to vasodilation, we will want to inhibit calcium entry into the cell. The correct choice is B.

59.

B is correct, ovaries. This is based entirely on our knowledge of hormones. Estrogen is produced primarily by the ovaries, and its target tissue is the general female reproductive structures. Its principal actions stimulate development of secondary sex characteristics in females and growth of sex organs at puberty. It also primes the uterus for pregnancy on a monthly basis. Hormone questions are not going to be tough in you know your hormones, where they come from, and what they accomplish. The correct choice is B.

60.

D is correct, '/^ of the original resistance. If we recall from physics or discussions of the cardiovascular system,
the resistance of a tube is inversely proportional to the radius of the tube to the fourth power. If we increase the diameter by a factor of two, we are increasing the radius by a factor of two. We are decreasing the resistance by 2 to the fourth power. Thus, the resistance is one sixteenth of what it was previously. The correct choice is D.

61.

C is correct, estrogen directly acts on vascular smooth muscle to cause relaxation. We know from the passage that the endothelial cells produce this EDRF which causes relaxation of the smooth muscle, and ultimately leads to vessel dilation. Since this is stated in the passage, wc have to take it as the truth and thus we can eliminate choices A and B. If one removes the endothelial layer, one is removing the ability to produce EDRF. If estrogen still acts as a dilator in this condition (no EDRF because no endothelial layer), then estrogen must have the ability to directly act on the smooth muscle to cause its relaxation and ultimately vessel dilation. Considering choice D, we run into a familiar problem. Estrogen cannot stimulate the production of something that cannot be produced without the presence of endothelial cells. Since these have been removed, estrogen must act directly on smooth muscle. The
correct choice is C.

Copyright by The Berkeley Review

263

The Berkeley Review Specializing in MCAT Preparation

Biology
62.

Reproduction St Development

Section IV Answers

B is correct, II only (introduction of estrogen leads to decreased catabolism of LDL-cholesterol.). Statement I is consistent with the idea the estrogen benefits cardiac tissue because less resistance to blood flow allows blood to flow easier, allowing your heart to work easier (assuming our heart was working harder than it should). In other words, more resistance to blood flow would force our heart to work harder, placing stress on the vital organ. Statement III is consistent with this idea because inhibiting the endothelial cell expression of adhesion molecules inhibits platelet aggregation and other adhesion that is normally seen in the early stages of atherosclerosis. This idea is that if less "stuff' can stick to the walls of the vessel, the less chance exists of blocking blood flow
Statement II is inconsistent because the statement claims a decreased breakdown, or catabolism of LDL cholesterol

is beneficial to the vascular system. An LDL particle is a low density lipoprotein which transports cholesterol from the liver to the body cells via the blood. The cells take up these particles through receptor mediated endocytosis. However, high levels of cholesterol will inhibit LDL receptor synthesis, and the particles will remain in the circulatory system. The build up of cholesterol in the blood is thought to be a prime cause of atherosclerosis, a condition where the arteries are no longer compliant and blood moves though with difficulty. This condition is
clearly not beneficial to the vascular system, making statement II inconsistent. The correct choice is B.

63.

C is correct, inhibits release of endothelin. Throughout the entire passage, we have seen how estrogen benefits the cardiovascular system. It is stated in the passage that endothelin is a potent vasoconstrictor. In keeping with the theme of the passage, it is most likely that estrogen will act to block the activity of endothelin. Answer choices A and B call for estrogen to stimulate this potent vasoconstrictor. This is no evidence for such an interaction, and choices A and B can be eliminated. Choice D wants us to believe that estrogen will not affect endothelin. Again,

this is not keeping with the theme that estrogen benefits cardiac tissue. Therefore, estrogen will most likely inhibit
the release of endothelin. This has been demonstrated in rabbit coronary arteries. The correct choice is C.

Passage X (64 - 69)


64.

Isoflavone Experiment

C is correct, an antagonist mimics a hormone, interacts with the hormone's receptor, and leads to a block of the

intracellular effects of the hormone. Choices B and D are incorrect, because they refer to agonists, not antagonists.
Choice B is the correct definition of an agonist. An antagonist acts against the action of a hormone and blocks its
effects. Choice A is incorrect. The correct choice is C.
65.

A is correct, the soy diet group has suppressed levels of FSH and LH compared to the controls. Examination of

Figure 1 indicates that both FSH and LH levels are decreased (suppressed) in the soy diet trial compared to the
control trial. The correct choice is A.
66.

D is correct, the subject would notovulate. Ovulation is triggered by a precise sequence of hormonal events. LH is

the direct stimulus for ovulation. The subject would not ovulate if LH were somehow suppressed (similar to the suppression of LH by the birth control pill). This isa theoretical question only. The amount of soy protein may never completely hall LH production. The focus of this question is more on ovulation than on soy protein. If the subject did not ovulate, she could not become pregnant. Choice Ais incorrect. Since she did not ovulate, there isa change in
the normal menstrual cycle. Choice B is incorrect. One cannot predict when menstruation would occur due to the
lack of influence of the corpus luteum. Choice C is incorrect. The correct choice is D.
67.

C is correct, exposure to estrogen is a probable determinant of developing breast cancer. From the passage, we learned that women who moved tocountries other than their native countries got breast cancer at rates comparable to
natives of the adopted country. This implies an environmental cause rather than a genetic cause. Choice A is incorrect. The fact of importance in this passage is women in different countries do not contract breast cancer at the same rate. Choice B is incorrect. Diet can modify estrogen levels, as seen in the experiment. Choice D is incorrect.
The correct choice is C.

68.

D is correct, 61%. This is taken directly from a tiny facet in the passage. The diet provided 98 grams of protein, of
which 60 grams was the soy protein. 60/98 = 61%. The correct choice is D.

69.

B is correct, the soy diet was hypocholesterolemic. This question tests our knowledge of terminology. Hyper means increased. Hypo means decreased. Iso means the same or unchanged. Choiesterolemic means level of cholesterol in the blood. From the data table, we can see that cholesterol levels in the blood were lowered by the soy-protein diet. The correct choice is hypocholcstcrcmic. Choices A and C are incorrect. Choice D is a nonsense answer and is
incorrect. The correct choice is B.

Copyright by The Berkeley Review

264

The Berkeley Review Specializing in MCAT Preparation

Biology
Passage XI (70 - 76)
70.

Reproduction St Development

Section IV Answers

Ovulation

Ais correct, corpus luteum. The corpus luteum secretes estrogen and progesterone after the egg is released from the follicle. The egg itself does not secrete hormones. Choice Bis incorrect. The corpus is a dying corpus luteum which
is no longer secreting hormones. Choice C is incorrect. The cervix secretes mucus, but not hormones. Choice D is
incorrect. The correct choice is A.

71.

D is correct, all of the above. This answer goes back to our understanding of basal metabolic rate. A basal

measurement is made at rest, before getting out of bed after a night's sleep, after all your food is digested, while
lying quietly. The correct choice is D.
72.

B is correct, I and III only. Look at Figure 1. We can see that LH and estrogen rise dramatically right before ovulation. Choice I and III are correct. However, progesterone is low before ovulation and rises dramatically
afterwards. Choice II is incorrect. The correct choice is B.

73.

C is correct, birth control pills. Both condoms and diaphragms are barrier methods of contraception. RU-486 prevents continued implantation of a fertilized egg. The levels of estrogen and progesterone in birth control pills
inhibit ovulation, so no egg is produced. The correct choice is C.

74.

D is correct, she is ovulating. Ferning is an indicator of ovulation. The woman is therefore not pregnant. Choice A is incorrect. She is not anovulatory (not ovulating). Choice B is incorrect. She is ovulating and therefore fertile.
Choice C is incorrect. The correct choice is D.

75.

A is correct, I only. Although FSHand estrogen play critical roles in preparing the follicle and positive feedback on
the hypothalamus, respectively, LH is the actual trigger for release of the egg from the follicle. The correct choice
is A.

76.

B is correct, cholesterol. The ring structure of cholesterol is the base molecule for the synthesis of sex hormones in men and women. Arachidonic acid is a long-chain fatty acid, a precursor of the class of molecules called prostaglandins. Phosphatidyl choline is a phospholipid, used in membranes. Sphingomyelin is a complex lipid found
in the brain. The correct choice is B.

Passage XII (77 - 83)


77.

Vertebrate Gastrulation

B is correct, the heart. The question is asking what structure would be affected if the embryonic mesoderm were defective. This is really a matter of memorizing which structures are derived from each of the germ layers (ectoderm, mesoderm, endoderm). Choice, A, the nervous system, is derived from ectodermal precursors, as is the skin and the lens of the eye. Choice C, the stomach lining, is derived from the endodermal germ layer, which also gives rise to other linings of the digestive and respiratory tracts, as well as differentiating into the major glands of the body such as the pancreas and the liver (choice D). This leaves choice B, the heart, as the structure which is mesodermal in origin (along with the notochord, skeleton, muscle, outer coverings of internal organs, and reproductive organs). The correct choice is B.

78.

D is correct, a transmembrane protein. From the passage, we learn that a morphogen is defined as a diffusible
"substance." Answer choices A, B, and C could all fit this broad definition. Answer choice D, however, is far from a

diffusible substance. Rather, a transmembrane protein is anchored into the plasma membrane and cannot either be secreted or diffuse anywhere except within the confines of the membrane. Such a confined protein would not be very effective at signalling distant cells. The correct choice is D.

79.

C is correct, I and II only. The question inquires about the processes which could bring about the extension, or the increase in length, of a sheet of cells. Statement I, cell shape changes, is a correct possibility. If individual cells within the cell sheet were to change their shape such that they were narrower and longer, the entire sheet would get

narrower and longer, resulting in extension. Statement II is also a valid possibility. Mitotic division of cells within
the sheet would cause an increase in the total number of cells in the sheet. If these cell divisions were in strategic

locations, the entire cell sheet would become longer and extension would occur. Statement III, on the other hand, is

not a valid explanation for sheet elongation. The unidirectional migration of every cell in the sheet would simply result in the entire sheet of cells moving somewhere synchronously. This would not result in the elongation and extension described in the question. The correct choice is C. 80. A is correct, epithelial cells. The question gives us some clues about the type of cells on the surface of the embryo. We learn that they have tight junctions, which are cell-cell adhesions which prevent molecules from slipping in
265

Copyright by The Berkeley Review

The Berkeley Review Specializing in MCAT Preparation

Biology

Reproduction St Development

Section IV Answers

between these cells, as well helping to establish an apical/basolateral polarity. We also learn that they serve a role of protecting inner embryonic cells from the external environment. From these clues alone, the most likely candidate would be answer choice A, epithelial cells. Epithelial cells can have tight junctions, often to aid them in their major role of protecting underlying tissues from arbitrary inward diffusion of lumenal, or gut, contents. We can also approach this problem via a process of elimination. Neuronal, smooth muscle, and connective tissue cells are all highly differentiated cell types. This question inquires about an embryo which has yet to undergo gastrulation. It is very unlikely that these advanced cell types would have differentiated at this stage. The correct choice is A.
81.

B is correct, primate embryos should not have gill slits. The question asks which statement is NOT supported by the observation that human embryos have gill slits and a tail at some time during their development. This observation does suggest that humans and the other vertebrates share a common ancestry (choice C) as well as supporting the theory that humans evolved from lower vertebrates (choice A). The fact that human embryos pass through stages which resemble the embryonic stages of lower vertebrates (fish and tailed mammals in particular) suggests that humans might have evolved from these species. Since these stages of development have not been drastically altered

from fish to human, it could be said that there is little selective advantage in altering these stages of embryogenesis.
This supports choice D. Since we are looking for the unsupported statement, by a process of elimination we are left

with choice B. Since primates are evolutionarily close to humans, we would expect that they would display similar embryonic stages. Therefore, they should have gill slits at some embryonic stage. Incidentally, the conservation of embryonic stages is often referred to as "ontogeny recapitulating phylogeny," that is, early developmental stages
reiterate the developmental stages of other species in the same phylogenctic tree. The correct choice is B.
82.

C is correct, a concentration gradient of morphogen could never be formed. First, let's think about what normally occurs. If the morphogen isproduced at a point source (i.e., the dorsal lip cells), it is degraded as it diffuses away. The result is a concentration gradient of morphogen with a maximum near the source, because morphogen molecules that get further away are more likely to be degraded because they have been present longer. If the morphogen were not slowly degraded, as the question postulates, what would happen? The gradient would never form (making choice B incorrect). This is because the morphogen would diffuse away from the dorsal lip cells and equalize in concentration throughout the embryo. This would affect gastrulation because proper signalling depends on the establishment ofa gradient ofmorphogen to which cells at different distances could respond to differently; therefore

choice A is incorrect. Choice D is likewise invalid because there would be no gradient, especially not a very steep
one. The correct choice is C.

83.

C is correct, the embryo would develop into two Siamese twin tadpoles, the result oftwo independent invaginations during gastrulation. This question is referring to Hilde Mangold's famous dorsal lip transplantation experiments. First of all, we must keep in mind that the function of the dorsal lip cells is to induce the invagination event that

introducing an additional dorsal lip would induce a separate invagination event, in essence causing the embryo to develop into two attached organisms (Siamese twins). Answer choice A is incorrect because not only would gastrulation occur, it would occur in two separate places. Answer choice D is incorrect because two dorsal lips in one embryo would lead to two invaginations. Answer choice B is a tempting one, but remember that an embryo at
this early stage doesn't have a differentiated immune system yet. The correct choice is C.

begins gastrulation. This is accomplished via the secretion of a morphogen gradient. From this, it follows that

Passage XIII (84 - 90) 84. 85.

Oxytocin St Labor

B is correct, hypothalamus. Oxytocin is produced within the hypothalamus by neurons and transported to the
posterior pituitary, where it is secreted. Choice A is incorrect. Choice C is incorrect. The placenta itselfdoes not
produce oxytocin, but the fetus docs. Choice D is incorrect. The correct choice is B.

B is correct, I and II only. If the uterus isresponding more strongly to normal, nonpregnant levels of oxytocin, it is
reasonable to expect that the number of oxytocin receptors has increased. This would mean that a small dose of

oxytocin would be very effective at stimulating uterine contractions. Choice I is correct. If oxytocin levels did not increase in the blood, there is the possibility that some sort of paracrine transfer is occurring. This means that hormone is secreted from one cell to another without actually moving through the bloodstream. Choice II is correct. Usually when things are conjugated by the liver, they are being processed for excretion. If oxytocin were increasingly conjugated, then there would be less present, leading to fewer contractions. Choice III is incorrect.
Choice B is the correct choice. The correct choice is B.

86.

C is correct, intravenous injection of human chorionic gonadotropin. You are looking for the FALSE answer. Injection of oxytocin (intravenously) would increase the circulating levels of oxytocin. This would increase
266

Copyright by The Berkeley Review

The Berkeley Review

Specializing in MCAT Preparation

BlOlOgy

Reproduction St Development

Section IV Answers

contractions. Choice Ais true. Intravaginal administration ofprostaglandins would increase contractions by acting
on the uterus, just as if the prostaglandins were produced by the endometrium. (This is the idea behind oral or

vaginal prostaglandin administration to allow abortion.) Choice B is true. Prostaglandins also work orally (see
above). Choice D is true. Human chorionic gonadotropin (hCG) is produced during early pregnancy to make sure
estrogen and progesterone levels remain high. Although it tapers off after about 10 weeks, it alone would cause no
uterine contractions. Choice C is FALSE. The correct choice is C.

87.

C is correct, administration of inhibitors of prostaglandin production. Both oxytocin and prostaglandins must be present for labor to occur. By eliminating one or the other from the equation, one can avoid an early labor in which the fetus is premature. A secretagogue encourages the secretion of something. This would not be favorable in a

person in whom labor threatens. Choice Ais incorrect. An oxytocin agonist would act on the oxytocin receptors just
as oxytocin would. Another unfavorable choice. Choice D is incorrect. RU-486 is used to promote abortion. Choice B is incorrect. Finally, inhibitors of prostaglandin production, such as indomethacin, can be used to prevent the
positive interactions between prostaglandins and oxytocin. Choice C is correct. The correct choice is C.

88.

D is correct, arachidonic acid. This is a bit of trivia, and you will either know it or not. Anyway, stearic is 18:0, myristic is 14:0, and linolenic is 18:2. Only arachidonic has the required 20 carbons (20:4). It is the precursor
prostaglandins. Choices A, B, and C are incorrect. Choice D is correct. The correct choice is D.

89.

B is correct, I and III only. Peptide hormones do not cross the lipid bilayer directly. They usually interact with cellular membrane receptors and signals are transferred to the intracellular medium by second messengers. Choices I and III are correct. If the hormone can't enter the cell, then it cannot stimulate mRNA synthesis directly. Choice 11 is
incorrect. The correct choice is B.

90.

A is correct, delivery can occur without stretch signals from cervix. A paraplegic woman may not be able to voluntarily contract her abdominal muscles to assist in labor. This is not a requirement. Choice B is incorrect. The whole passage discusses the importance of the interactions between oxytocin and prostaglandins. They are required.
Choice C is incorrect. The smooth muscle of the uterus contracts without voluntary stimulation. Choice D is

incorrect. What is missing in these women is the positivefeedback (through afferent nerves) from cervical stretching
that increases oxytocin secretion. Choice A is correct. The correct choice is A.

Passage XIV (91 - 95)


91.

Puberty

B is correct. Gonadotropin releasing hormone (GnRH). Estrogen is under primarily under the control of leutenizing hormone (LH) derived from the anterior pituitary. The production and release of LH, in turn, is under the influence of gonadotropin releasing hormone (GnRH). Therefore, estrogen production and secretion is indirectly stimulated through the indirect effect of GnRH. The correct choice is B.
B is correct, testosterone synthesis. The seminiferous tubules are located in the male testes. These tubules are the site of sperm production, or spermatogenesis. The important anatomical features of each seminiferous tubule are the Leydig cells, the basement membrane, and the Sertoli cells. The Leydig cells are found between the seminiferous tubules, in the interstitial space. These cells respond to luteinizing hormone (LH) and it helps regulate the conversion of cholesterol to testosterone. Therefore, testosterone synthesis does NOT occur within the seminiferous tubules. Once synthesized, testosterone can cross the basement membrane and influence the Sertoli cells. The Sertoli cells secrete androgen-binding protein which binds testosterone (an androgen). This helps to concentrate

92.

testosterone within the seminiferous tubules. The Sertoli cells also respond to FSH, which helps to control
spermatogenesis. The correct choice is B.
93. D is correct, I, II, and III. The seminiferous tubules are under the influence of LH, FSH, and GnRH. GnRH, derived

from the hypothalamus, stimulates the secretion of LH and FSH from the anterior pituitary. LH stimulates the interstitial cells of Leydig to produce testosterone. FSH and testosterone, in turn, stimulate the development of the
seminiferous tubules and spermatogenesis. The correct choice is I). 94. B is correct, LH. As mentioned in the previous answers, LH binds to receptors on the interstitial cells of Leydig and stimulates the synthesis of testosterone from cholesterol. The correct choice is B.

95.

D is correct, Insulin-like growth factor I (IGF-I). The clue to answering this question comes from the question itself. We are looking for the secretion of a hormone ovfactor thai can indirectly act on muscle and bone tissue. The word

factor might clue us into what we are looking for. Insulin is secreted from the (3-cells of the islets of Langerhans in the pancreas. Insulin has a direct effect on the cellular uptake of glucose, fatty acids, and amino acids, and their
Copyright by The Berkeley Review 267 The Berkeley Review Specializing in MCAT Preparation

Biology

Reproduction St Development

Section IV Answers

subsequent conversion into glycogen, triglycerides, and protein. Thyroxine (or T4, tetraiodothyronine) is a hormone that has a global effect on basal metabolic rate and is critical for normal growth. Thyroxine directly influences the secretion of growth hormone and helps it to promote protein synthesis and bone growth. Epidermal growth factor acts at the level of skin cells and promotes their growth. Insulin-like growth factor I (IGF-I), also called a

somatomedian, is similar to insulin in its structure. IGF-I is synthesized in the liver and acts as a mediator for growth hormone. In other words, GH stimulates the production of IGF-I which in turn promotes things like protein
synthesis, cellular division, and overall growth. The correct choice is D.

Passage XV (96- 100)

Testicular Cancer

96.

D is correct, tumor cells do not have a haploid chromosome number. We are looking for a statement that supports the idea that germ cell tumors arise from spermatogonia. In the process of spermatogenesis, spermatogonia will give
rise to primary spermatocytes. In this process, the genetic complement will be reduced. In other words, spermatogonia have 46 chromosomes while primary spermatocytes have 23. Therefore, if tumor cells did have their

origin in spermatogonia, they should be diploid cells. The fact that tumor cells do not have a haploid chromosome
number supports this theory. The correct choice is D.

97.

B is correct, at least 90% of testicular cancers are found in the seminiferous tubules. The passage tells us that only
5% of testicular cancers do not originate from germ cells. That means that about 95% do originate from germ cells.
Where are germ cells located? They are located in the seminiferous tubules of the teste. The correct choice is B.

98.

D is correct, infertile men run a greater risk for CIS. This answer could be arrived at by a process of elimination. Cryptorchidism is a risk factor. From the passage, we know that this condition arises from a developmental error.

This supports our theory. The fact that tumors have been shown to develop in gestation supports the theory that
germ cells tumors have their origins in fetal germ cells and have a misguided development. Furthermore, the

evidence that cancer genes act in fetal germ cells supports the theory that tumors come from fetal germ cells and have a disrupted developmental fate. The only statement which does not support the theory is that infertile men run a
greater risk for CIS. This really says nothing about misguided development and its connection to testicular cancer.
The correct choice is D.
99.

Dis correct, 92. This question is very straightforward. The number ofchromosomes in a human haploid cell is 23.
We are looking for a human cell that has a tetraploid number of chromosomes. Therefore, 23 X 4 = 92
chromosomes. The correct choice is D.

100.

C is correct, testicular cancer is rare among twins. The passage informs us that testicular cancer has little basis in genetics. We are looking for a statement that provides evidencefor this notion. The fact that testicular cancer is rare among identical twins is such a statement. If one twin developed testicular cancer, and the cancer had its basis in

genetics, it is likely the other twin develops the cancer. Since testicular cancer is rare among twins, this statements
supports the idea that testicular cancer has little basis in genetics. The correct choice is C.

Copyright by The Berkeley Review

268

The Berkeley Review

Specializing in MCAT Preparation

Biology
Section V
Endocrinology
and
A. The Endocrine System
1. Catecholamine Hormones

2. 3. 4.

Peptide Hormones Steroid and Thyroid Hormones Regulatory Mechanisms

Immunology

B.

5. The Pituitary Gland The Immune System


1. 2. 3.
4.

Cell Types Antibody Structure Antibody Action


T Cells

5.

Humoral and Cellular Immunity

Practice Passages and Answers

Berkeley XJ R-E-V-KE-W
Specializing in MCAT Preparation

Endocrinology and Immunology


Top 10 Section Goals
amiliar with catecholamine hormones and their actions.

Catecholamines like dopamine, norepinephrine, and epinephrine all contain a catecholring (abenzene Jjr Cate<

ring with two adjacent nydroxyl groups). They act at thelevel of the CNS and PNS.

Ql^ Be familiar with peptide hormones and their actions. j5r Peptide hormones contain two or more amino acids linked together by a peptide bond. These
* hormones are secreted by cells throughout the body and have a variety or actions.

>im Be familiar the actions of the G protein and adenylate cyclase.


*

__

jBr Understand what happens when ahormone like epinephrine binds to a cell surface receptor. Be
familiar with the action of the G protein and conversion of ATP to cAMP by adenylate cyclase.

Q'fm Understand in a general way how phosphorylation events activate proteins.

jSr Aclassic example involves the phosphorylation of glycogen phosphorylase by protein kinase A.
* Phosphorylated glycogen phosphorylase is activatedand canconvert glycogen intoglucose.

&* Be familiar with steroid hormones and their actions.

jjjr The common precursor of all steroid hormones is cholesterol. Steroid hormones are not stored after
I their synthesis, butinstead are used immediately. Know the major groups ofsteroid hormones.

@#H Be familiar with the mechanisms bywhich homeostasis is maintained.


jgr Understand the concept ofpositive and negative feedback. Understand what ismeant byendocrine,
neuroendocrine, paracrine, and autocrine regulation.

* synthesized and stored. Know why and how they are released into the circulatory system.
* ofleukocytes areneutrophils, eosinophils, basophils, monocytes, and lymphocytes.

Know the peptide hormones of the pituitary.

It is important to know where the 8 major hormones of the anterior and posterior pituitary are

@ijk Understand how a plnripotent stem cell gives rise to the cells ofthe immune system.
jar White blood cells, orleukocytes, are involved inthe body's immune response. The 5 different types

() <&* Understand the basic structure and function of an antibody. Jr Know the5 classes ofantibodies (IgA, IgD, IgE, IgG, andIgM) andwhat they do. Have a general
* grasp of their anatomy.

(g)<^ Be familiar with the differences between humoral and cellular immunity.
* Bcells, the different interferons, and howtheyeach interact with foreign antigens.

jar Understand the workings ofmacrophages, MHC receptors, the different interleukins, theTcells and

Biology

Endocrinology & Immunology

Catecholamine Hormones

Endocrinology
Catecholamine Hormones iis&yOne way that cells can communicate with one another over long distances is through extracellular products called hormones. Hormones can generally be divided into peptide hormones (e.g., insulin), amine hormones (e.g., epinephrine or adrenaline which are classified as catecholamines), and steroids (e.g., the male and female sex hormones). Hormones are released by endocrine organs into the blood and travel by way of the circulatory system to various target tissues. When a particular endocrine cell is stimulated to release a hormone into the
blood the concentration of that hormone increases. Once the stimulation is

terminated the hormone is no longer released the concentration falls back to some normal resting level. Hormones generally have a very short lifetime in the blood. Even though the time spent in the blood may be relatively short, hormones can act within seconds or they can take hours and quite possibly days to act on their target tissue. By comparison the action of the nervous system is
much faster.

When a hormone acts on a target cell it can either do so at the level of the cell's membrane by binding to a specific receptor in that membrane or by passing through the membrane itselfand binding with a specific targetproteinwithin the cell's cytoplasm. After the hormone has bound to its specific receptor that

complex undergoes a conformational change that allows for the synthesis of an intracellular messenger. This second messenger passesthe information mediated by the hormone (i.e., the first messenger) to some specific reaction within the cell. For example, the second messenger might convey instructions that allow a particular set of reactions to release glucose into the blood or it might even act at
the level of the gene to turn off or turn on gene expression.
Let's consider the action of the catecholamine epinephrine (adrenaline) on a

typical hepatic (liver) cell. When the body is under some type of stress like physical exercise or even fright, an increased need for glucose arises. Glucose is
stored in the body in the form of glycogen and through a series of reactions can
be mobilized and used as a source of energy during these times of stress.

Once a stress has been perceived the nervous system responds by signaling the adrenal medulla (part of the adrenal gland which sits on top of the kidneys) to release epinephrine into the extracellular fluid. Epinephrine diffuses into the blood and is carried to the hepatic cells where it binds to a specific cellmembrane

receptor called a ^-adrenergic receptor. This action causes the activation of the enzyme adenylate cyclase (bound on the cytoplasmic membrane surface) which
increases the concentration of the second messenger cyclic adenosine

monophosphate (cAMP)within the cell.

Thebinding of epinephrine, a water solublehormone, to the cellsurfacereceptor and the synthesis of cAMP within the cytosol is coupled through the action of a G protein. The name of this protein stems from the fact that it binds guanosine

triphosphate (GTP) and GDP. GTP and GDP arebothnucleotides justlike ATP
and ADP.

,Initially, before epinephrine binds to its receptor, GDP is bound to a particular subunit of the G protein. However, once epinephrine binds to its target receptor
Copyright by The Berkeley Review
271

The Berkeley Review

Specializing in MCAT Preparation

Biology

Endocrinology & Immunology

Catecholamine Hormones

this hormone-receptor complex interacts with the G protein and stimulates the exchange of GDP for GTP. The G protein-GTP complex diffuses through the membrane and activates the membrane bound adenylate cyclase enzyme. In turn, adenylate cyclase catalyzes the conversion of ATP to cAMP. These events are shown in the sequences (a) through (d) in Figure 5-1.
Hormone

(Epinephrine)

Extracellular Space
Membrane of Liver Cell

Cytosol

Extracellular Space
Membrane of Liver Cell .

Cytosol

Extracellular Space
Membrane of Liver Cell

Cytosol
Active

Extracellular Space

(d)

Membrane of Liver Cell

Cytosol

ATP
Figure 5-1
Action of adenylate cyclase.

cAMP + PPj

After a brief period of time the G protein hydrolyzes the bound GTP to GDP and Pi (not shown in Figure 5-1) thus returning the G protein back to its inactive state (Figure 5-la). However, during this period of activation (Figure 5-ld) the G protein-GTP complex was able to activate many adenylate cyclase enzymes thus forming many molecules of cAMP. In other words, by the time G protein is
Copyright by The Berkeley Review
272

The Berkeley Review Specializing in MCAT Preparation

Biology
many times.

Endocrinology & Immunology

Catecholamine Hormones

inactivated the hormonal signal induced by epinephrine has been amplified


Since cAMP is synthesized within the cytosol of the liver cell it can diffuse within that cytoplasm and interacts with a variety of molecules. One of the molecules

that cAMP interacts with is a particular type of protein kinase called protein kinase A (abbreviated as PKA). A protein kinase is simply an enzyme that transfers the end phosphate (the gamma phosphate) of ATP to a specific amino acid residue of a substrate protein. cAMP will bind to PKA and stimulate it to phosphorylate (add a phosphate group) an enzyme called glycogen phosphorylase. Once glycogen phosphorylase has become phosphorylated it is now an active enzyme and will catalyze the conversion of glycogen into glucose. This is shown in Figure 5-2. Glucose is released into the blood and travels throughout the body by way of the circulatory system. [Recall that glycogen is simply the storage form of glucose in mammals. Also, there are other reactions involved in this amplification process. Only the important ones have been shown
here.]

(cAMP)PKA
Inactive

Glycogen

I phosphorylase J

H ~ I phosphorylasey Active P( hGITger 1


Glycogen c

^ Glucose

Figure 5-2 Activation by phosphorylation.

Each enzyme in the cascade of reactions shown in Figure 5-2 promotes the activation of many more molecules in the next step in the sequence. This is exactly what we saw when epinephrine bound to its receptor. This rapid amplification process allows thebinding ofjusta few epinephrine molecules to release grams ofglucose into the blood. [In Figure 5-1 andFigure 5-2 thesites of amplification are indicated by asterisks (*).]
Cholera is an intestinal disorder caused by a bacterium (Vibrio cholerae). The

major symptom of this disorder is diarrhea and if left untreated will result in severe dehydration and eventual death. This toxin binds to the active state of the G protein and prevents the GTP from being hydrolyzed to GDP and Pf. This means that the adenylate cyclase enzyme is continually active and massive
amounts of cAMP are synthesized. cAMP causes the intestinal cells to secrete
(digestive) fluids.

Copyright by The Berkeley Review

273

The Berkeley Review

Specializing in MCAT Preparation

Biology

Endocrinology & Immunology

Peptide Hormones

Peptide Hormones

-;r <:..

The water soluble peptide hormone gastrin stimulates the secretion of HCl and pepsinogen from the stomach in response to stimulation from the vagus nerve and partially digested protein.

Gastrin(thefirst messenger) binds to a specific cellsurfacereceptor in the plasma membrane and activates a particular G protein (differentthan the one in Figure 5-3). Theactivated G proteininteracts with the membrane enzymephospholipase C (abbreviated as PLC) and induces that enzyme to hydrolyze phosphatidylinositol-4,5-bisphosphate (PIP2) to inositol-l,4,5-triphosphate (IP3) and 1,2diacylglycerol (DAG).
Hormone

(Gastrin) N
Extracellular Space

(a) :

(b)

(c)

Protein

Protein-P

(inactive)

(active)

HCl secretion
into the lumen of the stomach

Figure 5-3 Activation of a receptor by a peptide hormone.

Next, the second messenger, IP3, which was released into the cytoplasm,
interacts with the endoplasmic reticulum and stimulates the release of Ca2 into
Copyright by The Berkeley Review
274

The Berkeley Review

Specializing in MCAT Preparation

Biology

Endocrinology St Immunology

Peptide Hormones

the cytoplasm by some unknown mechanism. Meanwhile DAG, diffusing through the plasma membrane, interacts with a molecule called protein kinase C

and stimulates that kinase, with the help of Ca2 just released from the
endoplasmic reticulum, to phosphorylate an unknown protein which in turn
causes HCl secretion into the lumen of the stomach. These mechanistic events are

outlined in (a) through (c) in Figure 5-3. Insulin is a water soluble peptide hormone that binds to a specific trans membrane receptor in the cell membranes of liver, fat, and muscle cells. Once insulin binds to the receptor on the cell surface the cytoplasmic portion of the receptor is converted into a tyrosine kinase that autophosphorylates the amino acid tyrosine found within the cytoplasmic portion of the receptor. This acts to further enhance the activity of the tyrosine kinase. Presumably the insulin receptor can also internalize and (somehow) act as a second messenger. This action, along with enhanced tyrosine kinase activity, leads to the internalization of glucose into these cells (see Figure 5-4). The actual events in the insulin signaling mechanism that leads to the uptake of glucoseare somewhat obscure at the present time. [Recall that if glucose is in abundance (after a meal), it can be stored in the form of glycogen.]
Hormone

(Insulin) \
Extracellular Space
Receptor

Plasma Membrane

Cytosol
Tyrosine kinase activity

Phosphorylation activity
Figure 5-4
Insulin signaling mechanism.

Glucose uptake

Copyright by The Berkeley Review

275

The Berkeley Review

Specializing in MCAT Preparation

Biology

Endocrinology St Immunology

Steroid & Thyroid Hormones

Steroid & Thyroid Hormones


These are also lipid soluble hormones which can pass through the cell's plasma
membrane and interact with a receptor either in the cytosol or in the nucleus. Thyroid hormones, secreted from the thyroid gland, can diffuse across the

plasma membrane and into the nucleus where they bind with specific receptors.
The hormone receptor complex then activates transcription essential for certain

metabolic processes. Thyroid hormones help to regulate growth and differentiation and they can stimulate the breakdown of proteins, fats, and
glucose.

Steroid hormones all originate from a single precursor molecule, cholesterol. Two important classes of steroid hormones are the androgens (male sex hormones )

and estrogens (female sex hormones). Estrogen, which is lipid soluble, can diffuse across the membrane of a target cell and bind with a specific receptor with the cytoplasm. Thissteroid-receptor complex can then enter into the nucleus where it influences the transcription of mRNA forcertain protein products. The
general scheme for this action is shown in Figure 5-5.
Hormone-Receptor Complex

Hormone mRNA

Proteins
-C

S
o

Nucleus

Figure 5-5
General action of a steroid hormone.

Copyright by The Berkeley Review

276

The Berkeley Review

Specializing in MCAT Preparation

Biology

Endocrinology St Immunology
f,'- ihiii^' f'A

Regulatory Mechanisms

General Mechanisms

How is homeostasis maintained in the body? Homeostasis is simply the


maintenance of stability within the body. One way that homeostasis is maintained is by negative feedback. In this case there is some controlled variable which is sensed by a particular sensor in the body. Through a variety of mechanisms these sensors send input to control centers which in turn regulate the controlled variable (see Figure 5-6). For example, if a certain product was being produced in a large quantity before negative feedback was initiated, then after initiation of negative feedback a smaller quantity would be produced. Negative feedback reverses the output from effectors such as glands and
muscles.

Controlled Variable

j/ Sensor I
Negative Feedback

j/

Mechanism

(-)f
Figure 5-6
Negative feedback.

Endocrine Regulation
In the endocrine system we might have a gland secreting a hormone into the blood. This hormone, as we have seen, can influence other cells in the body. For example, let's consider the regulation of blood glucose levels. Why are the blood levels of glucose so important? If glucose supplies to the brain are interrupted for an extended period of time, brain damage can result. Glucose turns out to be the only fuel utilized by the brain. [That is, except during extended fasting in which case the brain can use compounds called ketone bodies.] Thus, the controlled variable in this regulatory mechanism is the concentration of blood glucose.

B Cells c=> Insulin c


Blood glucose
increases

Bl00d% Liver, Fat, &


Muscle cells

-S Uptake of:
Glucose

(-)

[Glucose]
()
Blood glucose
decreases

Bloodw

A cells

^ Glucagon

Liver & Fat cells

Release of:

^ Glucose
Fatty Acids

Figure 5-7 Effects of glucose on the p-cells and a-cells of the pancreas.

The pancreas secretes two hormones that are important in maintaining the proper levels of blood glucose. Both of these hormones are secreted from clusters
Copyright by The Berkeley Review
277

The Berkeley Review Specializing in MCAT Preparation

Biology

Endocrinology & Immunology

Regulatory Mechanisms

of specialized cells called the islets of Langerhans. Insulin is secreted by the (5 cells (or B cells) while glucagon is secreted by the a cells (or A cells). After a meal the levels of blood glucose begin to increase. This increase stimulates the P cells to release insulin into the blood. Insulin binds to specific receptors on liver, fat, and muscle cells and through a complicated mechanism promotes the uptake of glucose into these cells. This action tends to lower (by negative feedback) the blood glucose levels. See Figure 5-7. Conversely, if the levels of blood glucose levels begin to decrease below some normal value, then the a cells are stimulated to secrete glucagon. This hormone circulates in the blood and binds to specific receptors on liver and fat cells. Glucagon not only stimulates the liver cells to degrade glycogen to glucose but it also stimulates the fat cells to release fatty acids. Fatty acids can be metabolized and used in the Krebs cycle where they supply energy. This helps to alleviate the need for glucose as an energy source. Through these negative feedback actions the levels of glucose will begin to increase. See Figure 5-7.

Neuroendocrine Regulation
In this case the hormone is not release for an endocrine cell but rather from a nerve cell which releases its neurotransmitter in the form of a hormone into the

blood. For example, the adrenal medulla can receive sensory input from a sympathetic nerve, which tells it to release epinephrine into the blood. Other examples of neuroendocrine regulation involve the hypothalamus and the
pituitary gland. The pituitary gland can be divided into the anterior and posterior pituitary.

Paracrine Regulation
In paracrine regulation the chemical that acts as a signal is released from one cell

and influences a cellimmediately adjacent to it (Figure 5-8). An example of such a paracrine cell would be mast cells, which contain large amounts of histamine. The substances released by the paracrine cell are generally dumped into the extracellular space and not into the bloodstream. Other examples of a paracrine
signal would be neurohormones and neurotransmitters.
Secretory
Cell

Paracrine

Signaling compound

Autocrine

Figure 5-8 Paracrine and autocrine regulation.

Autocrine Regulation
In autocrine regulation cells can release certain chemicals which they can then respond to themselves (Figure 5-8). For example, certain cells (like tumor cells) can release growth factors which can then bind to specific receptors on the membrane of that same cell.Thus, the cells that released the growth hormone are stimulated to grow. Prostaglandins, which are lipid-soluble chemicals, also
appear to show autocrine regulation.

Copyright by The Berkeley Review

278

The Berkeley Review Specializing in MCAT Preparation

Biology

Endocrinology & Immunology


iipn

The Pituitary Gland

Pituitary Gland
Posterior Pituitary

The posterior pituitary releases oxytocin and antidiuretic hormone (Figure 5-9). These two hormones are synthesized in specific cells of the hypothalamus. The major effect of oxytocin is to stimulate female uterine contraction while antidiuretic hormone (ADH) stimulates water reabsorption in the kidneys and also helps to increase the blood volume (pressure).
Hypothalmic Secretory
Neurons

Pituitary
Stalk

CapillariesCZ^
Anterior

Artery

ftm \3 Capillaries

Pituitary

U/f^y Posterior ^^^ Pituitary


TSH, ACTH, FSH, LH GH.PRL

Oxytocin
ADH

Figure 5-9
The hormones released from the pituitary gland.

The ADH that is synthesized in the nerve cell bodies in the hypothalamus are packaged into vesicles and transported down the axon to the terminal bouton in the posterior pituitary. A nerve impulse (which propagated down the same axon) causes the release of these hormones into a system of nearby capillaries. ADH circulates in the blood and eventually reaches the kidneys where it stimulates water and Na reabsorption.
The nerve cell bodies in the hypothalamus where ADH was synthesized can

sense a change in blood volume (and a change in the concentration of Na). If


these cells sense a low blood volume, they release ADH into the blood. At the level of the kidneys ADH will stimulate the retention of as much water as possible. In other words, ADH prevents diuresis which is the excessive loss
Copyright by The Berkeley Review
279

The Berkeley Review Specializing in MCAT Preparation

Biology

Endocrinology &Immunology
increased.

The Pituitary Gland

urine. By preventing diuresis water can be reabsorbed and the blood volume

Anterior Pituitary
The anterior pituitary can secrete six hormones (Figure 5-39). They are thyroid stimulating hormone (TSH), adrenocorticotropic hormone (ACTH), follicle stimulating hormone (FSH), luteinizing hormone (LH), growth hormone (GH),
and prolactin (PRL). We will discuss the function of these hormones as we proceed in the course.

These hormones are regulated by a second set of hormones which are stored in

hypothalamic nerves. Forexample, let's consider TSH. Within the hypothalamus


are nerve cells that contain thyrotropin releasing hormone (TRH). When this nerve is stimulated it secretes TRH into a set of capillaries which extend into the anterior pituitary. TRH stimulates the synthesis and release of TSH from the anterior pituitary into the blood. TSH binds to specialized receptors in the thyroid gland and causes the release of thyroxine. This hormone, when released from the thyroid gland, influence metabolism and growth.
If the metabolic rate of the body is too low, TSH is released from the anterior pituitary and stimulates the release of thyroxine from the thyroid gland. Once thyroxine increases metabolism to just the right level (homeostasis) it acts back on the nerve cells in the hypothalamus and the cells in the anterior pituitary and inhibits the release of TRH and TSH, respectively. Again, this is another example
of negative feedback.

Copyright by The BerkeleyReview

280

The Berkeley Review Specializing in MCAT Preparation

Biology

Endocrinology St Immunology

Cell Types

Immunology
Every day your body is subject to countless attacks by microorganisms and viruseswhich can cause a variety of diseases. Bacteria can enter into your system
and disrupt cellular function. One example that we have mentioned is massive diarrhea caused by the bacterium Vibrio cholerae. The bacterial toxin from this organism can cause over a liter of fluid to be lost every hour. Parasitic viruses can also reap havoc on the body. Viruses are much smaller than bacteria and contain a nucleic acid core (either RNA or DNA) surrounded by a protein coat. Viruses lack their own metabolic machinery. Once they invade an organism they can take over the host's metabolic machinery for their own use and generate more progeny viruses which, after the infected cell lyses, can spread and infect other cells. However, there are certain conditions where the viral genome integrates into the chromosome of the host cell. In this case the viral genome can be replicated when the host chromosome is replicated. If this viral genome removes itself from the host genome, it can take over the metabolicmachinery of the host cell and produce more progeny viruses. Cancer causing viruses can insert their genetic material in to host chromosomes as well. Viruses can cause a variety of diseases ranging from small pox and influenza to measles and the common cold.

If we were to examine some blood under the microscope, we would find two different cells typesthe erythrocytes (red blood cells)and the leukocytes (white blood cells). Erythrocytes in adults are produced in the marrow of the sternum, ribs, and vertebrae. Leukocytes are produced partially in the tissues of the lymph and partly in bone marrow. We have already considered the erythrocytes in previous sections. Let's now briefly consider the leukocytes. There are 6 types of leukocytes found in the blood. The three types of leukocytes that we are interested in are the monocytes, neutrophils, and lymphocytes. The monocytes and neutrophils are considered to be phagocytes. These cells, along with mast cells and a variety of other cell types, participate in the immune
response.

Mast Cells: These cells are derived from leukocytes and then migrate out into the tissues where they reside. When mast cells are stimulated they release histamine which acts on endothelial cells and causes an increased permeability to cells like the neutrophils. This increased permeability allows the neutrophils easy access to the surrounding tissue in order to defend against foreign bacteria or viruses.
Phagocytes: As we have mentioned, monocytes and neutrophils are both phagocytes. When monocytes leave the blood through pores in the blood vessels and enter into the tissues, they can be transformed into macrophages. Neutrophils circulate in the blood. However, they too can leave the blood through pores in the blood vessels and enter into the tissues. The macrophages and neutrophils are the primary cell types that attack and destroy foreign bacteria and viruses. They do this by the process of phagocytosis, engulfing the foreign invader by endocytosis. Once inside the phagocyte lysosomal enzymes or hydrogen peroxide is released which can then degrade the foreign objects.

Copyright by The Berkeley Review

281

The Berkeley Review Specializing in MCAT Preparation

Biology

Endocrinology &Immunology

Cell Types

Lymphocytes: There are2 types oflymphocytes. Lymphocytes whichare derived from the thymus gland are called T lymphocytes(or T cells). The other type of lymphocytes are called B lymphocytes (or B cells). Let's briefly consider each
type.

T lymphocytes: These cells are responsible for cell-mediated immunity. These cells are responsible for the destruction of foreign microorganisms and other such agents harmful to the body. There are 3 types of T lymphocytes which we will be considering: (1) cytotoxic T cells (also called killer T cells), (2) helper T
cells, and (3)suppressor T cells.

B lymphocytes: These cells circulate in the blood and to the lymph organs like the spleen and lymph nodes and are responsible for humoral mediated immunity. [Humor is another way of saying fluid.] Upon infection the B lymphocytes can differentiate into plasma cells which have the ability to synthesize and secrete antibodies. Antibodies are proteins that are synthesized in response to an antigen that has been introduced into the body. An antigen is simply a foreign substance (protein or polysaccharide) which has a high molecular weight that has entered the body and induces a particular immune response. Antigens interact with specific antibodies.
General Immune Response Let's consider a general response to the invasion of a bacterium or even a virus. These invaders have on their surface an antigen. The cellular immune response begins with a macrophage engulfing a foreign particle and phagocytizing it. The antigenic fragments released into the cytosol of the macrophage are transported to the macrophage's membrane where they bind to a specific surface protein. This specific surface protein is called a major histocompatibility complex (MHC) protein (of the Class I type). The antigen is being presented or "displayed" on the surface of the macrophage.
Certain receptors on the class of T lymphocytes called cytotoxic T cells can recognize this antigen-surface protein complex on the macrophage. When the cytotoxic T cell binds to the antigen-Class I MHC protein complex of the macrophage, a growth factor called interleukin-1 is released by the macrophage. The cytotoxic T cell itself releases a growth factor called interleukin-2. Interleukin-1, interleukin-2, and interleukins released by helper T cells (discussed below) stimulate the synthesis of more cytotoxic T cells. These killer T cells proliferate and bind to the invading foreign cells bearing the antigen and induce them to lyse. This type of response is referred to as cell mediated immunity.

On the surface of B lymphocytes are Class II MHC proteins and antibodies. When a B lymphocyte finds an antigen that has specifically bound to its antibody, it engulfs that antigen-antibody complex. After degrading the antigenantibody complex the B lymphocyte transports a portion of that antigen to the Class II MHC protein and "displays" the complex on the surface of its
membrane.

Helper T cells with the right receptors are able to bind to the antigen-Class II MHC protein complex. This binding stimulates the helper T cells to release interleukins (a lymphokine). This stimulates the B lymphocytes to proliferate and form plasma cells. The plasma cells in turn produce a vast amount of

Copyright byThe Berkeley Review

282

The Berkeley Review


Specializing in MCAT Preparation

Biology

Endocrinology fir: Immunology

Cell Types

antibodies which are specific for the antigen. When these circulating antibodies bind to the antigen they actas a tag thatsignals circulating phagocytes to engulf the antigen-antibody complex and destroy it. This type ofresponse is referred to
as humoral mediated immunity.

The human immunodeficiency virus (HIV), thought to be responsible for the acquired immune deficiency syndrome (AIDS), acts at the level of the helperT
cells by infecting them.

Copyright by The Berkeley Review

283

The Berkeley Review Specializing in MCAT Preparation

Biology

Endocrinology St Immunology
fc*#^ Antibody Structure

Antibody Structure

Most human antibodies, or immunoglogulins (abbreviated as Ig), are composed of 4 subunits arranged in a "Y" configuration as shown in Figure 5-10. There are 2 light chains and 2 heavy chains. These subunits are joined to one another by disulfide bonds. Within each heavy and light chain are variable domains and
constant domains.

At the terminal ends of the heavy and light chains are variable (V) regions that can differ in amino acid sequence from immunoglobulin to immunoglobulin. The constant (C) regions of the heavy and light chains are found in the lower portions of the immunoglobulin. The antigen binding site for a particular antibody is located at the end of the variable regions of the heavy and light chains.

Two other regions of the immunoglobulin are important. They are the diversity (D) region and the joining (J) region. The genes that code for the J, D, and V regions of the variable domain greatly increase the diversity one finds among the immunoglobulins.

Antigen binding
site

V = variable

D = diversity J = joining

Antigen binding
site

4^

Light Chain

Heavy Chain

i[^>

Constant Domains Disulfide bond

B Cell Membrane

Figure 5-10 Generalized antibody structure.

Classes of Antibodies

We have 5 classes of immunoglobulins which differ in the composition of their heavy chains.They are (in alphabeticalorder) IgA, IgD, IgE,IgG, and IgM. Their
functions are:

Copyright by The Berkeley Review

284

The Berkeley Review

Specializing in MCAT Preparation

BlOlOgy
IgA:
IgD:

Endocrinology fif Immunology

Antibody Structure

Found in milk and helps to protect nursing infants.


Has an unknown function.

IgE: IgG: IgM:

Binds to mast cellsand is involved in the allergic reaction. The only antibody able to cross the placenta. It is also the most abundant and is produced withindays after the IgMantibody is secreted. Produced a few days after detection of an antigen and it is the first antibody produced in response to an antigen.

Combinatorial Diversification

The light chain contains at least 300 different genes that can determine the

variable region and about 4 different genes that can determine that joining
region. Therefore, there can be about (300) x (4) or 1,200 different combinations

for the lightchain. The heavy chain contains about 1,000 different genes for the variable region, about 12 different genes for the diversity region, and about 4
different genes for the joining region. Thus, we find that there are about (1,000) x (12) x (4) or 48,000 differentcombinations for the heavychain. If we associate all the different possiblelight chainswith all the different possible heavychains, we

will geta combinatorial diversification of about(1,200) x (48,000) or roughly 5.8 x 107 different possible antibody combinations. In other words, the immune
system can generate an antibody for practically any antigen that invades the

body. [The point behind this is the enormous antibody diversity that is
generated. Depending on how the calculation is done and what values are used

the diversity can be as high as 1.1x 1011.]

Copyright by The Berkeley Review

285

The Berkeley Review Specializing in MCAT Preparation

Biology

Endocrinology St Immunology

Antibody Action

Antibody Action
Antibodies do not destroy foreign antigens. They simply recognize and identify
them. Antibodies can do this is a number of ways. One method is by directly

blocking the foreign invader from gaining access to host cells. This is accomplished by the antibodies binding to the antigens of, say, a virus as shown
in Figure 5-11.

Figure 5-11
Direct block.

Complement is a rather complicated system for disposing of invaders. The essence of this process is as follows: An antibody has already recognized and bound to a specific antigen on a bacterial cell that is considered an intruder. A complement protein (a plasma protein) recognizes this antigen-antibody complex and binds to the Fc domain of the antibody. After a series of reactions the complement protein is activated and triggers an immune response. Further
reactions form a membrane attack complex (MAC) that inserts into the bacterial
cell's membrane and forms a channel that lets water into the cell. The bacterial

cell swells with water and eventually lyses (bursts). See Figure 5-12.
MAC

Activation

Complement
Protein

Complement

Water

FC
domain

Protein

Figure 5-12 Complement.

Antibodies can bind to specific antigens on the surface of a bacterial cell and coat
the cell surface. Once the antibodies have attached to the bacterial cell,

phagocytes and/or killer T cells can bind to the terminal portion of the Fc
domain of the antibodies and begin to engulf the foreign invader as shown in
Figure 5-13.
Figure 5-13 Cell surface coating.

Copyright by The Berkeley Review

286

The Berkeley Review

Specializing in MCAT Preparation

Biology

Endocrinology & Immunology


Wi$$

T Cells

The receptors for T cells are composed of two polypeptide chains, each with a constantand a variable domain. Within the variable domain in each polypeptide chain we find a variable (V) region and a joining (J) region. On one of the polypeptide chain is a diversity (D) region. See Figure 5-14. This is analogous to

the immunoglobulin structure in that a great amount of (antigen binding) diversity can be generated from different combinations of the genes that produce
these polypeptide chains.
Antigen binding
site

Variable Domain

V = variable
D =

diversity

J = joining

T Cell Membrane

Cytoplasm

Figure 5-14 T Cell receptor.

Cytotoxic T cells have cell receptors like the one shown in Figure 5-15. If a virus infects a host cell, then that virus will begin to take over the host's metabolic machinery. As this happens some of the viral antigens are transported to the surface of the host cell where they can complex with a Class I MHC protein receptor. Class I MHC receptors are found on almost every one of our cells.
MHC I receptor antigen complex

MHC I receptor

"Sc3
Viral

Jk

proteins

Figure 5-15 Cytotoxic T Cell interaction with MHC I receptor.

The antigen-Class I MHC protein complex is recognized by the cytotoxic T cell. Binding occurs between the cytotoxic T cell and the antigen-bearing host cell as shown in Figure 5-15. The cytotoxic T cells induce lysis in the host cell contain
Copyright by The Berkeley Review
287

The Berkeley Review Specializing in MCAT Preparation

Biology

Endocrinology St Immunology

T Cells

the viral particles in order to prevent more progeny viral particles from
spreading.

Immature helper T cells recognize macrophages which have presented an antigen on their Class II MHC protein receptors as shown in Figure 5-16. Binding induces the macrophage to synthesize and release interleukin-1 which acts on the immature helper T cell and causes it to synthesize and release
interleukin-2. Interleukin-2 further stimulates the immature helper T cell to

proliferate into a mature helper T cell. The mature form of the helper T cell
secretes interleukin-2 which can activate cytotoxic T cells, B cells, and more helper T cells.
MHC II receptor antigen complex

MHC II receptor CZ^


j= 2
60 *
ca a

<K^>
Viral

5 &
0 60
c5 C

v/
"V
X

Helper \^
T cel1 J
Activates:

proteins

^ A

cvxr

v-*. cytotoxic T cells, B cells, cytotox

helper T cells
Interleukin-1

Figure 5-16 Cytotoxic T Cell interactionwith MHC I receptor.

Copyright by The BerkeleyReview

288

The Berkeley Review

Specializing in MCAT Preparation

BlOlOgy

Endocrinology St Immunology
>rO

Humoral fir Cellular Immunity

Humoral & Cellular Immunity


Generalized Review

Let's summarize the basic events in humoral and cellular immunity. The
numbers in these steps willcorrespond to thenumbers in Figure 5-17.

1.
2.

A virus enters thebodyby theblood andis engulfed by a macrophage.


On the surface of the macrophage are Class I MHC receptors and Class
II MHC receptors.

3.
4.
5.
6.

A Class II MHC receptor presents the viral antigen to the receptor of a


helper T cell.

This causes the macrophage to release interleukin-1 (IL-1).


IL-1 stimulates the helper T cells to proliferate.
The helper T cell is stimulated to release IL-2 which enhances proliferation of helper T cells.

7.
8.

A Bcellwith a Class II MHC protein presents viralantigen to helper T


cell.

IL-2 releasedfrom helper T cellstimulatesB cells to proliferate.

9.

B cells produce plasma cells and memory B cells. Memory B cells


"remember" antigen and proliferate faster during a future invasion of
the same virus.

10.
11.

Plasma cells secrete antibodies specific for the viral antigen.


The antibodies respond by direct block, complement, and cell surface
coating.

12.

IL-2 from the helper T cells stimulates cytotoxic T cells which have bound to the Class I MHC protein-antigen complex of an infected cell to lyse that infected cell.
Interferon is secreted by the infected host cell and acts on the cytotoxic T cell to help enhance the immune response.
Cytotoxic T cell also make memory T cells (not shown) which will proliferate faster during a future invasion of the same virus.

13.
14.

Copyright by The Berkeley Review

289

The Berkeley Review Specializing in MCAT Preparation

Biology

Endocrinology & Immunology

Humoral fie Cellular Immunity

MHC Receptor CZ^ "

Class I

(2)

Complement
Mast

Cells

Histamine

Antibodies

(10)

Leaky
Endothelial

Cell Surface Coating


Figure 5-17
Review of humoral and cellular immunity.

Cells

Copyright by The Berkeley Review

290

The Berkeley Review Specializing in MCAT Preparation

Endocrinology and Immunology


15 Passages
100 Questions

Passage Titles
I. II. III. IV.
V. VI. VII.
VIII.

Questions
1 -7 8- 14 15-21 22-27 28-33 34-40 41 -46 47-54 55-61 62-67 68-75 76-83 84-90 91 -95 97 - 100

Antibody Structure Acquired Immunity Major Histocompatibility Complex (MHC) IgA Antibody Experiment
Complement System Myasthenia Gravis/Autoimmune Disease Glucose, Glucagon, & Insulin
Calcium, PTH, Calcitonin, and CaleitrioI

IX. X. XI. XII. XIII.


XIV.

Erythroblastosis Fetalis Septic Shock Calcitonin and Osteoporosis Vertebrate Immune System Insulin Receptor
Vitamin D3, PTH, and Calcitonin

XV.

Sepsis Syndrome

Berkeley
Specializing in MCAT Preparation

Suggestions
The passages thatfollow are designed toget you tothink ina conceptual manner about theprocesses of physiology at the organismal level. If you have a solid foundation in physiology, many of these
answers will be straightforward. If you havenot had a pleasant experience with the topic, some of these answers mightappearto come from thevoidpasttheOort field ofthe solarsystem. Picka few passage topics at random. Forthese initial few passages, do not worry about the time. Just focus on what is expected of you. First, read the passage. Second, look at any diagrams, charts, or graphs. Third, read each question and the accompanying answers carefully. Fourth, answer the questions thebestyoucan. Check thesolutions andsee howyoudid. Whether you got the answers right or wrong, it is important to read the explanations and see if you understand (and agree with) what is being explained. Keep a record of your results. After you feel comfortable with the format of those initial few passages, pick another block of passages and try them. Be awarethat timeis going to become important. Generally, you will have about
1 minute and 15 seconds to complete a question. Bea little more creative in how you approach this next

group. If you feel comfortable with the outline presented above, fine. If not, then try different approaches to a passage. Forexample, you mightfeel wellversed enoughto read the questions first and then try to answersomeof them, withouteverhaving read the passage. Maybe you can answer some of the questionsby just lookingat the diagrams, charts,or graphs that are presented in a particular passage.
Remember, we are not clones of one another. You need to begin to develop a format that works best for you. Keeping a record of your results may be helpful.

The last block of passages might contain topics that are unfamiliar to you. Find a place where the level of distraction is at a minimum. Get out your watch and time yourself on these passages, either individually or as a group. It is important to have a feel for time, and how much is passing as you try to

answer each question. Never let a question get you flustered. If you cannot figure out what the answer is from information given to you in the passage, or from your own knowledge-base, dump it and move on to the next question. As you do this, make a note of that pesky question and come back to it at the end, when you have more time. When you are finished, check your answers and make sure you understand the solutions. Be inquisitive. If you do not know the answer to something, look it up. The solution tends to stay with you longer. (For example, what is the Oort field?)
The estimated score conversions for 100 questions are shown below. At best, these are rough approximations and should be used only to give one a feel for which ballpark they are sitting in.

Section V Estimated Score Conversions


Scaled Score
>12 10-11

Raw Score

86-100
79-85

8-9
7

65-78 59-64
54-58

6
5 4

48-53 0-47

Biology
Passage I (Questions 1-7)

Antibody Structure

Passage I

Immunoglobulins fall into five classes (isotypes). The approximate percentage of total antibody in the serum
immunoglobulin pool is indicated in Table 1.
Table 1: Antibody Classes
Isotype
IgG IgA IgM IgD IgE

In order to protect themselves from pathogens, animals have developed an array of protective mechanisms within their immune system. Immunity can be divided into two types: innate (nonspecific) and acquired (specific). Innate immunity affords protection against a multitude of different pathogens and is not dependent on specific
recognition of foreign macromolecules. The skin, mucous membranes, and enzymes in secretions are all examples of nonspecific immunity.

% of plasma Ig
70-75 15-20
10
<1

= 0.005

Acquired immunity enhances the protection provided


by innate immunity and is the result of an initial encounter with a foreign macromolecule (immunization). If an

In humans IgG is the only antibody that can cross the placental membrane. IgA is primarily found in external secretions such as tears, saliva, colostrum, and milk. IgM

immune response is activated, the agent inducing that


response (an immunogen) will initiate a series of reactions that lead to the production of antibodies (proteins) that are

is the first antibody produced by the plasma cells during an immune response and is the predominant antibody produced by the fetus during development. IgD has an
unknown function, but since it is found as a surface

specific against the inducing agent. The ability of the inducing agent to combine with an antibody is called
antigenicity, and an agent with this characteristic is called an antigen. Vaccinations and a recovery from a disease

like the measles are examples of specific immunity.


Antibodies are globular glycoproteins that participate in the immune response. These immunoglobulins (Ig) are present in the y-globulin component of blood serum and are produced by plasma cells. All antibodies are related to one another through a common motif (Figure 1).

protein in many B cells, it may play a role in B cell differentiation into a more mature form like plasma cells or memory B cells. IgE is thought to confer immunity to certain parasites, such as worms. IgE can also bind to basophils and mast cells where, in mast cells, it promotes the release of histamine, a chemical that causes peripheral vasodilation and increased capillary permeability, leading
to anaphylactic reactions.

The fragment of the antibody that binds antigen is the antigen binding domain. The binding occurs between the paratope of the antibody and the epitope (antigenic determinant) of the antigen. An antigen may possess just a single epitope on its surface, making it a unideterminant, univalent antigen. Antigens may also be unideterminant
and multivalent, multideterminant and univalent, or multi-

Fab

Fab

determinant and multivalent. Antibody-antigen interaction can result in a variety of outcomes, including precipitation and agglutination. These complexes can be dissociated by high salt concentration or either high or low pH.

1.
Figure 1 Key for IgG
C
V
L H
=

Which of the following is NOT a component of the nonspecific immune system?

Fab

Fc

Constant region Variable region Light chain Heavy chain antigen binding fragment crystallizable fragment
Disulfide bonds

A. B. C. D.
2.

Keratinized and epidermal cells. Beating of epithelial cell cilia. Placental transfer of antibody. Normal body temperature.

Which of the following types of immunity would


result from a vaccination?

Papain cleaves Pepsin cleaves

A. B. C. D.
293

Active immunity that is naturally acquired. Passive immunity that is artificially acquired. Active immunity that is artificially acquired. Passive immunity that is naturally acquired.

Copyright by The Berkeley Review

The Berkeley Review Specializing in MCAT Preparation

Biology
3.
all of the following EXCEPT:
A. Covalent bonding.

Antibody Structure

Passage I

The binding of antibody to antigen occurs through

7.

Lysozyme inhibits peptidoglycan synthesis in the


cell wall of gram negative bacteria. Which of the following immunoglobulins would act agonistically
with lysozyme?
A. B. C. D. IgG IgA IgM IgE

B. C.
D.

Hydrogen bonding. Hydrophobic forces.


Van der Waals forces.

4.

A paratope can best bedescribed as that area of the:


A. B. C. D. antigen which reacts with the antibody. antibody designated by the Vh regions of the
Fab.

antigen which exhibits a multideterminant and


univalent expression.

antibody which has similar dimensions as the


antigenic determinant.

5.

Papain, a protease extracted from papaya plants, cleaves primarily on the carboxyl side of lysine and arginine amino acid residues. Treatment of IgG with papain before IgG is exposed to a unideterminant,
multivalent antigen will result in: A. cross-linking, because the antigen possesses just one type of determinant but many of them. an Fc fragment and a bivalent Fab fragment, which can participate in cross-linking. antigens that are not cross-linked, because
each Fab fragment is monovalent. two monovalent Fab fragments, which can participate in cross-linking.

B. C.
D.

6.

Treponema pallidium, an organism that causes syphilis, can cross the placental membrane and enter
into the fetal circulation. Which of the following immunoglobulins would be expected to increase in
concentration in the blood serum of the fetus?

A. B. C. D.

IgG IgA IgM IgE

Copyright by The Berkeley Review

294

The Berkeley Review Specializing in MCAT Preparation

Biology
Passage II (Questions 8-14)

Acquired Immunity

Passage n

10.

Acquired immunity is the resistance to disease that an organism develops during its lifetime. Immunity may be acquired actively or passively. Active immunity occurs when an organism is attacked by a foreign substance or a microorganism (an antigen). Antibodies and specialized lymphocytes are produced by this exposure that confer a "memory" when the foreign agent attacks again at a later time. Passive immunity occurs when an organism receives pre-formed antibodies from another organism. Acquired immunity may be achieved through natural or artificial means. For instance, a person may be exposed to the chicken pox virus, develop a rash, and become immune to chicken pox for the rest of his life. This immunity is actively acquired through a natural means. Conversely, a new chicken pox vaccine would provide artificiallyacquired active immunity. Vaccines do contain microorganisms that may be dead or weakened. The
substances in vaccines stimulate an active immune

The following diagram indicates the primary and secondary response to an antigen. IgG, a monomer, crosses the blood vessels easily, while IgM, a
pentamer, does not cross as well.
1000 n

E
100-

J5 'S
v.
V.

>

8
c

<

14

21

28

35

42

49

56

Time (days)

response. An example of passive natural immunity is the maternal provision of antibodies to the fetus via the blood
and to the neonate via breastmilk. The baby has an immature immune system, and these antibodies function for several months as the baby's system matures. An example of passive artificial immunity is receiving antibodies from a rabbit that was immune to a spider venom, as a treatment for a spider bite.

Which of the following statements is FALSE? A. B. A "booster shot" for an adult exposure to tetanus promotes formation of mainly IgG. Before the primary exposure, there are no antibodies to a particular antigen. Both IgG and IgM remain at high levels following a "booster shot". IgM provides the primary response to an antigen.

C. D.

8.

When a person is passively immune to a foreign agent, is this a permanent immunity? A.


B.

C. D.

Yes, receiving antibodies confers permanent immunity. Yes, making antibodies confers permanent immunity. No, receiving antibodies confers transient immunity. No, making antibodies confers transient immunity.

11.

A susceptible adult individual received pre-formed


antibodies isolated from the serum of an immune

individual. What type of acquired immunity does


this confer on the first individual?

A. B. C. D.

Artificially acquired active immunity. Artificially acquired passive immunity. Naturally acquired active immunity. Naturally acquired passive immunity.

9.

A newborn child is accidentally kissed by her cousin, who is contagious with measles. Why does the newborn stay healthy?
12.

The first vaccine by Edward Jenner in 1798

A.
B.

Maternal antibodies provided immunity to


measles in this situation.

protected against smallpox. He used a related strain


of microorganisms that caused cowpox as a vaccination against the microorganisms of smallpox. What type of immunity was conferred in this case? A. B. C. D. Artificially acquired active immunity. Artificially acquired passive immunity. Naturally acquired active immunity. Naturally acquired passive immunity.

The newborn child quickly made antibodies to


the measles virus.

C.
D.

The cousin provided antibodies by kissing the


newborn child.

Paternal antibodies provided immunity to


measles in this situation.

Copyright by The Berkeley Review

295

The Berkeley Review Specializing in MCAT Preparation

Biology
13.

Acquired Immunity

Passage n

Blood serum is subjected to electrophoresis in order to separatethe proteins.Antibodies are presentin the gamma globulin fraction. The following diagram shows serum proteins following gel electrophoresis:
Direction of migration
Cat tiode

Anode

(-)
r~~\
JS 60
s

(+)
.*.

.>' ''''''

2
H

v'/\

'\''-\

$$
*'..'/.
a
i I

LJ
Y
i

<:'.'

'.;.'
l

P
Globulins

Albumin

In this diagram, which protein is the largest, based on its migration pattern?
A. albumin

B.

a-globulin

C. D.

p-globulin y-globulin

14.

If a newborn is orphaned at birth, which process would provide more antibodies for the child?
I. Feed the child breastmilk from another nursing
mother.

II. Seclude the child in a sterile hospital unit. m. Vaccinate the child immediately to all
childhood diseases.

A. B. C.
D.

I only I and II only II and III only


I, II, and ffl

Copyright by The Berkeley Review

296

The Berkeley Review

Specializing in MCAT Preparation

Biology

Major Histocompatibility Complex (MHC)

Passage in

Passage III (Questions 15-21)

Endosomes are membrane vesicles that often contain

Transplantation antigen proteins expressed on the surface of a cell and recognized by the immune system are encoded by the major histocompatibility complex (MHC) genes. The MHC genes are of two fundamental types, class I and class II. While mouse and human populations carry more than one hundred forms of the molecules, only between three and six of each class are expressed.
Both classes of MHC molecules are involved with

surface proteins and their associated ligands. Endosomes are known to contain proteases, which are believed to
degrade the invariant chain and allow the molecule to

bind peptide. However, a small part of the invariant chain (known as CLIP) remains at the binding cleft until a DM
molecule (an MHC class H-like molecule) enters the endosome and binds CLIP, actively removing it from the
MHC class II molecule. At this time, the MHC class II

molecule, bound with peptide, is placed on the cell


surface.

antigen processing, which includes the ingestion of antigens, the fragmentation of antigens into peptides, and the binding of these peptides to MHC molecules. The formation of the MHC-peptide complex is a critical event
in the effective elimination of intracellular parasites. The

15.

The amino acid sequence of both the MHC class I


and MHC class II molecules should show:

peptides associated with MHC class I have invariably


been found to originate from a cell's own proteins, while

the peptides found bound to MHC class II are normally


located on the outer membrane.

A. B. C. D.

hydrophilic regions. hydrophobic regions. both hydrophobic and hydroapathetic regions. both hydrophilic and hydrophobic regions.

The MHC class I protein usually binds peptides that are eight to nine amino acids long. The two ends of the peptide chain bind to discrete binding sites located in the
cleft of the MHC class I molecule (Figure 1). The binding cleft of an MHC class II molecule is similar in shape to
that of an MHC class I molecule, but the MHC class II
16.

molecule usually binds peptides in the middle of the cleft


(Figure 2).

On the surface of a cell infected by a virus, the majority of MHC class I molecules have bound peptides that originate from:
A. B. C. D. the virus. a killer T cell.
a B cell. a host cell.

Peptide chains

17.

The two protein subunits that constitute the MHC class II molecule are MOST likely to contain a:
A. B. C. D. Shine-Dalgarno sequence. signal peptide sequence. pyrimidine-rich sequence. purine-rich sequence.

Figure 1

Figure 2

MHC class II molecules have been shown to assemble

18.

in the endoplasmic reticulum. However, immediately after their synthesis, the MHC class II subunits associate with a
third molecule known as the invariant chain. The

Endosomes are formed by which of the following processes?

invariant chain inhibits peptides from binding to the MHC


class II molecules and causes the MHC class II molecules

to leave the Golgi complex and fuse with endosomes.

A. B. C. D.

Megacytosis Exocytosis Endocytosis Transcytosis

Copyright by The Berkeley Review

297

The Berkeley Review Specializing in NCAT Preparation

Biology

Major Histocompatibility Complex (MHC)

Passage III

19. The protein DM is structurallysimilar to:


A. B. C.
D.

the invariant chain. MHC class II. CLIP. MHC class I.

20.

Which of the following statements is true regarding peptides bound to MHC class I and MHC class II proteins?
I. Peptides bound to MHC class I should exhibit a greater size variation. II. Peptides bound to MHC class II should exhibit a greater size variation. III. The amino acids binding to a cleft should be conserved despite the variety of peptides the
cleft can bind.

A. B. C. D.

I only II only I and III only II and in only

21.

Which of the following is LEAST likely to be a defense mechanism used by a pathogen to deter the antigen processing system? A.
B.

Suppression of MHC molecules early in an


infection Production of molecules that bind to MHC

C.
D.

class I molecules in the ER and prevent cell surface expression Production of a transcription factor that increases the transcription rate of the MHC gene early in an infection Suppression of the DM molecule

Copyright by The Berkeley Review

298

The Berkeley Review Specializing in MCAT Preparation

Biology
Passage TV (Questions 22-27)

IgA Antibody Experiment

Passage IV

In the following experiment, researchers studied the role of the cytokine, interleukin-6, as a factor in the response of immunoglobulin A (IgA) to foreign molecules in mice. IgA is the antibody group that is released from epithelial surfaces in secretions such as saliva or breast milk. IgA often represents a first line of defense against invading pathogens. Experiment 1

The graphs in Figure 2 represent the number of IgA secreting cells in mouse lung tissue 1 and 2 weeks following exposure to a virus carrier alone or a virus carrier plus the BL-6 gene.

22. Which of the following tissues are lined by epithelial


tissue? I. II. m. Mouth Small intestine Urethra

The IgA responses of mice unable to make IL-6 (IL-6") and control mice (IL-6+) were studied. Ovalbumin, an egg protein, was added to the mouse intestinal mucosa. Immunoglobulin response is depicted in Figure 1.
50i
u

A. B. C.
D.

I only I and II only n and m only


I, II, and ffl

IL-6+
IL-6"

40

30& 20-

= 10H
U 0

i
i

23. IgM
Figure 1

IgA

IgG

IgE

Which of the following statements is TRUE of Figure 1? A. IL-6" mice produce higher levels of IgA than
IL-6+ mice.

Experiment2
IL-6" and IL-6+ mice were immunized with a virus

B. C.
D.

IL-6" mice produce lower levels of IgA than


IL-6+ mice.

construct carrying the mouse IL-6 gene. The virus was theorized to insert the IL-6 gene into the DNA of the host
cells it infected.

The presence of IL-6 did not affect the production of IgA. IL-6" mice respond strongly to ovalbumin.

.2

100

IL-6+
|> 13

2
C/3vo

With virus carrier

With virus carrier

24. IgA is composed of what type of molecules?


A. Amino acids

alone
1

plus IL-6 gene


i 1

1 8.

"8 u

I-
12

I
25.

12

B. C. D.

Fatty acids Sphingomyelins Phospholipids

Weeks following innoculation of mice


With virus carrier

(J jn
.5

100
IL-6"

plus IL-6 gene

Which statement is TRUE of Figure 2?


A. The virus carrier alone transformed both strains of mice.

8 2
G/3\D

>>2 ~"
S 4)

B. C.
12 12

Restoring IL-6 in the IL-6" mice improved IgA


production.

IL-6" mice were hypersensitive to the virus


carrier.

Weeks following innoculation of mice

D.

IL-6+ mice did not respond to the virus carrier.

Figure 2

Copyright by The Berkeley Review

299

The Berkeley Review

Specializing in MCAT Preparation

Biology
26.

IgA Antibody Experiment

Passage IV

The researchers did not report BL-6 concentrationsin the blood. Why is this the case?

A.
B.

Cytokines are hormones and act while passing through the entire circulatory system.
Hormone concentrations cannot be measured
in blood.

C.

EL-6 never leaves the cell that produces it.

D.

Cytokines are local hormones and often act without passing through the entire circulatory
system.

27.

What term refers to the medical alteration of genes to correct an inherited or acquired disease?
A.
B. C.

Gene therapy
Vaccination Genetic immunization

D.

Pleiotropy

Copyright by The Berkeley Review

300

The Berkeley Review Specializing in MCAT Preparation

Biology
Passage V (Questions 28-33)

Complement System

Passage V

29. The proteolytic cascade described in the passage:


A. B. C.
D.

Complement was given its name because it complements the action of antibodies and is the principal means by which antibodies defend vertebrates against most bacterial infections. A system of serum proteins are activated to form a membrane attack complex (MAC) which forms holes in microorganisms. Complement also amplifies the defense system by dilating the blood vessels and attracting phagocytic cells to the site of infection. Individuals who are complement deficient also suffer from immune complex diseases, in which antibodyantigen complexes precipitate in small blood vessels in skin, joints, and brain causing the destruction of tissue.

focuses the complement system away from cell


membranes.

does not take place in the alternate pathway. provides a means of amplification, ultimately leading to many MACs. uses serine proteases at all serine residues in a
protein.

30.

The production of antibodies used in the complement process would be greatly affected by a
disease of the:

Complement consists of about 20 interacting proteins. The components involved in reactions are known as ClC9, factor B, and factor D. The rest of the proteins are involved in the regulation of this system. These proteins
circulate in the blood in an inactive form unless activated

A.
B.

lymph nodes.
bone marrow.

C. D.
31.

thymus. spleen.

directly by an invading microorganism or indirectly by an immune response. The final result of activation is assemblage of the late complement components (C5-C9) into a membrane attack complex.
The early complement components are activated by either antibodies bound to a microorganism or by polysaccharides on a microbial envelope. There are two distinct pathways of early component activation. CI, C2, and C4 belong to the classical pathway and is triggered by antibody binding. Factor B and D belong to the alternative pathway and are triggered by microbial polysaccharides. Both pathways will act on C3, a central component in the complement system. All early components and C3 are proenzymes that are activated by each other through proteolytic cleavage. As each proenzyme is cleaved, it is activated to generate a serine protease which cleaves the next proenzyme in the sequence. Each activated enzyme cleaves many molecules of the next proenzyme in the chain. The cleavage normally exposes a membrane binding site on the larger fragment and liberates a small peptide fragment into the blood stream. The C3 molecule is eventually cleaved, with its larger fragment binding both the cell membrane and C5. Activation of C5 will initiate the spontaneous assembly of C5 through C9, forming the membrane attack complex.

The classical pathway is usually activated by IgG or IgM antibodies bound to antigens on the surface of a microorganism. The CI complex most likely binds
to the:

A. B.
C.

antigenic determinant. variable region of the antibody.


cell membrane.

D.
32.

constant region of the antibody.

As stated in the passage, protease cleavage acts to expose a membrane binding site on the larger fragment. The most likely reason for this is to:
A. B. avoid precipitation of complement proteins. have the larger fragment act as a diffusible signal in the bloodstream. confine complement activation to the cell surface where it began.
inhibit the next reaction in the cascade sequence.

C.
D.

33.

During the proteolytic complement cascade, several small biologically active fragments are generated. One of the end results of these molecules' activity is an increase the permeability of local blood vessels. Which is the most likely explanation for such an
increase?

28.

According to the passage, complement may normally help:

A. B.
C.

These molecules stimulate the secretion of

histamine from T lymphocytes.


These molecules stimulate the secretion of

A. B. C. D.

produce antigen-antibody complexes. destroy antigen-antibody complexes. solubilize antigen-antibody complexes. precipitate antigen-antibody complexes.

histamine from basophils.


These molecules stimulate the secretion of

histamine from macrophage.


D. These molecules stimulate the secretion of

histamine from erythrocytes.

Copyright by The Berkeley Review

301

The Berkeley Review Specializing in MCAT Preparation

Biology

Myasthenia Gravis/Autoimmune Diseases


35.

Passage VI

Passage VI (Questions 34-40)

Myasthenia gravis is a rare, chronic, neuromuscular


disease. It is characterized by skeletal muscle weakness

Shown below is a diagram of a neuromuscular junction. Which number indicates an acetylcholine receptor?

and fatigability in response to repeated contraction. Resting partially restores muscle strength. The muscles of the eyes, face, jaw, and neck are usually affected first. As the disease progresses, weakness spreads to the extremities and the diaphragm. In severe cases, all the
muscles are weakened.

Research indicates myasthenia gravis is an autoimmune disorder in which antibodies are produced to the acetylcholine receptor that is present at the neuromuscular junction. Antibodies to the acetylcholine receptor have been found in 85% of patients with generalized myasthenia. Although the mechanism for antibody production is unclear, one hypothesis is that certain thymus cells that resemble muscle (myoid cells) are damaged by a virus. The virus may have a molecular region that mimics part of the acetylcholine receptor, such as the herpes simplex virus. A virus may damage myoid cells so that antibodies are produced against them directly. By whatever mechanism, the viral infection induces antibody production.
The actual interaction between the antibody and the receptor is not fully understood. The antibody may block the receptor, it may cause faster receptor breakdown, or it may promote complement-mediated damage.
A. B. C. D. I II HI TV

\v m

Autoimmune diseases as a whole are relatively common. In the following table is a list of some autoimmune diseases and the antibodies produced in the
disease state:

36. Which of the following is NOT an example of an


autoimmune disease?

A.
Disease

Type II diabetes.
Addison's disease.

Antibody Against
B cells of pancreas Thyroid stimulating hormone receptor Myelin basic protein (hypothesized)
Basement membrane

B.
Effects

C.
D.

Myasthenia gravis.
Graves' disease.

Type I
diabetes Graves'
disease

Destroys B cells
Stimulates TSH

receptor on thyroid Disrupts myelination

37. Which of the following is NOT a symptom of


Graves' disease?

Multiple
sclerosis

A.

Increased metabolic rate.

Glomerulo

nephritis

Destroys a variable of glomular capillaries number of glomeruli

B. C. D.

Weight loss. Lethargy. Hyperactivity.

34. Which drug could be given to counteract the effects of the antibody produced in myasthenia gravis?
A.
B.

38.

Secretion of which of the following from the pancreas is halted by antibodies to the B cells?
A.
B. C.

An inhibitor of acetylcholinesterase.
An immunostimulant.

Glucagon.
Insulin. Bicarbonate.

C. D.

A paralytic agent, like curare. An inhibitor of acetylcholine synthase.

D.
302

Digestive enzymes.

Copyright by The Berkeley Review

The Berkeley Review Specializing in MCAT Preparation

Biology
39.
disease?

Myasthenia Gravis/Autoimmune Diseases

Passage VI

How can a viral infection lead to an autoimmune

I.

The virus resembles a "self molecule, leading to antibodies that cross-react with other body
molecules

n. The virus damages a cell so that unrecognized

cell proteins are released, causing antibody


production
HI. The virus resembles a "non-self molecule,

leading to antibodies that cross-react with other body molecules


A. B. C. D. I only I and II only II only II and in only

40. Often patients with autoimmune diseases are treated


with corticosteroids to reduce immune responses.

Which organ in the body produces corticosteroids?


A. Pancreas.

B. C. D.

Adrenal gland. Pituitary gland. Thyroid gland.

Copyright by The BerkeleyReview

303

The Berkeley Review

Specializing in MCAT Preparation

Biology
Passage VTI (Questions 41-46)

Glucose, Glucagon, St Insulin

Passage VH

41.

The regulation and metabolism of carbohydrates in the body are controlled by the pancreas. The endocrine component of the pancreas, the islets of Langerhans, are specifically responsible for carbohydrate control. These
small clusters of cells imbedded within the exocrine

During times of stress the importance of having adequate levels of glucose, for utilization as energy, is dependent on hormonal secretions from the pancreas. Which choice below will BEST ready the body for stressful situations?
A. B. Increased levels of glucagon and insulin. Increased levels of glucagon and decreased
levels of insulin.

portion of the pancreas contain peptides with specific hormonal activity.


Glucagon, secreted by the a cells (or A cells), liberates glucose from storage areas in the body, stimulates glucose production, increases lipid concentration in the blood stream by releasing free fatty acids, and increases the production of ketones, a cells are stimulated to secrete glucagon during increases in plasma amino acid levels, Cortisol secretion, exercise, and sympathetic nervous

C.
D.

Decreased levels of glucagon and decreased


levels of insulin.

Decreased levels of glucagon and increased


levels of insulin.

system stimulation. Inhibition of glucagon is promoted by increases in plasma glucose, ketone, free fatty acids,
insulin, and somatostatin levels.

Insulin is secreted by the P cells (or B cells) and functionally is important in increasing the storage of glucose, fatty acids, and amino acids in the cells of target
tissues. Furthermore, insulin decreases the release of

glucose, mannose, amino acid, and glucagon plasma levels. Many intestinal hormones also stimulate insulin secretion. Parasympathetic stimulation of the (3 cells will increase insulin secretion, while sympathetic stimulation
will inhibit secretion.

42. Ketosis is developed from an increase in the conversion of free fatty acids to ketone bodies. These ketone bodies are an important source of energy in times of fasting. However, prolonged ketosis will lead to a plasma acidosis due to liberated hydrogen ions from ketone bodies. A patient with acidosis will develop shortness of breath, dehydration, hypervolemia, and hypotension. In severe cases the acidosis and dehydration will depress consciousness to the point of coma. Which choice below will lead to the development of ketosis
and acidosis?

Somatostatin is a peptide secreted by 8 cells (or D cells) in the islets. Somatostatin inhibits the production of both insulin and glucagon, and it acts as a regulator of
islet secretion.

A. B.

Increased levels of glucagon and insulin. Increased levels of glucagon and decreased
levels of insulin.

C.
D.

Decreased levels of glucagon.


None of the above.

The effects of insulin and glucagontarget very specific


regions of the body where their cellular actions occur. Insulin's effects are generally associated with muscle and

adipose tissue, leukocytes, fibroblasts, and mammary glands. Insulin does not directly affect brain and kidney tissue, intestinal mucosa, and red blood cells. Glucagon's
effects are targeted mainly on the liver.

43.

Symptoms reported by a patient include weakness, dizziness, confusion, and hunger. Furthermore, some
tremors, palpitations, and nervousness are also reported. These last symptoms are characteristic of

hypoglycemia. The development of these symptoms


is due to:

A. B.

abnormal increases in insulin secretion. abnormal decreases in insulin secretion.

C. D.

abnormally high levels of plasma glucose. abnormally low levels of free fatty acids.

Copyright by The Berkeley Review

304

The Berkeley Review Specializing in MCAT Preparation

Biology
44.

Glucose, Glucagon, St Insulin

Passage VD

What pancreatic hormonal response is expected after a heavy protein intake?


A.
B.

Increased levels of insulin and glucagon.


Increased levels of insulin and decreased levels

C.
D.

of glucagon. Decreased levels of insulin and glucagon.


Decreased levels of insulin and increased

levels of glucagon.

45.

From the data reported on glucose utilization of tissue and responsiveness of tissue to insulin, it may
be determined that the main function of insulin
secretion is:

A. B. C. D.

increasing glucose uptake in the brain. increasing glucose release from the liver. access and storage of glucose in cells of the peripheral tissues. increasing glucose loss in the kidney.

46.

Patients diagnosed as having diabetes mellitus are


said to be in "a state of starvation in the midst of

plenty." This analogy refers to:


A. B.
C.

extracellular glucose excess. extracellular glucose deficiency.


decreased effects of insulin on intestinal

mucosa uptake of glucose.

D.

deficiency of fatty acids in neural tissue.

Copyright by The Berkeley Review

305

The Berkeley Review

Specializing in MCAT Preparation

Biology
Passage VHI (Questions 47-54)

Calcium, PTH, Calcitonin, Calcitriol

Passage VII

Calcitonin acts to inhibit osteoclast activity and reduce


bone resorption. This hormone also inhibits calcium and phosphate reabsorption in the kidney and increases the
excretion of these ions in the urine.

Calcium and phosphate both play an important role in


the mineralization of bone in vertebrates. Calcium is

obtained from the diet and is largely absorbed at the level


of the intestine. The intracellular concentration of calcium

is about 10*7 mol/L while the extracellular concentration of calcium is about 10'^ mol/L. Blood calcium levels are primarily determined by bone metabolism and urinary
excretion.

47. Administration of PTH leads to changes in plasma


calcium and phosphate concentrations. Which of the

following graphs BEST represents these changes?


A.

Calcium metabolism is regulated through the actions of parathyroid hormone (PTH), calcitonin, and vitamin D3 (cholecalciferol). PTH is synthesized and secreted by the parathyroid glands, usually located in the central region of the thyroid gland near the trachea. Calcitonin is synthesized and secreted from parafollicular (or C) cells located in the thyroid gland. Cholcalciferol is synthesized
in an inactive form in the skin of animals from a

Phosphate

photolytic reaction involving UV light and a sterol


derivative. In order for this inactive hormone to be

0
B.

\fr

Time (hours)

activated, it must first be hydroxylated in the liver and then hydroxylated in the kidney. The activity of the hydroxylase enzyme in the kidney is regulated and increased by PTH.

PTH added

Bone is composed of an organic matrix consisting of collagen fibers and a ground substance composed of extracellular fluid and proteoglycans. The collagen fibers help to give bone its great tensile strength, while the ground substance helps to control the deposition of calcium salts, like hydroxyapatite (CastPO^OH). The calcium salts help to provide for the great compressional
strength found in bone.
C.

\/
PTH added

Time (hours)

The collagen matrix and ground substance is laid down by bone cells called osteoblasts. The tissue which is

formed, called an osteoid, can entrap some of the osteoblasts. Entrapped osteoblasts are called osteocytes. As the bone grows, calcium salts precipitate on the collagen fibers. Bone is also undergoing resorption by
cells called osteoclasts.

0
D.

\J
PTH added

Time (hours)

PTH causes calcium absorption from bone by stimulatingosteoclastic activity and transiently inhibiting osteoblastic activity. At the level of the kidney, PTH increases calcium absorption in the distal tubules and collecting ducts and greatly decreases the reabsorption of
phosphate at the proximal tubules.

The activated form of cholecalciferol (l,25-(OH)2-D3) has target receptors in the intestine, bone, and kidney (to
name but a few tissues). In the intestine this hormone

promotes absorption of calcium and phosphate (following as the counterion), while in bone it promotes resorption of both calcium and phosphate. In the kidneys this hormone promotes the reabsorption of both calcium and phosphate
so that little is excreted in the urine.

\/

Time (hours)

PTH added

Copyright by The Berkeley Review

306

The Berkeley Review

Specializing in MCAT Preparation

Biology
48.

Calcium, PTH, Calcitonin, Calcitriol

Passage VD

Which of the following structures BEST represents vitamin D3?


A.

49. Calcium and phosphate absorption in the intestines is stimulated by an increase in:
I.
n. IE.

l,25(OH)2D3
Calcitonin PTH

A. B. C. D.

I only I and II only Ill only I and III only

50.
B.

In order for the secretion of calcitonin to have a

greater effect on the concentration of calcium ions in the plasma, which of the following statements must be true regarding osteoclast activity and plasma
calcium levels?

A. B.
C. D.

Increased osteoclast activity hypercalcemic plasma. Decreased osteoclast activity hypercalcemic plasma. Increased osteoclast activity hypocalcemic plasma. Decreased osteoclast activity hypocalcemic plasma.

coupled with a coupled with a


coupled with a coupled with a

C.

51. Hypophosphatemic rickets is an X-linked dominant


trait that leads to decreased levels of phosphate reabsorption in the kidneys. Which pedigree shown below BEST represents this disease?
A.
B.

II

O-r-1

B-r-O

II

6-r-a D

rm
D.

c. I O-r-B

D.

#-r-a

in

6-r-a i i-r-o

oTTV

rv

rv

Copyright by The Berkeley Review

307

The Berkeley Review

Specializing in MCAT Preparation

Biology

Calcium, PTH, Calcitonin, Calcitnol

Passage VD

52. Familial hypophosphatemia is BEST treated by diet modification and supplying adequate amounts of:
A. calcium.

B. C. D.

phosphate. calcium and phosphate. phosphate and l,25(OH)2D3.

53.

Hypoparathyroidism is BEST characterized by:


I. II. III. IV. A. B. C. D. increased osteoblast activity. increased osteoclast activity. increased neural excitability. increased plasma calcium concentrations. I only H and m only m only I and IV only

54. In the graph shown below, all of the following


statements concerning the relationship between
PTH, calcitonin, and calcium are true EXCEPT:

Total Ca2+ concentration inplasma


A.

B. C.

PTH is a hypercalcemic hormone. calcitonin is a hypocalcemic hormone. a positive linear relationship exists between
calcitonin secretion and the concentration of

D.

plasma calcium. a positive linear relationship exists between PTH secretion and the concentration of plasma
calcium.

Copyright by The Berkeley Review

308

The Berkeley Review Specializing in MCAT Preparation

Biology
Passage IX (Questions 55-61)

Erythroblastosis Fetalis

Passage IX

56. Which of the following statements is TRUE?

Erythroblastosis fetalis (EF) is also known as hemolytic disease of the newborn. Hemolysis is rupture of red blood cells, so in hemolytic disease, anemia is common, due to the lysis of red blood cells. In the EF condition, maternal antigens cross the placental barrier, attack proteins on the surface of the red blood cells of the fetus, and lyse the cells..A very specific set of conditions
must exist for this disease to occur.

A. B.
C.

The placenta allows passage of all blood


products from the mother to the fetus.

The Rh factor is not a component of the ABO blood group system.


The mother makes red blood cells for the fetus

D.

in the placenta. Terminated pregnancies have no effect on future development of EF.

The mother must be Rhesus (Rh) factor negative, the fetus must be Rh factor positive, and the mother's immune system must be sensitized to the Rh positive antigen through previous full-term pregnancy or abortion. The Rh factor antigen is transmitted as a dominant trait, so that only people who are homozygous recessive are Rh factor negative.

57. What preventive measure could pruieui subsequent vvnai picvcuuvc measuie uuuiu protect fetuses if an Rh-negative mother gave birth to an Rh-positive fetus?
A. Give the mother a blood transfusion with Rh-

Roughly 90% of the cases of EF result from sensitivities to the D antigen on the Rh factor. When Rh positive blood enters the circulation of an Rh negative mother, antibody formation against D may be induced. This exposure may be during an accidental infusion, during pregnancy, delivery, or during a miscarriage or abortion. During a first pregnancy, there is usually little exchange of fetal and maternal blood, except near the end of the pregnancy or during delivery.
This time frame does not allow for antibody formation followed by an attack on the fetal blood cells. The problem lies in subsequent pregnancies. Small amounts of antigen, even the amount in 1 mL of fetal blood entering the mother's circulation, promote rapid increases in her anti-D antibody titer. IgG is produced, and it can easily cross the placental barrier into the fetal blood supply.
Even in the ideal conditions for EF, sometimes the disease.

positive blood.
B. Give the fetus a blood transfusion with Rh-

negative blood.
C. Treat mother with a set of antibodies directed

D.

against the anti-Rh antibodies. Treat fetus with antibodies against the anti-Rh
antibodies.

58.

An infant with severe EF has jaundice, a yellow coloring due to excess bilirubin, a breakdown

product of heme. In what tissue or organ is heme degraded into bilirubin?


A.
B.

does not manifest, due to variable physiological


conditions.

Spleen
Bone marrow

C.
D.

Liver All of the above

59. An Rh-positive mother is pregnant with an Rhnegative fetus. Will the fetus develop EF? 55. A Rh-negative woman and a heterozygous Rhpositive man have one child together. The woman
has never been pregnant before. What is the
likelihood that the child will be born with EF?
A. B. C. D. 100% 75% 50% 0%

A. B.

No, there are no maternal antigens to the Rh factor antigens. Yes, the mother can still make antigens to the
Rh factor of the fetus.

C.
D.

No, the fetal antibodies protect its red blood


cells.

No, but fetal antibodies attack the maternal red


blood cells.

Copyright by The Berkeley Review

309

The Berkeley Review

Specializing in MCAT Preparation

Biology
60.

Erythroblastosis Fetalis

Passage IX

What variables could affect the severity of

hemolysis in an Rh-positive fetus whose mother is


Rh-negative?
I. Amount of blood transferred.

II. Sensitivity of mother to D antigen. m. Number of pregnancies.


A. B. C.
D.

I only I and II only II and m only


I, H, and HI

61.

Which of the following clinical signs could be consistent with a diagnosis of EF in a newborn?

I. High levels of bilirubin in the blood. II. Low levels of hemoglobin in the blood. HI. Increased levels of erythrocytes.

A. B. C.
D.

I only I and II only H and ffl only


I, II, and HI

Copyright by The Berkeley Review

310

The Berkeley Review Specializing in MCAT Preparation

Biology
Passage X (Questions 62-67)

Septic Shock

Passage X

64. A defect in which of the following cells would


inhibit the production of soluble antibodies?
A. Mast cells

Septic shock, a disease characterized by hemodynamic derangements and multi-organ malfunction, has generally been associated with a gram-negative infection. However, it is becomming increasingly clear that gram positive organisms are equally responsible for sepsis. Many of these gram positive ogranisms release molecules known as superantigens. These superantigens can induce T cell proliferation without regard to the antigenic specificity of
the cell.

B.
C.

Cytotoxic T cells
Plasma cells

D.

Erythrocytes

65. In an experiment, a sepsis patient is treated with A common sign of septic shock is widespread activation of coagulation leading to widespread intravascular clotting. Microbial products activate Factor Xn, a molecule involved in blood clotting. Activation of this factor initiates the intrinsic coagulation pathway and also the bradykinin pathway. Bradykinin is a potent vasodilator and also increases the permeability of vascular
endothelial cells.

anti-factor XII antibodies. After treatment, one

would expect to see a: A. B. C. D. total lack of intravascular clotting. rise in the level of intravascular clotting. rise in the patient's blood pressure. decrease in the patient's blood pressure.

Cytokines, such as interleukin-1 and tumor necrosis


factor, activate tissue factor HI. This factor is found on the

outer membrane of macrophage and endothelial cells, and stimulates the extrinsic coagulatory pathway.

66.

Bradykinin acts to increase the radius of a given

blood vessel by a factor of 2. The flow of blood through the vessel should increase by a factor of:
A. B. C. D. 2. 4. 8. 16.

62. A Gram-positive cell differs from a Gram-negative


cell in that a Gram-positive cell:
A.
B.

does not have an outer membrane on its cell


wall.

67.

The activation of bradykinin most likely results in:

does have an outer membrane on its cell wall.

C.

D.

contains a thin peptidoglycan layer adjacent to the plasma membrane. contains no peptidoglycan layer adjacent to the plasma membrane.

A.
B. C. D.

hypotension in the patient.


cytokine release. macrophage activation. stimulation of the extrinsic coagulation pathway.

63. Macrophagesdestroy microorganisms through:


A. B.
C. D.

exocytosis, secreting toxins which eventually


form membrane attack complexes.

xxocytosis, surrounding the foreign particle with a lipid bilayer which protects the host
organism.

endocytosis, engulfing foreign particles which eventually will fuse with a lysosome. endocytosis, engulfing foreign particles which eventually will fuse with a peroxisome.

Copyright by The Berkeley Review

311

The Berkeley Review Specializing in MCAT Preparation

Biology
Passage XI (Questions 68-75)

Calcitonin and Osteoporosis

Passage XI

71. Which two amino acids form a disulfide bridge?


A. B. Methionine-threonine Methionine-methionine

Calcitonin (CT) is a polypeptide hormone secreted by the parafollicular cells of the thyroid gland in mammals. 32 amino acids make up the hormone, and a disulfide bridge links residues 1 and 7. The entire CT molecule and the disulfide bridge are required for full biological
activity.

C. D.

Cysteine-cysteine Cystine-cystine

CT works as an antagonist of parathyroid hormone (PTH). In response to small increases in plasma calcium, CT is released and acts on the kidney and bone to
decrease the calcium level. In the bone matrix, osteoblasts

72. Which of the following choices is a probable


explanation why salmon CT is 30 times more active
in humans than human CT?

synthesize bone, and osteoclasts catabolize bone. The main effects of CT are (1) inhibition of osteoclasts and (2) a transient increase in urinary calcium and phosphate.

I. U.

Salmon CT is more resistant to degradation by human enzymes. Salmon CT attaches more strongly to the DNA
of the osteoclasts.

Calcitonin is used pharmacologically as a treatment for osteoporosis. Salmon CT is commonly used. Although salmon CT differs markedly from human CT, it is about 30 times more potent when used in non-allergic humans to treat osteoporosis. Treatment with CT is not without its own side effects. CT treatment for osteoporosis increases plasma PTH and requires simultaneous calcium supplementation to avoid hyperparathyroidism.

m. Salmon CT attaches more tightly to the CT


receptor.

A. B. C.
D.

I only I and m only H and ffl only


I, H, and ffl

68. Osteoporosis is a disorder of bone characterized by a


decrease in bone quantity, most common in women following menopause and in elderly men and women. Which of the following conditions would

73. If a person begins a calcium supplement regimen


and doubles calcium intake, what would be the response in CT secretion?
A. B. CT secretion increases. CT secretion decreases.

lead to the GREATEST decrease in bone quantity? A. B.


C. D.

Decreased osteoblast activity, decreased


osteoclast activity.

Increased osteoblast activity, decreased


osteoclast activity. Decreased osteoblast activity, increased osteoclast activity. Increased osteoblast activity, increased osteoclast activity.

C. D.

CT secretion remains unchanged. CT secretion halts completely.

74. How is salmon CT, a polypeptide, administered?


A. B. C. D. Ingestion to avoid allergic reaction. Injection to avoid hydrolysis. Ingestion to avoid hydrolysis. Injection to avoid allergic reaction.

69.

What is the most abundant mineral in the human

body?

A.
B. C. D.

B12
Water Calcium Zinc

75. What are the symptoms of an allergic reaction to a


foreign protein in the bloodstream? I. Flushing or reddening of the skin. H. Skin welts (hives). m. Difficulty breathing. A. B. C.
D.

70.

Based on the passage, what is the role of PTH? A. B. C. D. PTH increases plasma calcium. PTH decreases plasma calcium. PTH increases urinary calcium. PTH increases urinary phosphate.
312

I only I and II only ffl only


I, H, and ffl

Copyright by The Berkeley Review

The Berkeley Review Specializing in MCAT Preparation

Biology
Passage XII (Questions 76-83)

Vertebrate Immune System

Passage XII

77.

In order to protect vertebrates from infection, the

When certain types of antibodies bind to their target cells, they attract a series of proteins collectively
known as complement. These proteins can then form pores which allow small molecules to freely diffuse
across the plasma membrane. What effect would this have on a target cell?

immune system has evolved both an antibody-based anda cell-mediated response to foreign antigens. B cells, which originate and develop in hemopoietic tissues (bone marrow and fetal liver),are responsible for producing and secreting antibodies which bind to antigen particles. T
cells also originate in hemopoietic tissues but later

A.
B. C.

The cell would die due to its inability to


initiate action potentials.

migrate to and mature in the thymus during early development. CytotoxicT cells are mainly responsible for mounting a cell-mediated defense by directly causing the death of infected cells. While B cells can be activated by
the binding of extracellular antigen to special receptors on
the plasma membrane, T cells must come in direct contact
with infected cells in order to become activated.

The cell would become hyperpolarized. The cell would lyse due to an upset water
balance.

D.

The cell would shrink due to an upset water


balance.

How the immune system differentiates between self

and foreign antigens has been the topic of much study. Immunologists in the first half of this century proposed
two main theories:
78.

Which of the following statements would be

Theory 1

consistent with BOTH Theory 1 and Theory 2?


I. A foreign cell line is injected into a mouse embryo. Further injections of the cell line into
the adult mouse do not elicit an immune
response.

Vertebrates inherit genes that encode receptors (present on the surface of B and T cells) that are capable of binding only foreign antigens. The immune system doesn't react against host tissues because it genetically lacks the receptors which bind self antigens.
Theory 2

The immune system is inherently capable of responding to both self and foreign antigens, but it becomes "tolerant" to self antigens during early development. Since foreign antigens aren't present during embryonic stages, the immune system does not develop a
tolerance to them.

n. Transplantation of organs between monozygotic twins does not result in organ rejection by the immune system. in. Cells transplanted between mice which are genetically identical (i.e., from the same inbred strain) are tolerated by the new host's immune
system.

A. B. C.
D.

I only II only II and III only


I, II, and ffl

76.

According to the passage, removal of the thymus from an adult human would most likely result in: A. B.
C.

drastically decreased antibody-mediated immune response. drastically decreased cell-mediated immune


response.

79.

D.

drastically decreased antibody and cellmediated immune response. little or no change in the effectiveness of either type of immune response.

Tolerance to self antigens breaks down in the human autoimmune disease myasthenia gravis, resulting in the production of antibodies against the patient's skeletal muscle acetylcholine receptors. Which of the following is a likely symptom of this disease?
A. B. C.
D.

Irregularities in heart contraction. Weakness and difficulty breathing. Paralysis of the gastrointestinal tract.
Dementia.

Copyright by The Berkeley Review

313

The Berkeley Review Specializing in MCAT Preparation

Biology
80.

Vertebrate Immune System

Passage xn

Recently activated B and T cells are examined via electron microscopy. Which of the following would
be the most likely observations?

83. Virally infected cells are usually killed by cytotoxic


T cells. The T cells can most likely target:

A.
B.
C.

The B cells have a greater number of


mitochondria than the T cells.

The B cells have considerably more rough

A. B. C. D.

most types of cells in the body. only cells lining the blood vessels. epithelial cells only. other blood cells only.

endoplasmic reticulum than the T cells.


Both the T and B cells lack nuclei.

D.

No differences between the two types of cells


are revealed at the level of electron

microscopy.

81.

A sample of B cells is removed from an adult mouse. A highly radioactive antigen X is added to the B cells, killing the few that bind strongly (<0.01%). The remaining B cells are injected into mice whose own B cells were destroyed by
irradiation. These mice can now make no antibodies

to antigen X but do respond to other antigens. Which of the following can most likely be concluded? A.
B. C. D.

Each B cell is predetermined to bind a specific antigen. B cells can recognize new antigens and "learn"
to bind to them.

T cells must interfere with B cell binding. Antigen X is normally non-immunogenic.

82.

Secreted antibodies often act collectively to bind to large target antigens and cross-link them into insoluble masses which are easily phagocytosed. Which of the following would NOT be consistent with this phenomenon? A. Antibodies each have two identical binding
sites.

B. C. D.

Antigens can be bound by only one antibody at


a time.

Antibody binding sites are connected by a flexible "hinge" region. Large antigens contain multiple antibodybinding sites.

Copyright by The Berkeley Review

314

The Berkeley Review Specializing in MCAT Preparation

Biology
Passage XUI (Questions 84-90)

Insulin Receptor
86.

Passage XHI

The response to glucagon is believed to involve the

The human insulin receptor is an integral transmembrane protein located in the plasma membrane of many cells throughout the body. Studies have shown that a functional receptor consists of two alpha and two beta subunits which are connected using disulfide bonding. The receptor consists of several domains, including one which binds insulin, a transmembrane binding region, and an intracellular tyrosine kinase phosphokinase (TPK) segment.
How does ligand binding create intracellular changes which bring about the effects of the insulin hormone? There is experimental evidence which suggests that the human insulin receptor is a insulin activated tyrosine kinase. While many details remain unclear, the binding of the hormone leads to both an autophosphorylation of two tyrosine residues on the TPK and a phosphorylation of cytoplasmic constituents. In addition, the binding of insulin to its receptor is believed to activate phospholipase C, which converts glycan phosphatidyl inositol phosphate to glycan inositol phosphate (GIP) and 2-diacylglycerol. GIP is known to mimic certain insulin responses and 2-diacylglycerol, in the presence of elevated calcium levels, is known to activate protein kinase C. Protein kinase C is known to influence the activity of other enzymes and metabolic pathways.

production of cAMP as a secondary messenger.


Which of the following graphs BEST describes the sequence of events associated with the binding of
glucagon to its receptor? [Note: CA = cAMP; PP =

protein phosphorylation; CR = cellular response.]


A.

a l
CO

Time B.

>
<

Time C.

84.

Which of the following procedures is used on cells to isolate the entire insulin receptor for experimental
studies?
A. Acidic wash.

52

Time D.

B. C. D.

Oxidizing wash. Detergent wash. Reducing wash.

>

X
ca

85. Through genetic manipulations, the insulin binding


region of the insulin receptor (IR) was fused with the kinase region of the epidermal growth factor (EGF) receptor (also a ligand activated kinase). The resulting hybrid receptor was transfected into a cell
and the addition of insulin resulted in a functional

<

Time

EGF kinase. From this experiment, it can be


concluded that IR and EGF have similar:
87.

In a type I, insulin dependent diabetes, the individual afflicted with the disease most likely has:

A. B. C. D.

ligand binding regions. signal transduction mechanisms. secondary messenger cascades. effects on carbohydrate, fat, and protein
metabolism.

A. B. C. D.

a mutated insulin binding site. a mutated membrane anchoring site. a normal insulin receptor. an inactive phospholipase C.

Copyright by The Berkeley Review

315

The Berkeley Review Specializing in MCAT Preparation

Biology
of GIP?

Insulin Receptor

Passage xm

Which of the following is consistentwith the actions


A. Inhibition of hexokinase.

B. C. D.

Inhibition of phosphofructokinase. Stimulation of glycogen breakdown. Stimulation of pyruvate dehydrogenase.

89. In cell #1, the addition of bivalent anti-insulin

receptor antibodies induces a response without the


addition of insulin. In cell #2, addition of a fragment

anti-receptor antibody (monovalent) induces no

response without insulin. To cell #2, antibodies against the monovalent anti-receptor antibodies are
added. This will result in:

A.

no response.

B. C. D.

decreased glucose uptake . increased CO2 production. increased insulin binding.

90. To disrupt the dimer in order to study its components, a researcher will most likely use:
A. B. C.
D.

oxidizing conditions. reducing conditions. high speed centrifugation.


high pH.

Copyright by The Berkeley Review

316

The Berkeley Review

Specializing in MCAT Preparation

Biology
Passage XIV (Questions 91-95)

Vitamin D3, PHI, &t Calcitonin

Passage XIV

In all vertebrates the maintenance of calcium (Ca2),

92. The primary precursor for l,25-(OH)2-D3 is located in which one of the following organs?
A. Liver

phosphate (P043e), and magnesium (Mg2) homeostasis


is governed primarily by the vitamin D3 derivative 1,25dihydroxycholecalciferol (l,25-(OH)2-D3), calcitonin
(CT), and parathyroid hormone (PTH).

B.

Intestine

C. Integument D. Kidney

Humans obtain vitamin D3 either by ingestion or through synthetic mechanisms that are initiated by cells in the epidermis of the skin. Vitamin D3 is hydroxylated first in the liver and then is transported to the kidney where the
enzyme 1-hydroxylase converts it to the active form of

93. Which of the following categories represent the three


hormones described in the passage?
I. Amine

II.
III. A. B. C.
D.

Polypeptide
Steroid

the hormone, l,25-(OH)2-D3. The activity of lahydroxylase is enhanced during hypocalcemia and hypophosphatemia. l,25-(OH)2-D3 passes into a target cell and then into the nucleus where it complexes with a receptor protein that has a DNA binding site. Gene transcription can either be enhanced or suppressed. Enhancement leads to the synthesis of calcium binding proteins (calbindins) in intestinal villi and crypt cells.

II only I and III only II and III only III only

Calbindin promotes the uptake of Ca2 from the


intestinal lumen. A similar mechanism allows for the

94. Which of the following conditions best describes a patient with a deficiency in PTH secretion?
I. II. III.
A. B. C. D.

absorption of phosphate and magnesium from the


intestinal lumen.

PTH is released from the chief cells of the parathyroid glands while CT is released from the parafollicular cells (or C cells) of the thyroid gland. Both hormones have
genes encoded in the short arm of human chromosome 11 and are synthesized as a preprohormone from different primary RNA transcripts.
PTH acts on bone and on the distal tubule of the

Hypocalcemia Hypercalcemia Hyperphosphatemia


I only III only II and III only I and III only

kidney to promote Ca2 reabsorption and inhibits the reabsorption of P043e in the proximal tubule of the
kidney. PTH also stimulates the synthesis of l,25-(OH)2D3 in the kidney. CT acts to lower both plasma calcium and plasma phosphate levels.

95. Which of the following curves BEST represents the relationship between blood levels of parathyroid
hormone (PTH) and calcitonin (CT) in terms of total

plasma Ca2 concentration levels?


A. B.

low

high

low

high

Total Plasma [Ca2+]


c.
D.

Total Plasma [Ca2+]

91. Ultraviolet light stimulates vitamin D3 production in which of the following cell types? A. B. C. D. Hepatocytes Keratinocytes Leukocytes Lymphocytes
6
low high
low high

Total Plasma [Ca2+]

Total Plasma [Ca2+]

Copyright by The Berkeley Review

317

The Berkeley Review

Specializing in MCAT Preparation

Biology
Passage XV (Questions 96-100)

Sepsis Syndrome

Passage XV

96. Which is the term for the process of white blood cells squeezing through blood vessels?
A. Mitosis

The normal response to an infection is a complex

system of activations and inactivations of the members of


the immune system. Neutrophils are the first to respond to an infection by squeezing through blood vessel walls and destroying pathogens directly by the release of toxic oxygen products, such as superoxide, hydrogen peroxide, and nitric oxide, and by the release of proteolytic enzymes. These white blood cells follow chemical attractants given off by the pathogen. Monocytes, another type of white blood cell, arrive next. They may be
transformed into macrophages in the tissue and engulf

B. C. D.

Diapedesis Phagocytosis Endocytosis

97. What purpose do proteolytic enzymes serve in


cellular defense?

pathogens and cellular debris. Monocytes and macrophages release cytokines, a powerful class of
chemicals that modulate responses of different members of the immune system. T cells and B cells are attracted and activated by cytokines at the infection site. Another set of cytokines are released to signal "stop" when the

A. B. C. D.

Inactivate bacterial lipid signals. Digest bacterial cell surface carbohydrates. Destroy bacterial DNA. Hydrolyze bacterial proteins.

pathogens are neutralized. This is the normal progression


of events when the body attacks an invading bacteria.
However, in the U.S. each year, about 175,000 people die from sepsis syndrome, in which the sepsis process is actually amplified rather than terminated. The cytokines for signal termination are not released correctly, leading to an imbalance with the amplifying cytokines. This attracts all the white blood cell participants to the area, which release more activating cytokines. Some of the substances that are released lead to increased permeability
of the blood vessels. In normal situations, this allows

98.

Which cell products are used in reactions to destroy pathogens?


A. Chemical attractants.

free radical

B. C. D.

Proteolytic enzymes. Cytokines. Toxic oxygen products.

WBCs to more easily enter the infected tissue. In excess, this leads to blood cell leakage and damage, and

positively feeds back to attract more WBCs. Ultimately, if this process is unchecked, blood vessels will deteriorate, leading to tissue death, organ death, or even patient death.

99.

Researchers are attacking sepsis syndrome from many perspectives. In one approach, synthetic receptors for tumor necrosis factor (TNF), a cytokine, are injected to bind to TNF and halt the cytokine cascade. What assumption does this model
make?

A.

B.
C.
D.

TNF halts the abnormal cytokine cascade of sepsis syndrome. TNF receptors enter cells by endocytosis and
bind intracellular TNF.

TNF is mainly present in the general


circulation.

TNF moves from cell to cell only through gap junctions.

100. What is the most abundant type of white blood cell?


A. B. C. D. Neutrophils Erythrocytes Leukocytes Macrophages

Copyright by The Berkeley Review

318

The Berkeley Review

Specializing in MCAT Preparation

Biology
Passage 1(1 - 7)
l.

Immunology St Endocrinology

Section V Answers

Antibody Structure

the upward movement (away from the lungs) ofmucous and particulate matter so itcan be expelled form the system. Normal body temperature is about 37 C. Many organisms (pathogens) simply do not grow well at this temperature. The only component which is not part of the nonspecific immune system is the placental transfer ofantibody. This is a passive process which occurs from mother to fetus via the placenta. The fetus acquires antibodies synthesized by
the mother. The correct choice is C.

called cilia that move particulate matter over the epithelial surface. In the respiratory tract the mucous membranes of the trachea and bronchi are lined with ciliated epithelium that beat towards the opening of the mouth. This allows for

from birth. In the second paragraph of the passage three examples ofnonspecific immunity were given. The skin consists ofan outer layer of dead keratinized cells and an internal layer of live epidermal cells. All of the body cavities and organs that open up to the skin surface are lined with mucous membranes. The epithelial lining ofthese structures contain goblet cells which secrete mucous and the epithelial cells themselves have hair-like projections

C is correct, placental transfer of antibody. Innate (nonspecific) immunity is what the individual has developed

C is correct, active immunity which is artificially acquired. In the first paragraph of the passage we see that acquired immunity is specific immunity. In the third paragraph we see that vaccinations and a recovery from a disease like the measles are examples ofspecific immunity. A vaccination is simply an immunization against a pathogen. It is something that is artificially acquired. The individual who receives the vaccination then begins to actively produce his or her own immunological agents against the attenuated (weakened) pathogen. Even though the pathogen has been made innocuous, itstill retains its antigenicity (i.e., its epitopes). Recovery from a disease like the example ofpassive immunity which is artificially acquired. Transfer ofIgG from the mother to the fetus through the
3.

measles is an example of active immunity which is naturally acquired. Antitoxin administration would be an

placental membrane would be an example ofpassive immunity which is naturally acquired. The correct choice is C.

Ais correct, covalent bonding. In the 7th paragraph ofthe passage itsays that antibody-antigen complexes can be dissociated by high salt concentrations or either high orlow pH. This must mean that the binding forces between an antibody and an antigen are relatively weak. All of the bonds and/or bonding forces listed in the answer choices arc

weak except for the covalent bond. We would not expect a covalent bond to be broken by a change in

(physiological) pH or by treatment with a high salt concentration. The correct choice is A.

Figure 1ofthe passage. This antibody can bind two epitopes ofthe same specificity (unideterminant). The portion of the antibody that binds the epitope resides near the N-terminus of both the Vh and Vl regions of the Fab. The epitope does not just bind the Vh regions alone. Wc can eliminate choice B. The average size of antigenic determinants is roughly equivalent to about 6 amino acids. Therefore, the size of the paratope must be similar in dimensions. If this were not the case, then binding between the epitope and paratope would be unfavorable. Remember, hydrophobic forces, Van der Waal forces, electrostatic forces, and hydrogen bonding all require
relatively close proximities between participating parties before they can be of any use. The correctchoice isD.
5.

an understanding of a paratope. In the 7th paragraph of the passage we find that the paratope is associated with the antibody. This allows us to immediately eliminate choices A and C. Look at the structure of the IgG molecule in

Dis correct, antibody which has similar dimensions as the antigenic determinant. This question is simply asking for

domains. It is a bivalent structure. As it stands, this antibody could participate in cross-linking with a
unideterminant, multivalentantigen (see the diagram below).

C is correct, antigens that are not cross-linked, because each Fab fragment is monovalent. In order to answer this question, we must consider the structure of the IgG molecule in Figure 1of the passage. Note that there are two Fab

However, after addition of the protease papain, the antibody is cleaved into an Fc segment and two Fab segments. The important point here is that the Fab segments are now independent ofeach other. They can now independently
bind to the antigen's epitope (see above). Cross-linking will not result.

In choice A the first part of the answer (cross-linking) is wrong, but the second part is correct. This makes the whole

answer choice incorrect. In choice B the answer is correct, but for treatment of IgG with pepsin, not papain. In
isC.

choice Dtwo monovalent fragments are produced, but they cannot participate in cross-linking. The correct choice

Copyright by The Berkeley Review

319

The Berkeley Review

Specializing in MCAT Preparation

Biology

Immunology St Endocrinology

Section V Answers

Fab

epitope

-
Before IgG is treated with papain.
After IgG is treated with papain.

C is correct, IgM. In the 6th paragraph of the passage we find that IgG is the only antibody to cross the placental
membrane. Even though this antibody enters into fetal circulation and can confer fetal immunity to certain diseases,

it is IgM which is the first antibody produced during an immune response. Also, it is the predominant antibody produced by the fetus during fetal development. The correct choice is C. B is correct, IgA. In the 6th paragraph of the passage we see that IgA is primarily found in external secretions such
as tears, saliva, colostrum, and milk. This question is asking us to recall a little information about lysozyme. In vertebrates, this enzyme is widely distributed in a variety of cells and secretions. The connection that needed to be made was that both lysozyme and IgA are found in bodily secretions. The actions of the other immunoglobulins are outlined in the passage. The correct choice is B.

Passage II (8- 14)


8.

Acquired Immunity

C is correct, no, receiving antibodies confers transient immunity. Passive immunity means receiving antibodies from another organism. The recipient does not make antibodies in this situation. Thus, choices B and D are incorrect. Injected foreign antibodies eventually degrade and are not replaced, since an immune response was not triggered in the recipient. The effect is not permanent. It is transient. Eliminatechoice A. The correct choice is C.
A is correct, maternal antibodies provided immunity to measles in this situation. The newborn has an immature

9.

immune system. She is protected by passive immunity via maternal antibodies she received in the uterus and in breastmilk, if she is fed that way. She cannot respond quickly to primary exposures of viruses until after about 1 month of age. Choice B is incorrect. Although antibodies may be present in saliva in an extremely small amount, this is not how passive immunity is acquired. Choice C is incorrect. Paternalantibodies stay with the father. Choice
D is incorrect. The correct choice is A.

10.

C is correct, both IgG and IgM remain at high levels following a "booster shot". We are looking for the FALSE statement. All of the questions can be answered by consulting the graph. A "booster shot" means receiving a
vaccination to boost the antibody effects of an earlier vaccination. On the second (and further) exposures, we can see

in the figure that IgG provides the primary response. Choice A is TRUE. Before a person is exposed to an antigen, there are no antibodies for that particular antigen. Choice B is TRUE. The primary response to an antigen is a rise in

IgM. This, as you can see in the figure, precedes a rise in IgG. Choice D is TRUE. Both IgG and IgM fall off in their levels following each expose. They do not remain at high levels. Choice C is FALSE. Since we are looking for the
FALSE answer, choose C. The correct choice is C.

11.

B is correct, artificially acquired passive immunity. Since the person receives antibodies from someone else who is immune to the particular antigen, then it must be passive. Choices A and C are incorrect. Passive natural immunity comes from a mother-baby relationship. This person is an adult, so it must be artificial. Choice D is incorrect. The
correct choice is B.

12.

A is correct, artificially acquired active immunity. A vaccine provides artificially acquired immunity. Choices C and D are incorrect. Active immunity is created by the immune response of the body to the antigens in the vaccine.

Copyright by The Berkeley Review

320

The Berkeley Review

Specializing in MCAT Preparation

Biology
The correct choice is A.

Immunology St Endocrinology

Section V Answers

When a second exposure occurs, the body has antibodies and lymphocytes ready to attack. Choice B is incorrect.

13.

D is correct, y-globulin. A gel electrophoresis retards larger molecules, while allowing smallerones to migrate more quickly. The larger molecules are the slowest. The y-globulin moved the least in a given time, so they must be the largest. For your information, the y-globulin fraction contains antibodies which can be used to confer passive
immunity. The correct choice is D.

14.

A is correct, I only. Breastmilk, especially colostrum, the breastmilk of several days immediately following birth, is important for all babies. This child could benefit from the antibody protection conferred by another mother's breastmilk. Choice I is correct. Seclusion would protect the baby from exposure to antigens, but it still would not make more antibodies. When the baby's immune system matures, it needs exposure to antigens in order to make antibodies. Eliminate choice II. Immediate vaccination would not produce considerably greaterantibodies, since the
baby's immune system cannot function really effectively at birth. Choice III is incorrect. The correct choice is A.

Passage HI (15- 21)

Major Histocompatibility Complex (MHC)

15.

D is correct, both hydrophilic and hydrophobic regions. The first sentence of the passage states that the proteins
which are encoded from the class I and class II MHC genes are expressed on the surface of a cell. This should strike a nerve in your brain. In particular, it should tell you that proteins expressed on the surface of the membrane must

have both a hydrophilic region and a hydrophobic region. The hydrophilic region must exist because, in this case, most of the protein is sitting in the extracellular medium, which will most likely be polar due to extracellular fluid. The hydrophobic region is necessary because the protein is anchored to thecell by the plasma membrane of thecell, which is of course lipid in nature. Therefore, the protein should contain both a hydrophobic and hydrophilic region. Knowing this information, choices A and B are merely incomplete. Choice C is redundant because hydroapathetic is
similar to hydrophobicity. The correct choice is D.
16.

D is correct, host cellular in origin. This question is not really a trick question, it simply requires a little thinking.
There are about one halfof a million to one million class I MHC proteins. There is no way you should know that number, but you should realize that there are a lot of these proteins on the surface. Furthermore, the passage states that the peptides found with class I MHC are invariably the cell's own proteins. When the cell is infected with virus, the viral proteins/peptides will be presented to stimulate an immune response. But this does not change the fact the
most class I MHC molecules will still have theirown proteins being presented on the cell surface. Choices B and C do not make any sense because thosecells are involved with the recognition/destruction of cells infected with virus. There is no evidence to suspect those cells will have their peptides presented. The correct choice is D.

17.

B is correct, signal peptide sequence. This question requires a solid knowledge of cellular action. It is stated in the passage thatclass II MHC molecules have been shown to assemble in the endoplasmic reticulum. This requires that the proteins get to the endoplasmic reticulum (ER). Proteins that are destined for the ER have a special sequence
called the signal peptide sequence which causes the ribosome they are being translated on to bind to the ER. The subsequent translation of the protein occurs, and the entire protein then finds itself within the ER. Consider the other

answers. Choice A is incorrect because a Shine-Dalgarno sequence is a segment of RNA found in prokaryotes that is
recognized by ribosomes as a place of binding. Therefore, choice A is incorrect. Choices C and D are incorrect because the question asks about proteins, not DNA sequences. The correct choice is B.

18.

C is correct, endocytosis. Recall that the ingestion of material by the invagination of the plasma membrane is called endocytosis. This is exactly the process that occurs for the formation of these endosomes. This is a very straightforward question. Consider the other answers. Choice A is a non-sensical answer and can automatically be eliminated. Choice B is the opposite of endocytosis, and would call for the expulsion of cellular material through the
plasma membrane. Transcytosis is the transport of material by a vesicle from one side of the cell to another. This
usually occurs in epithelial cells. The correct choice is C.

19.

B is correct, structurally similar to that of the class II MHC. The passage states that the CLIP part of the invariant
chain is actively removed by binding to the DM molecule. The clip used to be bound to the class II MHC molecule.

Based on this information, one could logically assume that the DM and class II MHC molecule are structurally similar, if they bind the same molecule. Considering the other answers, there is no evidence for any of the statements. The DM molecule is binding CLIP, which does not mean they are structurally similar. It just means they
Copyright by The Berkeley Review
321

The Berkeley Review Specializing in MCAT Preparation

Biology

Immunology St Endocrinology

Section V Answers

form a bond. Choice D is incorrect because class I MHC is not involved with DM. Finally, DM is not likely similar to the invariant chain. Remember, CLIP is merely a part of the invariant chain, and if DM is not like CLIP, there is
no reason to believe it will be like the invariant chain. The correct choice is B.

20.

D is correct, II and III only. A given class I MHC molecule will bind to a variety of peptides. However, the cleft where the peptides bind does not change for a given class I molecule. In other words, the amino acids that make up the cleft are invariant. The peptides that bind to the cleft can only do so if certain amino acids in their chain are able to bind to the amino acids which make up the cleft. That is, amino acids at certain positions in the peptides must be

highly conserved if a large variety of peptides arc able to bind to a class I molecule which does not change its amino acid composition. Therefore, choice III is correct. We find that choice II is correct based on the following logic: looking at Figure 1 and Figure 2, we can see that in class I molecules, the peptides are bound in the cleft by their
ends. Therefore, the peptides can only be so long, and as stated in the passage, abouteight or nine amino acids long. However, with class II, the binding occurs in the middle, and therefore we will see more of a variety of lengths in the

peptides which bind. The peptides which bind to class II molecules are not constricted to binding to the ends, so they
do not have to be of a certain length. The correct choice is D.

21.

C is correct, production of a transcription factor which increases the transcription rate of the MHC gene early in an
infection. If there was a transcription factor that promoted the transcription of the MHC gene early in infection, this

would promote an immune response against the pathogen. This isclearly not a defense mechanism for the pathogen.
All of theother answers result in a depression of the immune response, and therefore can be categorized as a defense mechanism used by a pathogen to disrupt the antigen processing system. The correct choice is C.

Passage IV (22 - 27)


22.

IgA Antibody Experiment

D is correct, I, II, and III. Any body surface that can contact the outside environment is lined with epithelial tissue. The mouth, is, of course, lined with epithelium. Choice I is correct. The entire GI tract is essentially "outside the

body." Itiscompletely lined with epithelial tissue. Choice IIiscorrect. The urethra also has contact with the external
environment. It, too is lined with epithelium. Choice III is correct. The correct choice is D.

23.

B is correct, IL-6" mice produce lower levels of IgA than IL-6+ mice. Read the bar graph. The IL-6" mice have almost no IgA compared to the IL-6+ group. Choice A is incorrect. Choice C is incorrect. The response to
ovalbumin was almost nil in the IL-6" mice. Choice D is incorrect. The correct choice is B.

24.

A is correct, amino acids. Immunoglobulins areproteins, made of amino acids. Fatty acids are partof many types of

lipid molecules. Choice B is incorrect. Sphingomyelins are special fats that are found in nervous system tissue.
Choice C is incorrect. Phospholipids are partof cell membranes. Choice D is incorrect. The correct choice is A.

25.

B is correct, restoring IL-6 in the IL-6" mice improved IgA production. The virus carrier alone did not change the genetic makeup of either strain of mice. Choice Ais incorrect. The IL-6+ mice simply responded to a viral infection by increasing IgA slightly. Choice D is incorrect. The IL-6" mice did not respond at all to the virus carrier. They
were hyposensitive. Choice C is incorrect. The correct choice is B.

26.

D is correct, cytokines are local hormones and often act without passing through the entire circulatory system.

Cytokines are special "local hormones" that do not always pass through the entire blood supply. Measurements made
on blood samples may not reflect what is happening at the tissue level. Choice A is incorrect. Hormone concentrations can be measured in blood samples by radioimmunoassay. Choice B is incorrect. IL-6 can and does leave the cell that produces it, like other cytokines. How could they interact with IgA if they never left the cell and IgA is present in secretions? Choice C is incorrect. The correct choice is D.

27.

A is correct, gene therapy. You may have read about the first gene therapy trials in humans during the last year or so. This was to alter genes in people with inherited, disease-causing genetic defects. Specific viruses carried the new

genetic material. Hopefully, these viruses will infect the host's DNA with a good copy of their defective gene. Vaccination is just plain, old immunization with a modified form of a pathogen. Choice B is incorrect. Genetic
characteristics in an individual. Choice D is incorrect. The correct choice is A.

immunization is a fake answer, and Choice C is incorrect. Pleiotropy having a gene that affects many different

Copyright by The Berkeley Review

322

The Berkeley Review

Specializing in MCAT Preparation

Biology
Passage V (28 - 33)
28.

Immunology St Endocrinology

Section V Answers

Complement System

C is correct, solubilizc antigen-antibody complexes. This question just requires one to read the passage carefully. The passage states that persons who are complement deficient suffer from immune diseases in which antigenantibody complexes form precipitates in the blood vessels of skin, joints and brain. These precipitates can go on to cause tissue damage. One can assume from this statement that complement plays a role in solubilizing antigenantibody complexes because when the complement is not present, the precipitatesform. The correct choice is C. C is correct, provides a means of amplification, ultimately leading to many MACs. The key word in this question is the word cascade. One should be familiar with the idea of biological cascades. Each activated enzyme in the pathway cleaves many molecules of the next proenzyme in the chain. The activation of early complement components consists of an amplifying proteolytic cascade where each molecule activated at the beginning of the
sequence leads to the production of many membrane attackcomplexes. The correct choice is C.

29.

30.

B is correct, bone marrow. This answer is arrived at through one's knowledge of immune system development. The question is asking about what disease would affect the production of antibodies. This would clearly be the bone marrow. The bone marrow is the site of B-cell birth and maturation. B-cells are responsible for the production of antibodies. Therefore, a disease of the bone marrow would greatly affect B-cell maturation and thereby affect antibody production. The thymus is the site of T-cell maturation, while the spleen and the lymph nodes are what are called secondary lymphoid organs, where mature B-cells and T-cells "hang out" to react with foreign antigens. The
correct choice is B.

31.

D is correct, constant region of the antibody. We know that there exists interaction/activation between the CI

complement component and either IgG or IgM. The question is asking what is the most likely means by which CI is
activated. CI does not change depending on the microorganism. In other words, we do not have a different CI for

different microorganisms. However, we do have different IgGs and IgMs for different microorganisms. The difference lies in the variable region of the antibody. That variable region binds to the antigenic determinant of the microorganism. The antibodies also have a constant region, which does not change (except for class switching) or depend on the particular microorganism. It most likely that the CI, which remains constant, will recognize and be activated by the constant region of the antibody. If this were not the case, CI would itself have to change to
recognize the variable region of the antibody. The correct choice is D.
32.

C is correct, confine complement activation to the cell surface where it began. The early components and C3 are proenzymes that are activated sequentially by limited proteolytic cleavage. When each proenzyme in the sequence is cleaved, it is activated to generate aserine protease which cleaves the next protein in the sequence. Many cleavages liberate a small peptide fragment and expose a membrane-binding site on the larger fragment. The larger fragment can now bind tightly to the target cell membrane and carry out the next reaction in the sequence. The reaction
cascade is constrained to the cell surface where it began, and where we want it to be. The correct choice is C.

33.

B is correct, these molecules stimulate the secretion of histamine from basophils. The question tells us that an
increase is seen in the permeability of blood vessels. From the answers given, we know this is due to histamine secretion. This histamine that is secreted increases the permeability of local blood vessels which will allow white blood cells, antibodies, and more complement to enter the site of infection. Thequestion becomes what cell secretes histamine. The answer is that mast cells and basophils secrete histamine. Most are familiar with mast cells, but basophils are a type of white blood cell. They fall under the category of granulocytes and they secrete histamine to help mediate inflammatory responses. The other types of cells in the answers are simply not involved in the
secretion of histamine. The correct choice is B.

Passage VI (34 - 40)


34.

Myasthenia Gravis/Autoimmune Diseases

A is correct, an inhibitor of acetylcholinesterase. Since the receptors are blocked or not as numerous, an increase in acetylcholine (ACh) should remove some of the effects. One way to accomplish this is to give an inhibitor of
acetylcholinesterase so the ACh is not degraded as quickly. This would give higher concentrations of ACh at the

receptors. An immunostimulant would further complicate the problem by encouraging more antibody production. Choice B is incorrect. A paralytic agent would add to the muscle weakness. Choice C is incorrect. Decreasing the
amount of ACh would be a bad idea, so choice D is incorrect. The correct choice is A.

Copyright by The Berkeley Review

323

The Berkeley Review

Specializing in MCAT Preparation

Biology
35.

Immunology &? Endocrinology

Section V Answers

C is correct, III. The nerve is the top or presynaptic part, while the muscle is the bottom, or postsynaptic part. Choice I indicates a vesicle containing ACh. Choice II indicated vesicle fused with the membrane and releasing ACh into the synaptic cleft. Choice III is the acetylcholine receptor on the muscle cell. Choice IV is a receptor on
the nerve cell. The correct choice is C.

36.

A is correct, Type IT diabetes. Type II diabetes is not caused by an autoimmune agent. It is mostly related to insulin resistance. Choices B, C, and D are clearly autoimmune diseases from the passage. The correct choice is A.
C is correct, lethargy. The effect of stimulating the TSH receptor causes the thyroid to secrete more thyroid hormone. This increases metabolic rate and can lead to hyperactivity in severe cases. Weight loss is probably connected due to increased metabolic rate. The person is not lethargic, which means tired and sluggish. The correct
choice is C.

37.

38.

B is correct, insulin. The beta cells of the pancreas produce insulin. Lack of insulin is the defect in Type I diabetes. You could figure this out from the table if you forgot. The other hormones are produced by the pancreas, but not in
the beta cells. The correct choice is B.

39.

C is correct, I and II only. A virus can resemble the structure of a body molecule and lead to autoimmune attack. Choice I is correct. A virus can damage a cell so that cell components that do not usually travel in the blood are

placed in the blood. This could also lead to an autoimmune response. Choice II is correct. Choice III is the opposite
of choice I and is incorrect. The correct choice is C.

40.

B is correct, adrenal gland. The adrenal cortex secretes steroid hormones. The correct choice is B.

Passage VII (41 -46)


41.

Glucose, Glucagon, St Insulin

B is correct, increased levels of glucagon and decreased levels of insulin. It is important to remember that glucose is needed by the body as an energy source. Thus, in times of stress, when the body needs energy to deal with the stressor, glucose plasma levels must increase. Sympathetic stimulation of glucagon secretion will increase glycogen breakdown and increase glucose plasma levels. This will make glucose available for tissues to utilize as an energy source. The sympathetic system will inhibit insulin secretion. The correct choice is B. B is correct, increased levels of glucagon and decreased levels of insulin. The major function of glucagon secretion is to increase the amount of products in the plasma that can be utilized for energy production. During fasting, blood

42.

glucose levels are low, stimulating glucagon secretion. Glucagon secretion will increase the release of free fatty
acids, which can be converted to ketone bodies and utilized for energy. However, if insulin levels are low, as in diabetes, ketone utilization is low and ketone products increase in the plasma beyond normal levels leading to
ketosis and acidosis. The correct choice is B.

43.

A is correct, abnormal increases in insulinsecretion. Hypoglycemia, by its name, means "low blood glucose." Thus,
abnormal increases in insulin secretion will decrease levels of blood glucose. Decreases in insulin secretion and high

levels of plasma glucose are signs of hyperglycemia. Low levels of free fatty acids may indicate increases in insulin
secretion or decreases in glucagon secretion. The correct choice is A.

44.

A is correct, increased levels of insulin and glucagon. Increases in plasma amino acid concentrations will stimulate

insulin secretion to promote protein production in cells. Furthermore, glucagon secretion from the pancreas is also increased. Glucagon secretion is needed to prevent hypoglycemia after a heavy protein meal since amino acids also
stimulate insulin secretion. The correct choice is A.

45.

C is correct, access and storage of glucose in cells of the peripheral tissue. Insulin stimulates the storage of glucose and eases the entry of glucose into tissue. The tissue affected most readily appears to be muscle and connective tissue of the periphery. The brain does not need insulin for glucose uptake, as brain tissue is not affected by insulin. Insulin stimulates glucose uptake and storage by the liver. Insulin will decrease glucose loss in the kidney by decreasing plasma glucose levels and increasing uptake of glucose by the tubular cells of the kidney. The correct
choice is C.

Copyright by The Berkeley Review

324

The Berkeley Review

Specializing in MCAT Preparation

Biology
46.

Immunology St Endocrinology

Section V Answers

A is correct, extracellular glucose excess. The lack of insulin secretion will decrease the uptake of glucose by cells.
Even with extracellular concentrations of glucose rising, cells cannot utilize the glucose because it cannot enter the

cells. Therefore, tissues die of starvation, with high concentration of glucose located just outside the cell. The
correct choice is A.

Passage VIII (47 - 54)


47.

Calcium, PTH, Calcitonin, Si Calcitriol

B is correct, (see the graph below). In the 5th paragraph we read the following: "PTH causes calcium absorption from bone by stimulating osteoclastic activity and transiently inhibiting osteoblastic activity. At the level of the kidney, PTH increases calcium absorption in the distal tubules and collecting ducts and greatly decreases the
reabsorption of phosphate at the proximal tubules." This means that at the time PTH is administered we would expect to see an increase in plasma calcium levels and a decrease in plasma phosphate levels. Since PTH is a hormone and goes everywhere the blood goes, we would also expect to see these effects happen at about the same time, which is exactly what we observe in the graph for choice B.

'LT

Time (hours)

PTH added

We would not expect to see the levels of phosphate or calcium lag behind one another. This immediately allows us to eliminate choices C and D. We can eliminate choice A simply because we know (from the passage) that the levels
of plasma calcium will increase. The correct choice is B.

48.

B is correct, (see the reaction below). The answer to this question is found in the first paragraph of the passage. We
are told that vitamin D3 is also called cholecalciferol and that it was synthesized from a sterol derivative in a

photolytic reaction involving UV light. A sterol derivative is based on the ring structure of cholesterol. A photolytic reaction involves the breaking of bonds. Think of "lytic" as something being "lysed" or broken. If you compare the four structures which are given as answers, you will note that choices B, C, and D are rather similar in that they contain just three complete rings. Choice A contains four complete rings. One of the rings (called the "B" ring) in choice A has been open up by the breaking of a bond. In this case that bond was broken by the action of UV light. Since vitamin D3 is formed from a photolytic reaction, we can eliminate choiceA as a possible answer. [It turns out
that choice A is called 7-dehydrocholesterol and is the precursorto vitamin D3.]
H,C

Vitamin D3 (cholcalciferol)

We are left with choices B, C, and D. It was stated in the passage that vitamin D3 is the inactive form of the hormone. It was activated by first hydroxylating it in the liver and then by hydroxylating it in the kidney. In other words, the active form of vitamin D3 (called lcc,25-dihydroxycholecalciferol (or l,25-(OH)2-D3)) has two more
Copyright by The Berkeley Review

325

The Berkeley Review

Specializing in MCAT Preparation

Biology

Immunology & Endocrinology

Section V Answers

hydroxyl groups on it than the inactive form. Therefore, we look for that structure which has not had two hydroxylation events. Based on this analysis we can eliminate choices C and D. The correct choice is B.
49.

D is correct, I and HI only. In the second paragraph it was mentioned that in order to synthesize the activated form of vitamin D3 (l,25-(OH)2-D3), the hydroxylase enzyme in the kidneys must be activated (stimulated) by PTH. Once the activated hormone is synthesized, calcium and phosphate can be absorbed across the mucosa membrane of the intestines. The phosphate ion follows the calcium ion across the membrane so electrical neutrality is maintained. We can eliminate choice C because PTH by itself has no direct effect on either calcium or phosphate absorption in the intestines. We can also eliminate choice B because calcitonin does not allow for intestinal absorption of calcium or phosphate either. Even though choice A is a correct answer, it is not the BEST answer. In order to get the activated form of vitamin D, PTH is needed. Therefore, an increase in both PTH and subsequently an increase in l,25-(OH)2-D3 is needed for absorption in the intestine. The correct choice is D.

50.

A is correct, increased osteoclast activity coupled with a hypercalcemic plasma. Be careful with what this question

is asking you to answer. The bottom line of whatis beingasked is: "Whatdoes it take in order to secrete calcitonin?"
Calcitonin acts to loser plasma calcium levels. In order to lower those levels they must first be high. What leads to high calcium levels? Increased osteoclast activity (i.e., bone resorption and the return of calcium and phosphate to the blood) and a hypercalcemic plasma (i.e., a plasma in which the level of calcium is above normal). These two conditions demand a decrease in the levels of plasma calcium. The secretion of calcitonin by the parafollicular cells of the thyroid gland reduces the concentration of calcium in the blood. Calcitonin does this by inhibiting osteoclast activity (i.e., inhibits bone resorption) and inhibiting calcium reabsorption at the level of the kidney. The correct
choice is A.
51.

C is correct, (see the pedigree below). The characteristic to look for in an X-linked dominant trait is that all of the daughters and none of the sons of males who carry the trait are affected. This is exactly what we see in this pedigree. Interestingly, the phosphate levels are not as low and the rickets are not as severe in an afflicted heterozygous female as in the afflicted male.

#J-D

n
II D-r-# B-i-O n O-r-D D

i-r-a D

DDEO 1 O-r-n
rv ODD
Autosomal Dominant

fm
Autosomal Dominant
Autosomal Recessive

If any son is affected or if any daughter is unaffected, then the inheritance is autosomal and not X-linked.
Remember, autosomal recessive is only expressed in homozygotes. The correct choice is C.
52.

D is correct, phosphate and l,25-(OH)2-D3. Familial hypophosphatemia refers to the fact that there is inadequate

phosphate in the plasma. As mentioned, this is an X-linked recessive disease and stems from the fact that reabsorption ofphosphate atthe level of the kidneys isimpaired. The addition ofcalcium will not affect the levels of phosphate and so wecan eliminate choice A. Addition of phosphate alone will not make that much of a difference because it will not have an adequate means by which to be absorbed. We can eliminate choice B. We cam also eliminate choice C for the same reasoning. The addition of l,25-(OH)2-D3 along with the phosphate insures adequate absorption by the intestines and (presumably) an increased reabsorption at the level of the kidney. The
correct choice is D.
53.

C is correct, increased neural excitability. Let's consider the listof possible answers. In orderto do this we need to know the meaning of hypoparathyroidism. The parathyroid glands secrete PTH and PTH allows for calcium

absorption from bone by stimulating the osteoclasts and transiently inhibiting osteoblasts. PTH stimulates calcium
reabsorbtion at the kidneys and indirectly allows for calcium absorption at the intestine through the action of the active form of vitamin D. A hypoparathyroid gland is one that does not secrete sufficient amounts of PTH. As a
Copyright by The Berkeley Review
326

The Berkeley Review

Specializing in MCAT Preparation

Biology

Immunology St Endocrinology

Section V Answers

result the levels of calcium in the body decrease below normal values. Not only is osteoclast activity decreased, but
because calcium levels are low, osteoblast activity is decreased as well. With this information we can eliminate

choices A, B, and D. We are left with choice C as the correct answer. Why would hypoparathyroidism lead to increased neural activity? A decrease in calcium in the extracellular fluid allows more sodium to flow through the membranes of nerve cells. Some nerves, like those in the peripheral nervous system, begin to spontaneously discharge. This will eventually give rise to tetany and if not correct immediately can lead to death. The correct
choice is C.
54.

D is correct, a positive linear relationship exists between PTH secretion and the concentration of plasma calcium. This statement is false. In order to make it true we would say that: "an inverse relationship exists between PTH secretion and the concentration of plasma calcium. We can see this from the graph shown below. The first three
choices are all true as discussed in the passage and in the previous answers. The correct choice is D.

Calcitonin

Total Ca2+ concentration in plasma

Passage IX (55 - 61)

Erythroblastosis Fetalis

55.

D is correct, 0%. If this is the woman's first child, she is very unlikely to have antigens to the Rh factor. Exposure
occurs during late pregnancy, delivery, abortion, or miscarriage. Even though a Punnett square would indicate a 50% likelihood of an Rh-positive child, the mother must also have the anti-Rh antibodies from a priorsensitization event.
In this case, she is probably not sensitized. The correct choice is D.

56.

B is correct, the Rh factor is not a component of the ABO blood group system. The placenta acts as a barrier for
some things and allows passage to others. There is not indiscriminate mixing of maternal and fetal blood. Nor does

the mother make the red blood cells for the fetus. Choices A and C are incorrect. As we learned in the passage, any
pregnancy, whether it is taken to completion or not, can trigger antibody formation in Rh- women who have Rh+
fetuses. Choice D is incorrect. The correct choice is B.

57.

C is correct, treat mother with antibodies against the anti-Rh antibodies. If the antibodies can be trapped and removed by other antibodies, then the mother is not sensitized against Rh antigen for subsequent pregnancies.
Treating the fetus would not help future fetuses. Choice D is incorrect. In extreme circumstances, the fetus, not the mother, receives a blood transfusion. This would not help future pregnancies, either. The correct choice is C.

58.

D is correct, all of the above. All three of these organs produce bilirubin, a break-down product of heme. The liver conjugates the bilirubin with glucuronic acid, so it is soluble, and secretes it into the gall bladder. The correct
choice is D.

59.

A is correct, no, there are no maternal antigens to the Rh factor antigens. The term positive means the antigen is present on the RBC. The mother does not have antigens to herself. The fetus is negative, so there are no Rh antigens
on the RBCs to make antibodies against. Choice B is incorrect. The newborn is not immunologically competent, able to make antibodies, until about 1 month after birth. It relies on the antibodies donated by the mother. Choices C
and D are incorrect. The correct choice is A.

60.

D is correct, I, II, and III. Hemolysis in the Rh-positive fetus depends on the amount of antibody produced by its
Rh-negative mother. If a large amount of blood has been transferred between the fetus and the mother, then she is

more likely to produce anti-Rh factor antibodies. Choice I is correct. How her antibody production responds to a given stimulus is also a factor. Choice II is correct. As the number of pregnancies increases, her exposure to Rh

Copyright by The Berkeley Review

327

The Berkeley Review

Specializing in MCAT Preparation

Biology
choice is D.

Immunology fie Endocrinology

Section V Answers

positive blood increases, and the likelihood of developing antibodies increases. Choice III is correct. The correct

61.

B is correct, I and II only. The primary problem in EF is lysis of red blood cells. This leads to anemia and lower hemoglobin level. Choice II is correct. If RBCs are destroyed, this does not lead to an increased number. Erythrocytes are RBCs. Choice III is incorrect. Bilirubin is a breakdown product of heme. It is elevated in neonates
and fetuses with EF. Choice I is correct. The correct choice is B.

Passage X (62 - 67)


62.

Septic Shock

A is correct, does not have an outer membrane on its cell wall. This question requires one to know a little about the differences between Gram-positive and Gram-negative cells. A Gram-positive cell does not have an outer membrane

on its cell wall. This information eliminates choice B. Furthermore, a Gram-positive bacterium has a very thick
peptidoglycan layer adjacent to the plasma membrane. The correct choice is A.

63.

C is corrrect, endocytosis, engulfing foreign particles which eventually will fuse with a lysosome. The question requires us to use our knowledge about how macrophages are functioning. They are phagocytes in the body along
with the neutrophils. The cells engulf microorganisms (this eliminates choices A and B) which then become phagocytic vesicles. These vesicles fuse with the cell's lysosome (eliminating choice D), an organelle with contains highly reactive molecules like superoxides. The fustion of thephagocytic vesicles with the lysosome exposes the foreign particles to these caustic molecules, acting to destroy the microorganism. The correct choice is C.

64.

C is correct, plasma cells. This question requires us to draw on our knowledge of the immune system and the cells which comprise that system. Antibodies are associated with B cells, which are lymphocytes. Antibodies can be found in two forms. One is the membrane bound form, attached to the B cell at its plasma membrane. When a B cell becomes activated, it will proliferate. Some of the new B cells will produce the antibody (it will be the same antibody) in a soluble form, enabling the antibody to circulate in the lymph and blood. B cells which make soluble antibody are called plasma cells. Therefore, a defect in plasma cells would affect production of soluble antibodies.
The correct choice is C.

65.

C is correct, rise in the patient's blood pressure. Anti-Factor XII antibodies will take out (render non-functional) some of the available Factor XII. This will affect the production of bradykinin and the intrinsic coagulatory pathway. Since not all Factor XII is non-functional, some intrinsic pathway leading to coagulation may occur. We still have the entire extrinsic pathway. Therefore, we should not see either a total loss (choice A) or a rise (choice B) in the intravascular clotting of the sepsis patient. As stated above, we are attenuating the role of bradykinin and one of its role is to increase the permeability of the vessel. Increasing the permeability leads to a loss of fluid and hypotension. Since we are attenuating the role of bradykinin, we should therefore see a rise in the patient's blood pressure. The
correct choice is C.

66.

D is correct, 16. The flow of blood through a vessel is proportional to the radius to the fourth power. If we see an increase in the radius of a factor of two, the increase in flow is 24 =16. The correct choice is D.

67.

A is correct, hypotension in the patient. One should realize from the passage that bradykinin acts in two ways. It increases the amount of blood flowing as a vasodilator, and it increases the permeability of the endothelial cells
making up the blood vessels. The increased permeability causes a lot of the plasma in the blood to leaves the vessel (through osmotic forces ) and enter into the tissue. The result of this is that the patient becomes hypovolumeic which will lead to a drop in blood pressure. Therefore, bradykinin will lead to hypotension in the patient. The correct
choice is A.

Passage XI (68 - 75)


68.

Calcitonin St Osteoporosis

C is correct, decreased osteoblast activity, increased osteoclast activity. When bone synthesis is low (decreased osteoblast activity) and bone breakdown is high (increased osteoclast activity), the amount of bone decrease is the greatest. The correct choice is C. C is correct, calcium. The question asks for a mineral. Eliminate choice B, water, because water is a compound, not a single atom. B \2 is a vitamin, not a mineral, so choice A is incorrect. Since calcium was mentioned in the passage,

69.

Copyright by The Berkeley Review

328

The Berkeley Review Specializing in MCAT Preparation

Biology

Immunology St Endocrinology

Section V Answers

consider it. Calcium is the major mineral making up bones. Our body contains about 1 kg of calcium. Although zinc

is part of many enzymes, our body contains only a lew grams of zinc. The correct choice is C.
70. A is correct, PTH increases plasma calcium. The passage states the CT is an antagonist of PTH. This means they have opposite actions. CT works to decrease plasma calcium, and PTH works to increase plasma calcium. Choice B is incorrect. To increase plasma calcium, the kidney decreases urinary calcium and phosphate. Choices C and D are
incorrect. The correct choice is A.

71.

C is correct, cysteine-cysteine. This is a biochemistry trivia question. Two cysteine residues link together via their sulfur atoms to form a cystine molecule that contains a disulfide bridge. Cysteine is the single amino acid, while cystine refers to a dimer of cysteine. The correct choice is C. B is correct, I and III. Choice II is incorrect. Since CT is a polypeptide hormone, it does not enter the cell, but rather acts on a receptor on the plasma membrane. Choice I is correct. The salmon CT is less recognizable to the human enzymes, so it resists degradation more and has a longer-lasting action. Choice III is correct. Salmon CT has a stronger interaction with the CT receptor and promotes a longer signal on the receptor. The correct choice is B. A is correct, CT secretion increases. In the beginning of a supplement program, a person would have increased calcium levels. In response to these increased levels, CT secretion would increase to bring the levels back down to
normal. The correct choice is A.

72.

73.

74.

B is correct, injection to avoid hydrolysis. Since CT is a polypeptide, it is subject to digestion (hydrolysis) in the stomach. Ingestion means to take orally, so choices A and C are incorrect. There is a possibility of an allergic reaction when proteins are injected, so choice D is incorrect. The correct choice is B.

75.

D is correct, I, II, and III. In a systemic allergic reaction, histamine, leukotrienes, and prostaglandins are released and lead to skin reddening, hives, and constriction of the bronchioles (leads to breathing difficulty), among other symptoms. The correct choice is D.

Passage XII (76 -83)


76.

Vertebrate Immune System

D is correct, little or no change in the effectiveness of either type of immune response. The question asks about the consequences of removing the thymus from an adult human. From the passage we learn that T cells are produced in the bone marrow and later mature in the thymus during early development. The thymus is therefore crucial during early development for the production of T cells, which are responsible for the cell-mediated response. If the thymus is removed from a mature adult, however, there will be little or no impact on the immune system since most T cell formation has already occurred. The tempting answer is choice B, but remember to read all of the answer choices before making a decision. A careful reading of the question in this case is also a must. The correct choice is D.
C is correct, the cell would lyse due to an upset water balance. When pores are formed in the membrane of the target cell, small molecules (i.e. charged ions) are suddenly able to flow freely along their electrochemical gradients. The effect of this would be to disrupt the osmotic balance maintained by the cell. This balance is necessary because the large number of negatively charged macromolecules in the cell (proteins, nucleic acids, etc.) would normally attract a large number of cations in order to balance the charge. The entry of such a large number of cations would draw water into the cell with them, a catastrophe which would cause lysis, or bursting, of the plasma membrane. In normal cells, this is prevented because the membrane is mostly impermeable to small charged ions, and the action of membrane transporters serves to maintain an appropriate osmotic balance. The action of the complement proteins is to poke holes in the membrane, allowing ions and water to rush into the cell, causing lysis; this is an efficient way of
killing invading microorganisms. Choice D is the opposite of what was just described and is therefore incorrect. Choice A is also wrong; an inability to initiate action potentials is not lethal to the cell. Additionally, most cells which are attacked by the antibody/complement system are invading microorganisms which do not have anything to do with the neuronal function of conducting action potentials. Choice B is wrong because if the cell is normally

77.

negatively charged inside, holes in the membrane would allow cations to rush in, leading to depolarization rather than hyperpolarization. The correct choice is C.
78. C is correct, II and III only. Statement II states that organ transplants between monozygotic twins do not result in immune rejection of the transplanted organ. This implies that the recipient's immune system regards the new organ

Copyright by The Berkeley Review

329

The Berkeley Review Specializing in MCAT Preparation

BlOlOgy

Immunology &Endocrinology

Section v Answers

as being "self." Since monozygotic twins are genetically identical (as opposed to dizygotic twins), this statement is

consistent with Theory 1, which states that individuals lacks the genes to produce a response against selfantigens. Since both twins have the same genetic structure, they both lack the genes necessary to mount an immune response
against theirown tissues; therefore, according to Theory 1 thetransplanted organis not rejected. Statement II is also

consistent with Theory 2, which inessence states that the immune system "learns" notto attack its own tissues early in development. Since both twins had theexactly the same tissues (the result of their identical genes), both of their immune systems "learned" not to attack the same tissues. This also explains why the transplanted organ was not
rejected. Statement HI is, for all intents andpurposes, an identical situation to that presented in Statement n. In this case, genetically identical mice from the same inbred strain are able to tolerate transplants from each other. An

inbred strain, incidentally, is a strain of mice that has been inbred formany generations, resulting in siblings which are genetically identical to one another. Statement I states thatcellsfrom a foreign cell line are injected into a mouse at theembryonic stage. When themouse matures, it does not mount an immune response to further injections of the same cells. Thissupports Theory 2 in that it provides evidence that the immune system "learns" during embryonic development not to attack cells that are present at the time (i.e., self cells). The immune system is fooled into thinking the foreign cells are selfcells. This contradicts Theory 1.According to this theory, the foreign cells would still be attacked even if they were injectedat the embryostage.The correct choice is C.
79. B is correct, weakness and difficulty breathing. The question states that antibodies are produced against skeletal muscle acetylcholine receptors. This would most likely result in muscle weakness (due to an inability of muscle fibers to receive signals from effector nerves). Difficulty breathing would also be likely because many skeletal
muscles (i.e., the intercostals) are involved in the respiratory process. This problem could also be solved via a process of elimination. Answerchoice A can be eliminated becausethe heart is made up of cardiac muscle. Since the
antibodies only attack skeletal muscle receptors, the heart should not be affected. Likewise, the smooth muscle of

the gastrointestinal tract should not be affected, ruling out answer choiceC. ChoiceD is unlikely to be true as well.
Neurons in the brain should not be affected by skeletal muscle receptor antibodies, and therefore no dementia should
occur. The correct choice is B.

80.

B is correct, the B cells have considerably more roughendoplasmic reticulum than the T cells. Since the function of activated B cells is to produce and secrete large amounts of antibody, it would be likely that under electron microscopy they would appear to have more rough endoplasmic reticulum than activated T cells, which do not have

as great of a secretory role. Rough ER is where proteins thatare to be secreted are mainly produced. The "rough" appearance of this organelle is due to the presenceof ribosomes dotting the convoluted membrane of the ER. These
ribosome produce the antibody proteins. Choice A is not necessarily a likely observation, as neither cell has an
especially great need for mitochondria relative to the other. Answer choice C is also incorrect as T and B cells both

have nuclei. Do not confusethem withanothertypeof blood cell which does lack a nucleus: the erythrocyte. Answer choice D is incorrect because electron microscopy is the most powerful means of visualizing cells and as such is
very likely to detect differences between B and T cells. The correct choice is B.

81.

A is correct, each B cell is predetermined to bind a specific antigen. First, let's go over the experiment in the question. A population of B cells is removed from an adult mouse and mixed with an effectively lethal antigen X. The B cells that have the appropriate surface receptors bind the antigen are killed due to the radioactivity. Very few B cells are killed, therefore very few out of the total population (<0.01%) recognize antigen X specifically. When the surviving B cells are injected into an irradiated mouse (lacking B cells), the mouse is now capable of responding to other antigens, but not antigen X. It can.be implied that the reason for this was the death of the B cells which specifically recognized antigen X. The remaining population of B cells had no members which recognized antigen X, though they could recognize other antigens. Therefore we can conclude that each B cell is predetermined to bind a specific antigen. B cells can not recognize new antigens and "learn" to bind to them, making answer choice B incorrect. Answer choice C is wrong because the above experiment implies nothing about T cell interference with B cell binding. Answer choice D is wrong because an antigen, by its very definition (antibody generator), is immunogenic. Therefore antigen X does elicit an immune response normally. The correct choice is A.
B is correct, antigens can only be bound by a single antibody at a time. The question is asking which condition would not hold true if antibodies were capable of "teaming up" and cross-linking antigens together. We can approach this problem by first theorizing how antibodies could accomplish this feat. First, if antigens could be bound by more than one antibody, and each antibody had two antigenbinding sites connected by a flexible "hinge" region, cross-linking could easily occur. Each antibody would bind two antigen particles, which would then bind an additional two antibodies, which would then bind another two antigen particles, etc. The flexible hinge region would allow greater flexibility during the cross-linking process. The above description would make answer choices A, C,

82.

Copyright by TheBerkeley Review

330

The Berkeley Review


Specializing in MCAT Preparation

Biology

Immunology St Endocrinology

Section V Answers

and D all consistent with the cross-linking phenomenon. This leaves choice B as the answer. If antigens could only
be bound by a single antibody at a time, the above scheme would not be possible and there could be no crosslinking. The correct choice is B.
83.

A is correct, most types of cells in the body. The key concept to remember here is that viruses can infect most types of cells in the body. Since the question states that cells infected with viruses are usually killed by cytotoxic T cells, it logically follows that the T cells should be able to target most cells in the body. The other answer choices mention
specific cell types and are therefore incorrect. The correct choice is A.

Passage XIII (84 - 90)


84.

Insulin Receptor

C is correct, detergent wash. The passage informs us that the insulin receptor is an integral transmembrane protein.
The receptor is thus embedded in the lipid bilayer and actually spans the entire layer from the extracellular to the intracellular face. These proteins arc tightly bound to the lipid bilayer by hydrophobic forces and harsh conditions are needed to remove them. In particular, detergents must be used. The reason is that these molecules are amphipathic, containing both a polar and non-polar region. The non-polar region binds to the hydrophobic regions of the membrane proteins, disrupting the lipid bilayer. The polar region creates a water soluble micelle with the membrane protein (hydrophobic portion) in the middle. The receptor is thus isolated and ready to study. The
correct choice is C.

85.

B is correct, transduction mechanisms. We know from the question that we have a hybrid receptor which is made up of a insulin binding region and a EGF tyrosine kinase segment. From the passage, we know that the insulin receptor is believed to be a insulin activated tyrosine kinase. Recall that the binding of insulin leads to tyrosine kinase activity. The mechanism which brings about this activity is termed the signal transduction pathway. With our hybrid receptor, the binding of insulin leads to kinase activity in the EGF segment. This information indicates a similar
signal transduction mechanism. The correct choice is B.

86.

C is correct. The question tells us that the binding of glucagon will result in the production of the secondary messenger cAMP. The values of cAMP should thus rise with time. Using this piece of information, we can eliminate choices A and B. The question then becomes one of knowing the sequence of events. To answer this, one has to use previous knowledge and/or a little common sense. The secondary messengers produced lead to changes in the cell. Recall that to truly bring about change for the cell, there must be some change in proteins, as proteins are the key to function. One very common way to regulate proteins is through their phosphorylation and their dephosphorylation. It is the secondary messenger that promotes the phosphorylation of certain peptides, converting these proteins into their active forms. The active forms of the proteins bring about the change. From this information, the correct sequence of graphs can be seen. The correct choice is C.
C is correct, a normal insulin receptor. The question refers to a type I, insulin dependent diabetes. These individuals do not produce insulin. That is why they are dependent. This should clue us into the fact that their receptors more than likely have nothing wrong with them. The problem is that these individuals have no beta cells in the pancreas which release insulin. These individuals take insulin and their body responds appropriately. From this information, we can conclude that the receptors are more than likely normal. The correct choice is C.
D is correct, stimulation of pyruvate dehydrogenase. We know from the passage that GIP mimics certain insulin responses. Therefore, we should look for an insulin response. Choice D indicates a stimulation of pyruvate dehydrogenase, the enzyme converting pyruvate into acetyl CoA. This would indicate an acceleration of glycolysis and the Krebs cycle. This is certainly one of the effects of insulin, as the hormone promotes the use of glucose as a
source of fuel. The correct choice is D.

87.

88.

89.

C is correct, increased CO2 production. The question tells us that the addition of bivalent anti-insulin receptor antibodies induces a response without insulin. What will antibodies cause? The key word here is bivalent. The bivalent structure will induce cross-linking, leading to a clustering of insulin receptors. This conclusion is bolstered by the next sentence of the question which states that monovalent antibodies, which cannot produce cross-linking, do not induce a response in the absence of insulin. However, if we add antibodies against these monovalent antibodies/fragments, we should be able to induce cross-linking and thus clustering. Therefore, we should be able to induce an insulin-like response, which involves the promotion of glucose as a fuel. When glucose is oxidized, the
carbons are released as carbon dioxide. The correct choice is C.

Copyright by The Berkeley Review

331

The Berkeley Review Specializing in MCAT Preparation

Biology
90.

Immunology St Endocrinology

Section V Answers

B is correct, reducing conditions. This question is answered from our understanding of the conditions which are necessary to form or break disulfide bonds. When two cysteine residues are oxidized, the bond is formed and they form cystine. In order to break the bond, cystine must be placed in reducing conditions. From the passage, we know that the dimer insulin receptor is held with disulfide bonds. To isolate a component of the receptor for study, reducing conditions would be used to breakthese disulfide connections. The correct choice is B.
Vitamin D3, PTH, & Calcitonin

Passage XIV (91 -95)


91.

B is correct, keratinocytes. Hepatocytes are cells found in the liver. The liver has numerous functions, such as the

regulation of metabolism of carbohydrates, lipids, and proteins. The liver stores glycogen and is the primary site of gluconeogenesis. The liver is a major source of cholesterol in the body and is a major storage site for iron. The liver
is also the site where many hormones are degraded and where many toxins are inactivated. Leukocytes (white blood cells) are a cellular constituent of blood, along with erythrocytes and platelets, and they play an important role in the body's defensive system. Leukocytes come in five classes: neutrophils, eosinophils, basophils, monocytes, and lymphocytes. Lymphocytes can be categorized as being either B-cells, which mature in the bone marrow, or T-cells, which mature in the thymus. Lymphocytes participate in the immune response. Keratinocytes are skin cells of the epidermis. This cell type (of all the cell types listed) is closest to the exposure of ultraviolet light and allows for the conversion of 7-dehydrocholesterol to previtamin D3. In the skin previtamin D3 can be converted to vitamin D3, which is the precursor to the active form, l,25-(OH2)-D3. The correct choice is B.
92. C is correct integument. In the passage it states that humans obtain vitamin D3 either by ingestion or through synthetic mechanisms that are initiated by cells in the epidermis of the skin. If we ingest vitamin D3, it will end up being absorbed by the cells of the intestine. However, vitamin D3 is not the primary precursor for l,25-(OH2)-D3. In the skin (integument) the primary precursor (7-dehydrocholesterol) reacts with UV light to form previtamin D3, which in turn will form vitamin D3. The correct choice is C. C is correct, II and III only (polypeptide and steroid). In the second paragraph of the passage we are told that 1,25(OH)2-D3 passes into a target cell and then into the nucleus where it complexes with a receptor protein that has a DNA binding site. This is the general mechanism for action of steroid hormones. We can eliminate choice A as a possible answer. In the third paragraph we find that both PTH and CT have genes encoded in the short arm of human chromosome 11 and are synthesized as a preprohormone from different primary RNA transcripts. If they are both synthesized as a preprohormone from an RNA transcript, it must mean that amino acids are being linked together in peptide linkages at the ribosome. In other words, a polypeptide is being synthesized. The mature PTH polypeptide contains 84 amino acids while the mature CT polypeptide contains 32 amino acids. These three hormones fall under the steroid and polypeptide classes of hormones. The correct choice is C. 94. D is correct, I and III only (hypocalcemia and hyperphosphatemia). What does PTH do under normal circumstances? Based on information in the passage we find that PTH acts on bone and on the distal tubule of the

93.

kidney to promote Ca2 reabsorption and inhibits the reabsorption of PO4-* in the proximal tubule of the kidney.
PTH also stimulates the synthesis of l,25-(OH)2-D3 in the kidney. We saw that l,25-(OH)2-D3 promotes the uptake

of Ca- and PO^ from the intestinal lumen. If we did not have adequate levels of PTH secretion, we would expect low levels of Ca2 reabsorption (hypocalcemia) and a decrease in urinary PO^ excretion. A decrease in urinary PO^ excretion means that PO^ is being reabsorbed by the proximal tubules of the kidney. This will lead to high (hyperphosphatemia) plasma levels of PO^. The correct choice is D.
95. B is correct, (see the graph below). Be careful when reading the graph. Along the x-axis we see the total plasma

[Ca2], ranging from low to high values. What is going to happen when the plasma levels of Ca2 are low? We are going to want to place more Ca2 in the plasma. How can we do this? We increase the levels of PTH. In the fourth
paragraph of the passage it states that PTH acts on bone and on the distal tubule of the kidney to promote Ca2

Copyright by The Berkeley Review

332

The Berkeley Review Specializing in MCAT Preparation

Biology

Immunology St Endocrinology

Section V Answers

reabsorption. As the levels of Ca2 begin to increase the levels of PTH will begin to decrease. This is exactly what
we see in the graph for parathyroid hormone.

low

high

Total Plasma [Ca2+]

What happens when plasma Ca2 levels are high? The parafollicular cells begin to secrete calcitonin which acts on
osteoclasts of bone tissue. As the calcium levels are lowered (by a mechanism that is not completely understood) the calcitonin levels begin to drop. This is exactly what is shown in the graph for calcitonin. The correct choice is B.

Passage XV (96 - 100)


96.

Sepsis Syndrome

B is correct, diapedesis. Although the answer is not given in the passage, let's use our test-taking skills. Choice A is incorrect because mitosis is the process of duplication of genetic material and make 2 cells in somatic cells. Phagocytosis and endocytosis involve a cell engulfing some outside particle. Choices C and D are incorrect. Finally, the correct term for WBCs squeezing out is diapedesis. The correct choice is B. D is correct hydrolyze bacterial proteins. Think of what enzymes do and how they are named. A lipase hydrolyzes lipids, so choice A is incorrect. An amylase or glucosidase hydrolyzes carbohydrates, so choice B is incorrect. A nuclease would hydrolyze DNA, so choice C is wrong. A protease or proteolytic enzyme hydrolyzes proteins. The
correct choice is D.

97.

98.

D is correct, toxic oxygen products. Toxic oxygen products are toxic because they are free radicals or decompose to form free radicals. The passage explains the communication role of cytokines, so eliminate choice C as incorrect. The proteases directly attack bacteria protein and hydrolyze bonds, but do not use free radicals. Eliminate choice B. Chemical attractants signal neutrophils to move to infection area, so eliminate choice A. The correct choice is D.

99.

C is correct, TNF is mainly present in the general circulation. In the question, we are told that binding TNF halts the abnormal cytokine cascade. Therefore choice A is incorrect. Usually proteins that enter cells by endocytosis are degraded in lysosomes. If the receptors were taken up in this manner, then they would be degraded and not able to interact with TNF. Choice B is incorrect. If the TNF receptors are injected in the general circulation and work, then TNF must be present in the general circulation. This means choice C is correct and must contradict choice D. The
correct choice is C.

100. A is correct, neutrophils. By correct reading, we can eliminate choices B and C. Erythrocytes are red blood cells, so choice B is incorrect. Leukocyte is simply another name for white blood cell, so choice C is incorrect. Neutrophils are 60-70% of the WBC population, making them the most abundant. The correct choice is A.

Copyright by The Berkeley Review

333

The Berkeley Review Specializing in MCAT Preparation

Physiology

Diagnostic Passages
and Questions
Set I

Berkeley

JJr-e-v-le-w

Specializing in MCAT Preparation

DIRECTIONS: A descriptive passage precedes many of the questions found in the Biological Sciences section of the exam. The questions are arranged into groups. Examine a passage before selecting the one best answer choice to each question in the group. There will be some questions which areindependent of a descriptive passage and independent of each other. The one best answer choice must be selected for these questions as well. If you are not sure of an answer, eliminate the alternative choices that youknowto be incorrect and then choose an answer from the remaining alternatives. Indicate your choice by darkening the corresponding bubble on your answer sheet. A periodic table of the elements is provided for you at the end of this book, and you

Tear Along The Perforated Line

Tear Along The Perforated Line

to o

00

o\

K)

<o

00

t/l

*s>

Hi

<s>

<a>

<s> (>

<>

4*.

U>

vo

00

-J

<
^

<*> <Sl

<*> t

w U>

u> fcs>

w l-i

^O

te

00

&

<|

^>

</l

t>>

K>

U>

fcO

N>

N>

h*

N>

&9 QTQ S3 O
**
**

id
CA
**

C/5 QTQ
Ul O 4^ ^ 00 4^ n] ^ 9\

4*. U)

4* 4^

4^ U

4* N

4* H

<>

<g> @

<>

Physiology

Diagnostic Passages
and Questions
Set I

7 Passages with 39 Questions 13 Independent Questions


52 Questions Total

70 Minutes

Berkeley

Ur-e-v-i^e-w

Specializing in MCAT Preparation

Passage I (Questions 1-5)


The thyroid, one of the largest endocrine glands in the human body, is responsible for secreting thyroid hormone (thyroxine). This hormone is primarily responsible for

Radioactive iodine treatment is extremely effective while having relatively few side effects. This is mainly
because:

regulating basal metabolic rate. Excess secretion of thyroid


hormone leads to an abnormally high rate of metabolism, resulting in a range of symptoms including weight loss, increased appetite, and profuse sweating. The secretion of thyroxine is stimulated by thyroidstimulating hormone (TSH), a peptide hormone. TSH secretion, in turn, is stimulated by thyrotropin-releasing hormone (TRH), a peptide containing 3 amino acids. The
interaction of TRH, TSH, and thyroxine is shown below in Figure 1.

A. B. C. D.

most cells of the body can take up iodine. once inside a cell, ,31I is non-toxic. iodine is only absorbed by thyroid cells. cancerous thyroid cells lose their ability to take up iodine from the blood.

Surgical removal of the .thyroid without hormonal supplementation might lead to: A. B. C. D. increased appetite and food intake. weight gain. profuse sweating. increased oxygen usage by the tissues.

Hypothalamus

TRH IC
Thyroid hormone is most likely a derivative of the
amino acid:

Anterior Pituitary
TSH

A.

threonine.

I
"8
H H 0

B.
C. D.

tyrosine.
histidine. methionine.

Thyroid Gland

Radioactive iodine treatment is most effective several


I
I

I
I

II

C-C-C-OH

weeks after the surgical removal of the cancerous thyroid gland. This is most likely because by this time: A. cancerous thyroid cells have had a chance to divide and spread further. thyrotropin-releasing hormone levels have dropped. thyroxine levels have risen, allowing increased iodine uptake. thyroid-stimulating hormone levels have risen, allowing increased iodine uptake.

NH2

Thyroxine

B. C.

Figure 1

D.
TSH stimulates the cells of the thyroid gland to take up iodine from the blood for use in the synthesis of thyroxine. Due to this iodine uptake, patients with cancer of the thyroid usually have a good prognosis even if the cancer has spread. This is because after surgical removal of the cancerous

thyroid, radioactive l3,I can be administered, effectively


killing (via radiation) the remaining cancerous thyroid cells
that absorb it.

Can radioactively labeled TSH be used as a thyroid cancer therapy?

A. B. C. D.

Yes; TSH selectively enters cells of the thyroid, allowing them to be destroyed by the radiation. Yes; TSH stimulates the production of radioactive thyroxine. No; TSH binds to surface receptors on thyroid cells, thereby never actually entering these cells. No; TSH is taken up by many tissues other than the thyroid.

Copyright by The Berkeley Review

340

GO ON TO THE NEXT PAGE

Passage II (Questions 6-11)


Over-the-counter pregnancy tests which are simple to use and fairly accurate have been available for several years. Although there are a large variety of such tests, all of them
share a set of common principles.

8.

As described in the passage, the home pregnancy test:


A.
B.

lacks positive and negative controls.


is only effective after at least one month of
pregnancy.

C.
D.

is inaccurate because hCG can be produced


normally in the non-pregnant mother.
is inaccurate because hCG is sometimes not

These home pregnancy tests are basically immunoassays which detect the presence of the peptide hormone human chorionic gonadotropin (hCG). hCG is produced by the fertilized ovum and released into the mother's bloodstream, preventing the degradation of the corpus luteum and thereby preventing menstruation. By the time of the mother's first missed period, hCG levels in the urine are high enough to detect.
The home pregnancy test generally involves dipping a stick containing immobilized hCG monoclonal antibodies into a urine sample. If hCG is present in the urine, it will bind to the monoclonal antibodies on the dipstick. The dipstick is then placed in a solution containing a second monoclonal antibody which recognizes the bound hCGantibody complex on the surface of the dipstick. This second antibody is conjugated to colloidal gold particles which change color when they are immobilized, indicating a positive test result (i.e., pregnancy).
9.

produced by the fertilized zygote.

Which of the following can be concluded from information given in the passage?
A.
B.

hCG is also secreted by the mother.


hCG does not enter the mother's bloodstream.

C. D.

The tubules of the kidney do not reabsorb all the hCG filtered in the glomerulus. The actions of progesterone are counteracted by
hCG.

10.

The same techniques used in the home pregnancy test can be implemented in other clinical tests. Which of

the following could NOT be detected using such


techniques?

A.
6.

Strep throat caused by a particular strain of


bacteria

Why isn't the presence of progesterone used as an indicator of pregnancy in the home pregnancy test?
A. Antibodies to steroid hormones can't be made.

B.
C.

Autoimmune diseases such as


arthritis HIV infection

rheumatoid

D.

Tryptophan blood levels

B.

Progesterone is secreted too late in the pregnancy


to be of any diagnostic value.

C.
D.

Progesterone is not made exclusively by the


fertilized zygote.

Progesterone is not present in the bloodstream.

11.

The monoclonal pregnancy test:

antibodies

used

in

the

home

A.
7.

come from a single T-cell clone and bind to a single antigen site on the hCG molecule.
come from
molecule.

The mechanism by which hCG acts on its target cells is


most similar to that of:
A. testosterone.

B.

several different T-cell clones and

bind to multiple C. D.

antigen sites on the hCG

B. C.
D.

glucagon. progesterone.
estradiol.

come from a single B-cell clone and bind to a single antigen site on the hCG molecule. come from a single B-cell clone and bind to multiple antigen sites on the hCG molecule.
GO ON TO THE NEXT PAGE

Copyright by The Berkeley Review

341

Questions 12 through 15 are NOT based on a


descriptive passage.

14.

Colchicine is a drug which prevents the polymerization


of microtubules. Which of the following processes would be most affected by colchicine? A. B.
C. D.

12.

Whales are capable of diving to great depths and remaining underwater for extremely long periods of time. Which of the following would LEAST contribute
to these abilities in whales?

DNA synthesis Protein synthesis


Mitosis Muscle contraction

A. B. C.
D.

Large muscle myoglobin concentrations High basal metabolic rate High cellular tolerance for CO2
Large lung capacity

15.

Which of the following represents a physiological effect of breathing air with a slightly increased partial pressure of CO2?
A. B. C. D. Increased breathing rate Decreased breathing rate Increased blood pressure Decreased blood pressure

13.

Which of the following digestive enzymes are NOT produced by the pancreas?
A. B. C. D. Pepsin Chymotrypsin Lipase Carboxypeptidase

Copyright by The Berkeley Review

342

GO ON TO THE NEXT PAGE

Passage III (Questions 16-20)


The bacterium Vibrio cholerae is responsible for causing the intestinal disease cholera, a major cause of mortality in developing nations. The enterotoxin secreted by the bacterium causes an increase in chloride ion secretion by intestinal crypt cells. Water and sodium follow chloride into the intestinal lumen, dehydrating the victim's blood and causing diarrhea. The resulting water and electrolyte loss can
lead to death within hours.

16.

After administration of the standard ORT solution, the volume of fluid lost through toxin-induced diarrhea:
A. increases due to a net flow of water out of the blood and into the intestinal lumen.

B.

increases due to the increased osmolarity of the


fluid in the intestinal lumen.

C.
D.

does not change, because ORT exactly replaces lost fluid withoutaffecting the diarrheadirectly.
decreases, because more fluid is reabsorbed into

the blood than is lost through diarrhea.

Studies of the intestinal sodium/glucose symport led to the development of oral rehydration therapy (ORT). This therapy involves administering liquids with the components
shown in Table 1.
17.

In addition to dehydration and diarrhea, cholera victims

are most likely to experience which of the following


symptoms?
Ingredients
Sodium Chloride

Grams /Liter of H20


3.5 2.9
1.5 20.0

A. B. C.
D.

Trisodium Citrate"*"
Potassium Chloride

Increased glomerular filtration and urine production Increased blood pressure Increased rate of breathing
Metabolic alkalosis

Glucose

"'"Note:

trisodium citrate is used in modern ORT solutions instead of the more unstable sodium bicarbonate.

18.

Table 1: Standard ORT solution

The most important function of trisodium citrate in the


standard ORT solution is to:

The standard ORT solution, when administered early in the disease, prevents dehydration of cholera victims by using

A.
B. C. D.

provide Na+ ions for co-transport with glucose.


help raise the pH of the cholera victim's blood. increase the osmolarity of the ORT solution. decrease the osmolarity of the ORT solution.

the sodium/glucose symport to co-transport Na+ and glucose


from the lumen of the intestine into the blood. Water follows

by osmosis, exactly replacing the fluid that is lost due to the


toxin-induced diarrhea. In order to avoid an increase of

water flow into the intestinal lumen, the ORT solution has an

19.

osmolarity (i.e., solute concentration) that equals that of normal blood. This makes it hypotonic relative to the cholera victim's partially dehydrated blood.

Would the addition of more glucose and sodium to the ORT solution result in increased net fluid uptake by the
blood?

Yes; increased Na+ and glucose would increase


co-transport and draw more water into the blood.
B.

No; increased Na+ and glucose would have no


effect on fluid uptake. Yes; the increased glucose would provide ATP for the active transport of water.

C.
D.

No; an increase in Na+ and glucose would


increase the osmolarity of the ORT solution.

20.

Potassium chloride is added to the ORT solution

primarily to: A. B.
C.

help maintain muscle function and membrane potentials. increase intestinal sodium absorption.
boost the low chloride concentration of the intestinal lumen.

D.

activate K+/C1* antiporters in the intestinal


epithelium.

Copyright by The Berkeley Review

343

GO ON TO THE NEXT PAGE

Passage IV (Questions 21-25)


When the bacterium Clostridium botulinum is grown

22.

The physiological effects of botulinum toxin on


muscles can best be mimicked by:

anaerobically, it produces botulinum toxin (BT), one of the most potent natural poisons. For adults, BT is lethal in doses as small as 2-3 |ig. The active portion of the toxin is a protein with a molecular weight of 150,000 that is conjugated to a variety of accessory proteins. This protein complex functions by disrupting calcium-mediated acetylcholine release, causing an irreversible blockage of
synaptic transmission.

A. B.

addition ofCa2+ directly to the space between the


endplate membrane and the presynapticterminal.
addition of acetylcholinesterase directly to the space between the endplate membrane and the presynaptic terminal.

C.
D.

injection ofCa2+ into muscle fibers.


injection of acetylcholine into muscle fibers.

Despite being a dangerous poison, botulinum toxin has recently been used as a therapeutic agent in the treatment of
a series of conditions collectively known as focal dystonias.

Focal dystonias are neuromuscular disorders caused by the involuntary, sustained contraction of localized (focal) regionsof skeletal muscle.The resulting twisting, turning, or squeezing movements can result in physical impairment (as
in severe "writer's cramp", a debilitating focal dystonia).
23.

As a therapeutic agent, highly diluted botulinum toxin is injected intramuscularly at the site of the focal dystonia. Within a short period, abnormal muscle contraction ceases and the patient experiences considerable relief. Histological studies show that BT causes the atrophy of nerves fibers innervating the affected muscles, thereby preventing contraction and relieving the focal dystonia.

Botulinum toxin most likely exerts its effects directly


on the:

A. B. C.
D.

presynaptic nerve terminal. postsynaptic nerve terminal. calcium-sequestering organelles in muscle cells.
nodes of Ranvier.

24.

According to information given in the passage, focal dystonias are most likely caused by:
A.
B. C. D.

a defect in the sarcoplasmic reticulum of affected


muscle fibers.

constant overstimulation by afferent nerve fibers. a malfunction in the central nervous system. an oversecretion of acetylcholinesterase.

21.

Injection of botulinum toxin for the treatment of focal dystonia often provides relief which only lasts several weeks. Reinjection after this time provides renewed relief for most patients, but some no longer respond to the therapy. Which of the following best explains why these particular patients lose responsiveness to botulinum toxin injections?
25.

A. B. C. D.

Neurotransmitters other than acetylcholine are used by the effector nerves in these patients. These patients produce antibodies against the active portion of the botulinum toxin. Nerves regenerate over the course of several weeks in these patients. Affected muscle fibers become permanently disabled in these patients.
344

Food poisoning can often be caused by Clostridium botulinum. Which of the following would carry the greatest risk of botulinum toxin contamination?
A.
B.

Dairy products.
Fruit.

C. D.

Canned food. Food left uncovered

and

unrefrigerated

overnight.
GO ON TO THE NEXT PAGE

Copyright by The Berkeley Review

Passage V (Questions 26 - 32)

28.

Which of the following would represent an effective


medical treatment for hypovolemic shock?

Circulatory shock occurs when blood flow through the circulatory system is inadequate to meet the oxygen demands of the tissues. Without physiological compensation
or emergency medical intervention, shock can rapidly lead to
death.

A. B. C. D.

Administration of diuretic drugs. Transfusion of blood plasma. Injection of epinephrineantagonists. Injection of acetylcholine antagonists.

Hypovolemic shock, a type of circulatory shock, occurs when the volume of the blood is too low, resulting in low blood pressure and decreased cardiac output. Since arterial
blood pressure is directly proportional to blood flow, the

immediate result of hypovolemic shock is inadequate blood


flow to the organs and tissues.

29.

Another physiological compensation for hypovolemic shock involves the production of ADH and
aldosterone. These hormones: A. increase urine volume.

Sensing the drop in blood pressure, baroreceptors in the circulatory system trigger a series of sympathetic responses to hypovolemic shock which help to compensate for the low blood flow. One such response involves the vasoconstriction of arterioles leading to most tissues other than the brain and heart. This affects the total peripheral resistance (i.e., the
resistance to blood flow encountered in all of the vessels in

B. C. D.

decrease salt reabsorption. increase blood pressure. inhibit the sympathetic nervous system.

the circulatory system). Blood pressure (BP) is related to cardiac output (CO) and total peripheral resistance (TPR) as
shown below:

BP oc (CO) (TPR)

30. Anaphylactic shock can occur during severe allergic reactions, resulting in the widespread release of the
vasodilating hormone histamine. Histamine causes this potentially deadly type of circulatory shock by:

Resistance to blood flow is inversely proportional to the radius of the blood vessel raised to the fourth power. Cardiac output is equal to the heart rate times the stroke volume of
the ventricles.

A. B. C. D.

increasing total peripheral resistance. decreasing total peripheral resistance. increasing cardiac output. increasing blood pressure.

26.

Which of the following would most likely NOT result in hypovolemic shock?
A. B.
C.

31.

Which of the following represents a likely physiological compensation for hypovolemic shock?
A. B.
C.
D.

Severe hemorrhaging Dehydration


Heart failure

Extensive sweating. Dilation of arterioles leading to the skin.


Decreased stroke volume.
Increased heart rate.

D.

Blood loss due to injury

27.

In order to prevent any further decrease in blood volume during hypovolemic shock, which of the following should most likely occur?
A. B. Sympathetic stimulation of the renal afferent
arterioles.

32.

Despite the majority of blood volume being in the venous system at any one time, the mean venous pressure is considerably lower than the mean arterial
pressure due, in part, to the:

Parasympathetic stimulation of the renal afferent


arterioles.

C.
D.

Sympathetic stimulation of the renal efferent


arterioles.

A. B. C. D.

greater ability of the venous walls to expand. greater ability of the arterial walls to expand. presence of one-way valves in the veins. larger water permeability of the veins.

Parasympathetic stimulation of the renal efferent


arterioles.

Copyright by The Berkeley Review

345

GO ON TO THE NEXT PAGE

Questions 33 through 36 are NOT based on a


descriptive passage.

35. Progesterone is primarily secreted by which of the


following structures?
A. B. C.
D.

33.

In order to maintain a constant body temperature,

Primary follicle Secondary follicle Corpus luteum


Ovum

humans must employ regulatory mechanisms to cope with an increase in temperature. Which of the following would NOT represent such a mechanism?
A.
B.
C.
D.

Dilation of arteriole sphincters leading to skin


capillary beds Activation of sweat gland secretion Decrease in skeletal muscle activity
Piloerection of hairs on the skin surface

34.

In severe asthma, a patient's bronchioles can become chronically inflamed and obstructed, increasing the effort necessary to inflate the lungs with air. Which of the following might be a consequence of severe
asthma?

A.
B. C. D.

Enlargement of chest accessory muscles.


Reduction in the size of the diaphragm. Low CO2 levels in the blood. High O2 levels in the blood.

36. Which of the following are NOT components of cellmediated immunity?

A.
B.

Macrophages
Killer T-cells

C.
D.

Helper T-cells
B-cells

Copyright by The Berkeley Review

346

GO ON TO THE NEXT PAGE

Passage VI (Questions 37-41)


In order for physiological processes to function correctly, the pH of blood must be precisely maintained. If a stress on the body works to change blood pH, certain
mechanisms act to compensate for the stress. One such

37. Which of the following might lead to metabolic


alkalosis?

A.
B.

Vomiting
Diarrhea

mechanism is known as the CO2/HCO3' buffer system:


H + HCO3 =
0

C. D.

Rapid breathing Slow breathing

H2C03

H20 + C02

Although effective when changes are small, this

bicarbonate buffering system can be overwhelmed by a large


acid or base change in the blood. The result can be either

38. The decrease in blood pH associated with acidosis


would most likely NOT alter the:

acidosis (lower-than-normal blood pH) or alkalosis (higherthan-normal blood pH). Respiratory acidosis or alkalosis
results from impaired pulmonary gas exchange or an abnormal rate of breathing. Metabolic acidosis or alkalosis

refers to changes in pH caused by non-respiratory factors. Acid/base disorders trigger renal and/or respiratory compensation mechanisms that tend to return blood pH
towards normal levels (Figure 1).

A. B. C. D.

tertiary structure of blood proteins. secondary structure of blood proteins. oxygen affinity of hemoglobin. isoelectric point of endocrine peptide hormones.

39. In response to respiratory alkalosis, renal compensation


45

can help restore normal pH through increased bicarbonate excretion in urine. Which point on the graph in Figure 1 shows complete renal compensation for respiratory alkalosis?
A. B. C. D. Aj B2 C\ Dj

40

,J 35

30

0C?
U

DC S, 25 -

40.

20 -

A patient's blood bicarbonate levels are 45 mmol/L while his blood pH is 7.4. According to the information given in the passage, this patient most likely:

A.
15 -

is perfectly healthy.
suffers from metabolic acidosis.

B.

C.
D.

has elevated pC02 levels.


has blood alkalosis.

10

J
7.0 7.1

I
7.2

I
7.3

I
7.4

I
7.5

I
7.6

I
7.7

I
7.8

pH
Figure 1: Effects of renal and respiratory compensation on blood

41.

pH. N = normal pH and HCO3 levels. A = respiratory acidosis.


B = respiratory alkalosis. C = metabolic acidosis. D = metabolic alkalosis. Letters with subscripts depict compensatory effects.

Which of the following statements is NOT true with regards to the graph shown in Figure 1?

A.
B. C. D.

The metabolic acidosis depicted by point C may be caused by an abnormal excretion of


bicarbonate.

The underlying cause of the acid/base disorder has been cured at points A2 and B2. Point A2 represents respiratory acidosis with complete renal compensation. Point Ci represents metabolic acidosis with partial renal compensation.

Copyright by The Berkeley Review

347

GO ON TO THE NEXT PAGE

Passage VII (Questions 42 - 47)

42.

Based on information given in the passage, which of the following would NOT be a likely symptom of
multiple sclerosis?
A. Muscle spasms

Multiple sclerosis is a debilitating disease which affects


over 250,000 Americans. During the course of the disease, neurons in the white matter of the brain are progressively

demyelinated, resulting in axons which are unsheathed, or


naked.

B. C.
D.

Paralysis Fatigue
Numbness

Recent evidence suggests that multiple sclerosis may be


an autoimmune disease. Autoimmune diseases are caused by

a malfunction of the patient's own immune system, resulting in an immune response to self (i.e., non-foreign) tissue. The

progressive steps leading to nerve demyelination during


multiple sclerosis are outlinedbelow:

1. The patient is infected by a virus which contains an antigen that closely resembles a component of the myelin
sheath. Through exposure to this virus, the patient's immune

system is somehow provoked into attacking the myelin component which is mimicked by the viral antigen.
2. T cells which recognize the viral antigen are activated

and proliferate. They then travel through the blood stream to


the brain, where they cross the blood-brain barrier.
3. Once in the brain, the T cells recognize antigens present

43.

According to the passage, which of the following would NOT induce multiple sclerosis-like symptoms in
wild-type, normal mice?

on the surface of the myelin sheaths. These T cells then secretedamaging peptidecytotoxins which gradually destroy the myelin,exposing the naked axon.
4. T cells also stimulate B cells to proliferate and secrete

A.
B.

Injection of antibodies against components of the


myelin sheath. Infection with a virus whose antigen mimics a

C.

antibodies against oligodendroglial cells, which are normally responsible for repairing the myelin covering.
Because infection by viruses which mimic neural

D.

component of the myelin sheath. Autoimmunization against components of the myelin sheath. Injection of cytotoxins in the vicinity of neural
cells.

antigens doesn't always trigger the development of multiple


sclerosis, it has been suggested that there is a genetic

predisposition to the disease. In other words, those who are predisposed may be more likely to develop multiple sclerosis
after exposure to viral antigens.

44.

An individual with multiple sclerosis receives a

transplantof oligodendroglial cells from her unaffected


identical twin. Would such a transplant cure the
disease?

A.

B.
C.
D.

Yes; the transplanted oligodendroglial cells would prevent further autoimmune attack. Yes; the transplanted oligodendroglial cells would repair the damaged myelin sheaths. No; the transplanted cells would be rejected by the patient's immune system because they come
from a foreign source.

No; the transplanted cells would eventually be destroyed by the same autoimmune attack that originally caused the disease.
GO ON TO THE NEXT PAGE

Copyright by The Berkeley Review

348

45. Based on the passage, which of the following would


represent potential treatment(s) for a patient with multiple sclerosis?
I. II. Injection of antibodies against secreted T cell
cytotoxins.

47. For a virus, which of the following would be an evolutionary advantage of mimicking a host antigen?
A. B.

Such mimicry would cause the death of the host due to multiple sclerosis.

Injection of antibodies against viruses which mimic myelin-related antigens. III. Injection of antibodies against T cells which recognize myelin-related antigens.
A. B. C.
D.

The mimicked antigen would act as a decoy, drawing attack by the immune system but leaving
the virus unharmed.

C.

The virus could evade attack by the host's immune system because the antigen would be
considered as "self."

I only II only I and III only


I, n, and III

D.

There would be no evolutionary advantage; it is purely coincidental that these viral antigens
resemble host antigens.

46.

Rheumatic fever is a condition in which the immune

system attacks and scars the patient's heart valves.

Based on information given in the passage, what might


trigger this autoimmune disease?

A.

Increased blood pressure due to bacterial


infection.

B.
C. D.

Demyelination of nerve fibers innervating the


cardiac tissue.

Infection by an agent which displays antigens similar to those present on the heart valves. Direct infection of the heart valves by viruses.

Copyright by The Berkeley Review

349

GO ON TO THE NEXT PAGE

Questions 48 through 52 are NOT based on a


descriptive passage.

50.

During embryogenesis, the separation of cells into the


three individual germ layers (endoderm, mesoderm,
and ectoderm) first occurs during:
A. cleavage.

48. Individual cardiac muscle cells are electrically linked with one another through:

B. C.
D.

gastrulation. organogenesis.
neurulation.

A. B.
C.

synaptic junctions. gap junctions.


axonal extensions.

D.

acetylcholine-mediated neurotransmitters.

49.

The renal afferent arteriole is of a larger radius than the


efferent arteriole. What effect does this difference in

radius have on the functions of the kidney?

A.
B. C.

Reabsorption in the tubules is decreased.


Glomerular filtration rate is increased. ADH secretion is decreased.

D.

Blood flow to the kidney is decreased.

51.

Vertebrates are evolutionarily well adapted to life on land. Which of the following adaptations is LEAST likely to contribute to the vertebrates' land dominance?
A. B. C.
D.

Internal lungs Impermeable outer skin Short loops of Henle


Internal fertilization

Copyright by The Berkeley Review

350

GO ON TO THE NEXT PAGE

52. During the digestive process, the stomach and intestines undergo peristaltic contractions which help to push food along the digestive tract. These
contractions are:

A.

caused by the stimulation of skeletal muscle


fibers.

B. C. D.

inhibited by the stimulation of the vagus nerve. increased during times of fear or stress. enhanced by the actions of the parasympathetic
nervous system.

Copyright by The Berkeley Review

351

STOP, YOU ARE FINISHED

Biology
Passage I (Questions 1 - 5)

Physiology Diagnostic Set I

Answers

Thyroid Hormone

C is the best answer. When treating thyroid cancer, the goal is always to destroy the cancerous cells while leaving normal

body cells unharmed. Radioactive iodine is such an effective treatment because only thyroid cells allow it to enter them. When it gets inside these cells, the radiation given off by the iodine isotope destroys the tumorous cells. Other body cells don't take up iodine, so they are left unharmed by the radioactivity. Radioactive iodine represents a "magic bullet," selectively entering and destroying only thyroid cells, no matter where they have spread throughout the body. Let's consider
each of the answer choices in light of this knowledge. Choice A can be eliminated because it would result in extremely

severe side effects. If most cells of the body could take up radioactive iodine, then treatment with 13II would result in the destruction of most normal body cells, in addition to the tumorous thyroid cells. Choice Bcan be ruled out because if l3II
were non-toxic once inside cells, it would be an extremely ineffective treatment for cancer. It would be unable to kill the

cancerous thyroid cells at all. Choice D can likewise be eliminated. If cancerous thyroid cells can't absorb iodine from the
blood, treatment of thyroid tumors with radioactive iodine (which chemically behaves exactly the same as regular iodine)
would be useless. The 131I wouldn't enter the tumor cells and therefore wouldn't kill them. The best answer is C.

B is the best answer. Surgical removal of the thyroid gland would result in a decline in thyroid hormone (thyroxine)

production. From the first paragraph ofthe passage, we learn that thyroxine regulates basal metabolic rate. We are also told
that excess thyroxine secretion causes weight loss even though food intake is increased. This implies that thyroxine tends to increase metabolic rate, leading to a higher rate of cellular respiration. The excess heat produced in this process may lead to

the profuse sweating which is the other symptom of excess thyroid hormone secretion. Knowing what an excess of thyroxine may cause, we can infer what a decline in thyroxine production would do. Lower thyroxine levels would result in
a lower metabolic rate. This lower rate of cellular respiration would use less ATP. Therefore, the excess calories that a

person consumes would not be "burned" by this lower metabolism, resulting in weight gain. Normally, when the thyroid is surgically removed, thyroxine pills (i.e., hormonal supplements) are taken to maintain the basal metabolic rate at appropriate
levels. Let's consider the other answer choices. Choice A represents the consequences of increased thyroxine production. A

higher metabolic rate would lead to an increased appetite and food intake (necessary to fuel the increased metabolism).
Since we are looking for the effect of decreased thyroid hormone production, eliminate A. Choices C and D can be
eliminated for the same reasons. Profuse sweating is the result of excess thyroxine secretion (as mentioned above).

Likewise, excess oxygen usage by the tissues would be the result of increased cellular respiration caused by excess
thyroxine levels. The best answer is B.

B is the best answer. Figure 1 in the passage gives us the structure of thyroxine (a.k.a. thyroid hormone). Notice the hydroxylated benzyl rings, reminiscent of the structure of tyrosine, shown below:
II II ()

I
HO

II

-C-C-C-OH
NH,

NH2

Tyrosine structure of thyroid hormone:


ii

Thyroxine

It can be seen how thyroxine might be derived from tyrosine. The other answer choices, shown below, do not resemble the
ii ii o

II

II

II

C-C-C-OH

H,C SC - C - C - C - OH
I
H

II

i
The best answer is B.

I
H

I
NH,

I
H

I
NH,

Histidine

Methionine

order to produce thyroxine. Therefore, high TSH levels would be beneficial to l3lI treatment ofthyroid cancer because TSH

D is the best answer. From the passage, we learn that TSH stimulates thyroid cells to increase their uptake of iodine in

would increase the uptake of radioactive iodine by cancerous cells. After the surgical removal of the thyroid, thyroxine levels drop offover the course ofa few weeks. According.to Figure 1. if thyroxine levels drop, there would be less of an inhibitory effect on TSH secretion. This means more TSH would be secreted, resulting in more effective radioactive iodine therapy. This is why the therapy is most effective several weeks after the removal ofthe thyroid. By this time, TSH levels have risen, promoting the uptake of 13II by any remaining cancerous cells. Choice Ais invalid because ifcancerous thyroid cells spread further, the iodine treatment would be less effective at killing them all. Choice B can be eliminated because
352

,@ Copyright by The Berkeley Review11

The Berkeley Review

Specializing in MCAT Preparation

Biology

Physiology Diagnostic Set I

Answers

decreased thyroxine levels (as the result ofthyroid removal) would not lead to a decrease in TRH (see Figure 1). Choice C is wrong because thyroxine levels do not rise when the thyroid is removed. In any case, rising levels ofthyroxine would not
stimulate iodine uptake. The best answer is D.
5.

C is the best answer. In order for a radioactively labeled compound to destroy a cell, it must first actually enter the cell. In the passage, we learn that TSH is a peptide hormone. Recall that peptide hormones can't cross the plasma membrane of target cells. Generally, they act by binding to receptors on the surface of target cells. This binding then triggers second
messenger production, which leads to changes in the cell (e.g., gene transcription). In the case of radiolabeled TSH, the

radioactive molecule would never be able to enter its target (i.e., cancerous thyroid cells); rather, it would bind to a receptor
on the surface of these cells. This is not sufficient to destroy the tumorous cell. Let's consider the other answer choices.

Choice A can be eliminated because TSH does not actually enter the cells of the thyroid, as mentioned above. Choice B is

invalid because radiolabeled TSH can't make radiolabeled thyroxine, and even if it could, thyroxine's targets are not
cancerous thyroid cells. Choice D can also be ruled out because the only target of thyroid-stimulating hormone is the
thyroid. The best answer is C.

Passage II (Questions 6-11) 6.

Home Pregnancy Test

C is the best answer. In order to be indicative of pregnancy, a hormone must only be produced either by a mother who is pregnant or the fertilized embryo itself. Otherwise, false test results might be possible. Progesterone is normally produced by females who are not pregnant during the course of their monthly menstrual cycles. It is not purely a hormone associated
with pregnancy. The other answer choices do not offer valid explanations why progesterone can't be used. Choice A is

incorrect because antibodies can be made to almost any molecule, as long as that molecule will fit into the antibody binding site. Choice B is incorrect because progesterone is secreted by the corpus luteum throughout pregnancy. Choice D can be
eliminated because progesterone is definitely present in the bloodstream; this is how endocrine hormones reach their target
cells. The best answer is C.

7.

B is the best answer. From the passage we learn that hCG is a peptide hormone. Peptide hormones act by binding to
receptors on the plasma membranes of their target cells. This binding then triggers a signaling cascade which results in the

transcription of specific genes. Peptide hormones can't cross the plasma membrane directly because they are not lipid
soluble. Glucagon is a peptide hormone secreted by the alpha cells of the pancreas; it therefore uses a similar mechanism to

that of hCG. The other answer choices can be eliminated because they are steroid hormones. These act by directly crossing
the plasma membrane and binding to receptors either in the cytoplasm or in the nucleus. The best answer is B.

8.

A is the best answer. The test, as described in the passage, simply assays for the presence of hCG in the urine. The person using the test would either obtain a positive or negative result. Whether or not this result is valid remains in question because there are no experimental controls involved. In other words, there is nothing to test whether the test reagents are faulty or the procedure was done incorrectly. A positive control would involve using a sample liquid containing hCG and testing to see whether the pregnancy test could actually detect it. If it can't, then something is wrong. A negative control might involve using a urine sample with no hCG in it and seeing if the test gives a false positive result. Such controls are not included in the test as described in the passage. The other answer choices can be eliminated. Choice B is wrong because the passage states that hCG levels are high enough to be detected by the time of the first missed period. This could be anywhere from 7-14 days from the time of fertilization. Choice C can be eliminated because the mother does not produce hCG. If she did, the test would be entirely useless. Only the embryo produces the hormone. Choice D is also incorrect because the embryo must produce hCG in order to prevent menstruation and maintain the corpus luteum. If it doesn't, menstruation will occur and the embryo will be sloughed off with the uterine lining. The best answer is A. C is the best answer. In the passage it is stated that hCG is detected in the urine. In order to get there, the hormone must be filtered from the blood into the kidney tubules (via the glomerulus). The tubules must then not reabsorb all of it back into
the blood; instead, the hCG must be allowed to leave in the urine. The other answer choices can be eliminated after

9.

consideration. Choice A is wrong because hCG is only secreted by the fertilized embryo; if it was also secreted by the mother, the pregnancy test would not be effective because hCG could be detected in non-pregnant mothers. In either case, the passage does not imply that the mother makes hCG. Choice B can be eliminated because the passage directly states that hCG is released into the mother's bloodstream. Choice D is wrong because hCG works to maintain the corpus luteum, which secretes progesterone. If anything, hCG works to maintain the production and function of progesterone, not
counteract it. The best answer is C.

10.

D is the best answer. Answering this question simply requires picking the answer choice that doesn't have anything in common with the others. Choices A, B and C all represent conditions in which specific antibodies are produced which might be tested for using the same technique described in the passage. After all, antibodies are proteins, and monoclonal antibodies can be made to detect them. HIV infection can be detected by assaying for the antibodies against HIV. Arthritis can be detected by looking for antibodies against "self" proteins. Strep throat can be detected by directly assaying for the

Copyright by The Berkeley Review

353

The Berkeley Review Specializing in MCAT Preparation

Biology

Physiology Diagnostic Set I

Answers

presence of bacterial proteins. The same technique which was used to detect hCG can be used in these instances. Tryptophan blood levels, however, can not be determined using the same methodology. For one thing, tryptophan (an amino acid) does not elicit antibody production. Secondly, the actual levels of substances in the blood can't be detected by the type
11

of test described in the passage. This test yields either a"present" or "not-present" result. The best answer is D. C is the best answer. Antibodies are made by B-cells, not T-cells (eliminate choices Aand B). Monoclonal antibodies, as

antibodies. Monoclonal antibodies only strongly recognize and bind to one specific antigenic site. The hCG molecule, being

their name implies, are produced by a single clone of B-cells. In other words, a single B-cell is provoked to divide, producing a clone of identical cells. These cells then crank out one single type of antibody. These are monoclonal

a polypeptide, may have multiple antigenic sites that can be recognized by antibodies, but a given set of monoclonal
antibodies only recognizes one ofthese sites. Eliminate choice D. The best answer is C.
Not Based on a Descriptive Passage

(Questions 12 -15)

12.

Bis the best answer. The question asks what would LEAST contribute to the whale's ability to remain submerged for long

would be detrimental to long stays underwater where no new oxygen can be obtained. Stores of oxygen obtained on the surface would more rapidly be consumed by a high metabolism, decreasing the ability to remain underwater. The other answer choices all represent advantages for prolonged submersion. Choice A can be ruled out because large muscle

periods of time. Ahigh basal metabolic rate represents a large demand for oxygen by the rapidly respiring tissues. This

myoglobin concentrations would help to store oxygen and deliver it most efficiently to the muscles which need it during the

dive. Choice C can be eliminated because a high cellular tolerance for CO2 would be beneficial to a diving animal, as CO2

would build up during the dive since itcould not be exhaled. Choice Dcan also be thrown out because a large lung capacity
would facilitate a better initial oxygen intake before the dive, allowing more oxygen to be available during submersion. The
best answer is B.

13.

A is the bestanswer. Pepsin is a protease secreted by the chief cells lining the stomach (initially in the form of the zymogen

pepsinogen). All of the other answer choices represent enzymes secreted by the pancreas. Lipase aids in the digestion of fats. Chymotrypsin and carboxypeptidase are both proteases (i.e., they break down proteins). The pancreas secretes its
enzymes intothe small intestine. The best answer is A.
14. C is the best answer. Colchicine affects microtubules, the strands of polymerized protein which make up part of the

cytoskeleton. We must therefore pick the answer choice that is associated with some sort of microtubule-driven process. DNA and protein synthesis are not such processes. These do not depend on microtubules and therefore would not be affected by colchicine (eliminate choices A and B). Muscle contraction depends on actin and myosin, not microtubules
(eliminate choice D). This leaves us with mitosis as the answer. During mitosis, chromosomes are attached to microtubules which serve to line them up and pull them apart. In fact, the spindle apparatus is predominantly made up of microtubules.

The polymerization and depolymerization of microtubules is vital to mitotic division. Colchicine thereby prevents mitosis.
The best answer is C.

15.

A is the best answer. Slightly increased levels ofCO2 in the air we breathe stimulates an increase in breathing rate. This is because breathing such air causes arterial levels ofCO2 to rise, telling the breathing centers of the brain that CO2 buildup is occurring. The brain then stimulates an increase in ventilation (hyperventilation) to try to exhale as much of the blood COt as possible. Choice B is therefore incorrect. Choices C and Dcan also be eliminated because small changes in CO2 levels
would not significantly affect blood pressure in any way. The best answer is A.
Oral Rehydration 1 herapy

Passage III (Questions 16 - 20)


16.

C is the best answer. The question asks whether administration of an oral rehydration therapy solution would reduce the extent of fluid loss through diarrhea. From the last paragraph of the passage, we learn that the standard ORT solution outlined in Table 1exactly replaces the fluid that is lost due to toxin-induced diarrhea. This means that the diarrhea is still occurring and fluid is being lost, but whatever is lost from the body is replaced by intestinal absorption of the ORT solution. Since the volume of liquid leaving through diarrhea is unchanged, we can conclude the ORT does not reduce the extent of diarrhea; it simply prevents the patient from experiencing a net loss of fluid while the diarrhea is continuing. A cholera

patient who is given an ORT solution still experiences the same amount ofdiarrhea; the ORT solution simply prevents total dehydration by keeping blood fluid and electrolyte levels fairly constant in the face of this massive loss. We can eliminate

choice A because ORT does not promote a net flow of water out of the blood and into the intestinal lumen. Rather, the

passage tells us that it promotes the bloodward flow of water and electrolytes by taking advantage of sodium/glucose cotransport mechanisms. Additionally, the ORT solution is hypotonic (i.e., it has a lower osmolarity. orsolute concentration)
relative to the dehydrated (hypertonic) blood; water flows by osmosis from a hypotonic solution to a hypertonic one (or

Copyright by The Berkeley Reviewr

354

The Berkeley Review

Specializing in MCAT Preparation

BlOlOgy

Physiology Diagnostic Set I

Answers

from a solution oflow osmolarity to a solution ofhigh osmolarity). Since the ORT solution does not increase the osmolarity
of the intestinal lumen, we can eliminate choice B. Choice D can be ruled out because in the passage we learn that ORT
exactly replaces fluid that is lost through diarrhea. The best answer is C.

17.

C is the best answer. Diarrhea causes massive fluid loss. This lost fluid contains intestinal secretions (i.e., electrolytes and
bicarbonate, HCO3"). Recall that the pancreas produces bicarbonate ions and secretes them into the lumen of the intestine. In the case of cholera, however, this bicarbonate is excreted at a high rate in the diarrhea. Removal of bicarbonate alters the

acid/base composition of the blood; the blood becomes more acidic (less basic) and acidosis results. This type of acidosis is considered metabolic because it is not caused by respiratory problems (as in respiratory acidosis). In order to compensate for the increased acidity of the blood, the breathing rate becomes faster. This allows CO2 to be exhaled at a greater rate,
lowering the acidity of the blood. The other answer choices can be eliminated after consideration. A cholera victim would

be dehydrated. This means that the victim's blood volume would be low, lowering blood pressure. This directly eliminates answer choice B and indirectly eliminates answer choice A. Recall that glomerular filtration rate (GFR) would decrease if the hydrostatic pressure of the blood were lower, as it is in cholera. This means that a cholera victim would not experience
an increased GFR, and therefore there would be no increase in urine production. Choice D can be ruled out because, as we
discussed above, loss of bicarbonate (a basic anion) lead to metabolic acidosis, not alkalosis. The best answer is C.

18.

B is the best answer. The question asks for the mostimportant function of trisodium citrate. Our major clue comes from the
footnote to Table 1, which states that trisodium citrate is used instead of the more unstable sodium bicarbonate. As we

know, bicarbonate is a basic anion which is an important part of the blood buffering system. In cholera, vast quantities of
bicarbonate are lost due to diarrhea, and the patient's blood becomes acidotic. The ORT solution must address this acid/base imbalance. It does so by including trisodium citrate, which dissociates to form basic citrate anions, removing some of the excess H+ in the blood by forming citric acid (the conjugate acid of citrate). We can infer that this is the function of trisodium citrate by thinking about what bicarbonate would do if included in the ORT solution. Bicarbonate would raise the

pH of the blood by combining with H+. It is not included in the ORT solution because it is unstable; since trisodium citrate
is its replacement, we can infer that trisodium citrate has a similar pH-raising function. Answer choice A is extremely tempting; after all, trisodium citrate has plenty of sodium to put into solution. This is not its most important function, however. Sodium chloride provides most of the sodium needed in the solution. Trisodium citrate's most important function is to fix the blood's abnormally low pH. Choice C can be eliminated because although trisodium citrate does increase the osmolarity of the ORT solution (by increasing the concentration of solute), this is not its most important function (nor is it a function exclusive to trisodium citrate). Every component of the solution raises the osmolarity of the solution, but only trisodium citrate alters acid/base balance. Choice D can be thrown out because adding more solute causes an increase, not a
decrease, in osmolarity. The best answer is B.

19.

D is the best answer. Recall from the passage that the osmolarity, or solute concentration, of the standard ORT solution is equal to that of normal blood. This makes the ORT solution hypotonic relative to the dehydrated blood of the patient. From our knowledge of osmosis, we know that water flows from a hypotonic solution to a hypertonic solution. The oral rehydration therapy is successful because water is not drawn into the lumen of the intestine by osmosis. The lower solute concentration of the ORT solution relative to the higher solute concentration of the blood prevents this flow (known as an osmotic penalty). Since we are trying to increase the bloodward flow of water and electrolytes, such an osmotic penalty would be counterproductive. This is why the concentrations of glucose and sodium in the ORT solution can't be much higher than what is given in Table 1. If they are, the osmolarity of the solution will exceed that of the blood and water will flow into the lumen of the intestine from the blood. Even though the sodium/glucose co-transporter would function at a higher rate (due to the higher concentrations of its substrates), it would not entirely make up for this osmotic penalty. If we don't consider this, we might have picked choice A; knowing that increased sodium and glucose concentrations would not result in net increased fluid uptake (due to osmotic penalty), we can eliminate choice A. Choice B can also be ruled out because the passage states that sodium and glucose do affect fluid uptake. Choice C is incorrect because water itself cannot directly be actively transported. The best answer is D. A is the best answer. Potassium is one of the major electrolytes lost through cholera-induced diarrhea. Potassium

20.

replacement is important because K+ is vital in maintaining membrane potentials and muscle function (recall that action
potentials involve potassium and potassium channels). The other answer choices can be eliminated after careful consideration. Choice B is incorrect because neither K+ nor CI"can increase Na+ absorption in the small intestine. Choice C
can be thrown out because the lumenal chloride concentration in cholera victims is already too high. Recall that cholera toxin induces crypt cells to secrete chloride ions into the intestinal lumen. There is no therapeutic need to further boost the

concentration of CI" in the lumen. Choice D can likewise be eliminated because there is no K+/C1* antiporter. Additionally,
why would we add KC1 just to activate these hypothetical antiporters? This would accomplish nothing of therapeutic use.
The best answer is A.

Copyright byTheBerkeley Review

355

The Berkeley Review


Specializing in MCAT Preparation

Biology
Passage IV (Questions 21-25)
21.

Physiology Diagnostic Set I

Answers

Botulinum loxin

B is the best answer. Recall from the passage that the active portion of the botulinum toxin is a large protein conjugated to a series of accessory proteins. Certain patients, upon initial injection of BT, respond well but become tolerant to further

acetylcholine secretion from the presynaptic terminals of the neuromuscular junction (causing these terminals to atrophy

injections, making the therapy ineffective in the long run. What could be happening in these patients? One explanation is that upon initial injection, the immune system recognizes the protein toxin as foreign and begins synthesis ofantibodies against it. By the time the antibodies are made, the botulinum toxin has already served its function of irreversibly preventing
from lack of use). This initially relieves the focal dystonia by preventing the muscle contractions which cause it. Over time, however, new terminals sprout to replace the old damaged ones, and the muscle eventually becomes renervated. This is why BT therapy only lasts a few weeks. For the patients who produced antibodies to the toxin, further injections ofBT would be
useless because the antibodies would bind to and inactive the active portion of the toxin before it could perform its function. These individuals have become tolerant to the therapy. Let's eliminate the other answer choices. Choice A is incorrect

because acetylcholine is the only major neurotransmitter used in the neuromuscular junctions to trigger muscle contraction.
Even if another transmitter were used, this would not account for the initial effectiveness of BT therapy in these patients.

Choice C is a true statement; atrophied nerves do regenerate over the course of several weeks (this causes the return of the focal dystonia by restoring innervation of the affected muscles). This would not, however, explain the tolerance to BT that
causes it to become ineffectual in certain patients. After all, several weeks is a long time for nerves to regenerate; we would

expect BT to cause alleviation of symptoms for at least that long. Eliminate choice C. Choice D can easily be thrown out
because if affected muscle fibers became permanently disabled in these patients, there would be no need for further therapy because the dystonia would be cured. The best answer is B.
22.

B is the best answer. From the passage, we learn that botulinum toxin acts by causing an irreversible blockage of acetylcholine release. Recall that acetylcholine is normally released from the presynaptic terminal; after crossing the synaptic cleft, it binds to receptors on the endplate membrane (on the muscle side of the neuromuscular junction). This binding triggers a series of depolarizations that ultimately lead to muscle fiber contraction. If acetylcholine doesn't bind to its receptors on the endplate membrane, muscle contraction can't occur; this is the normal physiological effect of botulinum
toxin. In order to mimic this effect, we must prevent acetylcholine from binding to its receptors. One way to do this is to

introduce a large amount ofacetylcholinestera.se directly into the space between the endplate membrane and the presynaptic
terminal. Recall that acetylcholinesterase breaks down acetylcholine. Adding extraneous acetylcholinesterase would prevent

acetylcholine from binding to its endplate membrane receptors, effectively mimicking the effects of botulinum toxin. We can also approach this problem through a process of elimination. Choice A can be ruled out because calcium, when
introduced into the neuromuscular junction, would not cause cessation of acetylcholine release, nor would it prevent

acetylcholine from binding to its receptors. Choice C is invalid because injection of Ca-+ into muscle fibers would cause
contraction of the muscle fibers; remember that calcium binding allows interaction between actin and myosin. Muscle

contraction is normally prevented by botulinum toxin, so injection of calcium would not mimic BT's effects. Choice D can be eliminated because acetylcholine, when injected into muscle fibers, would have no effect because there would be no
23.

binding to outer membrane receptors. The best answer is B. A is the best answer. Recall from the passage that botulinum toxin exerts its effects by blocking the calcium-mediated

release of acetylcholine. Acetylcholine is normally released from the presynaptic terminal underthe influence of a changing
intracellular calcium concentration; therefore, BT must exert its effects here. Choice B is incorrect because acetylcholine is

not released from the postsynaptic terminal (or endplate membrane, in this case). Choice C can be ruled out because

calcium-sequestering organelles in muscle cells (i.e., sarcoplasmic reticulum) have no influence on acetylcholine secretion. They are instead responsible for altering intracellular calciumconcentrations in order to regulate muscle contraction. Choice
D can be ruled out because the nodes of Ranvier do not secrete acetylcholine. They simply facilitate faster transmission of nerve action potentials. The best answer is A.
24.

C is the best answer. Focal dystonias are localized, sustained abnormal contractions of muscle fibers. Whatever causes them must continuously stimulate the muscle fibers to contract. From the passage, we learn that botulinum toxin, which operates on the level of the presynaptic terminal of the neuromuscular junction, can relieve the abnormal contractions encountered in focal dystonias. The cause of the dystonias must therefore lie somewhere at or before the level of the presynaptic terminal. Since signals for muscle contraction originate in the central nervous system and then travel to the

neuromuscular junction, a defect at the level of the CNS could cause the focal dystonia. A defective contraction signal sent
from the CNS would be blocked at the level of the presynaptic terminal (via acetylcholine blockage) by botulinum toxin.
We can eliminate the other answer choices. Choice A is incorrect because a defect in the sarcoplasmic reticulum would lie at the level of the muscle fibers themselves. This can't be the case, because we learn from the passage that botulinum toxin

counteracts focal dystonias by preventing acetylcholine secretion. BT would have no effect if the dystonia were caused by a
defect in the muscle fibers themselves. Choice B is a tempting answer, but we must make sure we understand the terms

used. Afferent nerves lead away from the muscles and towards the central nervous system (as opposed to efferent nerves
Copyright by The Berkeley Review,
356

The Berkeley Review

Specializing in MCAT Preparation

Biology

Physiology Diagnostic Set I

Answers

which lead from the CNS to the effector organs or muscles). Overstimulation by afferent fibers would therefore affect the

CNS, not the muscle fibers; this is most likely not a cause of the sustained muscle contraction seen in focal dystonias. Choice D can be eliminated because an oversecretion of acetylcholinesterase would cause increased degradation of
acetylcholine and therefore would induce paralysis, not contraction, of muscle fibers. This can't be the cause of focal
dystonias. The best answer is C.

25.

C is the best answer. According to the first paragraph of the passage, botulinum toxin is produced when the bacterium C. botulinum grows anaerobically. In order to be contaminated with BT, a food product must be present in an environment that does not contain oxygen, forcing any C. botulinum that are present to grow anaerobically (without O2). This would cause the production of botulinum toxin. Canned foods are packaged such that there is very little oxygen inside the can. This is an
ideal anaerobic environment. Incidentally, the potentially fatal disease resulting from ingestion of BT is called botulism. Choices A and B can be ruled out because these foods are generally exposed to some oxygen. Choice D can be eliminated because food left uncovered is also exposed to oxygen. None of these answer choices represent foods that would provide a good anaerobic environment. The best answer is C.

Passage V (Questions 26 - 32)


26.

Circulatory Shock

C is the best answer. Recall from the passage that hypovolemic shock results from a low blood volume which tends to lower blood pressure and cardiac output. The less blood volume present in the system, the less blood the heart is able to pump at any given time, thereby lowering cardiac output: since blood pressure is directly proportional to cardiac output, blood pressure is lowered as well. The ultimate result is decreased blood flow to the tissues, resulting in circulatory shock. Since hypovolemic shock is the result of lowered blood volume, we can eliminate responses that tend to lower the volume of the blood. Choice A, hemorrhaging, refers to excessive bleeding as the result of some sort of damage to blood vessels. Since hemorrhaging results in blood loss, it decreases blood volume. Eliminate choice A. Choice B, dehydration, involves the loss of water from the blood plasma. This also would tend to lower the volume of the blood, so eliminate choice B. For the same reasons, choice D can be ruled out because blood loss due to injury (similar to hemorrhaging) could cause hypovolemic shock due to decreased blood volume. This leaves us with choice C, heart failure, as the correct answer. Although heart failure may lead to some type of circulatory shock (i.e., inadequate blood flow to the tissues), it would not reduce the volume of the blood present in circulation. Therefore, it would not directly cause hypovolemic shock. The best
answer is C.

27.

A is the best answer. The question asks about preventing any further decrease in blood volume during hypovolemic shock. Recall that the kidneys are normally very active in regulating the volume of the blood through both the reabsorption and excretion of water (as urine). The kidneys can lower blood volume by excreting more urine or they can maintain blood volume by excreting less urine (and reabsorbing more water in the tubules). In this case, we want to prevent blood volume loss and therefore we want to prevent excessive urination. This can be accomplished by constricting the renal afferent arterioles. Recall that these arterioles bring blood into the glomeruli for filtration into the glomerular capsule. If these arterioles are constricted, less blood can get to the glomeruli and produces a lower glomerular filtration pressure, thereby filtering less fluid and resulting in a decrease in urine production. From the passage, we learn that the physiological

responses that compensate for hypovolemic shock are sympathetic in nature. We can therefore conclude that sympathetic
stimulation of the renal afferent arteriole would cause its constriction, thereby reducing the amount of urine produced in the kidneys. Remember that the parasympathetic nervous system does NOT innervate the kidney at all, and therefore can't be an answer choice. If we couldn't recall this fact, we might also remember that parasympathetic stimulation generally has a relaxing effect anyway, and probably wouldn't cause constriction of the renal arterioles. Eliminate choices B and D. With regard to choice C, constriction of the efferent arteriole would not have as great an effect on urine production. This is because the efferent arteriole takes blood away from the glomerulus. Constricting it wouldn't affect urination as greatly as constriction of the afferent arteriole (which takes blood to the glomerulus). The best answer is A.

28.

B is the best answer. Recall that hypovolemic shock is caused by a decrease in blood volume. In order to treat it, we might try to bring the volume of the blood back up. One way to do this is to transfuse (inject) more blood plasma into the person suffering from the shock. This would add more to the volume of the blood, raising blood pressure and increasing cardiac output. Choice A is incorrect because administration of diuretic drugs would cause diuresis, or water loss due to urination. If
more water were lost from the blood, the blood volume would decrease further. This would not be an effective treatment; it

would in fact be very harmful. Choice C is also invalid because epinephrine (a.k.a. adrenaline) is a major effector molecule of the sympathetic nervous system; adding an epinephrine antagonist would decrease the activity of epinephrine, thereby decreasing sympathetic function. Recall from the passage that the mechanisms by which the body tries to compensate for hypovolemic shock are mainly sympathetic in nature. By blocking these beneficial responses, the patient suffering from shock would actually get worse. Choice D, injection of acetylcholine antagonists, does not represent a valid treatment for hypovolemic shock either. By blocking acetylcholine (a neurotransmitter), the patient might become paralyzed, a situation
of no benefit in this case. The best answer is B.

Copyright by The Berkeley Review1

357

The Berkeley Review Specializing in MCAT Preparation

Biology
29.

Physiology Diagnostic Set I

Answers

C is the best answer. Recall that ADH (anti-diuretic hormone) and aldosterone are both hormones which tend to decrease water excretion in the form of urine. These hormones thereby increase blood pressure by promoting the reabsorption of

water in the kidney (and preventing urine production). We could answer this question even if we had forgotten the functions of ADH and aldosterone. Recall that the question states that secretion of these hormones represents a physiological

compensation for hypovolemic shock (i.e., low blood volume). Raising blood pressure is the only answer choice that would compensate for the loss ofblood volume and blood pressure encountered in shock. Choice Ais incorrect because if the two
hormones increased urine volume, they would increase water secretion and therefore lower blood pressure even further. Choice B can be ruled out because if salt reabsorption were decreased, the excreted salt would cause water to follow,

lowering blood volume and pressure further. Aldosterone actually promotes an increase in salt reabsorption. Choice D can be eliminated because inhibiting the sympathetic nervous system would not represent a physiological compensation for hypovolemic shock. Recall from the passage that the sympathetic system actually helps to compensate for shock; inhibiting
it would cause more harm than good for the patient suffering from hypovolemic shock. The best answer is C. 30. B is the best answer. In the question, we are told that histamine is a vasodilating hormone. This means that it promotes the dilation (expansion) of blood vessels. Anaphylactic shock is a severe allergic response (e.g., to a bee sting) which results in body-wide histamine release. This release promotes the dilation of many blood vessels in the body. According to the last paragraph of the passage, an increase in the radius of a blood vessel (as occurs during dilation) results in a decrease in the resistance of that vessel. From the equation relating blood pressure to cardiac output and peripheral resistance, we can see
that this decrease in resistance would cause a drop in the blood pressure. Since histamine causes a lot of blood vessels to dilate, the total peripheral resistance is greatly reduced, causing blood pressure to plummet, resulting in inadequate blood
flow to the tissues and thus shock. With this knowledge, we can eliminate choice A, because histamine does not cause an

increase in total peripheral resistance. Choice C can be eliminated, because an increase in cardiac output would not cause
shock. Likewise, we can rule out choice D, because an increase in blood pressure would not necessarily cause shock. Dilation of blood vessels by histamine neither raises cardiac output nor increases blood pressure. The best answer is B.

31.

D is the best answer. In order to compensate for hypovolemic shock, we must somehow try to increase blood pressure so that the tissues may be adequately provided with blood. One such way to do this is to raise the heart rate. From the last paragraph of the passage, we learn that cardiac output is equal to the heart rate times the stroke volume. If heart rate increases, cardiac output goes up. Since blood pressure is directly proportional to cardiac output, blood pressure would rise as well. Therefore, increasing the heart rate might be an effective physiological compensation for hypovolemic shock. Choice A is actually counterproductive because extensive sweating would rob the blood of yet more water, decreasing its volume even further. Choice B would result in decreased total peripheral resistance. As we learn from the equation given in the passage, decreasing such resistance leads to a further lowering of blood pressure. Choice C can be ruled out because a
decrease in stroke volume leads to a reduction in cardiac output which leads to a decrease in blood pressure. The best
answer is D.

32.

A is the best answer. Even though there is more blood volume in the veins and venules at any given time, they have a lesser pressure than the arteries and arterioles, which have less blood volume in them at any given time. One reason (among several) for this is that the venous blood vessels have a greater compliance; that is, they can expand more easily. When they expand, their radius increases and the resistance to blood flow decreases, thereby lowering the pressure according to the various relationships detailed in the passage. The arterial vessels are less compliant; they don't expand as easily (due to their more extensive smooth muscle component) and their radii are thereby small, increasing the pressure of the blood inside them. Knowing this, we can eliminate choice B. Choice C has little to do with the decreased venous blood pressure; the one way valves simply keep blood flowing back to the heart, preventing backflow (due to the low pressure of venous blood). Choice D is a smoke screen; veins aren't especially permeable to water. The best answer is A.

(Questions 33 - 36)
33.

Not Based on a Descriptive Passage

D is the best answer. This question asks which mechanism of thermoregulation does NOT defend the body against elevated temperatures. Let's consider the answer choices. Choice A represents an effective defense against increased temperature. If the sphincters regulating blood flow to the skin capillary beds were to dilate, or open, blood would rush to the surface of the
skin, carrying heat with it. This heat could then be dissipated to the environment, cooling the body. Choice B is also an effective cooling mechanism. Sweat glands secrete liquid onto the surface of the skin. When the liquid evaporates, it carries heat away with it. Choice C would also help to cool the overheating body. If activity in the skeletal muscles were to decrease, this would lead to less metabolic heat production. This leaves us with choice D as the correct answer. Piloerection

of hairs on the skin surface (i.e., goosebumps) result in hairs which stand perpendicular to the skin. This interferes with the flow of air over the skin, resulting in a conservation of body heat. Goosebumps form when the body is trying to keep warm,
not when it is overheating. The best answer is D.

Copyright by The Berkeley Review**

358

The Berkeley Review Specializing in MCAT Preparation

Biology
34.

Physiology Diagnostic Set I

Answers

A is the best answer. In severe asthma, it takes more effort to inflate the lungs. Recall that such inflation is an active process, carried out by contractions of the diaphragm and chest accessory muscles (such as the intercostals). If it is more difficult to carry out inflation (or inspiration), these muscles have to work harder, and as a result they eventually become
larger (hypertrophy). This gives certain asthmatics a "barrel chested" look. Choice B therefore can be eliminated, because the size of the diaphragm will not be reduced if it must work harder all the time. Choice C can be ruled out because

asthmatics have a difficult time breathing; this would lead to higher levels of CO2 in their blood because their lungs can't efficiently excrete it. Choice D is likewise incorrect because asthmatics can't breathe efficiently enough to raise their blood
O2 levels above normal. The best answer is A.

35.

C is the best answer. The corpus luteum is formed from the secondary follicle after the egg is released during ovulation. After its formation, it begins to secrete progesterone. The other answer choices do not represent structures that secrete appreciable amounts of progesterone. The primary follicle will later differentiate into the secondary follicle, which will
eventually release the ovum, or egg. As a note, when the secondary follicle bursts and releases the ovum, a small amount of progesterone is released. This amount is far less than what is later secreted by the corpus luteum, however. The best
answer is C.

36.

D is the best answer. Cell-mediated immunity is the branch of the immune response that involves direct cellular attack on invading organisms or infected body cells. This is opposed to humoral, or antibody-mediated immunity. B-cells secrete antibodies; they are the main source of humoral immunity. B-cells do directly participate in cell-mediated immune responses. Macrophages are lymphocytes which phagocytose foreign material. Killer T-cells are activated by helper T-cells to destroy cells of the body which are infected. Both participate in the cell-mediated response. Incidentally, helper T-cells also serve a role in activating B-cells, and are therefore part of the humoral immune response. The best answer is D.
Acid/Base Disorders

Passage VI (Questions 37-41)


37.

A is the best answer. Vomiting involves ejecting the contents of the stomach. These stomach contents contain a large
amount of H+ due to the acidic secretion of HCl that occurs in the stomach. When these acidic contents are lost, more HCl

must be secreted to offset the loss, causing the blood H+ concentrations to drop; this raises the pH of the blood, causing
alkalosis. We refer to this condition as metabolic alkalosis because it results from a problem that does not have to do with

the lungs (as in respiratory alkalosis). Incidentally, notice that by lowering the concentration of H+ in the blood, the concentration of HCO3" increases according to the equation given in the passage. This occurs because the removal of H+ (a
reactant) shifts the entire equation to the left, producing more HCO3" (which is also a reactant). To confirm this, take a look at point D on the graph. This point depicts uncompensated metabolic alkalosis. Notice that pH levels are raised, as are HCO3" levels. The other answer choices can be ruled out after consideration. Choice B can be eliminated because diarrhea

would result in the excretion of large amounts of bicarbonate (HCO3") , thereby lowering the pH (by increasing the |H+] )and causing metabolic acidosis. Choice C, rapid breathing, would lead to respiratory alkalosis. This is because CO2 would be lost in large amounts during rapid breathing, shifting the equation shown in the passage to the right, lowering both the H+

and bicarbonate concentrations (see point B on the graph in Figure 1). Since the [H+] is lower, the pH is raised. Choice D,
slow breathing, is also incorrect because this would lead to respiratory acidosis. Slow breathing allows CO2 to build up in

the blood, shifting the bicarbonate equation to the left, producing more H+ and bicarbonate (see point A on the graph in
Figure I). The best answer is A. 38. D is the best answer. Endocrine hormones, by definition, circulate in the blood in order to reach their target tissues. A decrease in pH associated with acidosis would not affect the isoelectric point of these peptides, though. Recall that isoelectric point is an inherent property of a peptide, determined by the pKa's of the various side chains. The isoelectric point is the point at which a protein has no net charge. By lowering the pH of the blood, we do not directly change the isoelectric point of individual peptides; the only way to do that is to change the amino acid sequence of these peptides. Let's consider the other answer choices. Both the secondary and tertiary structures of proteins can be altered by changes in blood pH. This is because secondary structure is largely determined by hydrogen bonding and ionic interactions. If the medium becomes "more acidic, more side chains may become protonated and hydrogen bonding and ionic interactions can be altered. This alteration of secondary structure (i.e., alpha helices and beta-pleated sheets, etc.) can lead to a change in the tertiary, or folded, structure of the protein. Therefore, eliminate choices A and B since the question asks for the answer choice that would NOT be affected by a lower pH. Choice C can also be eliminated because hemoglobin's affinity for oxygen is definitely altered by a drop in pH. Recall that hemoglobin binds oxygen less tightly at lower pH's. This is called the Bohr Effect. Th The best answer is D.

Copyright by The Berkeley Review .if!

359

The Berkeley Review Specializing in MCAT Preparation

Biology
39.

Physiology Diagnostic Set I

Answers

B is the bestanswer. Recall that respiratory alkalosis involves an increase in blood pH that is due to an increase in the rate

of breathing. Such an increased breathing rate would excrete an excess amount of blood CO2, shifting the bicarbonate

equation given in the passage to the right (because CO2 levels are low). This lowers both the H+ and HCO3" concentrations
in accordance with Le Chatelier's principle. In other words, since we have taken away products (CO2) we must shift the

equation to the right to compensate for this stress to the equilibrium system; reactants (H+ and bicarbonate) are consumed. The drop in H+ levels raises the pH. The question states that the kidney can help to compensate for this higher pH by
excreting HCO3" (renal compensation). Such excretion would cause the equation given in the passage to shift to the left, raising H+ concentrations. According to Figure 1, point Bdepicts uncompensated respiratory alkalosis (notice the higher pH
and lower bicarbonate levels). Letters with subscripts represent compensatory effects. Point B2 represents complete renal

compensation for respiratory alkalosis. We can tell this because at point B2, the blood pH has returned to a normal 7.4, while the HCO3" levels have dropped even further (due to excretion by the kidney). We can rule out the other answer
choices. Point A2 depicts compensation for respiratory acidosis. Eliminate choice A. Point Ci represents partial compensation for metabolic acidosis. Eliminate choice C. Point D\ represents partial compensation for metabolic alkalosis.
Eliminate choice D. Remember to carefully read the legend to Figure 1 in order to stay on the right track. The best answer
isB.

40.

C is the best answer. The best way to approach this problem is to use the values given in the question and locate the corresponding point on the graph given in Figure 1. Bicarbonate levels of 45 mmol/L and a pH of 7.4 correspond to point A2 on the graph. Notice that at this point, blood pH has normalized but HCO3" levels are above normal. What could be going on with this patient? From the legend to Figure 1 we learn that point A represents uncompensated respiratory acidosis, while the letters with the subscripts represent compensatory effects. Point A2 therefore depicts an underlying respiratory
acidosis that has been compensated for by an increase in bicarbonate levels (shifting the reaction shown in the passage to the right and lowering the abnormally high H+ levels; see answer explanations to previous questions). Recall that respiratory acidosis involves the failure of the lungs to excrete enough CO2. This leads to a buildup of CO2 in the blood, shifting the bicarbonate reaction (shown in the passage) to the left, increasing the H+ and bicarbonate concentrations, thereby lowering

the pH and causing acidosis. In this patient, although the pH levels have normalized, the underlying high blood CO2 levels
are still present; their effect is simply minimized by the compensation of higher bicarbonate levels. The patient is therefore not perfectly healthy; eliminate choice A. He also does not suffer from metabolic acidosis. His problem is respiratory in
nature; eliminate choice B. Likewise, he does not suffer from alkalosis; rule out choice D. The best answer is C.

41.

B is the best answer. It might be helpful to refer to the answer explanations for the previous questions in order to get a better background for approaching this one. Points A2 and B2 depict areas where the pH has been normalized by

compensatory mechanisms that either increase or decrease bicarbonate levels in response to the underlying acidosis or
alkalosis. While these bicarbonate compensation mechanisms do return the pH to its normal value, the underlying disease that disrupted the pH in the first place has not been cured. Its effects are simply being masked by compensation mechanisms (such as renal compensation mentioned in an earlier question). An example of this is given in the previous question about the patient who has underlying respiratory acidosis with complete compensation. Let's eliminate the other answer choices, keeping in mind that the question is looking for the FALSE statement. Choice A is a true statement and can therefore be eliminated. Metabolic acidosis (point C) may be caused by an abnormal HCO3" excretion (as in chronic diarrhea). This
would raise H+ levels, thereby lowering the pH and causing acidosis that is not related to respiratory problems (hence

making it metabolic acidosis). Choice C is a true statement because point A2 depicts a normal pH with elevated bicarbonate levels, representing respiratory acidosis with complete renal compensation (the kidneys reabsorb more bicarbonate, raising HCO3" levels and lowering H+ levels). Eliminate choice C. Choice D is also a true statement. Point C\ represents a partial renal compensation for the metabolic acidosis depicted in point C. Eliminate choice D. The best answer is B.

Passage VII (Questions 42 - 47)


42.

Multiple Sclerosis

A is the best answer. The question asks for the answer choice which does NOT represent a potential symptom of multiple sclerosis. Recall from the passage that the disease causes the destruction of the myelin sheaths which surround the axons of neurons in the white matter of the brain. If the axons are no longer wrapped in myelin, the efficiency of the conduction of action potentials decreases. Therefore, muscles are stimulated less, resulting in fatigue and eventual paralysis (choices B and C). Likewise, incoming nervous impulses from skin sensory organs would be disrupted once they arrived at the "naked" axons. This might lead to choice D, numbness. This leaves us with choice A as the correct answer. Muscle spasms would

Copyright by The Berkeley Review

360

The Berkeley Review Specializing in MCAT Preparation

Biology

Physiology Diagnostic Set I

Answers

not be a likely symptom of multiple sclerosis because they would require excessive nervous stimuli to the muscle fibers, something that would be impossible if action potentials were not conducted efficiently. In the case of multiple sclerosis,
such excessive nervous stimuli could not occur due to the demyelination of nerve axons. The best answer is A.

43.

B is the best answer. Notice that the question asks about inducing multiple sclerosis symptoms (i.e., demyelination) in wild-type, normal mice. From the last paragraph of the passage, we learn that individuals with multiple sclerosis may have been genetically predisposed to acquire the disease. Such individuals would get symptoms after being infected by a virus
which mimics a myelin antigen. Normal individuals would not get the disease after infection by the same virus. Therefore, if we can extrapolatethis information to our animal model, the mouse, we can safely assume that wild-type animals would not be genetically predisposed to acquiring an autoimmune attack on myelin. Therefore, infection by an antigen-mimicking virus such as the one described in the passage would have no effect (on the myelin sheaths, at least). We can eliminate the other answer choices because they all represent methods that would induce multiple-sclerosis like symptoms in mice.
Choice A can be ruled out because injection of antibodies against components of the myelin sheath would cause demyelination. Choice C and choice D both also mimic the effects of multiple sclerosis by causing the destruction of myelin. The best answer is B.

44.

D is the best answer. It is essential that we understand what the question asks: would transplantation of healthy oligodendroglial cells from an identical twin result in a cure for multiple sclerosis? Although such a transplant may represent a treatment, it does not represent a cure. The underlying autoimmune disease is still present, and the newly transplanted oligodendroglial cells would eventually be attacked by the host's immune system in much the same way that the original cells were attacked. In order to actually cure the disease, we must address the autoimmunity issue; this is not a simple task, and thus multiple sclerosis remains incurable at present. Let's eliminate the other answer choices. Choice A can be ruled out because nowhere in the passage is it implied that oligodendroglial cells prevent immune system attack. They do not cure the underlying autoimmune disease. Choice B can be eliminated because although the new oligodendroglial cells might assume their normal role of repairing myelin sheaths, they would eventually be destroyed by the same autoimmune attack that killed the original cells. Thus, such a transplant represents a treatment (which may temporarily eliminate some symptoms) but not a cure for the disease. Choice C is incorrect because it states that the patient's immune system would destroy the newly transplanted cells because they came from a foreign source. These transplanted cells would not be considered foreign because they are genetically identical to the host's original cells; recall from the question that the cells were transplanted from an identical twin. The best answer is D.

45.

46.

C is the best answer. In this case we are asked about potential treatments, that is, methods that would temporarily relieve the symptoms of multiple sclerosis. Let's consider each of the statements in the context of the information given in the passage. Statement I suggests that the injection of antibodies against secreted T cell cytotoxins might pose a potential treatment. From the passage, we learn that these cytotoxins are responsible for myelin sheath degradation. We also learn that they are peptides; therefore antibodies can be manufactured which bind and inactivate the peptide cytotoxins. This might indeed constitute and effective treatment, as it could prevent degradation of the myelin sheath. Statement III is likewise a valid treatment option. T cells which recognize myelin-relatedantigens are the ones which, according to the passage, travel to the brain and attack myelin-covered neurons. Injecting antibodies which bind to and inactivate these T cells might slow or prevent the progression of the disease by preventing demyelination. Statement II, on the other hand, is not a valid treatment option. Although these antigen-mimicking viruses may be responsible for triggering the disease (according to the passage), they are not directlyresponsible for the demyelination that causes the symptoms of multiple sclerosis. Therefore, if someone already has the disease, injecting antibodies which inactivate the virus that initially triggered autoimmune attack would not accomplish anything; the damage has already been done by the virus at this stage. Such antibody injection might constitute a possibleprevention of the disease, but it would not constitute a treatment. The best answer is C. C is the best answer. The key to answering this question correctly is picking up on the word autoimmune. Rheumatic fever
is said to be an autoimmune disease. Therefore, we can eliminate answer choices which have nothing to do with the attack

47.

of the immune system on "self tissues. Choice A is therefore incorrect. Choice B is invalid because demyelination of nerve fibers innervating the heart would not cause scarring of the heart valves. Choice D, direct infection of the heart valves by viruses, is invalid because this is does not represent an autoimmune attack on the valves. Choice C is therefore the correct answer. Infection by an agent which mimics antigens present on the heart valves might provoke the immune system into mistakenly attacking the heart valves. We know this is a possibility because from the passage we learn that a similar mode of action may trigger multiple sclerosis. Recall that the question states, "Based on information given in the passage, what might trigger this autoimmune disease?" Choice C best answers this question. The best answer is C. C is the best answer. Recall that an evolutionary advantage is anything that allows an organism to better survive, adapt, and reproduce. Viruses that mimic host antigens probably evolved because they had some selective advantage. In normal individuals who are not affected by any autoimmune disease, self antigens are ignored by the immune system. Therefore, if a virus could mimic these self antigens, it might also be ignored by the immune system of its host. This would be an enormous evolutionary advantage because the virus could then reproduce and survive unchecked. We can eliminate the

Copyright by The Berkeley Review

361

The Berkeley Review


Specializing in MCAT Preparation

Biology

Physiology Diagnostic Set I

Answers

other answer choices. Choice A is invalid because antigen mimicry, from the last paragraph of the passage, does not always

lead to multiple sclerosis. Additionally, killing the host would not be the most evolutionarily advantageous event for a virus;
if the host dies, it is most likely that the virus will die too. Choice B is incorrect because the mimicked antigen appears to be a self antigen to the immune system; therefore, it couldn't act as a decoy because it would be normally ignored by the immune system. Choice Dcan also be eliminated because there is an evolutionary advantage to mimicking host antigens, as
we have already discussed. The best answer is C.

(Questions 48 - 52)
48.

Not Based on a Descriptive Passage

B is the best answer. Cardiac muscle in considered syncytial, that is, the cytoplasms of each of the individual cardiac cells can "communicate" electrically and chemically with one another almost instantly. This ability is due to gap junctions, which are transmembrane protein channels connecting each of the cardiac muscle cells to each other. We can eliminate the other answer choices on the basis that each would require that the cells be connected to each other by nerves; instead, the cells

directly contact one another and are electrically-coupled by gap junctions. The best answer is B.
49. B is the best answer. The renal afferent arteriole leads to the glomerulus, while the renal efferent arteriole leads away from it. If the afferent arteriole has a larger radius than the efferent arteriole, blood flowing into the glomerulus must leave it through a smaller vessel. This can be thought of as trying to "squeeze" a lot of liquid through a small pipe; a back-pressure is created. This pressure increases the force with which the blood is filtered through the glomerulus. The more force, the greater the rate of filtration. The other answer choices can be eliminated after consideration. Choice A is incorrect because
the diameter of the renal afferent and efferent vessels should have little effect on reabsorption in the tubules; if anything,

reabsorption would increase because more material is being filtered by the glomerulus. Choice C can be ruled out because
ADH secretion has little to do with renal arteriole diameters. Choice D is wrong because blood flow to the kidneys is increased because the afferent arteriole bringing blood in has a larger radius. The best answer is B. 50. B is the best answer. Answering this question requires background knowledge of the stages of embryonic development. Recall that after fertilization occurs, the zygote undergoes a series of cell divisions (via mitosis), dividing the embryo into a series of subcompartments. This division stage is referred to as cleavage. The next stage of development involves the formation of the blastula, a hollow ball of cells with a fluid-filled interior (the blastocoel). After this stage, gastrulation occurs. During gastrulation, the embryo is subdivided into the three germ layers: endoderm on the inside, mesoderm in the middle, and ectoderm on the outside. Hence, it is during gastrulation that the separation of cells into the three individual germ layers occurs (making choice B the correct one). After gastrulation, the formation of the notochord and the dorsal nerve chord begins. This stage is known as neurulation. Organogenesis occurs afterwards, during which the three germ

layers differentiate into the various organs of the mature organism. The best answer is B.
51. C is the best answer. Vertebrates are extremely well adapted to life on land mainly because they have solved the problem of maintaining water inside their bodies, eliminating the need to be surrounded by it. Recall from renal physiology that the loops of Henle are responsible for concentrating urine; the longer the loops, the more the urine can be concentrated and the less water is needlessly excreted. Hence, longer loops of Henle are an evolutionary advantage for life on land, as conservation of water is extremely important when one does not live in a body of water. Shorter loops of Henle would not conserve water and concentrate urine as effectively, making them least likely to contribute to the success of the vertebrates' land existence. Let's consider the other answer choices. Choice A, internal lungs, would definitely represent an adaptation to living on land versus living in the water. Internal lungs are protected from the elements and do not lose as much moisture. Choice B also represents an advantageous land adaptation, protecting the organisms internal tissues from desiccation (drying out). Choice D, internal fertilization, does indeed represent an evolutionary adaptation to life on land. In the sea, external fertilization is easy, but on land, gametes must be protected from the harsh external environment. The best answer
isC.

52.

D is the best answer. The parasympathetic is responsible for stimulating digestive functions such as the peristaltic
contractions of the smooth muscle of the stomach and small intestine. Let's consider the other answer choices. Since the

muscles responsible for the contractions of the digestive tract are predominantly smooth muscle, we can eliminate choice A. Choice B can be ruled out because the vagus nerve is parasympathetic in nature, and stimulation of the gastrointestinal tract by the vagus actually increases digestive activity. Choice C can be eliminated because during times of fear and stress, the sympathetic nervous system takes over and inhibits digestive function (so blood can be diverted to the skeletal muscles and brain: a "fight or flight" response). This means that digestive function is decreased during times of fear and stress. The best
answer is D.

,<g Copyright by The Berkeley Review^

362

The Berkeley Review Specializing in MCAT Preparation

Biology

Physiology Diagnostic Set I

Answers

Passage Topics Physiology Diagnostic Set I

Passage I Passage II Passage III Passage IV Passage V Passage VI Passage VII

Thyroid Hormone Home Pregnancy Test Oral Rehydration Therapy Botulinum Toxin Circulatory Shock Acid/Base Disorders Multiple Sclerosis

Score
>13 11-12

Physiology
45-52
40-44

10 9 8
7

37-39 34-36 31-33 28-30


24-27
22-23

6
5 <4

0-21

Copyright by The Berkeley Review

363

The Berkeley Review Specializing in MCAT Preparation

Tear Along ThePerforated Line

Tear Along ThePerforated Line

fc)
*a

fcJ cn

W CM

fc> fc)

fc) O

CO

U)

|J

O
^

CO

01

Oi

01

K3

eS) eg) eg) eg) eg) eg) eg) eg) eg) eg) eg) eg) eg) eg) eg) eg)

ck> ck> cs> cs> ck>


eg) eg) eg) eg) eg) eg) eg) eg) eg) eg) eg) eg) eg) eg) eg)

eg) eg) eg) eg) eg) eg) eg) eg) eg) eg) eg) eg) eg) eg) eg)

eg) eg) eg) eg) eg) eg) eg) eg) eg) eg) eg) eg) eg) eg) eg)

eg) eg) eg) eg) eg) eg) eg) eg) eg) eg)

eg) eg) eg) eg) eg)


2

cn o

CM

CM
si

CM

CO

00

0>

fc)

cn

^ ^

4* CM

4* fc)

^ N-

CM CO

00

CM 0*

CM cn

4*

CM CM

CM fc)

CM *-

CM o

CO

fc) 00

fc)
si

fc) 0)

eg) eg) eg) eg) eg) eg) eg) eg) eg) eg)
eg) eg) eg) eg) eg)

eg) eg) eg) eg) eg) eg) eg) eg) eg) eg) eg) eg) eg) eg) eg)

eg) eg) eg) eg) eg) eg) eg) eg) eg) eg)
eg) eg) eg) eg) eg)

eg) eg) eg) eg) eg) eg) eg) eg) eg) eg) eg) eg) eg) eg) eg)

eg) eg) eg) eg) eg) eg) eg) eg) eg) eg) eg) eg) eg) eg) eg)

I I

si Cn

si

s|

Oi
si

cn

cn
si

cn

cn

01

fc)

*-

0) CP

0) 00

0) 0)

cn

4*

CM

0) fc)

0) **

0) o

cn cp

00

cn cn

4*

cn CM

cn

cn

g) eg) eg) eg) eg) g) eg) eg) eg) eg)

eg) eg) eg) eg) eg) eg) eg) eg) eg) eg) eg) eg) eg) eg) eg)

eg) eg) eg) eg) eg)


eg) eg) eg) eg) eg) eg) eg) eg) eg) eg)

eg) eg) eg) eg) eg) eg) eg) eg) eg) eg) eg) eg) eg) eg) eg)

eg) eg) eg) eg) eg) eg) eg) eg) eg) eg) eg) eg) eg) eg) eg)
o

g) @

eg) eg) eg)

CO
si

CD

CO
cn

CO

CO
CM

00
si

o o

CO CO

CO 00

CO fc?

CO N-

00 o

00 CO

00 00

00 0)

oo cn

00

00
CM

00
fc)

00
N-

si

si

si

oo o

CO

00

si si

0>

g) eg) eg) eg) eg)

eg) eg) eg) eg) eg)


eg) eg) eg) <g) eg)

g) eg) eg) eg) eg)


g) eg) eg) eg) eg)

eg) eg) eg) eg) eg) eg) eg) eg) eg) eg)
eg) eg) eg) eg) eg)

eg) eg) eg) eg) eg) eg) eg) eg) eg) eg)
eg) eg) eg) eg) eg)

eg) eg) eg) eg) eg) eg) eg) eg) eg) eg) eg) eg) eg) eg) eg)

eg) eg) eg) eg) eg)

Notes

;....

!!

C-

::J vv v;

t---i

-;'

,.

Notes

Notes

The

ERKELEY
R'E'V-I-E-W

PERIODIC TABLE OF THE ELEMENTS

H
1.0

He 4.0
4

5 B 10.8

8 O

9 F
19.0

10

Li
6.9 11

Be 9.0 12

C
12.0 14

N 14.0 15
P

Ne 20.2 18 Ar 39.9
36

16.0 16
S
32.1 34

13
Al
27.0 21
22

17

Na
23.0

Mr
24.3
20

Si
28.1

CI
35.5 35 Br 79.9
53 I

31.0
33

19
K

23 V

24

25 Mn 54.9 43 Tc

26 Fe 55.8
44

27

28 Ni 58.7

29

30

31

32

Ca
40.1 38

Sc
45.0 39

Ti 47.9 40

Cr 52.0
42

Co
58.9 45

Cu
63.5
47

Zn 65.4

Ga
69.7
49 In 114.8
81

Ge
72.6 50

As 74.9 51

Se
79.0
52 Te 127.6
84

Kr 83.8
54

39.1
37

50.9
41

46 Pd
106.4 78
Pt

48 Cd
112.4

Rb 85.5
55

Sr
87.6
56 Ba

Y 88.9

Zr 91.2
72 Hf 178.5 104

Nb 92.9
73 Ta
180.9

Mo
95.9
74

Ru
101.1
76

Rh
102.9
77 Ir

Ag
107.9
79

Sn
118.7 82 Pb 207.2 114

Sb
121.8
83

Xe 131.3 86
Rn

(98)
75 Re

126.9
85
At

Cs
132.9

571 LaT
138.9

80

W 183.9

Os
190.2 108 Hs

Au

Hg
112

Ti 204.4 113

Bi
209.0 115

Po

137.3 88 Ra

186.2 107 Bh

192.2 109 Mt

195.1
110

197.0 200.6
111

(209)
116 Uuh

(210)
117 Uus

(222)
118 Uno

87 Fr

89 R

105

106

Ac

(223) (226) (227)

Db Rf Sg (261) (262) (266) (264)

(277) (268)

Rg (271) (272)

Ds

Cn

Uut

Uuq
(289)

Uup
(291)

(277)

(287)

(292)

(292)

(293)

58

59
Pr

60

61

62

63 Eu

64

65

66

67

68 Er

69 Tm

70

71

Ce
140.1 90

Nd

Pm

Sm

Gd

Tb

Dy
162.5 98

Ho 164.9 99
Es

Yb 173.0
102 No

Lu 175.0 103

140.9 91
Pa

144.2
92 U

(145) 150.4 152.0 157.3


93
94
Pu

158.9
97 Bk

167.3
100 Fm

168.9
101 Md

95 Am

96 Cm

Th

Np

Cf

232.0 (231) 238.0 (237) (244)

(243) (247)

(247) (251)

(252) (257)

(258) (259)

1(260)

Lr

Specializing in MCAT Preparation

Biology

If you study it, it will come!

SM

Anda mungkin juga menyukai